Вы находитесь на странице: 1из 499

..

Física
Primer Nivel

4ta Edición
Revisada, corregida y aumentada

Dirigido por .

Félix Aucallanchi Velásquez


- Potencia

OBlETlWS
1.- Entender el concepto de trabajo y recono-
cer su principal impontancia en la genera-
ción de los movimientos.
2.- Conocer y aplicar el concepto de poten-
cia en los diferentes campos de la vida
y la tecnología.

odas las máquinas -desde un abridor de latas hasta un subma-


rino nuclear, máquinas que cosen, que cortan o trituran, que
tiren o golpeen, que levanten o lancen, giren o doblen -y
todos los mecanismos que producen movimiento hacen trabajo. A pesar
de las gotas de sudor en su frente, después de horas de laboriosos empu-
jones sobre la cabeza de un elefante terco, Usted no habrá hecho ningún
trabajo sobre la bestia si ésta no se mueve, al menos en lo que se refiere
a la Física y al elefante. Arrastrar un arado en la tierra, ejercer cientos
de newtons sobre kilómetros de tierra con un costo en el-combustible o
en el quintal del arroz producido es el lado práctico de todo ésto. JAMES WAlT
(1736 - 1819)
ID! CONCEPTO DE ~AJO Ingeniero, natural de Esco-
cia. Fuéhijo de fabricantes de
Por propia experiencia sa- Instrumentosy máquinas, de
bemos que necesitamos fuerza gran talento e inventiva. En
para alterar la rapidez de un ob- 1706 dIseñóuna máquina de
vapor mejorado, pero no la
jeto, para vencer el rozamiento, Distancia había hecho funcionar hasta
para comprimir un resorte, para que fué presentado ante Mat-
moverse en contra de la grave- thew Boulton, el más grande
dad; en cada caso debe realizar- IndustrIalInglés de finales del
sIglo XVIII. Watt se sintió
se trabajo. En tal sentido, el tra- encantado de poder em-
bajo es vencer siempre una resis- plear los artesanos e Instru-
tencia. Luego, entendemos por mentos de la factoría de
trabajo a la facultad que tienen Boulton. No tardó mucho la
las fuerzas para generar movi- máquIna de Watt en subir el
agua que inundaba las
miento venciendo siempre una profundas mInas inglesas.
resistencia, sea ésta una fuerza Boulton y Watt se convirtieron
o bien la propia inercia de los en losprimeros fabricantes de
cuerpos, y sólo habrá trabajo máquinas de vapor eficIentes
sobre un cuerpo si éste se des- alardeando:
«Vendemos lo que todo el
plaza a lo largo de la línea de mundo quiere: iPotencia!•.
acción de la fuerza aplicada.
Fig 10.1
202 Física-Primer Nivel Félix Aucal/anchi V.

CUIDADO!
•• TRABAJO REALIZADO POR UNA FUERZA
Se dice y con razón, que _CONSTANTE
el trabajo es un concepto
relativo, es decir, que de- Si una fuerza mantiene siempre el mismo valor (módulo) y la misma o-
pende siempre del sistema
de referencia desde donde rientación (dirección), se dice que es constante. Ahora, cuando el punto de apli-
se haga la observación del cación de la fuerza se desplaza, se dice que la fuerza realiza trabajo, cuyo valor
movimiento. Así,una misma dependerá de la componente de la fuerza paralela a la dirección del movimiento
fuerza para distintos obser- y de la distancia recorrida. Así:
vadores ubicados en siste-
mas de referencIa también
distintos podrían tener opi-
I W=F.d, coso I (10.1)
niones diferentes para el tra-
bajo realizado por aquella. o, IW=Ftrd I (10.2)
Unidades en el S.I.:
F = newton (N)
Á' ~7J
d= lI!ctor DupUJr,amimto
d = metro (m)
F.= FIIOVJ Paralela al Dup/ar.amimto
W = joule (1) = 1J = 1 N.m
Fn= F. Cose

Significado físico de 1 Joule Fig 10.2


ATENC/ON !! Un agente realiza un trabajo de 1 joule si aplicando una fuerza
de 1 newton sobre un cuerpo logra desplazarIo una distancia de 1 m.
Sila trayectoria es curva, se
debe señalar otimeto; el Observaciones: Analizando la relación (10.1) podemos descubrir que el tra-
vector desplazamiento d , Y bajo puede ser positivo, negativo o nulo. Veamos
luego proceder a encontrar
F,I' a) Trabajo positivo.- Se le llama también trabajo motriz, y se presenta cuando
la fuerza actúa a favor del movimiento tal que:
F ti J , y , e = 00 ~ W=+F.d

b) Trabajo negativo.- Lo denominamos también trabajo resistente, y aparece


cuando la fuerza actúa en contra del movimiento, ésto es:
F iJ,J , y , e = 1800
~ W = -F .d
_ TRABAJO NULO
Desde que el trabajo de-
pende de tres factores: F, d Y a) F
~----_-:;:-,t)----'
(![}_
F )
;---O----'
cose, tendremos tres causales . )
para que el trabajo sea nulo: . h
DEBES SABER QUE: =
1m Cuando F O, el cuerpo se , d=O
mueve debido a su inercia. WHombre=O
Cuando un movimiento
curvilíneo experimenta una
2QQ Cuando d =
O, el cuerpo b)
fuerza tangenclal de mó-
puede recibir fuerza pero no
dulo constante (FJ, se se mueve (Fig 10.3.a).
verifica que para una 3m Cuando cose = O, es decir,
distancia recorrida (e), el
trabajo realizado por esta
=
e 90°, en este caso la fuerza
actúa'perpendicularrnente a v WNon.I=O
viene dado por:
la dirección del movimiento L!~~-~-~-~-~''-<:>-~~~!,,!,!!!~!!..,.,.----.-;....J
(Fig 1O.3.b). Fig 10.3
Trabajo-Potencia 203

OJO!
• TRABAJO DEL ROZAMIENTO
El valor y signo del trabajo
Cuando un cuerpo resbala so- I ~ neto nos proporciona indi-
bre una superficie áspera, experi- f. N mov rectamente una informa-
menta un trabajo por parte de la I e '-C>- ción sobre el movimiento:
fricción cinética. En los casos (a) y -.::.;.i--T.:l:-¡¡¡::="""'Ti"i~ (a) así:
(b) de la Fig. 10.4, la fuerza de roza- - =r=:=: 1] Wne10 = (+) => El movi-
miento cinético actúa en contra del miento es acelerado.
movimiento, lo cual nos permite
asegurar que su trabajo es negati- 2) Wneto = O => El movi-
vo para los observadores indicados, miento es uniforme.
verificándose en ambos casos que:
3] Wne10 = (-] => El movi-
! 'Wroz' =- fc.d.~ -pJYd I (10.3) (b)
miento es desacelerado .

• TRABAJO NETO
Llamaremos trabajo neto o total a aquel que se consigue sumando
los trabajos que varias fuerzas realizan sobr- un mismo cuerpo para un
RECOMENDACIONES
desplazamiento determinado. Así por ejemplo, de la Fig. 10.5, el trabajo
neto vendrá dado así: Para los ejercicios de
trabajo neto, dispones hasta
aquí de tres relaciones:
A --~ l:W: Rd; mad,
<,
-, pero:
,-,
= F¡d + F2d + ...+ Fnd -, 1) Usa: Rd, si entre los datos
' <, _-y figuran principalmente
= (F¡ + F2 + ...+Fn) d fuerzas.

~ IWneto=R.d I (10.5)
AB
R =
=d
LF
(Desplazamiento)
=ma
2] Usa: mad, si entre los
datos figura la acele-
ración.
o I Wneto = madi (10.6) Fig 10.5

•. GRAFICO FUERZA - VS - POSICION


En el ejemplo de laFig. 10.6, "-<>-
el bloque parte de la posición PARA NO OLVIDAR!
x = Om, y desde allí es arrastrado --:1-1-:-11--::+-1 ~I ~ :>
por una fuerza F = ION constante, O 1 2 3 4 S x(m) Cuando trabajes con grófi-
F (N) cos Fuerzas-vs-Posición,
que al llegar hasta x = 5 m, habrá debes tener en cuenta que
realizado un trabajo: las óreas tienen signossegún
la región que ocupen, así:
W = F. Llx = ION. 5 m = 50 joule
w
Al graficar el comportamiento
que tuvo F a lo largo de la recta x F
O x(m)
obtendremos una gráfica, tal que:
Area=(lON)(5m)=50 Jou1es (+)
el área bajo la línea coincide con x
el trabajo realizado. Area=Trabajo

Fig 10.6
204 Física-Primer Nivel Félix Aucollonchi V.

KILOWATT - HORA
El kilowaft es una unidad de
11I& CONCEPTO DE POTENCIA
potencia que equAlale a mil Cuando se contrata un trabajo, sin importar el tiempo que tarden
'MJffs,yel ki/oNatt-hora es una en hacerlo, se compra sólo trabajo. Por ejemplo, si contratamos a una
unidad que por naturaleza persona para que lave nuestra ropa sin indicarle el tiempo, ella lo podrá
le corresponde al trabajo, realizar en una hora, en un día o en un año, la única condición es que lo
pero es más usada como
unidad de energía eléctrica. lave todo. Pero si la condición es realizar todo el lavado en un día (día
Un kWh corresponde a 1 000 laboral de 8 horas) la persona contratada deberá hacer las cosas lo más
W liberados contínuamente rápido posible. En en el fondo, lo que pretendemos es conseguir una
durante una hora. Así pues, cantidad de trabajo por hora. Este es el lenguaje práctico de la industria
se tendrá que: y la potencia es justamente ésto, la rapidez de hacer trabajo. En una
1 kWh = (1 000 W) (3 600 s)
sociedad industrializada,las máquinas se seleccionan por la potencia
= 3,6.1()6 J que desarrollan. Si por ejemplo la máquinaA tiene mayor potencia que
la máquina B, lo que queremos decir es que: En el mismo tiempo la
máquina A desarrolla mayor trabajo que B ,0, La máquina A realiza el
mismo trabajo que B pero en menor tiempo.
11II POTENCIA MEDIA
La potencia media es aquella que nos indica sin mucha precisión
la rapidez con que en promedio se efectuó un trabajo determinado.
SIGNIFICADO F ISICO DE Luego, su valor se define así:
1 WATT . Trabajo Realizado ~ .. W
Una máquina desarrolla una
Potencia = ~lempo
T' 1 ad tc'__
emp e o en nacer
lo ::::> Pot = -t (10.7)
potencia de un watt si,
realiza un trabajo de un *) En el S.L la unidad de potencia es el watt (W), que se define como
Joule en un tiempo de un
unjoule de trabajo en cada segundo: 1W = 1 l/s.
segundo.

_ POTENCIA INSTANTANEA
Es el tipo de potencia que nos informa de la rapidez con que se
realiza trabajo en un intervalo de tiempo muy corto. Si la potencia es
mecánica, su valor instantáneo se determina así:

(10.8) IPot.= ,F.v.cosOl a = Angulo entre F yv


lB EFICIENCIA
El trabajo útil o la poten-
cia que salede una máquina
TABLA DE EFICIENCIAS nunca es igual a la de entrada.
Máquina TI ("lo) Estas diferencias se deben en
parte a la fricción, al enfria-
miento, al desgaste, contami-
Motor eléctrico s 95 nación, ..., etc. La eficiencia nos
Compresora 85 expresa la razón entre lo útil y
lo suministrado a una máquna:
Automóvil < 15 Pot •••••=PoI6til+Potpu4

Locomotora 5 - 10 (Pot) , ti.)


de vaoor 't}
%
= (Pot)suministrada
u .100 Fig 10.8
Trabajo-Potencia 205

PROBLEMAS 'RESUELTOS

Probo 1.- Un muchacho tira de un bloque de manera que éste se desliza sobre el suelo con
velocidad constante, como se muestra en la figura. Si la fuerza de rozamiento entre
el objeto y el suelo es de 20 newtons, ¿Qué trabajo realiza el muchacho para llevar el
objeto a una distancia de S metros?

A) SOjoules O) 100joules

B) 2Sjoules E) 130joules

CJ 2Sjoules UNI 84-2


Resolución.-

Descomponiendo la fuerza F que ejerce el muchacho, y


considerando que el bloque está en equilibrio dado que Yi ~e/,
se mueve con velocidad constante, tendremos: N.:-4~_-I-_-----y"/
FJ.
LFx =O ::::} F" =f ::::} FII = 20 N
F¡¡
Luego, el trabajo del muchacho estará dado por la
relación 10.2. f
p
w = + Fil. d = 20 N • 5 m::::} ~ = + 100 J RPTA.D

Probo 2.- Elcuerpo mostrado en la tigura tiene 4 newtons de peso y se desplaza con velocidad
constante una distancia de 10m sobre una superficie horizontal (coeficiente se fricción
J.l = 0.4) por acción de las
fuerzas F1 paralela al plano, y
F2 de 2 newtons, inclinada un
ángulo de 300 con respecto
a la horizontal. El trabajo
realizado por la fuerza F1 en
joules es:
A) 0,27.10-2 O) 1027.101
B)0,27.10-1 E) 10,27.102

CJO,27.1(JÓ UNI87

Resolución.-

En base al D.C.L. indicado y aprovechando que el sistema


está en equilibrio, encontraremos el valor de F1:
LFy = O ::::} N = 4 + 1 ::::} N =5 N N=SN

::::}f = !le N = 0,4 (5) ::::}f = 2 N


LFx = O::::} FI +J3 = f ::::}FI = 2 - J3 0,27 N j=2N
se
----C!>iL· -W~~-
Y finalmente determinaremos el trabajo de esta fuerza: FI
P=4N
W = FI • d = (0,27 N) (lO) ::::} W = O,27.1o-t Ioules RPTA.D
206 Física-Primer Nivel Félix Aucallanchi \1.

Probo 3.- Un cuerpo con 2 kg de masa está inicialmente en reposo en un plano horizontal y sin
fricción. Si se aplica una fuerza horizontal de 10 N por un tIempo de 10 segundos,
¿Cuál es el trabajo en joules realizado por está fuerza?

A)2500 B) 1 500 C)500 O) 4500 E) 5 000 UNI86


Resolución.-

Calculemos primero la aceleración del bloque y a continua-


ción determinaremos la distancia recorrida. Lo primero lo
averiguaremos por Dinámica, y lo segundo por Cinemática.
Veamos:
Del D.C.L. aplicaremos la relación 9.1, siendo R = ION N

ma =R ~ (2 kg) a = 10 N ~ a = 5 m/s2 ~.-


Luego, por la relación 4.8 tendremos:
1
d = 2at2 .... (vi = O) ~ d = 21 (5) (10)2 ~ d = 250 m p ~LISO
y por último aplicaremos la relación 10.1 para encontrar el trabajo.

W = F. d = (10 N) (250) ~ W = 2 500 J RPTA. A

Probo 4.- Una fuerza F actúa sobre un cuerpo de


masa 2 kg. En el dibujo se muestra dicha IF(N)
fuerza en función de la posición. Sabiendo
. que la fuerza F tiene la misma dirección y
sentido que el movimiento, detemine el
trabajo (en joules) realizado por la fuerza
entre las posiciones x = O m y x = 3 m.

A) 7 O) 10
B) 5 E) Ó x (m)
3 >-
C)8 UNI 94-1 2

Resolución.-

Se sabe que el área bajo la cuva F -vs- x nos dá el trabajo realizado. Luego:
(3m)(4N)
W = Area del triángulo = 2 ~ W = 6 Ioules RPTA. E

Probo 5.- Un cuerpo puntual de masa m se mueve


en una trayectoria circular bajo la acción A
de una fuerza constante F y una velcx;idad
instantánea V, siendo ambas tangentes a
la trayectoria. El trabajo realizado por la
fuerza sobre la masa para ir de A a B es:

A) 2 mv/RF O) mvRF
B) mv2F E) rrRF
C) mv2/R UNI87
B
Trabajo-Potencia 207

Resolución.-

Debemos reconocer que la distancia recorrida de A hasta B es igual a la mitad de la circunferencia. Luego:
e = 1tR. Seguidamente calcularemos el trabajo realizado por la fuerza tangencial en base a la relación:
Wt = + F. e (signo positivo porque F i iv)
Wt=1tRF RPTAE

Prob, 6.- Uno cocino eléctrico consume 550 vatios. y está en uso durante 15 minutos codo
mañana. ¿Cuál es el gasto diario que ocasIono, si se pago $ 0,12 por kWh?
A) $ 0,004 4 B) $ 0,042 C) $ 0,036 O) $ 0,016 5 E) $ 0,048 UNFV84
Resolución.-

Reconociendo los datos, y pasando los vatios a kilowatt y los minutos a horas, tendremos, por la relación
10.7, que el trabajo o energía consumida (W) así:

= 550 W = 0,55 kW }
POI

t = 15min = h ¡ W = Por-t = (0,55 kW)


(l )
¡h =:} W = 0,1375 kW.h

Seguidamente encontraremos el costo en soles, utilizando la siguiente regla de tres simple:

IkWh------- $0,60 }x= O,1275kW.$ 0,6


O,1375kWh---- x l kWh
=:} x = $ 0,016 S RPTA.D

Probo 7.- Uno grúa es capaz de levantar uno masa de 100 kg o uno altura de 15 m en 5 s. ¿Qué
potencio expresado en wotts suministro lo máquina? (g = 9,80 m/s2).
A) 1470 B) 2 800 C) 3 450 O) 2940 E) 7500 UNMSM94
ResoluciÓn.-

Considerando que la grúa logra levantar el bloque de B


-----------1------------
,
pesoP = =
(lOOkg) (9,8 mls2) 980N, Ycon velocidad ,
constante diremos que la fuerza F que aplica la grúa
será F = 980 N. A continuación calcularemos el traba-
jo realizado por aquella utilizando la relación 10.2:
W = (980N).(l5m) = 14 700J. Seguidamente encon-
traremos la potencia de la grúa empleando la relación
d= 15m
tv=cte t=5s
10.7 :
p
14700 J ,
Pot=--::--
5s
Pot = 2940W RPTA.D
l
,
,,
L _

Probo 8.- Lo eficiencia del motor de un yate cuyo potencio es 150 kW es 30%. Calcular lo fuerzo
en kN que debe desarrollar lo hélice propulsara poro que el yate puedo desplazarse
en el mor o rozón de 75 km/h.
A) 2,22 B) 2,20 C) 2,18 O) 2,16 E) 2,14
208 Física-Primer Nivel FélixAucallanchi V.

Resolución.-

Sea F m la fuerza propulsora de la hélice. Luego, ésta


es la que genera la potencia útil del sistema impulsor.
Por lo tanto empleando la relación (10.9) encontrare-
mos dicha potencia:

11.Pot 30.150kW
Pot
u
= ---100s = ---- 100

Pot u = 45. kW
A continuación emplearemos la relación (10.8) considerando que el ángulo formado por la fuerza impulsora
(F m) y la velocidad (v) es 0°, tendremos que:

km
45kW=Fm .75
h

Fm=2,16kN RPTA.D

Probo 9.- Unmotor eléctrico de 80% de eficiencia moviliza una grúa de 25% de rendimiento el
mismo que debe levantar cargas de 80 kg a razón de 2 rrüs. ¿Qué potencia (en kW)
debe recibir el motor eléctrico para que el sistema funcione según lo indicado?
(g = 10 mls2).

A}2 B}4 C}6 D} 8 D} 10

Resolución.-

Analizando el movimiento de la carga consi-


deraremos que la tensión (T) del cable es igual
al peso de aquella: T = mg. Luego de acuerdo
con la relación (10.9) tendremos:

Para la grúa:

Pot3 T v mg .v
11 =--.100 :=}Pot =-'-.100=--.100
g Pot2 2 110 110

80.10.2
:=} Pot2 = 25 .100:=} Pot2 = 6 400 W = 6,4 kW
Para el motor eléctrico:

Pot2
11M =-P .100 Pot. =8kW RPTA.D
ot1

Probo 10.- ¿Qué potencia tiene el motor de una bomba que eleva 18 000 litros de agua por
hora de un pozo que tiene 30 m de profundidad? Dar una respuesta aproximada.
Considere (g = 10 mls2) (1 HP = 746 watts)

A}10HP B}3HP C}1.5HP D}2HP E} 1.6 HP UNI87


Trabajo-Potencia 209

Resolución.-

Dado que la potencia del motor se desarrolla


de manera permanente, será indiferente tomar
el todo del agua o una parte de él. Por comodi-
dad utilizaremos solo a un litro de agua, recor- _--1===,-_agua
dando que l litro de agua equivale a 1 kg.
•••.•• J •
Luego, por una regla de tres simple encontra-
remos el tiempo t que emplea en subir los 30 ,'.
111 del pozo a la bomba, así tenemos:
.': ,..' ; mov
J F·····!· ..···
. . 1: .... 1,' .1: .... 1;.'.: •

.' ,
18000kg ------ Ih}t= l kg.lh l 3600 '-
'.,
18000 h= 180005
• 1',

1 kg ------ t 18000kg

Ahora, calculamos el el trabajo que realiza el motor en ese tiempo: ... ,'. b=:-::lI=:~=rI

W = F . d = P . d = ION. 30 m ~ W= 3001
, .
Finalmente la potencia del motor será: -,

3001 1HP
Pot= = 1 500 W. 746 W Pot= 2,01 HP RPTA.D
l
55

Probo 11.· Unamáquina tiene la particularidad de proporcionar una potencia equivalente al


25% de la potencia que pierde por calentamiento y obsolescencia. ¿Cuál es la
eficiencia de dicha máquina?
A)20% 8)30% C)40% 0)50% E) 75%

Resolución.-

De acuerdo con los datos se tiene que:

25
Potútil = 100 Potperd ~ Potperd = 4 Potútil

Luego, por lo visto en el esquema de potencias de la Fig. 10.8, podemos establecer que:

Potútil + Potperd = Potsum ~ Potútil +4 Potútil = Potsum


~ 5 POlútil = Potsum

Finalmente, empleando la relación (10.9) para la eficiencia, tendremos:

Potútil
5P otú til
.100 TI % = 20% RPTA.A
210 Física-Primer Nivel FélixAucallanchi V

Probo 12.- Uncajón de 5 kg se encuentra inicial-


mente detenido en un piso horizon-
tal áspero, con el cual !lk = 0,5. De
pronto es afectado por una fuerza

-r
constante F = 50 N que logra
ponerlo en movimiento. Se desea
averiguar el trabajo neto realizado
sobre el cajón cuando éste se haya
trasladado desde "A"hasta "B" (g =
10 m/s2).

A) 220J B) 330J C) 120J

O) 225J E)50J

Resolución.- 30N
Haciendo el D.C.L. del bloque y aplicando lo visto N
en los capítulos de estática y rozamiento, tendremos:
P = mg = 50 N , N = 20 N, f = !le' N = ION
f 40N
R= LFx=40N- ION
~~====~~~--------------
~ R=30N
p
Luego por la fórmula (10.5) del trabajo neto, ten-
dremos:
Wnelo = R . d .cos 0° = 30 N . 7,5m . 1

W neto = 22SJ RPTA.D A~====~==========~B


~
~ ~d ~~

Probo 13.- Del problema anterior, se desea averiguar la potencia media con que obraran las
fuerzas en conjunto durante el desplazamiento de "A"hasta "B"?
A)450w B) 451 w C)452w 0)453 w E) N.A

Resolución.-

Averiguemos el tiempo que duró el traslado del bloque "A" hasta "B", usando para ello la fórmula (4.8) del
M.R.U.V. y la segunda ley de Newton, se puede establecer que:

...... (Vi = O)

~ 75m= -l (30N)
-- (2
2 5kg

~ (=5s

y de la relación (10.7 ) para la potencia media, tendremos:

Por = W 2250 J
~ Pot=4S0 W RPTA.A
( 5s
Trabajo-Potencia 211

1.- Señalar verdadero (V) o falso (F) según corres- . C)


ponda:
() El trabajo es una magnitud escalar. F
() La potencia es una magnitud vectorial.
() La eficiencia nunca es mayor del 100%.
A) VFV B) VVV C) VFF D) VVF E) FVF

2.- Elige las palabras que completen mejor la si-


guiente oración: «La existencia de trabajo se confir- v
ma si permanentemente vencemos una y
como consecuencia de ello producimos », 6.- Dado el siguiente esquema, en donde el bloque se
A)Fuerza, aceleración D) Masa, velocidad mueve desde P hasta Q, se establece que:
B) Resistencia, movimiento E) Potencia, eficiencia () W, > O Fl
C) Inercia, equilibrio () W2 =O
3.- Dadas las siguientes afirmaciones: () W3 < O
1) El trabajo no depende de la trayectoria.
11) El trabajo es independiente del sistema de refe- Señalar verdadero (V) o falsotf')
rencia. A) FVV B) FFV C) VVV D) VFF E) FFF
11I) El rozamiento siempre hace trabajo negativo.
7.- Una persona de 60 kg sube uniformemente por
Señale lo incorrecto. una escalera en forma de caracol, y asciende verti-
calmente 611I. Luego:
A) I B) 11 C) 11I D) 1 Y 11 E) Todas
1) Su peso hizo un trabajo de -3600 J.
4.-lndicar la afirmación correcta: 11) La reacción en sus pies hizo un trabajo de + 3600J.
11I) El trabajo neto sobre la persona es nulo.
A) El kilowatt-hora es una unidad de potencia.
B) La fuerza centrípeta hace trabajo nulo. Señalar lo correcto.
C) Las fuerzas de acción y reacción hacen trabajos A) Todas B) 1 C) 11I D) Ninguna E) 1 Y 11
iguales.
8.- Sabiendo que el bloque de 5 kg desciende con
D) Cuando sostenemos un libro con nuestras manos velocidad constante desde A hasta B, hallar el trabajo
este recibe trabajo positivo. neto.
A
E) Si subimos por una escalera, la reacción normal
sobre nuestros pies hace trabajo negativo. A)+ 300 J

S.- ¿Cuál de las siguientes gráficas Fuerza-vs-veloci- B)- 150 J


dad representa mejor la potencia constante de un
C)- 300 J
motor de automóvil?
D)+ 150 J

E) Cero
R _ - D
8m
9.- Una máquina absorbe uniformemente una po-
tencia de 600 Wy eleva cargas de 31 kg a razón de 2
mis. ¿Cuál es su eficiencia?
A)96,7 %B) 103 % C)69 %
D) 81 % E) Los datos son incorrectos
212 Fisica-Prinier nivel Félix Aucal/anchi V

. PROBLEMAS PROPUESTOS

NIVELl 06.- Determinar el trabajo que realiza el peso de un


cuerpo de masa 111 = 8 kg, cuando éste cae desde A
01.- Sabiendo que en el caso mostrado el bloque se hasta B.
desplaza con velocidad constante desde A hacia B, hallar
el trabajo (en]) que realizael rozamiento, si F = 20N.
(AB = 10111). -
A)-300}
B) 400 J
-r~Crm
I

C) 200 J
A)-200
B) 150
C) 200
D)-150
E) -250
A B
D) 300 J
E) -400}
JmB'
02.- En el caso mostrado el bloque se desplaza con 07.- Determinar la potencia media, que desarrolla
velocidad constante desde A hacia B, hallar el trabajo una podadora durante los primeros 205. Si en dicho

_d'.
(cn J) que realiza F = 50 N. (AB = 10111). tiempo la fuerza que aplica varía con la posición
según como se indica.
A)·WO D) 50
A) 6 W
F(N)

I
xoo

2I2J~~)
BI E) 500
B) 7 W
CHO A B C) 8 W
(U.- Si el bloque de 5 kg se desplaza con velocidad D)9 W
constante desde A hacia B, hallar el trabajo (en ])
E? 10 'W
que rcaliva F (AB = 30 1Il).\J..l~ = 0,2 20
'i00 D) 350 ~ 08.- Encontrar la potencia media de una persona

r¿-+
A)
durante 50 s de aplicar una fuerza que varía con la
posición según como se indica.
~:::~ E) 300 A B
A) 10 W

B) 20 W 40~--..,.
o.t.- Sabiendo que el bloque de 5 kg se desplaza desde
I
A hacia B, hallar el trabajo (en ]) que realiza F = 20N. C) 30 W

ol----.-L
(AB = 15/1/). D) 40 W
x(m)
A) 240 D) 160 E) 50 W 30 ~
20
B) 200 E) 180
09.- En el caso mostrado, se sabe que la potencia de-
C)210 sarrollada es de 480 W. Si : F = 50 N /\ V = 10/1//5.
¿Cuál es la medida del ángulo O?
05.- Si el bloque tiene una masar» = 1 kg Y F = 15 N,
determinar el trabajo neto sobre él, sabiendo que el
movimiento es de A hasta B. No hay rozamiento.
A) 60}
,B
B) -30} ,
,, 10.- Una máquina pierde 60 W por medio de calor. Si
,,
C) 15} la potencia aprovechable llega a ser 140 W ¿Cuál es
13 m la eficiencia de dicha máquina?
D) -40} ,
E) 45} A) 50 % B) 70% C) 75% D) 80% E) 85'7c
A
Trabajo - Potencia 213

NIVEL 2 A) -40]
B) 40]
11.- Un cajón debe moverse 2 /1/, "jalado" por una
fuerza de ION, que forma un ángulo constante de 37° C) 60]
con la horizontal; hallar el trabajo (en}) efectuado por
O) -60]
dicha fuerza.
E) 100]
A)16 0)10
B) 14 E)20 17.- Una persona de 60 kg sube 20 m por las escale-
ras de un edificio en 411lil/.¿Qué potencia desarrolló
C)8 (en W)? (g = 101ll/s!)

12.- Un vendedor de hamburguesas le aplica una fuer- A) 55 B) 40 C) 80 O) 60 E) 50


za de 20 N a su carrito inicialmente en reposo y éste
adquiere una aceleración de 2m/s!. ¿Qué trabajo neto 18.- Un bloque se deja caer de la parte superior de
(en J) desarrolló en 10 segundos'! un plano inclinado 30° con la horizontal y liso, la
masa del bloque es 10 kg Y su velocidad media para
A) 200 B) 300 C) 500 O) 2 000 E) 3 000 todo el recorrido es 4 III/S; hallar la potencia media
(en wattsi desarrollada. (g = 101ll/s-)
13.- Un hombre empuja un bloque de 20kg, partien-
do del reposo alcanzando una velocidad de 10 m/s en A) 400 B)500 C)200 0)300 E) 100
5 segundos; si el movimiento es uni formemente va-
riado; hallar el trabajo realizado (en J). 19.- Encuentra la potencia (en kW) de una grúa sa-
biendo que eleva 60 sacos de harina de 100 kg cada
A) 200 B) 500 C)420 O) 400 E) 300
uno hasta una plataforma ubicada a 311I de altura en
14.- Determinar el trabajo (en}) que desarrolla el I minuto. (g = 10m/s!)
peso de un cuerpo de 10 kg, si el movimiento es de
"A" hacia "B". A)2

A) 450
B)4

B)500 C)6

C)490 0)3

O) 800 E)7
E) 200 20.-·Un motor consume una potencia de 1,2 kWy es
capaz de elevar cargas de 108 N de peso a 10 III/S.
15.- En la figura mostrada, ¿Qué trabajo realiza el ¿Cuál es la eficiencia del motor?
hombre para subir el paquete de 8 kg hasta una altu-
ra de 5 m con velocidad constante? (g = 10/1//s2) A) 20cm B) 60% C) 70cm O) 90% E) 75%

A) 200] 21.- Hallar 1;)eficiencia de una máquina sabiendo


que la potencia perdida equivale al 25% de la poten-
B) 300] cia útil.
C) 250] A) 60% B) 70% C) 90'7c O) 50% E) 80lff

O) 400] 22.- Un bloque de 2 kg resbala por un plano inclina-


do que forma un ángulo de 37° con la horizontal, si
E) 205] parte del reposo y recorre 6//1 en 2 segundos; hallar
el trabajo (en J) realizado por la fuerza de rozamien-
to. (g = 10 III/S2)
16.- El bloque de 4 kg, desciende con una velocidad
constante. ¿Cuál es el trabajo desarrollado por la A) -36 B) - 40 C) -52 0)-54 E)-71
fuerza" P' sobre el bloque en una distancia de 3 /I/?
(g = 10 lII/s2).
214 Física-Primer nivel Félix Aucallanchi V.

NIVEL 3 'rnen inicial del gas fue de 200m3.

23.- Calcular el trabajo neto sobre el bloque cuando se gas


desplaza desde" A" hasta" B". m = 4 kg ; ~k = 1/4;
F= 100 N; AB = 10 m. (g = 10 mIs!)
B
F : ·t
A) 150 J

B) 250 J

C) 300 J A) 200 B) 50 C) 100 D) 150 E) 120


D) 450 J 28.- Un hombre empieza ajalar una pesa de 10 kg me-
diante una fuerza constante de 60 N. Si ~= 0,6 y 0,5;
E) 500J
hallar el trabajo neto sobre la pesa transcurridos los
primeros 4 segundos. (g = 10 1I1/s1)
24.- Una fuerza de módulo constante F = 10 N, es
aplicada siempre tangencialmente a la trayectoria A) 350 J
circunferencial de radiaR = 21/1, que describe el cuer-
po sobre la cual actúa. Hallar el trabajo realizado por B) 320 J
dicha fuerza cuando el cuerpo ha dado 10 vueltas.
C) 340 J
A) I 260 J B)1256J C)1215J
D) 400 J
D) I 813 J E) I 918 J
E) 100 J
25.- En la figura, calcular el trabajo (en kJ) desarrollado
por la tensión sobre el bloque "A" desde la posición
mostrada hasta que uno de los bloques, llegue al piso. 29.- Un motor de eficiencia 11 J= 80%, acciona a una
mA = 45 kg ; /116 = 55 kg . (g = 10 m/s-) bomba centrífuga de eficiencia 11 ,= 60%; hallar la
eficiencia del sistema instalado si se sabe que ambos
A) 9,9 se encuentran trabajando simultáneamente.

B) -9,9

C) 8,5

D) -8,5
]!B .~
A) 40% B) 60% C) 48% D) 50'70

30.- Si la potencia que se pierde es el 400% de la


potencia que se emplea para el trabajo, cuál es la
eficiencia de la máquina?
E) 72%

E)6 A) 20% B) 10% C) 25% D) 30% E) 40%

26.- Calcular el trabajo (enJ) realizado por la fuerza 31.- Un tren invierte el 50% de su potencia en vencer
constante" F" de 50 N al trasladar la esferilla a lo el rozamiento y el resto en aumentar su velocidad
largo de la trayectoria curva" A BC" .. .Cuál es el coeticiente de rozamiento entre los rieles
y las ruedas del tren, si este acelera a razón de Im/s2
A) 100 C (g = 10 m/s2)
B) 150 A) 0,5 B) 0,10 C) 0,02 D)0,05 E) N.A.
C)240 3m
32.- Un ascensor pesa 6 OOON Y puede des-plazarse
D) 250 a razón de 1m/s cuando trabaja a plena carga. El
'-----:-----DA motor que lo hace funcionar, trabaja con una potencia
E) 300
de 19,5 kW. Cuál es el máximo número de pasajeros
que dicho ascensor puede trasladar? (Peso promedio
27.- La fuerzaF= 10Ncomprime al pistón tal como
de los pasajeros = 700N) (g = 10 m/s2)
se muestra en la figura hasta que el volumen del gas
sea de 100 m3. Si el área del pistón es 101/12; hallar el
A) 5 B) I C) 2 D)9 E) 10
trabajo(en J) realizado por dicha fuerza, si el volu-
Energía
OBJETImS

1.- Interpretar correctamente el concepto de


energía y reconocer sus distintos tipos.

2.- Describir los movimientos a partir de


cantidades escalares como el trabajo y
la energía cinética.

3.- Conocer la primera Gran Ley de Con-


servación: La «Ley de conservación de
la Energía»

na de las principales preocupaciones del hombre en la actua-


lidad es la de conseguir nuevas fuentes de energía; esto ha
dado lugar incluso a enfrentamientos armados. pues resulta
vital la obtención de energía para el mundo moderno en que
vivimos.Buena parte de nuestros bosques han sido dilapados para
"
obtener energía de la madera; se extrae gas y petróleo de las
profundidades de la Tierra y del mar; se almacena el agua de las lluvias'
para generar energía eléctrica; con sofisticados procedimientos se extrae
la energía de los átomos. llamada energía nuclear •...• etc.Todo ésto
con una finalidad: generar movimiento. y la energía es la clave.

DII ~ONCEPT<? DE ENERGIA JAMES PRESeOTr JOULE


Es importante reconocer que la noción de energía es un invento (1818-1889)
de la imaginación humana. pero reúne una serie de características que
le da unidad. pues al utilizarla en la explicación de los fenómenos los Nacido en Inglaterra.
Estudió en la Universidad de
hace entendibles. La energía tiene la especial característica de pasar Manchester, en donde fué
de un cuerpo a otro. o. cambiar de forma. Decimos que un cuerpo tiene discípulo de Jhon Dalton.
energía si puede realizar trabajo; así. la energía se mide por el trabajo Durante más de quince
realizado. años, el execéntrlco tctxt-
. cante de cerveza y c/entífi·
En el ejemplo de la Fig.ll.l. ea aficionado a la Físlcc
el hombre al empujar 5m al ar- luchó gran parte del tiempo
contra el consenso de le
mario aplicándole 20 N de época: La existencia dei
. fuerza. realiza un trabajo de: calorlco. A finales de 1 85e
20N.5m= 100 joules . el cuidado y rigor de sus
Luego. diremos que en térmi-' Investigaciones le permitió
nos de energía: .. establecer la equlvalencie
del trabajo y el ootot.str,
a) El hombre ha perdido 100 J duda, estos trabajos sirvie-
ran de base para el estable-
de energía. cimiento del Principio de
b) El armario ha ganado. 100 J Conservación de la Energía.
de energía llamada cinética.
Fig 11.1
216 Física-Primer Nivel FélixAucallanchi V.

TRABAJO INTERNO
Se sabe que si un cuerpo
11II TIPOS DE ENERGIA
realiza trabajo, pIerde una De acuerdo con su naturaleza, la energía puede ser mecánica,
cantidad equivalente de calorífica, eléctrica, magnética, luminosa, solar, nuclear, química, bioló-
energía. Al analizar el caso
del hombre sostenIendo
gica, ..etc. Entre las energías mecánicas más conocidas, tenemos: la
unas de las pesas (Fig. 10. 3bJ energía cinética, la potencial gravitatoria, la potencial elástica, la hidráu-
se comprobó que este no lica (agua), la eólica (viento), la mareamotríz (mareas) ...etc.
realizaba trabajo sobre las
pesas; sin embargo, su can-
sancio y su sudor nos sugie- _ ENERGIA CINETICA (Ee>
ren que perdió ener;;¡ía. Este
consumo de energla se ex-
plica porque al tensar los Cuando un cuerpo experimenta movimiento de traslación, se dice
músculos, éstos disminuyen que tiene energía, es decir, puede hacer trabajo gracias a su movimiento.
su longitud haciendo un tro- Esto lo podemos ver en el ejemplo de la Fig. 11.2, en donde el coche
baJo interno al que realizan de masa m se desplaza con relación al piso con una velocidad v. A
las fuerzas internas de una
parte de un sIstema sobre continuación choca contra la plataforma suspendida, y aplicándole una
otra. fuerza F lo empuja la distancia x hasta detenerse finalmente.Sin duda.
ésto es una prueba de que el coche hizo trabajo en virtud a su
movimiento. Entonces, llamaremos Energía Cinética a la capacidad
de un cuerpo para efectuar trabajo gracias al movimiento de traslación
que experimenta.
Se verifica que la ener-
gía cinética es siempre posi- +-d--+
tiva, depende del sistema de
referencia, y su valor resulta
se directamente proporcional
con la masa del cuerpo y con
el cuadrado de su velocidad.
"Gracias a su movimiento el coche
empuja laplataforma"
I Be = !mv2j (11.1) Fig 11.2

I!IIENERGIA POTENCIAL (EP)


Este nombre fué sugerido por William Rankine (1820- 1872), Y
es un tipo de energía almacenable y recuperable que espera ser liberada.
Esta energía existe en virtud de la posición de un cuerpo con relación
a otro, entre los que existen fuerzas de interacción. La propia experiencia
nos confirma que esta energía se va almacenando al mover un cuerpo
CUIDADO !! en contra de una fuerza, la misma que continúa actuándo incluso después
La relación (11.1) pera la e- del desplazamiento cuando el cuerpo se detiene. Por ejemplo una liga
nergía cinética solo tiene estirada contiene EP elástica, una lámpara que cuelga del techo tiene
validez silo velocidad con que
se desplaza el cuerpo es mu- EP gravitatoria, un cartucho de dinamita contiene EP química, una
cho menor que la velocidad nube cargada tiene EP eléctrica, dos imanes separados que intentan
de la luz. (e), enfrxlces su ener- unirse tienen EP magnética, los misiles nucleares tiene EP nuclear.
gía cinética esfcrá dada por: Debemos aprender a reconocer que : «La energía potencial está
Ee = (m - mJc2 almacenada en un sistema de objetos que interaccionan entre sí, de
manera que esta energía no es propiedad de cada objeto, sino del
donde: m =moso en
movimiento
sistema».
mo = masa en reposo
Energía 217

IIIIENERGIAPOTENCIAL GRAVITATORIA (EPG)


Si levantamos un macetero de 1 kg desde el piso hasta una repisa
que está a una altura de 1,2 m habremos realizado un trabajo igual a :
ION. 1,2m =12joules, para vencer la fuerza de gravedad. Esto significa
que hemos invertido 12 J en levantar el macetero, y éste ha ganado 12J
de energía, que quedará almacenado en él hasta que algún agente externo
lo libere. Así pues, cuanto más trabajo se invierte en levantar un cuerpo,
mayor es la energía que éste almacena, a la que llamaremos energía
potencial gravitatoria. No cabe duda que cuando un cuerpo libera su
energía potencial gravitatoria, ésta le permite realizar trabajo; vale
decir, el cuerpo devuelve la Energía que se invirtió en él para levan-
tarJo (Fig .11.2). Llamamos"
pu~s energía potencial ~ravita-
tona, a aquella que tiene un
cuerpo gracias a su peso (P) y
1'¡
, <K'
I'·j··.
g
I~.'''''
.
(.. -,
.
.

i:;victU:d~~~s:e~!~i~~ •·1, J I f 1h
1
~~n~~~: ••
a un nivel de referencia deter- ....•
minado Así, su valor viene dado
II
por la siguiente ecuación: __ ___
f .
(11.2) Nivelde 1« errmcia

Fig 11.3 PARA NO OLVIDAR


Observacién.: El signo de h será positivo si el C.G. está por encima del Nivel Una trayectorIa curva es
de Referencia (N.R.). El nivel cero de energía potencial se presenta cuando el cerrada sIcomienza yterml-
cuerpo se ubica en el N.R. no en el mIsmo punto. A
partir de ésto se establece
11II ENERGIA POTENCIAL ELASTICA (EpE)
que el trabajo que real/ze
una fuerza conservatlva es
una trayectorIa cerrada es
Si estiramos o comprimimos un resorte, nosotros le aplicamos nula.
un fuerza para vencer su resistencia, y así cuando el punto de aplicación
de la fuerza se desplaza una distancia determinada, diremos que se ha
realizado un trabajo, notándose ahora un resorte deformado. Hasta aquí
todo bien; pero:¿Quién se quedó con nuestro trabajo?
Indudablemente,lo almace-
nó el resorte; por ello llamaremos F
energía potencial elástica a aque-
lla que poseen todos los cuerpos
elásticos en general cuando se en-
cuentran deformados. Para el caso
de un resorte como el de laFig.ll.4,
podemos encontrar la energía EI?E
almacenada en base al trabajo
realizado, encontrándose que:

(11.3)
Fig 11.4
Observacién.« Este tipo de energía es siempre positiva, sea que el resorte
esté estirado o comprimido.
218 Física-Primer Nivel Félíx Aucallanchi V.

DEBES SABER QUE:

En las casos en que los


lIIfi ENERGIA MECANI(;A TOTAL (Em)
sistemasfísicosno presenten Si sumamos las energías mecánicas que posee un cuerpo f) sistema
cuerpos elásticos, la ener- en un punto de su trayectoria, habremos establecido una de las más
gía mecánica viene dada importantes definiciones que permitirá entender fácilmente el Principio
así: de Conservación de la Energía. Así pues, queda establecido que:
I Em =Bc + B~ +EpE 1 (11.4)

11 •
TEOJ;lEMADEL TRABAJO Y LA ENERGIA
CINETICA
Resulta conocido el hecho de que un cuerpo altere !I valor de su
velocidad por causa de la aplicación de una fuerza resultante, tal como
se explicó en el Capítulo 9 de Dinámica.Sin embargo, aplicando los
OJO! conceptos de energía cinética y trabajo podemos reconocer que' «Si
un cuerpo o sistema físico recibe un trabajo neto, experimentará un
En la relación (11.5), el cambio en su energía cinética igual al trabajo recibido». En el ejemplo
trabajo neto Incluye a todas
las fuerzasque actuán sobre de la Fig. l 1.5 el bloque experimenta u.za fuerza resultanteR. que desa-
el cuerpo o sistema físico rrolla sobre aquel un trabajo neto que viene dado or:
elegido, y aunque la de-
mostración se ha hecho Wneto Rd= mad =
considerando que el movi- donde por Cinemática:
miento es rectilíneo y uni-
formemente variado: este ad = (v: - ~~ ) /2.
teorema es stemote váll-
do,cualqulera sea la forma 2 2
de la trayectoria. mv mv
f i
Luego: Wneto =-2- - -2-

! W••••~B; • B~ J (11.5) Fig 11.5


ATENCiÓN !!
_FUERZAS CONSERVATlVAS
Los valores de la veloci-
dad y los desplazamientos Diremos que uua fuerza es conservativa si el trabajo que realiza
verticales del ejemplo de la dentro de un sistema permite cambiar las energías componentes de forma
Fig.11.ó se han formado de
la tabla salvadora mostrada tal que la energía total se mantiene constante. Asimismo, estas fuerzas
en la página IV' 97. Ahora, se caracterizan porque el trabajo que realizan no dependen de la tra)'ecto-
analizando fa participación ria; solo depende de la posición ini- - --

¡.
del peso P=20N, tenemos cial y de la posición final. Entre las ~ •.•.••
'-" " 'P!-__
que en el trayecto AB: fuerzas conservativas que encontra- v=O '-r'l!._"-v '"'PI.)-tVV
Atv\

W;!, = 20 N. 5 m = 100 J mos en la naturaleza tenemos: Las s",I P


fuerzas gravitatorias (peso), las
El valor de este trabajo coin- .fuerzas elásticas (resortes) y las ~: •
cide con el aumento produ- fuerzas electromagnéticas. Puede .. B . P v=lo.Il .(=lOO;.e:a-=300
cido en la energía cré.ca y
también con lo disminución probarse que el trabajo realizado por
que expirementó lo energía estas fuerzas se calcula así:
potencial grovitotoria:
E:(j - E;c =300J-400.1=-100J IW Peso =. ABPG :l (11.6) la:
IS", :
: wocIo

E:-400;E;'o=o
I
... 'IV=2011111
.. W~eso = -[ E:c-E;c 1 (11.7) ,
~ WIle8ork= ABpE
c: Wpeso = - 6 EpG Fig 11.6
Energia 219

CONSERVACION DELAENERGIAMECANICA MUY IMPORTANTE


En los ejemplos de la Fig.II.7 se observa que mientras disminuye En los ejemplos de la Fig.

1:¡:
la energía potencial gravitatoria del cuerpo, su energía cinética va en 11.7 puede notarse que a
aumento, de manera que la energía mecánica en A,B y C tiene el mismo pesar de existir fuerzas no
conservativas como la ten-
valor; esto se debe a' que el cuerpo se mueve en el vacío, y solo está siónen la cuerda (Fig.11.70),
sujeto a una fuerzaconservativa como es su peso. En los ejemplos de la y la normal de losplanos in-
clinados (Fig. 11. lb), la
Fig.11.7,elpénduloliberadoenAy energía mecánica se con-
la esferilla en P pueden oscilar de ,
serva, ello debido a que
manera que si no existe rozamiento, ambas no realizan trabajo
los cuerpos siempre regresan al nivel (a) A .... B por serperpendiculares a la
horizontal de los puntos de partida. '! dirección del movimiento.
Así pues, «si todas las fuerzas que e En tales circunstancias se
dice que los sistemasestán
realizan trabajo son conservativas, aisladosy son conservativos.
la energía mecánica de un sistema
se conserva».

Fig 11.7

PRINCIPIO DE CONSERVACION DE LA
ENERGIA
En 1 842 unjoven alemán de nombre Julius Robert Mayer daría el
siguiente paso de gigante en la construcción del gran edificio de la
Física, publicando un primer ensayo en el cual propuso que las distintas
formas de energía «son cuantitativam ente indestructibles y
cualitativament e convertibles». Así estableció que «Todas las
manifestaciones de la energía sal! transformables unas en otras, y la
energía como un todo se conserva».
Estoequivale a decir.«La Energía no se creani se destruye, solo se transforma».

TEOREMA DEL TRABAJO Y LA ENERGIA


ME CANICA
En el teorema del trabajo y la
Energía Cinética se utiliza el tra-
bajo neto que se desarrolla sobre
un sistema el cual incluye el trabajo
que realizan las fuerzas conser-
vativas (W c) y las no conservativas
(WNC)' Luego se establecerá que:
ltl h,
DEBES SABER QUE
Una fuerza es no conset-
votivo cuando el trabajo
que realiza depende de la
WNC + WC = ~ Ec ..... ( I )
.-' -----J,-------- trayectoria, y su presencia
puede aumentar o dismi-
Y en base a las relaciones (11.6) Y nuir fa energía mecánica
si E: =25OJ y E~=400J de un cuerpo o sistema
(11.7) el trabajo de las fuerzas con-
físico. Entre las fuerzas no
servativas estará dado así: =:) WNC= 40OJ-250J=150J conservativas más comu-
nes tenemos a la fuerza de
"La Energía Mecánica aumenta rozamiento emético. cuyo
solo si existen fuerzas no conser- princi-pal efecto es el de
I vativas haciendo trabajo" disminuir la energía mecá-
--.J
!
nica, por lo que también se
Fig 11.8 le llama fuerza disipativa.

. "';
220 Física-Primer Nivel FélixAucallanchí V.

Reemplazando (2) en (1) y despejando WNC :

(l1.8)

Resultando que se interprete así: «El cambio que experimenta la energía mecánica de un cuerpo
o sistema físico es igual al trabajo que realizan sobre él las fuerzas no conservativas».

A
Ejercicio llustrativo 1
,1
SI el bloque mostrado es dejado en libertad en A y 1
1
desciende por un plano Inclinado donde ~c = 1/4, 1
1
¿Qué velocidad poseerá el pasar por B? 1
: 301m
Resolución.- 1
1
Resolveremos este ejercicio por tres métodos diferentes con 1
la finalidad de demostrar la versatilidad de las leyes de la cl. 37"_
Dinámica vistas hasta aquí - B
ler Método.- Utilizando el D.C.L. del bloque aplicaremos la
Segunda Ley de Newton :

ma = LF - LF :::)ma = 6m - 2m :::) a = 4m/s2


a favor en contrar
de "a" de "a"

Ahora, reconociendo que se trata de un MRUV, aplicaremos


la relación (4,7) para el cálculo de la velocidad final, siendo
e=AB = 50m:
Ve = & = J2.4.50 :::) v,= 20 mIs Rpta.

2dl1 Método.- Ahora aplicaremos el Teorema del Trabajo y la Energía Cinética, para lo cual es necesario
reconocer a la resultante R de todas las fuerzas, lo cual se obtiene del D.C.L. del bloque:
R = 6m - 2m = 4m , donde m = masa del bloque.
A continuación calcularemos el trabajo neto sobre el bloque Wne10 = R.d = (4 m)(50) = 200 m. Y finalmente
aplicamos la relación (11.5):
F 1 1 2 1 2
Wnc1o=Ec -Ec :::) 2oom=2"mvr -2"m(O) :::) v,=20m/s Rpta.

3JlIMétodo.- En este último método aplicaremos el Teorema del Trabajo y la Energía Mecánica, para ello
necesitamos reconocer las fuerzas no conservativas que actúan sobre el bloque y seguidamente el trabajo
que estas realizan. Del D.C.L. podemos apreciar que son dos la fuerzas no conservativas: la reacción normal
(N) y la fuerza de rozanamiento (j), y sólo esta última realiza trabajo: Luego:

WNC = -f d= - (2 m) (50) = -100m


y eligiendo como ni vel de referencia la horizontal que pasa por B aplicaremos la relación (11.8).

WNC= E!-E~ :::) -IOOm=[!mv: +mg(0)]-[!m(0)2 +mg(30)]

2
l 2 r V
:::) -100m=2" mVr -300m:::) 2oom=moT:::) v,=20m/s Rpta.
Energía 221

PROBLEMAS RESUELTOS
Probo 1.- Una masa de 100 kg Inicialmente en reposo tiene al cabo de 5 segundos en moviento
por un plano sin fricción una energía cinética de 20.1 cf3joules. Entonces, el valor de la
fuerza constante que provoca este movimiento es en newtons :
A) 800 B) 700 C)600 0)500 E) 400
ResoluciÓn.-

En primer lugar calcularemos la velocidad final que adquiere el cuerpo utlizando la relación (11.1); así:

!mV}=E[ ~ !.lOO.V} =20.10 ~ vf=20mls

Seguidamente encontramos la aceleración que experimentó, durante el movimiento, para lo cual utilizaremos
la relación (4.5) del M.R.U.V.

a=-t-VI-Vi =-5-
20-0 ~ a=4m1s

y finalmente encontraremos la fuerza que provocó esta aceleración utilizando la 2da Ley de Newton.

F'e ma = (100)(4) RPTA.E

Prob. 2.- ¿Cuánto trabajo es requerido para levanforvertfco/mente un bloque de O,1 kg partiendo
del reposo hasta una altura de 2 metros, de manera que llegue a dicha altura con
una velocidad de 3 mis? (g = 9,8 mls2).
A) 4,32jaules B) 2.41 jaules C) 3,28 jaules O) 5,15 joules E) 1,36 jaules
UNI93-2
Resolución>
r----'
Sea W el trabajo que debe efectuarse para levantar al bloque
y WpeTtrabajo del peso. Entonces, al utilizar la relación (11.5)
podemos decir que:
,---' __.1 f=~.8""."
~=O.lkg
ti = 2/11 Q/II
P=0.98 N

~ WE-(0,98N)(2m) = ! (0,1 kg) (3 mls)2 ~


J _____
¡---¡
,
L
,
1<..---'

RPTA.B

Probo 3.- Un cuerpo de 10N de peso es levantddo desde el reposo a una altura de 10 m por una
fuerza constante de 20N. La velocidad final de la masa en mis es: (g = 10 mls2)

A) 100 B) 150 C)1OJ2 O) .J130 E)10JW UNI86


222 Física-Primer Nivel FélíxAucallanchi V.

r-----,
Resolución- ---: 1

De acuerdo con los datos reconocemos que la masa del cuerpo


es: m = Plg = 1 kg. Asimismo, por el D.C.L. mostrado
podemos reconocer que la fuerza resultante es:

R = LFy = F-P = 20 -10 =:) R = 10 N.


el = lO 11I
A continuación encontraremos el trabajo neto utilizando la 1'= ION
relación (10.5):

Wnc10 = R.d = 10 N . 10 m =:) Wne10 = 100 J

Y finalmente empleando la relación (11.5) determinaremos r---'


la velocidad final: l :"i :::O
v,= IOJ2m/s
RPTA.C

Probo 4.- Un cuerpo de 1 kg de masa se encuentra sobre una superficie /Isohorizontal atado a
un resorte cuya longitud natural es de 10 cm y de constante elástica 104 Ntrn. Si el
cuerpo es desplazado 10 cm desde la posición de equilibrio y luego soltado;
determinar la energía cinética (en joules) del cuerpo cuando la longitud del resorte es
35 cm.
A) 37,5 B) 637,5 C) 187,5 O) 112,5 E) 73
UNI88
Resolución,-

En base al esquema elaborado podemos reconocer que el


sistema es conservativo; por ello diremos que la energía k I'¡=O
mecánica se conserva. Además, la deformación inicial del
resorte es: Xi = 10 cm = lO·lm, y su deformación final: xf = ~
5 cm = 5.10.2 m. Luego. ~1
1- 40 (:11I .: 10 cm"
I

I
1
I
~I
1 P.E.
11I:::::\ :

l.
:. EF=37,5} RPTA.A : 5 CI/I 1
e ~35cm _." al

Probo 5.- Una bola de 200 gramos cae a partir del reposo. Su velocidad es de 15 mis
depués de haber caído 20 metros, ¿Cuánta energía se perdió debido a la friccIón
del aire? (g = 9,8 m/s2).
Al 11-2 J B) 16.1 J C) 12,2 J O) 21,3 J E) 32,5 J
UNI93-2
Energía 223

Resolución.-
Calculemos la energía mecánica de la bola en cada punto.
Así:

En A : E~

~
= tm(0)2
EA = 39 2J
+ mghA = (0,2 kg ) (9,8 m/s2) (20 m)
~'.
:
'l' =":
:.., '.. .., .t.
.
.
,,.
'Q '.:
.
.
I Vi~~ .: ·:

.. .~.

.'¡ .':
m ' .'
1 .. K

t
I .
h = 20/11. 1
En B: E! = ttrV} + mg (O) = (0,2 kg) (15 m/s)2 • 1

. .. . I

~'~;~'~f~~~1
. I
:;:> EB = 22 5 J
m '

Luego, la energía perdida (E) será: i ~;~.'


.~.
.NIVEL OE.
i
.... '.'
mIs

REFERENCIA . .,;
. .... . .. PISO' : "
~\\\\\\\\\\\\ \\\\\ \ \.\ \\.\ \ \\\\\\\\\ \\\\\\\\ \\ \\\\\
E= 16,7 J RPTA.B

Probo 6.- Una pelota es lanzada desde una altura de 4 m. Si rebota sin pérdidas de energía
hasta una altura de 5 m. ccémo fue el lanzamiento? (g = 10 mls2).

A) Verticalmente hacia arriba con v = 415 trvs


B) Verticalmente hacia abajo con v = 215 rrü:

C) Horizontalmente con v = 215 rtvs


O) Saltado con v = O
E) N.A. UNFV-88-1

Resolución>
Dado que la pelota rebota hasta alcanzar una altura
máxima de 5m, donde su energía cinética se hace nula,
concluímos que el lanzamiento fué vertical.
Consideraremos que el tiro fué vertical y hacia abajo.
asimismo, elegimos un nivel de referencia que pase
por A, y dado que el choque con el piso ño altera la
energía de la pelota, 'concluimos que la energía
mecánic~ se conserva; Luego:

• s A B' '1 2
2mvA
»

Em = Ein ~ =mghB

RPTA.B
224 Física-Primer Nivel FélixAucallanchl V.

Probo 7.- En la figura un carro de una "montaña rusd' parte del reposo en el punto A. Halle la
altura h, sabiendo que IQvelocidad en el punto e es 20 mis (g = 9,8 mls2).
A) 9,59 m
B) 19,18 m
"
e
C) 28,97 m 3011I

O) 8,18 m
E) N.A. UNFV 89 - 2

Resolución.-

Asumiendo que el sistema está libre de toda fricción, diremos que la energía mecánica se conserva. Utilizando
ahora un nivel de referencia que pasa por B y D, tendremos que:

e (20)2
A
Em = Em ~ "21m(O) 2
+ mghA = "21 mv A2 + mgh ~ (9,8) (30) = -2- + (9,8) h

h =9,59 m RPTA.B

Probo 8.- Si un pequeño cuerpo de masa m parte del reposo y se desliza sin rozamiento
por la superficie como se muestra en la figura, entonces la distancia recorrida (x)
viene dada por:
A)x =H -h

8)x = 2h

C)X JFih
=
--- --
O)x=2Jh{H-h)

E) x = J2 Hh UNMSM 89
. 1- x --l
Resolución.-

Utilizando la conservación de la energía mecánica, obtendremos la velocidad v de salida de la rampa.

Ahora, del movimiento parabólico estudiaremos los movimientos vertical y horizontal por separado.

De la caída libre vertical: h = !


gt2 ...••. (vjy = O) => t = J2 (h/ g) ..... (2)

Del movimiento horizontal: x = v.t ..... (3)

Finalmente de (1) y (2) en (3) : x = J2 (h/ g) .~ =>. x = lJh(H-h) RPTA.D


Energía 225

Probo 9.- En la figura, m1 = 4 kg Y m2 = 1 kg; h = 2 m. si el sistema empieza a moverse dei


reposo, ¿Cuál es la magnitrJd de la velocidad de las masas cuando se encuentran?
(g = 10 mls2).

A]4J15 mis

B] 12 mis

C] 4..[3{j mis

O] 12./2 mis

E] Tiene valores diferentes.

UNI84 -1
Resolueíén-

Dado que se trata de un sistema conservativo, diremos que la


energía mecánica se conserva en todo el movimiento. Ahora,
los bloques suben o bajan las mismas distancias en el mismo
tiempo, lo cual nos permite asegurar que ambos tienen siempre
la misma velocidad v. Utilizando el esquema adjunto y la con-
servación de la energía mecánica, tendremos:

(Eel + Ee2) + (EPGl + EPG2) = E~G


g = 10 ml.r2

y reemplazando datos : v = l2 mis RPTA.B

Probo 10.- Una esfera peq.;eña de masa m se deja erJ


libertad en el piJlto A Y recorre la stp3IfJCie
lisa. Halla la tecxxxx: normal de la Sl..PeffICie
sobre la esfera coxxx: ésto pase (:XX el ixreo
B.
A]mg

B]2mg

C]3mg

D]4mg
E] N.A. UNFV91
T26 Física-Primer NiveL FélixAucallanchi V.

ResoluciÓn.-
Para determinar la reacción normal en B (NB) se necesitará
calcular primero la velocidad tangencial (v) que la esfera tiene
al pasar por dicho lugar, y ello se podrá determinar utilizando
la conservación de la energía mecánica, en vista de que no
existen fuerzas disipativas. Así pues tendremos:

E!=E~ ~ tmy2+mg(O)= !m(O)2+mgR

~ y2=2gR ..... (1)


A continuación aplicaremos la relación (9.7) que es la 2!I;LLey
de Newton para movimientos circulares, y en base al D.CL.
indicado tendremos:
2
v
mac = I.F - I.F ~ m R = NB - O ..... (2)
van al salen del
centro centro

y de (1) en (2) : RPTA.B

Probo 11.- Unapequeña esfera se desliza a par-


tir del reposo desde el punto A. Halle
la reacción normal en el punto C.

A]mg O]3/2 mg

B] (313/2] mg E] N.A.

C]2mg
UNFV90
ResoluciÓn.-
Procediendo de un modo similar como lo hicimos en el pro-
blema anterior tendremos:
1) Cálculo de la velocidad (v) en "C" .- Por conservación de la o!'..~<!..---- o
energía mecánica: A \ 30" I

\ N 60" 1RJ2
E~ = E~ ~ t my2 = mg( f) ~ v2 = gR ~ ,C DI _
11'~ NIVEL DE
2) Cálculo de la reacción normal (N) en "C".- Utilizando la Segun-
da Ley de Newton para el movimiento circular [relación (9.7)]:
60"'~ REFERENCIA fg
<,

mg mg
ma =
e
I.F - I.F N -""2 .....(2)
van al salen del
centro centro

m.gR --N- mg
Y reemp 1azan d O ( 1) en (2) , encontramos: R 2 ~ N=3/2mg RPTA.D
Energía 227

Probo 12.- Una esfera de masa m resbala sin - - - ----_._------------,


fricción desde el punto A. ¿Cuál es

r
la fuerza resultante que actúa sobre
el punto 8? La esfera parte del
reposo en el punto A.
\
A]mg J17 O] 17 mg

8]mg

C] 15mg
115 E] N.A.

UNFV 89-1
'1 ----~
Resolución.-
En base a los dos ejercicios anteriores procedere-
mos a calcular la velocidad en B en base a la con-
servación de la energía mecánica:

EB=EA
m m
~ lmv
2 =mg(2R)
2 ~ v2=4gR

Seguidamente encontramos la reacción normal N


en "B" utilizando la relación (9.7):

v2
m¡¡ =N-O ~ N=4mg

y finalmente hallaremos la fuerza resultante (F)


en "B" en base al Teorema de Pitágoras. por t!:.a-
tarse de dos fuerzas perpendiculares (N y P)
-'--~
RPTA.A

Probo 13.- Unbloque ose pa1e del reposo res-


bala por una rampa y pierde entre
A y 8 el 10% de su energía
-----o
mecánica, por efecto del ro-
zamiento.Sien el punto de má>dma
altura su ve/ocido::f es de 5 mLs. su
altura máxima es: (g = 10 mil]

A]7m D]9J5m

8] 7J5m E] N.A.

C]9m UNFV 87
Resolución.-

Considerando un nivel de referencia horizontal que pase por B, diremos por condición del problema que:
228 Física-Primer Nivel FélixAucaflanchi V.

Asimismo, considerando el trayecto parabólico Be, donde la energía mecánica se conserva en vista de no
existir fuerzas disipativas, tendremos que la altura máxima que buscamos es la altura del punto C. Veamos:

1 9 52 9
2" mv~ + mghe = 10 mghA => 2 + 10 . he = 10. 10 . 10

he =7,75 m RPTA.B

Probo 14.- Determinar el ángulo e que define la


posición donde la esferllla lisa,
abandona la rampa circular, sa-
biendo que partió del reposo en A.

A] arc cos (1/4] D] arc cos (1/5)


B] arc cos (1/2) E} arc cos (2/3]
C) arc cos (1/3)
Resolución.-

A partir de la conservación de la energía mecánica


podemos establecer que en el trayecto de A hacia
B se verifica que:
VA =0

=> 1mv~ = mgh donde h = R (1 - cos e)

=> v~ = 2 gR (1 - cos e) ..... (1)

y del movimiento circular, aplicaremos la relación


(9,7) en el punto "B" :

v2
=> m. ; = mg cos e - o ..... (2)
van al salen del
centro centro

Luego de (1) en (2) :

2gR (l-cos e)
R = g cos e=>2 - 2 cos e = cos e

=> cos e-- 13 e = are cos (213) RPTA.E


Energía 229

11~ AUTOEVALUACION

1.- Indicar verdadero (V) o falso (F) según 5.- Considerando que es posible colocar un obs-
corresponda: táculo en 1 ó en 2, el péndulo que es liberado en A
ascenderá:
() Toda variación de energía cinética implica la reali-
zación de un trabajo. ( ) Por encima de la hori-
() La energía potencial gravitatoria es la misma, cual- zontal si el obstáculo
quiera sea el nivel de referencia elegido. está en 1.
() El trabajo de las fuerzas conservativas es siempre
nulo. () Por debajo de la hori-
zontal si el obstáculo
A) VFF B) VFV C) FVV D) FFV E) FFF está en 2. 2

2.- Elige las palabras que completen mejor la si- ( ) Siempre hasta la mis-
A
guiente oración: «La energía está almace- ma horizontal de A,
nada en un sistema de objetos que interaccionan entre cualquiera sea el lugar
sí, de manera que ésta energía no es propiedad de que ubiquemos al
cada objeto si no del », obstáculo.
A) Cinética; cuerpo D) Eléctrica; aire Señalar verdadero (V) o falso (F):
B) Potencial; sistema E) Nuclear; átomo
C) Química; líquido A) VFV B) VVF C) VVV D) FFV E)FFF

3.- Cuando un carrito de juguete empuja una pared 6.- En el esquema, el vagón se desplaza con una
agotando toda su batería se dice que: velocidad v2 = 7 mis. Si el hombre A lanza una pelota
de 2 kg con una velocidad VI = 5 mis respecto de su
1.- No hizo trabajo sobre la pared. cabeza, se afirma que:
n.- La energía se transmitió totalmente de la batería
a la pared.
IlI.- Hubo trabajo interno en el carrito.
Señala lo incorrecto:
A) I B) II C) III D) I Y n E) II Y III

4.- Los gráficos de energía cinética (Ee)' potencial


gravitatoria (EPG) y mecánica (Em) se han graficado
en función de la altura que va ascendiendo un cuerpo
que ha sido lanzado verticalmente hacia arriba. Se
pide señalar la relación que no corresponde:
1) Ee - B
( ) Para "A" el trabajo realizado es de + 25 J.
II) EPG - e ( ) Para "B" la energía cinética de la esfera es de
IlI) Em - A +49J.
( ) Para "B" el hombre" A" realiza un trabajo nega-
IV) Ee - D
tivo durante el lanzamiento.

A) I B)II Indicar verdadero (V) o falso (F)

C) III D) IV A) VFF B) VVV C) VFV D) FVF E) FFF

E) N.A. h
230 Física-Primer nivel Fé/ix Aucallanchi V.

PROBLEMAS PROPUESTOS

NIVEL! A)IO B)15 C) 20 D)30 E) 40

01.- Calcular la energía cinética del coche, si su masa 06.~Encontrar la variación (en]) que experimenta la
es m = 5 kg Y su velocidad es v = -Zi (m/s) energía cinética del bloque de masam = 5 kg cuando
pasa de A hasta B. AB = 10 m.
A) 20 1
B) 10 1
C) 151
D) 25 1
A ~'-,l---Bo--
E) 301
A) 350 B) 300 C) 250 D) 200 E) 150
02.- ¿Qué cantidad de energía potencial gravitatoria
posee el cono macizo y homogéneo de la figura, 07.- Si soltamos un bloque en A, encontrar la
respecto del piso? Masa del cono = 3 kg variación que experimenta la energía cinética del
A) 50 1 bloque de masa m = 'S kg cuando pasa de A hasta B.
AB = 10 m.
B) 40 1
A) 700 1
C) 30 1 A
D)20 1 B) 600 1 I
I
I

E) 10 1 C) 500 1 I
I

Liso :
03.- Encontrar la energía mecánica de la esfera de
O) 4001 ________
D
masa m = 4 kg Y velocidad v = 5 m/s según como se E) 3001 8m
muestra para el nivel de referencia B.

A) 80 1
--f---c------- 8.- El trabajo neto desarrollado sobre el bloque
mostrado al ir desde A hacia B es de:
4m v
B) 60 1 I

C) 50 1 B m=4kg

D) 40 1

E) 30 1

04.- Encontrar la variación que experimenta la energía B


cinética del bloque de masa In = 5 kg cuando pasa de
A) 500 1 B) 600 1 C) 700 1
A hasta B, si durante el trayecto estuvo afectado de
una fuerza resultante constante F = 20 N. A B = 10m. D) 800 1 E) 9001

09.- La energía mecánica de un sistema es de 4001, a


continuación recibe un trabajo de 3001 por parte de
fuerzas no conservativas y -200 1 de trabajo por
parte de fuerzas conservativas. Cuál es la energía
mecánica (en ])del sistema al final izar dicho proceso?
A) 8001 B) 7001 C) 6001 D) 5001 E) 2001
A) 600 B) 700 C) 800
OS.- Una paloma se encuentra volando a40maltura de
modo que su energía mecánica es el triple de su energía D)900 E) I 000
cinética. ¿Cuál es la rapidez (en m/s) con que viaja?
Energía 231

NIVEL 2 A) 2
B)3
10.- Desde cierta altura se deja un cuerpo de 2 kg
¿Cuál es su energía cinética (en k1) al cabo de 4 C)4
segundos? (g = 10/11/52)
D)5
A) 1,6 B) 1,8 C) 1,799 D) 2 E) 3,2
E)6
11.- Si el cuerpo de 2 kg demora 4 segundos en llegar 17.- Un cuerpo es soltado desde la posición mostrada.
al piso ¿Cuál es su energía potencial (en 1) respecto Hallar su rapidez cuando pase por el punto más bajo
del piso, 3 segundos después de haber sido soltado de su trayectoria. La cuerda mide 2 m (g = 10 11I/52)
(g = 10 m/s2)? rrtz:

A) 600 f· A) 2JS m/s

B) 500 B) JS /11/5
'60·
C) 800 C) 5 mis

D) 900 D) 5 J2 m/s
E) 700
E) 3 JS m/s """'---0
12.- Si un cuerpo de 2 kg se lanza verticalmente
hacia arriba con rapidez de 60111/5. Hallar su energía 18.- El bloque parte del reposo sin velocidad inicial
potencial (en /...1) respecto al nivel de lanzamiento por el camino mostrado que distancia" d" recorre en
cuando su rapidez sea de 40 m/s (g = 10 11I/52) la parte rugosa hasta que se detiene. (g = 10/11/52)
A) 2,8 B) 3 C)2 D) 0,5 E) 0,2

13.- Un estudiante de 60 kg parte del reposo en una


moto acelerando a razón de 2/11/52. Calcular su energía
cinética (en kJ) al cabo de 5 segundos.
A)3 B)2 C) 1,8 D)5 E) 1,7

14.- El bloque mostrado está apoyado en el extremo


~---- d >:
de una mesa, ¿Qué trabajo (en 1) realiza la fuerza" F'
para deformar al resorte de constante k = 4 kN/m? A)2m B) 3 m C) 4 /11 D) 5 /11 E) 6 m
Se sabe que:
19.- En la superficie cilíndrica de 5/11 de radio, el
- Longitud original del trabajo que realiza la fricción es de -900J. Hallar la
resorte = 0,7 m masa del bloque: VA = 30 mIs y vB =10 ml s
(g = 10 m/s2)
- Longitud final del resorte
=0,2 m A) I kg
A
A) 500 B) 600 C) 800 D) 250 E) 300 B) 2 kg
15.- ¿Qué trabajo (en J) se debe realizar sobre un C) 3 kg R
resorte para deformarlo 20 cm? k = I 000 N/m
D) 4 kg
A)80 B)20 C)60 D)40 E) 12
E) 5 kg B
16.- Se deja caer un cuerpo desde la posición
mostrada. ¿Cuál es la rapidez (en m/s) cuando pasa
por el punto más bajo del hoyo? Además R = 20 cm.
(g = 10 m/s2)
232 Física-Primer nivel Félix Aucallanchi V.

NIVEL 3 no inclinado. Si ambas superficies son igualmente


rugosas. Determine el coeficiente de rozamiento J..l.
20.- Una esfera de masa "m" se deja en la libertad en la
posición "A". Hallar la máxima distancia que logra A) 1/2
recorrer sobre la superficie horizontal rugosa (J..l = 0,2) B) 1/5

A)2 m

B) 3 m
·f·O C) 2/3
D) 1/3
80 cm E) 1/4
C)4m

D) 5m
.L fl 26.- La figura muestra un resorte de constante
k = 81 Nlm, que está unido en "A" a un collarín de
E) 6m 0,75 kg de masa ,el cual se mueve libremente a lo
largo de una varilla horizontal. La longitud natural
del resorte es Lo = 1m, si el collarín se deja en liber-
21.- Un bloque de masa "M" se lanzadesde "A"con
tad en "A" , qué rapidez tendrá en el punto "B"
una rapidez inicial v = J6ii mis, Hallar Hlh para
que el bloque se detenga en "B". No hay rozamiento A) 9 mis B
(g = 10 mls2)

A)2 Q /i B) 8mls

B)3 A t ---:J l C)7mls

C) 4 h liso H D) 6mls

D) 5
E) 1,3
J l E) 5mls
0,9 m
e
27.- Se lanza una esfera con 15 mls tal como se
22.- Un conductor aplica los frenos cuando su auto indica. Si su masa es 2 kg,determinar la reacción
viajaba a 201llIs.¿Qué distancia recorre antes de dete- normal de la superficie cuando en la esfera pasa por
nerse si el coeficiente de rozamiento entre las llantas y "B". Además R = 5 m. (g = 101l/1s2)
el suelo es O,5? (g = 10 mls2)
A) 50N
A) 50 m B) 3011I C) 20 m D) 1011I E) 4011/
B)20N
23.- Una bala de 20g con rapidez de 500mls, penetra
C) 30N B
25 cm dentro de un bloque de madera hasta detenerse
¿Cuál es la fuerza media en (kN)producida sobre di-
D)40N
cha bala? (g = 10 mls2)
A)5 B)IO C)40 D)8 E) 3 E)60N A

24.- Encuentre la rapidez de lanzamiento "v" del blo- 28.- El bloque de la figura de masa ore 2 kg es soltado
que de 1kg sobre el piso áspero (J..lk = 0,5) de manera desde A, y cuando el resorte presenta una deformación
que el resorte sufra una de formación máx ima de O,2m, .r = 10 CIII, la velocidad de aquel es de 16 mis.
siendo k = 1 kNlm (g = 10 mls2) Determinar la altura h (k = 400 Nlm).
A)3/Smls A) 10,0 11I
B) 4.J5mls
B) 12,0 m
C) 6/Smls
C) 13,9 m
D)7/S//1ls f- . ,
3,8m D) 13,6 m
E) Tmls
E) 13,7 m
25.- Un objeto es soltado de un plano inclinado y al
llegar a la parte inferior continúa deslizándose hasta
detenerse en una distancia igual a la longitud del pla-
Cantidad de Movimiento

OBJETIVOS
1.- Establecer un nuevo modo de enfocar los
problemas dinámicos en donde participan
dos o más cuerpos que interactúan entre
sí.

2.- Conocer y aplicar el Principio de Conser-


vación de la Cantidad de Movimiento.
3.- Reformular las leyes de Newton de la
Mecánica, en términos de Cantidad de
Movimiento e Impulso.

uando estudiamos los movimientos mecánicos, su genera-


ción, su transmisión, su conservación o variación, vemos RENE DESCARTES
que las leyes de la Mecánica estudiadas hasta aquí se ven (1596 - 1650)
muy limitadas si el número de cuerpos que participan e interactúan es
Este notable filósofo y mate-
grande, pues el estudio se vuelve complejo, de manera que predecir mátIco francés nació el 31
resultados de velocidad o aceleración finales es toda una «proeza». En de Marzo de 1596 en La Ha-
esta parte del curso incluiremos nuevas herramientas que harán más ya (Touroine). Se formó en el
sencillos y directos el análisis de tales casos; para lo cual recurriremos colegio de JesuItas de la
Fleche. Debido a su delicado
a los conceptos de Cantidad de Movimiento, Impulso y Centro de Masa. estado de salud, se le permi-
Asimismo, veremos como las tres leyes de Newton para la Mecánica tió de Joven permanecer en
se vuelven a reformular con tales conceptos. cama todo el tiempo que
deseaba antes de Ir a la es-
lID MEDIDAS DEL MOVIMIENTO cuela. A los treinta años fué
ofIcIal de campo con T11ly;
vMó después de largos viajes
Luego de un tremendo debate iniciado por los filósofos del siglo en los Paises Bajos. En 1649
XVII, se concluyó que el Universo se regía por dos grandes leyes de marchó a Estocolmo por
conservación: El de la Energía y el de la Cantidad de Movimiento, Invitación de la reIna Cristina
de Suecia. Es el prIncipal fun-
siendo ambos las dos medidas del movimiento. dador de la Nueva FIlosofía.
a) Energía Cinética (mV2/2).- Magnitud escalar cuya transmisión es tem- Los sistemas de coordenadas
poral y se hace por vía del trabajo. La energía resulta ser la medición que él Introdujo se cortan en
angulos rectos, y se designan
más genérica del movimiento, y el trabajo es la medida de su variación. con las letras; x Y-z: llevan su
b) Cantidad de Movimiento (m v). - Magnitud vectorial cuya transmisión nombre. Tuvo una prolija vida
en el campo de las matemá-
es instantánea, y se hace por vía del impulso. La cantidad de movi- tIcas. Se le atribuye la Idea Ii-
miento resulta ser la medida directa del movimiento mecánico, yel neal/zada de la inercia, deno-
impulso es la medida de su variación. mInada Momenfum Lineal;
- Todo cuerpo que se mueve
11II CANTIDAD DE MOVIMIENTO (¡;) tiende a continuar su movi-
miento en línea tecr». /o que
aporecló en sus-Principios de
Esta magnitud es conocida también como Momentum Lineal, y
Filosofía- (1644). Su obra
a lo largo del tiempo se la relacionó con el movimiento de un cuerpo. cumbre fué -Discours de la
En un inicio se le midió multiplicando el peso por la velocidad (Galileo), Methode-.
234 Física-Primer Nivel Félíx Aucallanchi V.

DILEMA el volumen de materia (masa) por su rapidez (Descartes), y finalmente


la masa del cuerpo por su velocidad (Newton). Así quedó establecido
-Escomún observar cómo
determinados cuerpos en
que la cantidad de movimiento es una magnitud vectorial que mide el
movimiento finalmente pier- grado de oposición (inercia) v = O 01 mis
den su velocidad y se detie- que presenta un cuerpo para ~
nen. SI hacemos uno exten-
sión o todo el Universo, cambiar 'u movimiento. Su ~::
concluiremos que éste de-
valor resulta ser directa-
be estor muriendo, lo cual mente proporcional con la
resulto Inaceptable, pues si masa y la velocidad de un
el Universo es obro de Dios, cuerpo:
él debe ser eteav».
Este fué el dilema que
surgió o mediados del siglo
I p = m. v 1 (12.1)
XVII entre los filósofos
europeos, lo que los empujó Unidad (S.I.): (P) = kg.m/s
o aceptar lo existencia de
uno magnitud, que relacio- Observacián.« Con la cantidad
nado con el movimiento de movimiento, el concepto de
debía ser constante(eterna). "La bala y el tren tienen la
El problema para ellos era
inercia (masa) adopta una misma cantidad de movimiento"
ponerse de acuerdo en naturaleza vectorial debido a
¿Qué se conservo: lo ener- la inclusión de la velocidad.
gía clnétlco o lo cantidad de Fig 12.1
molimiento?
Esteasunto quedó resuelto
01 descubrirse que lo ener-
I!II
IMPULSO (J)
gía (entre ellos lo cinétlca) Es conocida también como impulsión, y viene a ser una magnitud
podía cambiar de formo de tipo vectorial que aplicado a un cuerpo hace que éstos experimenten
pero en esencia seguía sien- algún cambio en su movimiento. El impulso nos indica el grado de
do constante.
efectividad que posee una fuerza para poner en movimiento a un cuerpo
o para detenerlo, en un intervalo de tiempo definido. Así pues, su valor
es directamente proporcional
con la fuerza F aplicada y con
el tiempo (M) que duró su ~
OJO!
aplicación:
Tonto en el Trabajo como
en el Impulso, lo fuerzo resul-
to ser el agente motriz, y su
IJ= F.
Át 1
(12.2)
presencio solo se explico
por lo Interacclón existente
Unidad (S.I.): (1) = N.s.
entre los cuerpos. Observaciones.« En relación al
ejemplo de la Fig 12.2. se puede
decir que:
1) Los impulsos pueden trans-
MUY INTERESANTE mitir movimientos en intervalos
de tiempo muy cortos.
SI analizamos los unidades
del Impulso, descubriremos 2) Al graficar una Fuerza ver-
que ellos son equivalentes o sus el Tiempo podemos com-
los unidades de lo Cantidad
de Movimiento. Veamos:
probar que el área bajo la curva
nos proporciona el impulso que
newton.segundo = (kg.'; ]s recibe un cuerpo en un intervalo
de tiempo dado.

~ N.s = kg.
m
s
rATea = Impulso I Fig 12.2
Cantidad de Movimiento 235

ATENCION!!
TEOREMADELIMPULSOYLACANTIDAD
• DE MOVIMIENTO 1) Por razones pedagógi-
cas, la relacIón (12.3) se
_ Según el ejemplo de la Fig 12.2, el lanzador aplica un impulso ha demostrado conside-
(J) ~ la bola de masa m, y debido a ello le cambia su velocidad de rando un movImiento
rectilíneo (Flg. 12.30).
v¡ = O a vr; esto significa que el impulso provocó un cambio en la
cantidad de movimiento de la bola. En base_al ejemplo de la Fig. 12.3a 2) En la Fig 12.3b se pueqe
podemos apreciar que la fuerza resultante R es aplicada en un tiempo notar que el Impulso ( J )
Il.t, luego, utilizando la 2da Ley de Newton tendremos: señala hacIa el Interior de
la curva (concavIdad). Su
má e R; m.
~v -
~t =.R => m(vr-v¡)=R .~t
valor puede encontrarse
mediante la relacIón (3.2)
para un vector diferencIa:
=> mVr -mv¡ =J => Pr -p¡ =} ; ó; I~=J I (12.3)
«Si sobre un cuerpo o sistema de partículas actúa un impulso
externo éste tendrá un valor igual al cambio producido en la cantidad
de movimiento del cuerpo o sistema».
(b)
:--~tr p,
: ~ p¡

p¡-rriV¡
¡--c=¡ J
: ~
, 1:>'
pr=mvr
Fig 12.3

11II SISTEMA DE PARTICULAS


Cuando un sistema físico está compuesto de varios cuerpos o
partículas, el estudio de su movimiento se simplifica si encontramos la
Cantidad de Movimiento Total del Sistema (PT) Y el de su Centro de
Masa. Resulta que el movimiento del Centro de Masa es equivalente CENmo DE MASA (C.M.]
al que realiza todo el sistema. Luego, a partir de ésto se definen:
Esel lugar del espacIo en
a)Cantidad de Movimiento del Sistema.- Por ser la cantidad de movi- donde se consIdera con-
miento una magnitud vectorial, se establece la siguiente suma: centrada toda la masa de
un sistema de partículas. La
(12.4) IPT=p.+P1+: •.+PDI ; ó ; [PT=mil+mi"l+ ...+mnvnl ubicacIón del C.M. se
determIna por medIo de las
relacIones vIstas en el item
b)Velocidad del Centro de Masa.- Se demuestra que la cantidad de 8.13.
movimiento del Centro de Masa coincide con la Cantidad de Movi-
miento del Sistema. Luego: _ Lm¡x¡

(12.5) IM T·V CM =P T I donde MT = Lm¡


x=--
Lfl1¡

- I.m¡y¡
_ Ln¡v. 1 y=--
(12.6) )'CM=~ Lm¡
1
236 Física-Primer Nivel FéJixAucallanchi V.

FUERZAS INTERNAS c)Aceleración del Centro de Masa.- La velocidad del Centro de Masa
Se considera que una varía a través del tiempo solo con la presencia de una aceleración, y
fuerza es interna sies la que ésta se relaciona con la masa de todo el sistema y con la resultante de
aplica una partícula del sis- todas las fuerzas externas del siguiente modo:
tema sobre otra que tam-
bién pertenece al mismo (12.7)
sistema. Lasfuerzas Internas
I
MT.iiT =R donde: R "LF 1,"Lm¡ii¡ = =
se caracterizan además
porque: !miil
1)La resultante de todas (12.8) QT =l'.ñt
ellas es nula dentro de un ~ I


sistema.
2) Pueden producir variacio- PRINCIPIO DE CONSERVACION DE LA
nes en la cantidad de CANTIDAD DE MOVIMIENTO
movimiento de las partí-
culas que componen un
sistema, pero no alteran De acuerdo con lo establecido en el Teorema del Impulso y la
la cantidad de movimien- Cantidad de Movimiento, se tiene que:
to del mismo.
R .!1t= Pf - y si: p¡,
R= O
(Resultante de las fuerzas externas)

FUERZAS EXTERNAS
~ 0= Pf - p¡ ~ !i¡'= ir I (12.9)
Lo que equivale a decir que: «Si la resultante de las fuerzas externas
Son aquellas que ejercen
los agentes externos al que actúan sobre un cuerpo o sistema de partículas es nula, entonces
sistema sobre una partícula la cantidad de movimiento total se conserva».
del mismo. Las fuerzas
externas pueden variar la Observaciones> Dado que la cantidad de movimiento (p ) es una cantidad
cantidad de movimiento to- vectorial, entonces el principio establecido por la relación (12.9) puede expre-
tal de un sistema. sarse de distintos modos:

111) !"L(mv)antes ="L(mv)dapuis~ (12.10)


SISTEMA AISLADO
Se dice que un sistema de 211) Al trabajar en el plano, se tiene que: v = vx +v y' Luego:
partículas es aislado si la
resultante de las fuerzas
externas a él es nula. Si un
sistema es aislado, entonces
(12.11) ! "L(mvx)antes ="L(mvx)~ 1\

desde un marco de referen-


cia inerclal se observará
que su Centro de Masa se
(12.12) I "L(mvy)antes = "L(mvy)despu& I
encuentro en reposo
(v = O J, o moviéndose con a)
velocidad constante. vA=4m1s "8=3m1s vA=3m1.r l'¡¡=4mIs
--<=>- -.....c::::- ---c:::---....c::-
Los casos mostrados en la
Fig. 12.4 están libres de fric-
ción, y por tanto correspon-
,B (B) ~;f-!32 }!¿
den a sistemas aislados, de Antes
manera que en ellos se cum-
ple que la cantidad de b)
movimiento total se con-
serva.
En el caso (c) de la Flg.
12.4, los componentes del
sistema aislado parten del
reposo, y por ello la coor-
denada x del Centro de
Masa se mantiene Inaltera-
ble en todo momento. Fig 12.4
Cantidad de Movimiento 2 7

nlOflLEMAS RESUELTOS (ptA NRTE)

Probo 1.- Una raqueta logra golpear una pelota de tenis


de 200 g desviando su dirección según como
se Indica. Si v1 = 35 mis y v2 = 75 mis. ¿Qué
fuerza media experimentó la pelota, si la
duración del contacto fué M = 0,04 s?
A) 300 N O) 100 N
B)500 N E)600 N
C)200N
Resolución.-

En base a los datos podemos reconocer las cantidades de mo-


vimiento inicial y final:
Ipll = m,vI = (0,2 kg) (35 mis) = 7 kg.mls
IP21= mV2 = (0,2 kg) (75 mis) = 15 kg.mls
Luego, utilizando el Teorema del Impulso y la Cantidad de
Movimiento dado por la relación (12.3) tendremos que el
impulso (1) que experimenta la pelota se calcula como un
vector diferencia:

::) 111 = 20 N.s

A continuación calculamos la fuerza (F) que aplicó la raqueta y que propició el impulso (J ) obtenido.
Luego, empleando la relación (12.2) que define a este último, tendremos que:

- J
F=-
/1t
IFI=!.{!=
/1t
20N.s
0,04 s
::) IF' = 500 N RPTA. B

Probo 2.- UnacrnetrcikXJorads¡xro txias de 40 g CXX'llJ)(] vekx:;icJocJ de 5aJ mis. ElsoIdcx::Jo q.Je mCTJ-
tienesujetala crnetrcikXJoraCXX'l1asmeros p.,I€iCk3 ejerceruna fuerzamáxima de 200 N sxxe
la crnetrdkXJora.DetermrYJ el rroaro rirneIO de txxs q.Je puede cisxra en lTI minuto.
A) 400 B) 500 C) 600 O) 300 E)200
Resolución.-
Se sabe que las balas inicialmente están en reposo (v¡ = O),
Y luego de recibir el impulso de la ametralladora
adquieren la velocidad de ve = 500 mis. Luego, las .Ifl·;t(.o,,:iún

cantidades de movimiento inicial y final de cada bala


es: p¡ = O, y:
Pr -= mv = (0,04 kg) (500 mis) ::) Pr = 20 kg.mls
Asimismo, reconocemos que el intervalo I:!.t que existe
entre disparo y disparo es: I1t = 1 minln = 60 s/n,
siendo "n" el número de balas percutadas en un minu-
to. Luego, por condición del problema se tendrá que
si "n" es máximo, la fuerza (F) que puede sujetar a la
ametralladora produce el impulso (1), que estará dado
por la relación (12.2):
J = F.1:!.t ::) J = (600 N). 60 s
n
238 Física-Primer Nivel FélixAucallanchl V.

Seguidamente aplicaremos el Teorema del Impulso y la Cantidad de Movimiento dado por la relación (12.3):
- 60s m m
J = Pf - p¡ ~ (200 N) -
n
= 20 kg . - - Okg . -
s s
~ n = 600 RPTA. C

Probo 3.- Si un cuerpo de 2 kg en el IQstantet = O s ,


tiene una velocidad v = - 4i (mis); se pide F(N)
encontrar su velocidad en el Instante t = 8 s, si 10 -------------
durante ese tiempo experimentó una fuerza
variable paralela al eje x y que viene dada por
el gráfic.? adjunto Fuerza - vs - Tiem~. 6
A) - 10 I (mis) D) + 22 ¡(mis)
- -
B) + 12 I (mis) E) - 52 i (mis) t(s)
C) - 14 I (mis) O 4 8
Resolución.-

En el item 12.3 se explicó que el área bajo la curva Fuerza- F(N)


vs- Tiempo nos dá el impulso (l); luego, por el esquema adjunto
podemos decir que:
6+10)
J = Area trapecio + Area triángulo = ( -2- 4 + --2-
(lO) (4)
¡
,,
~ J = + 52 N.s ~ J = 52 1 (Eje "x" positivo)
6
10!
Luego, por condición del problema, este impulso dá lugar a : t(s)
un cambio en la velocidad, y por consiguiente en la cantidad
de movimiento del cuerpo, siendo vf su velocidad final. o 4 (8)
Ahora, si utilizamos el Teorema del Impulso y la Cantidad de Movimiento, tendremos:

J = Pf - p¡ ~ J =m (Vf-V¡) ~ 521 =2 [Vf-(-41)] ~ vr =22 ¡ (mis) RPTA.D

Probo 4.- Uncuerpo, según la figura, reposa sobre una


superficie perfectamente lisa (sin fricción) y
horizontal. Su centro de gravedad está en G
como indica la figura. Siel cuerpo se inclina .G
ligeramente, cae al piso. ¿Dónde quedará
su centro de gravedad G .
A)EnP
B)Dependiendo de hacia qué lado se haya
producido el impulso, en Q o S
p Q R s T

C) En T D) En R E)Muy lejos de dichos puntos, pues no hay fricción. UNI88


Resolución.-
Despreciando el impulso inicial que permitió la
inclinación de la barra, diremos que ésta no expe-rimenta
fuerzas externas en el eje horizontal (x); por tanto, el
Centro de Masa (C.M) de la barra inicialmente estaba en
reposo en dicho eje: v =0, y conservará dicho estado en
virtud al Principio d; Conservación de la Cantidad de
Movimiento establecido para un sistema de partículas. Uso
Así:
p T
Si: Rx = LF" = O ~ Px = constante = M,vCM
Pero: vCM = Vx ~ vCM = O ~ XCM = constante' RPTA. D
Cantidad de Movimiento 239

Observación.- En el eje vertical (y) la barra experimenta fuerzas externas: Peso y Normal, que por no estar
equilibradas producen una resultante Ry que impulsa la barra hacia abajo. Luego, en dicho eje el Centro
de Masa experimenta un movimiento acelerado.

Probo 5.- Unhombre y un muchacho que pesan 800 N Y400 N respectivamente, están sobre un
piso sinrozamiento. Sidespués de que se empujan uno al otro, el hombre se a/~a con
una velocidad de 0,5 mis respecto al piso, ¿Qué cñstcmctoto: separa despues de 5
segundos?
A) 7,5 m B) 9 m C) 6 m D) 10,5 m E) N.A.

Resolución.-
El sistema formado por el hombre y el muchacho constituyen un sistema aislado, dado que sobre ellos no

el Principio de Conservación de la Cantidad de Movimiento, tendremos:

(Phombre + Pmuchacho)antes
M (O) + m (O) = M (- vH) + mVrn
= ( Pbornbre

=>
+ Prnuchacho)dcSPUéS
vm = (M/m) vH
~1
hay fuerzas externas que produzcan impulso neto, pues solo están sujetos a fuerzas internas. Luego, aplicando

H - ~ 111

.r: ANTES
donde: M = 80 kg, m = 40 kg Y vH = 0,5 mis. Luego: I'h .• '.' .........•..... ....." ~ v

vrn = (!~)0,5 => vrn = 1 mis =:}--~:;;_''';~'''':''~+~iSÓ''::¡


~
- i -- ..
A continuación calcularemos la distancia que los separa " "~- : - = DES/'U ES
,T.J",,",.. '..' =~~'"""'"",..,.."*~~~
al cabo det = 5 s, utilizando para ello el esquema adjunto
y la relación (4.1) para los espacios recorridos:
,'!l'_.,t.c, ..~.', :~' .• ' ..
t--
f.-, ... ~.• ,"". ri:
..
el! •.• -===__ ---"
d = eH + ern = (vH + vm) t = (0,5 + 1) 5 => d = 7,5 m RPTA.A

Probo 6.- Un hombre de 60 kg que vtojo de pie sobre una plataforma de 240 kg avanzan en
línea recta y horizontal a rozan de 20 mis. Siel hombre empieza a correr respecto de
la plataforma y en el mismo sentido de su movimIento a razón de 5 mis, ¿Cuál será la
velocidad de la plataforma durante el evento? Despreciar el rozamiento de la
plataforma con el piso.
A) 19 mis B) 18 mis C) 15 mis D) 12 mis E) 10 mis
Resolución.-

Dado que el sistema conserva su cantidad de


movimiento, diremos que la velocidad que tiene
la plataforma durante el evento es vp' Luego, el
hombre respecto del piso tendrá una velocidad
vp+u. Ahora, utilizando la relación (12.9) ten-
dremos:

Pantes = Pdespué s => (M +m)v = M vp +m (vp +u)

y reemplazando datos:

(240 + 60) 20 = 240 vp + 60 (vp + 5)

vp= 15m!s RPTAC


240 Física-Primer Nivel Fé/lxAucallanchi V.

Probo 7.- Marlon y Rocío se lanzan al agua simultáneamente desde una balsa. Losmódulos de
sus velocidades respectivas son iguales a 6 mis y 5 mis. y sus masas son 75 kg Y 52 kg
respectivamente. ¿Con qué velocidad se moverá la balsa, si la joven lo hace hacia el
Norte y el joven hacia el Sur? Masa de la balsa = 95 kg.
A) 2 mis: hacia el Sur B) 1 mis: hacia el Norte C] O mis: no se mueve
O) 1 mis: hacia el Sur E)2 mis: hacia el Norte
Resolución.-

Considerando despreciable la fuerza de rozamiento de la balsa con el agua, diremos que el sistema compuesto
por la balsa y los jóvenes experimentan una fuerza resultante nula (el peso del sistema se equilibra con el
empuje del agua). Luego, podemos decir que éste es un sistema aislado, y por consiguiente conserva su can-
tidad de movimiento. Asimismo, podemos reco-
nocer que antes del lanzamiento de los jóvenes el
sistema estaba en reposo (v = O); luego, aplicando M I!R
el Principio de Conservación de la Cantidad de
Movimiento dado por la relación (12.9), tendre- SUR NOHTE
mos:
~-_. -

=:) O=mM vM +mB vB +mR vR


y escribiendo escalarmente la ecuación obtenida
tendríamos:

75 (6) - 52 (5)
95 RPTA.E

Probo 8.- Una partícula de masa m es lanzada verticalmente hacia abajo desde una altura h
con una velocidad Inicial v. Si co/lslona elásticamente con una mesa de altura c < h
puesta sobre el piso. ¿Cuál es el módulo de su velocidad, justo después que rebota?
A)vclh B)v C] J2g(h-C) O) J2ghc E) J2g(h-c)+v2 UNI94-2
Resolución.-

La partícula llega a la mesa luego de caer libremente la


altura (h - e); luego, la velocidad con que incide sobre la
mesa estará dada por la relación (5.2):
-----l-r. ~g

J v--+-2-g-(-h-:"'-c-)
t~;~'=t
r (h-c) :
vi = v2 + 2 g (h - e) =:) Vi = 2

y por tratarse de un choque elástico (e = 1) Y frontal, se h


verificará que la velocidad de rebote (vr) será:
e
RPTA.E -+-
Cantidad de Movimiento 241

lfB COLISIONES DEBES SABER QUE:

Cuando un bate golpee


Llamamos así a aquellos fenómenos de corta duración, y que se una pelota de beisbol, cade
producen cada vez que dos cuerpos con movimiento relativo interactúan uno recibe del otro une
por contacto, generándose entre ellos fuerzas impulsivas variables y fuerza variable, de corto
duración, y que puede
muy intensas, las mismas que originan deformaciones y aceleraciones tomar valores considerable·
muy grandes, lo cual produce variaciones considerables en la veloci- mente grandes.
dad de los cuerpos. A pesar de ello, y por lo visto hasta aquí, las fuerzas
impulsivas son fuerzas internas, y por consiguiente la cantidad de movi-
miento total de los cuerpos es la misma antes y después de la colisión.
Según la dirección de los movimientos, las colisiones pueden ser:

A) Colisiones directas.- Se les conoce también como choques frontales


o en una dimensión, y son aquellos en donde los cuerpos se mueven
sobre una misma recta antes y después del choque (Fig. 12.5a).

B) Colisiones oblicuas» También se les conoce como choques oblicuos


. o en dos dimensiones, y se caracterizan porque los cuerpos se mueven
en direcciones o rectas distintas antes y después del choque (Fig.
12.5b).
FUERZA MEDIA

Ant~* !~.
La fuerza media que se
produce durante una coli-
sión se define como la fuer-
za constante que actuando
durante el mismo tiempo
que empleó el contacto,
produciría el mismo Im-
pulso.
Pv
Des~~ ~ Fuerza
IPn= Pn I ~
F -- Impulsivo

F -- -- -:---+--+---,
a) b

o
11II COEFICIENTE DE RESTITUCION (e)
Se le denomina también coeficiente de percusión, y viene a ser (*) Lasáreas bajo las dos cur-
vas deben ser iguales.
un número adimensional propuesto por Isaac Newton para poder rela-
cionar las velocidades relativas de dos cuerpos antes y después de cho-

de manera que: v-=-


W ~
~
{0
car. Así, el coeficiente de restitución es la razón entre estas velocidades,
i/21
--c;-
e Velocidad relativa de alejamiento
Velocidad relativa de acercamiento crE
1
v
(2
Antes
FUERZAS IMPULSIVAS

V2f -vu Dura,·te Se les denomina también


e= v -v (12.13) fuerzas impulsaras, y son las
1i 2I

En general se verifica que:


Después W
VII que se producen durante
una colisión o explosión. Su
duración es muy breve, y sus
valores son relativamente
O:S;e:S;l Fig 12.6 grandes.
242 Física-Primer Nivel Félix Aucallanchi V.

¿COMO MEDIR e?
Un procedimiento experi-
11&1 TIPOS DE COLISIONES
mental y sencillo para medir El valor de e está íntimamente vinculado con la pérdida de energía
el coeficiente de restitución cinética Así entonces, las colisiones según el valor de e pueden clasificarse en:
(e) entre dos cuerpos de
materiales A y 8 consiste en a) Colisiones elásticas.- Son aquellas en donde los cuerpos luego de la
aplicar /0 siguiente re/ación. colisión conservan la misma energía cinética. Asimismo, la deforma-
ción experimentada por los cuerpos durante el choque solo es tempo-
ral, observándose que cada uno recupera su forma original terminada
la colisión. Además se verifica que:
le=1}
b)Colisiones inelásticas.- En estos choques los cuerpos presentan de-
formaciones luego de su separación. Esto es una consecuencia del
trabajo realizado por las fuerzas impulsivas, lo que conduce a una
disminución de la energía cinética total de los cuerpos. Además se
observará que:
10<e<11
c)Colisiones completamente inelásticas.- Se les llama también choques
plásticos, y se caracterizan porque los cuerpos durante la colisión
reciben un trabajo por parte de las fuerzas internas que los obliga a
mantenerse unidos y continuar su movimiento en esa forma. Esto
nos sugiere que la energía cinética total de los cuerpos es menor des-
pués del choque, y ello debido a una fuga de energía bajo la forma
de calor. Asimismo, se comprueba que:
I e=O 1
1111!1 LEY DE REFLEXION EN LAS COLISIONES
Durante una colisión oblicua de un cuerpo con la superficie de
otro de mayor masa, como por ejemplo una pared o el piso de una
habitación, se verifica que las direcciones del movimiento cambian
dependiendo de dos factores:

1) Del coeficiente de rozamien-


to entre las superficies en
contacto (1-1).
2) Del coeficiente de restitución
existente entre los cuerpos
(e).
Así pues, puede probarse que
los ángulos de incidencia (Li)
y de reflexión (Lr) están rela-
Le
cionados así: v, - Velocidad InIciIJI
v, - Yelocldad de Rebote

Fig 12.7

(*) Esta relación es válida: 'V i> O°. Si i = 0° => r = 0°


Cantidad de Movimiento 243

ATENCiÓN
IllllVELOCIDAD DE REBOTE
Durante un choque como el
Utilizando el ejemplo general mostrado en la Fig. 12.7, podemos mostrado en la Flg. 12.7, se
determinar el módulo de la velocidad con que rebota la pelota a partir verificará que la Normal, la
Recta de incIdencIa y la
de datos específicos, por 10 cual se presentan los siguientes casos: Recta de reflexión, se
encuentran ubIcados en un
a) Dados los valores de 11y e, los ángulos de incidencia (l) y de reflexión mIsmo plano el cual será a
(r), y la velocidad de incidencia (vi)' se tendrá que la velocidad de su vez perpendicular a la
superficIe de reflexIón.
rebote (vr) es:

b) Si el piso es liso (11 = O) Y son conocidos el ángulo i, y el coeficiente


e, se tiene que:

=
e) Si las superficies son lisas (11 O) Y el choque es elástico (e=I), la
velocidad de rebote no depende del ángulo i :
l"r=V¡
=
d) Si el ángulo de incidencia es i 00, la pelota choca frontalmente con
el piso, entonces, independientemente de la aspereza relativa (11) de
los cuerpos se cumplirá que:

REFORMUlACION DE LAS LEYESDE


• NEWfON
Sir Isaac Newton, en su obra cumbre: «Principios Matemáticos
de la Filosofía Natural», postuló entre otras cosas, sus tres leyes de la
Mecánica, las cuales se han presentado en los capítulos 7 y 9. Sin em-
bargo, la esencia de los mismos en su forma original estuvieron susten-
tadas en los conceptos de Impulso y Cantidad de Movimiento vistos
recién aquí; de este modo estas leyes se enunciarían así:
llí1) Ley de la Inercia.- «Si sobre un cuerpo no actúa ningún impulso
externo, dicho cuerpo conservará su cantidad de movimiento».
Si J = O => =
P constante
2¡!¡¡)Ley de la Fuerza.- «El ritmo de cambio de la cantidad de movi-
miento de un cuerpo es igual a la fuerza neta aplicada, y tiene
lugar en la mi-sma dirección».
·li = !!.p ;ó : li = Mmv)
Al Al
3m) Ley de la Acción y la Reacción- «Los impulsos que experimentan dos
cuerpos que interactúan entre sí producen en ellos la misma variación
en su cantidad de movimiento, pero en direcciones opuestas».
- -
J acción = - J reacción
244 Física-Primer Nivel Féllx Aucal/anchi V.

PROBLEMAS RESUELTOS (ZN PARTE)

Probo 9.- Una pelota cae vertIcalmente al piso, y al rebotar en él se observa que solo se eleva
hasta la mitad de la altura inicial. SI la velocidad justo antes del choque es de 20 rnts.
¿Cuál es la velocidad después del Impacto?
_ A] 12,1 rnls 8] 13,1 mts C] 14,1 rtüs O] 15,1 mts E] N.A. UNFV 91

Resolución.-

De acuerdo con los datos se tiene que las alturas inicial (h.) y final (hf) logradas por la pelota son tales que:
h, = h(2. Luego, el coeficiente de restitución entre la pelota y el piso estará dada por la relación experimental:

e= {h; = Jh¡12 e=-


J2
V-';; h¡ 2

Seguidamente calcularemos la velocidad del rebote (vr), reconociendo que la velocidad de incidencia es v. =
20 mis y que el choque es frontal, luego por la relacion dada en el item 12.11 para estos casos, tendremos:

=> vr - 14,1 mis RPTA.C

Probo 10.- La figura muestra la colisión de los


bloques 1 y 2. Entonces, el coefi- 20mls 12m/s 16m1s
ciente de restitución entre los ---<:>- FO ---<:>- ---{:::>
bloques es:

A] 0,1 E] 0,5 1 2 1
0]0.4 C]O,3
8]0,2 UNI94-2
Resolución.-

Aplicando directamente la relación (12.13) para el cálculo del coeficiente de restitución (e) tendremos:

V2! - vlf (16m/s) - (12m/s)


e= = e =0,2 RPTA.B
(20m/s) - (Om/s)

Probo 11.- Dosmasas m y 2m se desplazan con movimiento uniforme sobre una misma recta, coll-
. '.~ .sIonando elásticamente. SIpara la masa 2m la velocidad final es el doble de la inicial, la
relación (velocidad finaljl{ve/ocidad inic/alj para la masa m en valor absoluto es:

A] 1/5 8] 112 C] 1 O] 2 E]4 UNI94-1


Resolución.-

=
Sean mi =.m y m2 = 2 m las masas de los bloques; asimismo, sean Vii' vIP v i = V Y v2f 2v sus velocidades
inicial y final respecti vamente para cada bloque. Ahora, utilizando primero ¡a relación (12.13) para el coefi-
ciente de restitución (e = 1), y luego el Principio de Conservación de la Cantidad de Movimiento, tenemos:
Cantidad de Movimiento 245

=> v¡i+V¡f=3v ..... (I)

b) m¡vli + m2v2i = m¡vlf + m2v2f


=> mV¡i + 2 m.v = m,vlf + 2 m.2 v

=> v¡i-vlf=2v (2)

y resolviendo (1) y (2) obtenemos: vii = %v y v¡! =~ RPTA.A

Probo 12.- Dos cuerpos Inelástlcos tienen uno masa total de 12 kg, moviéndose en sentidos
opuestos con velocidades de 4 mIs y -6 rtüs. Colislonon y adquieren uno velocidad
común de + 114rrüs. ¿Enqué relación están los masas de los cuerpos?
A) 614 B) 513 C) 813 O) 615 E)514 UNI 83 - 1
Resolución>

Utilizando directamente la relación (12.10) para el Principio de Conservación de la Cantidad de Movimiento,


y siendo el choque completamente inelástico (los cuerpos quedan unidos), tendremos que:

m¡lI¡ +~v2 = (m¡ +~) v => m¡ (+4)+~(- 6)= (m¡ +~) (+¡)

mI _.!
15m¡ = 25~ mz - 3 RPTA.B

Prob. 13.- Uno bolo de masa m se disparo contra un bloque de masa M como se muestro en
lo figuro. Después de lo colisión el centro de masas del conjunto (m + M) se desplazo
hasta uno altura h. Encuentro lo velocidad de lo bolo en función de m, My h.

A) m+M J2 gh O) m+M JQii


m m

E) M J2 gh
m

C) J2~gh UNI89

Resolucién>
I I
Averigüemos primero la velocidad v· que adquieren el bloque y la bala juntos inmediatamente después del
im-pacto de ésta sobre aquella, para lo cual utilizaremos la Conservación de la Energía Mecánica, dado que
el sistema no experimenta trabajo por parte de las fuerzas externas no conservati vas (por ejemplo, la tensión
en las cuerdas). Luego:

E~ = E;' => t(m+ M) v; = (m+ M) gh


246 Física-Primer Nivel FélixAucallanchi V.

~ Vi = Jz gh ..... (1)

y ahora, emplearemos el Principio de Conservación de la


Cantidad de Movimiento para el momento del impacto de la
bala contra el bloque:

(Pbala + Pbloque ) antes = (Pbala + Pbloque) despué s

~ mv+M(O)=(m+M)v¡ (2)

Reemplazando (1) en (2) y depejando la velocidad de la bala


(v), tendremos: (M+ m)

RPTA.A

Probo 14.- Sabiendo que el sistemaporte del reposo


¿Conqué velocidad abandona el bloque
a la rampa que es curva? No hay
rozamiento y además: h= 5 metros,
M = 9 m y g = 10mfs2.
r
h
m

A) 8 mis D) 9 mis
B) 18 mis E)4 mis
C) 16 mis
Resolución.-
En primer lugar debemos reconocer que por la ausencia de una fuerza resultante externa, el sistema constituído
por el bloque y la rampa conserva la cantidad de movimiento, tal que su velocidad vr estará dada así:
M (?) + m (O) = M ( - vr) + mv
m
~ vr = M v ..... (1)

Asimismo. notamos que el sistema no experimenta


trabajo por parte de fuerzas no conservati vas, por con-
siguiente la energía potencial gravitatoria se convierte
en energía cinética de la rampa y del bloque mismo.
Así pues:

l 2 l 2
mgh = -mv
2 +-
2 Mv r (2)

Reemplazando (1) en (2) y despejando v, tendremos:

v=(~)J2ih
M+m
~ V = 9 mis RPTA.D
Cantidad de Movimiento 247

1.- Se tienen tres cuerpos moviéndose en línea 6.- Suponga que usted está en el centro de un lago
recta, de manera que: helado. Entonces, para salir de él:

A: mA = 6 kg, vA = 5 mis 1. Camina hacia afuera.


n. Se dá volantines en el piso.
B: mB = 3 kg, vB = 9 mis
III. Se saca el abrigo y lo lanza hacia adelante.
C: me = 12 kg; ve = 2,5 mis Indique lo incorrecto:
Entonces, el cuerpo que menos se resiste a cambiar
la dirección de su movimiento es: A) I Y III B) n y III C) 11I D) 1 Y n E) Todas

A) A B)B C)C D)AyB E) AyC 7.- Complete correctamente la siguiente oración:


«Un cuerpo puede poseer y no necesa-
2.- Usted debe recibir entre sus manos una biblia riamente »,
de 1 kg soltada desde 2 m de altura. Entonces, para
A~ Velocidad; energía.
que el impacto le produzca el menor dolor, el contac-
to con ella.deberá ser: (elija un valor) B) Energía; cantidad de movimiento.
C) Aceleración; impulso.
A)lO-ls B)lO-4s C)2.1O-2s D) 10-2 s E) 10-3 S
D) Cantidad de movimiento; energía.
3.- Una persona está parada sobre una balanza y E) Masa; energía.
ésta da una lectura L. Luego la persona toma un im-
pulso hacia arriba y salta. Entonces: 8.- Si las esferas mostradas son idénticas y elás-
ticas, después del choque las bolas que se moverán
( ) L no cambia de valor. son:
) L primero aumenta y luego se anula. A) D,E

) L disminuye de valor durante el impulso. B) C, D, E


C) B,C,D,E
Señalar verdadero (V) o falso (F):
D) E
A) FyV B) VFV C) FVF D) FFV E) FFF
E) Faltan datos
4.- Un hombre aplica con sus manos una fuerza
neta de 200 N sobre una pared durante 0,45 s. Luego, 9.- En la figura se muestran dos bolas de billar. Si
el impulso recibido por la pared es en N.s: "B" está en reposo, ¿qué gráfico indica mejor las
trayectorias antes y después del choque elástico?
A)9 B)90 C) 900 D)45 E) O

5.- ¿Por qué un buen futbolista obtiene la máxima


velocidad y recorrido con un tiro suave?

A) Por una menor fuerza.

B) Por una mayor fuerza.

C) Por un mayor tiempo de contacto. D)


D) Por un menor tiempo de contacto.
E~
~
E) Por un menor impulso.
248 Física-Primer Nivel Félix Auco/lonchi V.

PROBLEMAS PRO'UESTOS

NIVEL! B) La velocidad del centro de masa. (en mis)

01.- Una fuerza de 20N actúa sobre un cuerpo durante 8 mis 5 mis'
0,5 segundos. Luego, el cambio (en kg.mls) que
produce en la cantidad de movimiento de aquel
(r------- ··15
será . A) 1;3 B)2;3 C) 3;2 O) 6; l,5 E) 26; 1
A) II B)9 C) 10 O) 12 E)20
08.- Dado el siguiente sistema de partículas de ma-
02.- Un cuerpo de 6 kg se desplaza rectilíneamente sas iguales (m = 2 kg), se pide encontrar para el
con una velocidad IV 11 = 4 mis. Si luego su velocidad instante mostrado:
se convierte en 1v,l = 3 mis, ¿Cuál es el impulso A) La cantidad de movimiento total. (en kg. mis)
(en Ns) que recibiÓ si : VI tt 2 v B) La velocidad del centro de masa. (en mis)
A)-6 B)12 C)6 0)18 E)-20
A) 5;10
03.- Un cuerpo de 6 kg se desplaza rectilíneamente B) 14;4 ...~
con una velocidad Iv 11 = 4 mis. Si luego su velocidad . 10m/s
se convierte en Iv 21 = 3 mis, ¿Cuál es el impulso C) 12;3
/~ 37°
(en Ns) que recibió si : V 1 .1 v 2 O) 3;7 ~ ......• ,e .

A) 15 B) 30 C) 24 O) 18 E) 36 E) 4;17 8m1s
04.- Un cuerpo de 6 kg se desplaza rectilíneamente 09.- Calcular el coeficiente de restitución (e) en la
con una velocidad Iv 11 = 4//1ls. Si luego su velocidad siguiente colisión:
se convierte en Iv 21 = 3 mis, ¿Cuál es el impulso
6 mis 2 mis 3 mis 5 mis
(en Ns) que recibió si : VI t.L v 2 "'--C:> -<J.---J"< -<:--'" "'--C:>
A) 40 B) -36 C) -18 0)36 E)-42 (i) (2) (i) (2)
05.- Un cuerpo de 6 kg se desplaza rectilíneamente Antes Después
con una velocidad Iv 11 = 4//115. Si luego su velocidad
se convierte en Iv ,1 = 3 mis, ¿Cuál es el impulso A)0,5 B) I C)O,4 O) 0,2 E) 0,8
v
(en Ns) que recibió si : V 1 Y 2 forman 60°
10.- Si el piso es liso, hallare en el choque mostrado.
A) 2 m B) 6 C) Ji O) 6Ji3 E) 4
A) 1/2
06.- Si un sistema físico experimenta una fuerza B) 6/11
externa F que varía con el tiempo según como se
muestra en la figura, hallar el impulso (en Ns) que C)4/9
recibe el sistema entre I = Os y I = 105. O) 5/8
A)6 E) 9/16
B) 1811: F~N)
6 ------- 11.- Si el rebote es elástico, hallar el coeficiente de
C) 611: rozamiento (J-1) en el siguiente choque.
0)36
E) 36 rt O 12
t (s)
~
A) 1/2
B) /3/2
~"cb 1i¡11
07.- Dado el siguiente sistema de partículas de masas C) /3 /4 -,
,
i.
iguales (m = 2 kg), se pide encontrar para el instante
;,,6
, I

mostrado: O) 1/4 30°


A) La cantidad de movimiento total. (en kg. mis) E) 2/3
Cantidad de Movimiento 249

NIVEL 2 A) 8 mis

12.- Para detener a un cuerpo es necesario aplicarle B) 12 mls


un impuso de 300 N.s. Si solo se dispone de una
fuerza de 1,5 N, ¿Durante qué tiempo será necesario C) 41111s
aplicar la fuerza para conseguir dicho propósito?
O) 14 II1ls
A) 200 s B) 250 s C) 100 s O) 150 s E) 300s
E) 6 mis
13.- Un balón de futbol de 0,5 kg se lanza con una
velocidad de 20 II1ls. Un guardavallas desea retenerlo 18.- Dos bolas chocan de modo que luego del impacto
pero sin sentir mucho dolor, para lo cual lo "embolsa" v = 4 mis y v., = 6 mis, siendo r», = 5 kg Y mJ = 4 kg.
retrocediendo un par de pasos. Si el proceso duraü.c s, J:uál es la medida del ángulo 8, si las trayectorias
¿Qué fuerza media recibió el guardavallas? son las indicadas?

A) 20 N B) 25 N C) ION O) 35 N E) 30 N A) 35°

14.- Un cuerpo de 6 kg es afectado por una fuerza B) 36°


constante durante 10 s, tal que al final de la apli-
cación adquiere una velocidad de 20 mis. ¿Qué valor C) 37°
posee dicha fuerza?
O) 38°
A) II N B) 13N C) ION O) 12N E)N.A
E) 39°
15.- Una bola atada a una cuerda gira con M.C.U.,
siendo su velocidad tanaencial de 4TlIls. Si su masa es 19.- Una bola golpea a otra que se encontraba
de 2 kg, ¿Qué impulso ~'ecibe por parte de la cuerda inicialmente en reposo, de modo que cada una se
al pasar de A hasta B? separa con velocidades de 3m!~ y 5 mis y de manera
que sus direcciones forman 60°. ¿Cuál es el valor de
vo? Las masas de las bolas son iguales.
A) 2.16 N.s
B A) 51/11s
B) 3 2 N.s

C)2J3N.s B) 4mls
\
\
\
O) 5 2 N.s 1 C) 6mls
1

E) 8J2 N.s O) 71111s

16.- Sabiendo que el choque es inelásticos con e = 0,6 E) N.A


Y las bolas son idénticas, ¿Qué velocidad presenta la
bola 2 después del choque? No hay rozamiento. 20.- Dos coches' de igual masa se desplazan con
velocidades VI = 12mls y VJ = 16mls por rectas que
A) 5,2mls (-7) se cortan perpendicularmen-te. Si ellos colisionan de
modo que quedan unidos. ¿Qué ángulo El forma la
B) 6,2mls
o
(~) dirección de su movimiento con el eje y después del
choque?
C) 6,41111s (-7) Reposo
Yl
le / "
A) 35°
O) 7,4l11ls (~) VI
\L-...c> ~/
B) 37° ,/

E) N.A
-- -----.i-'-----------
: x

v,jé
C)36°
17.- Sabiendo que las masas de la pluma y el bloque
son iguales y que ellos quedan adheridos después 0)38°
de la colisión, ¿Cuál es la velocidad final del sistema?
E) 39°
250 Fisica-Primer nivel Félix Aucallanchí V.

NNEL3 25.- Un carro de masa M = 500 kg va con una


velocidad v = 15 I/I/S. ¿Con qué velocidad v· en m/s
21.- La figura muestra tres partículas de masas iguales debe dispararse en el mismo sentido a una masa
(1 kg) sobre las que actúan las fuerzas indicadas: m = 50 kg para que el carro retroceda en la misma
I~I= 15 N, I~I= 3 N, I~I= 6 N. Lucgo,la acele- recta y con la misma rapidez inicial? (No hay fricción y
todas las velocidades se miden respecto de la Tierra).
ración del centro de masa es: (en m/s2)

A)(3; J2) y A)150 ~

B)(2;1) B) 205

C) (1 ; 2) C) 580

0)250
M
0)(2; 2)

E) (6; 3) E) 315

x
22.- Una granada se desplaza horizontalmente a razón 26.- Si e = 0,5, determinar el número de impactos y
de 10 II//s, y explota según como se indica. Si el la velocidad al cabo de 5 segundos (El tiempo se
fragmento central se mueve a razón de 18/11/s, ¿Qué toma a partir del primer impacto) J.l = O.
velocidad en /II/s posee el fragmento inferior?
A) 10

B) 8
y:
:3
I ¿ m
A) 1 ; 8 /II/S

B) 2 ; 4 m/s
16m/s
~ : / C) 3 ; 2/11/s ~
C) 12
_~: ~~~1!°
__ ~2_ 0)4;1/11/5
el
~ :'~600 ~
O) 16
mI', t 8m +-
E) N.A. i '0.:::/3 E) 5 ; 0,5 m/s

~ '\ 27.- En la figura, las masas están en reposo y el


23.- Una granada estalla en el aire dividiéndose en choque es elástico. Si no hay rozamiento, ¿Cuál es la
dos fragmentos de masasm, y 1/12 (1/1, = 31/1,). Ambos altura h que alcanza el bloque M? (M = 3 m).
son disparados verticalmente con velocidades vI y
v, el primero hacia arriba y el segundo hacia abajo. A)H
La aceleración del centro de masa luego de la explosión
será:
A) g/3 B) g/S C) 4g/5 O) 2g/5 E)g
B) 3 H/8

C) 3 H/4 t 1
24.- Un proyectíl es disparado y tiene una trayectoria
parabólica. Al llegar al punto más alto B de su
trayectoria, el proyectil se parte en dos fragmentos
idénticos. Uno de ellos, "el primero", por efecto de
0)H/4

E) H/3
t ~·_M~--~----l
_
la explosión retorna a la posición inicial cuya trayec-
toria coincide con la de ida. Entonces, el "segundo" 28.- Si en el problema anterior el choque fuera
cae en C cuya abscisa es: . completamente inelástico, ¿Qué alturah alcanzarán
los bloques juntos?
A) 2 a
B) 4a
y

..
B !g A)H/2 O) 4H/5

C) 3 a /
.- .. ,,
<,
-, -, B) HII6 E) 2 H/3
/ -,
D) 5 a I
I -, , C) H/4
\ x,.,
E) N.A. A Q---4 C
Gravitación
OBlETlKJS
1.- Reconocer y comprender las causas que
originan la gravedad y su poderosa
influencia en la configuraciones del
Universo
2.- Entender y aplicar las leyes que gobier-
nan el movimiento de los planetas y
del os satélites.

1permanente afán del hombre por descubrir las causas y las


razones que podrían explicar la maravillosa organización
del Cosmos lo introdujeron en inimaginables aventuras,
acumulándose así miles de horas - hombre en el planteo de teorías y la
realización de investigaciones, pero poco a poco se fueron obteniendo
las pistas que conducirían a una explicación simple y probadamente
cierta, que llegaría por medio del genio de un hombre llamado Isacc
Newton, quien basándose en los trabajos de sus antecesores como Tycho
Brahe, Copérnico, Galileo y Kepler, logró descubrir la Ley de
Gravitación Universal, publicada en sus "Principios matemáticos de
JOHANNES KEPLER
Filosofía Natural" (1686/05/8).

11II NEWTON,LAMANZANA Y LA LUNA


(1571 - 1630)
Este gran matemático y
Se dice (sin haber quedado confirmado) que la idea de la gravita- astrónomo alemán nació
ción le sobrevino a Newton a raíz de la caída de una manzana cuando en Wurtenberg. Fué director
descansaba bajo un manzano. Lo importante de esta popular anécdota del Observatorio Astronómi-
co de Praga, sucedléndole
es que Newton nos propuso lo siguiente: «La misma causa que hace en el cargo a su maestro
caer a los cuerpos en general es la misma que mantiene a la Luna en Tycho Brahe (1546 - 1601),
orbita alrededor de la Tierra». de quien heredó sus traba-
jos de mediciones astronó-

afirm~~~~f~~'n~~~~~~~:;:!
que la Luna abandone la tan-
II~ --.lu~-f)~.,:;~::?;¡;.--~ -
.l· I ••.
/~~~. )
micas, las que luego de un
minucioso y prolongado
estudio le permitieron des-
cubrir las leyes que descri-
gente es que ella experimente I / .> Tangente ben el movimiento de los
una aceleración, la misma que [ /ts.
.»:
r: //
planetas y llevan su nom-
bre,las mismas que, luego
s?lo se exPflicaríadsi la Tie.~a 1:' /'J- le permitieron a Newton
ejerce una uerza e atracción
sobre aquella. De este modo,
en el mismo tiempo en que cae
I
I

¡-'
I

'....:1
--..
••.

t,..JY-.t.
••
h Lineas
de visión
descubrir
Gravitación
la Ley de la
Unlvesal.Sus
conclusiones se publicaron
con los títulos de «Mysterium
la manzana la alturah. la Luna Cosmographicum. (1596)
cae la distancia y, los dos con al :> ~ «Astronomla Nouvc» (1609)
movimientos acelerados, aunque ------->..~----'-~..•....... ~F=:-ig--:-13::--'.1 y «De Harmonice Mundi.
con distintas aceleraciones. (1619).
252 Física-Primer nivel

CONSTANTEG

110 años después de la


publieac/ón de la Ley de la
Gravitación de Newton se
.¿ A QUE SE D~BE LA GRAVEDAD
Félix Aucallanchi

?'
Denominamos gravedad al efecto mismo de la atracción existente
entre los cuerpos; sin embargo, la causa o el porqué sucede ésto ha que-
V.

pudo efectuar la medición dado un tanto sin explicación hasta principios del siglo XX, Actualmen-
de G y, el primero en hacer- te los fí~icos teóricos plantean que jl} I <\
la fue Henry Cavendish en
1797, utilizando para ello el espacio se deforma por la presen- ~.
una balanza de torslón.EI cia de grandes masas, generándose (f )
valor de G en el S./. es Igual
a 6,67.10'11 N.m2/kgl.
así la gravedad. Esta deformación X.
del espacio es como la que origina
una bola pesada al centro de un man- ~ _ _ ~
tel como el de la Fig. 13.2. Fig. 13.2
11II LEY DE LA GRAVITACION ÜN!VERSAL
Esta ley establece que: « Toda f--'..~=, ==.,."".,
partícula material del Universo ser
atrae con cualquier otra partícula, con [
INGRAVIDEZ
fuerzas de igual intensidad pero de:
Un cuerpo se encuentra en direcciones opuestas, y cuyo valor ese
estado de ingravidez cuando directamente proporcional con elr
está en caído libre, de mane- producto de sus masas e inver-¡r
ra que toda balanza bajo
dicho cuerpo o dlnamómetro
samente proporcional con el cuadrado I
que lo sostenga Indicará cea: de la distancia que las separa» . I
Una nave espaclalorbltando ¡
alrededor de la nerra está su-
jeta a una única fuerza y ésta
es la fuerza de gravedad por
(13.1) I
L=~~~~~~~~~~~
lo tonto se encuentra en caí- Fig 13.3
da libre. Esto significa que la donde G es la Constante de Gravitación Universal, De la Fig. 13.3, vemos
nave se encuentra en estado
de Ingravidez; luego, todos los
cuerpos dentro de la nave
• flotarán •.
DI ·
que la Tierra atrae a la Luna, y de igual modo la Luna a la Tierra.
VARIACION DE LA GRAVEDAD CON iA..
ALTURA . .. .

Newton demostró que la atracción gravitatoria que ejerce la Tierra


sobre los cuerpos se traduce en un movimiento acelerado cuando éstos
son dejados caer libremente, verifi- '.'
cándose que la aceleración (g) de
la caída será menor cuanto más lejos
DEBESSABERQUE: nos encontremos de la superficie te- ¡ ,',;.
rrestre.Si consideramos a la Tierrar'/:~~ O
Para una esfera maciza o como una esfera, el valor de g en la~",:::::..
uniformemente hueca, New- superficie y a una altura h respec-. . ·'~:Ti,.
ton demostró que para efec- tívamente estará dada por: i,' ..;. ,c:..,-'-"" .. ,
tos externos se puede consi-
derar que toda su masa está
concentrada en su centro
geométrico. A esta equiva-
I gs=G¡rl (13.2)
lencia se llama masa pun-
tual. Asimismo, toda partícula
dentro del cascarón estará I g = G (R ~h)21 (13.3)
en estado de imponderabi-
lidad, es decir la fuerza de
gravedad del cascarón so- donde: s, = 9,81 m/s2 Fig 13.4
bre él es nula.
Gravitación 253


VARIAC/ON DE"g" CON LA
VARIACION DE LA GRAVEDAD CON LA »amx: (h)
LATITUD (O)
(En la latitud 9=45°)
El movimiento de rotación de la r------------,
Tierra sobre su eje produce un cambio h(km} g(m/s2}
en el valor de la aceleración de la gra-
=
vedad.DelaFig. 13.5: go gravedad si O 9,81
9.75
la tierra no girase, lie = Aceleración 20
40 9,69
centripeta, y g = (g o -lie) es la gravedad
60 9,63
efectiva del lugar. En realidad, 10 que
80 9,57
nosotrosmedirnos y experirnentunoses
g; y si soltunos una plomada, ésta se 100 9,51
orientará en la dirección de g .Solo en el 200 9,22
polo desaparecen los efectos de la
aceleración lie, verificándose allí que:
g = s; Fig. 13.5

11II ENERGIA POTENCIAL GRAVITATORIA

o
....
Cuando un cuerpo de masa m se VARIAC/ON DE"g" CON LA
encuentra a una distancia d considera- ~ LATITUD (9)
ble con relación al centro de la Tierra, -, -c
(A nivel del mar)
la energ.ía
ambos potencial
tienen se definegrav~tatoria
aSI: que ;;:.;.M.. // »>
Latitud

I
g(m/s2}

EPG = - G M;r-I (13.4);~" <:/ / . 00


200
9.780
9.786
dond~ EPG también coincide con el ':\', \ / / d 400 9,802
trabajo de un agente externo para traer ~ - V 60 ° 9,819
a m desde el infinito hasta el punto "A". '-----'~--....:::..:c~~-~'='r>?_Z 80° 9,831
900 9,832
__ VELOCIDADES ORBITALES y DE ESCAPE
Cuando el hombre intentó lanzar satélites artificiales para ponerlos
a or-bitar la Tierra, encontró que éstos podían describir una circunferencia,
una elipse, una parábola o una hipérbola, 10 cual quedaría definido por el
impulso inicial. Llamaremos velocidad orbital o primera velocidad
cósmica (VI) a aquella que le permite a un satélite dar vueltas en
circunferencias alrededor de la Tierra, y cuyo valor viene dado por:
DEBES
SABERQUE:

Los gravímetros sensitivos


y l\amaremos velocidad de escape (v ) a la que deberíamos dar son aparatos que permiten
a un satélite para que no vuelva al punto de fanzamiento, de modo que medir pequeñisimas varia-
ciones de la gravedad. Sus
siguiendo una trayectoria parabólica se aleje para siempre de la Tierra. valores se expresan en la
----~=.....
Su valor está dado por: unidad gol :

= ~2 Gt! = J2
1 gol = 1 cm/s2 = 1O-3g.
v2 VI = 11.16 km/s
Subira un cerro de 300 m
de altura produciría una
Cuando el lanzamiento se realiza con una velocidad Vo tal que: variación de 9 del orden de
VI < Vo < v2' las trayectorias son elípticas, de manera que los proyectiles 0,1 gol.
254 Física-Primer nivel Fé/ix Aucal/anchi V.

lanzados con velocidades mayores que v2 describirán trayectorias hipe r-


DEBESSABER QUE
bólicas.
Llamaremos Apogeo al
punto de la elipse que se I Vuelo Libre
encuentra más alejado de
la Tierra, y Perlgeo al punto a) .",..1 b)
de la elipse que se encuen- -.
tra mós próxImo a la Tierra.
Estos mismos puntos se de-
nomInan Afelio y Perihelio
'\.\ "o
respectivamente cuando en
el foco de la elipse se en-
cuentra el Sol. Fin de
____
'_-\ Propulsión
t Vuelo Motor
Punto de lanzamiento

MUY INTERESANTE
Fig.13.7

Un satélite seró estacio-


nario sI en todo momento se
encuentra frente a un mismo
11II MOVIMIENTO PLANETARIO
lugar de la superficie terres- El descubrimiento de las leyes que rigen el movimiento de los
tre, debido al hecho de tener
un período de rotación igual planetas fué uno de los procesos más interesantes en la evolución de la
a 24 horas. ciencia, y en particular de la Física. Los griegos consideraron al hombre
como el centro del Universo,y fue por ello que asumieron que la Tierra
se encontraba en el centro geométrico del mismo, de manera que los
demás astros giraban alrededor de él, y en el siguiente orden: La Luna,
Mercurio, Venus, Sol, Marte, Júpiter, Saturno y las estrellas fijas. Esta
creencia fué reforzada por Claudio Ptolomeo con su teoría "Geocéntrica",
teoría que duro hasta mediados del siglo XVI, en que surge la teoría
Heliocéntrica de Nicolás Copérnico, que consideraba al Sol como el
centro del Universo, desde el cual la trayectoria de los planetas son líneas
más sencillas.Esta teoría fué reforzada por observaciones astronómicas
hechas por Tycho Brahe, las mismas que fueron utilizadas por Johannes
Kepler para el descubrimiento de sus leyes.
En la actualidad se tiene pruebas de la existencia de nueve planetas
que conforman el Sistema Planetario Solar los cuales son:
TEORIAGEOCENTRICA (1) MERCURIO (2) VENUS (3) TIERRA (4) MARTE (5) JUPITER
Fué sustentada por Clau- (6) SATURNO (7) URANO (8) NEPTUNO (9) PLUTON
dio Ptolomeo (100-17B),
astrónomo de Alejandría.
Según esta teoría, la Tierra
es el centro del Universo, y
11II LEYES DE KEPLER
cada planeta se mueve con Luego de 20 años de obsesionado estudio, Kepler logró corregir
M.C.U. con relacIón a un el modelo Heliocéntrico , y pudo establecer con sencillez asombrosa
punto, y dicho punto con que el movimiento de los planetas así como el de los satélites se rigen
relacIón a la Tierra tiene un
segundo movimIento circu- por tres leyes:
lar. La trayectoría así obtenI-
da la denominó epicicloide.
lni) Ley de las órbitas.- Todo planeta gira alrededor del Sol describiendo
Esto fué descrito en su traba- una orbita elíptica, ocupando él uno de los focos.
jo titulado Almagesto. 2da) Ley de las áreas.- La recta que une un planeta cualquiera con el
Gravitación 255

TEORJA
Sol (radio vector) describe áreas iguales en tiempos iguales. HELlOCENTRJCA

JB) Ley de los períodos.- El cuadrado del periódo (T) de un planeta Estateoría fué publicada
es directamente proporcional con el cubo del radio vector medie en el libro «De Revolutionibus
Orbium Coeiesttirr» el año
(semíeje mayor de la elipse). 1542. por el monje polaco
Nicolás Copérnico (1473-
TI _ "1 1543). quien en su afán de
=> T.. - r...: (13.4) encontrar una explicación
~ ., ~ más sencilla al movimiento
r-' ---, de los planetas llegó a la
a) inevitable conclusión de
que el Sol debía situarseen
el centro del Universo.y el
resto de los planetas de-
bían girar alrededor de él
l--------lA a distancias cada vez
P
mayores.

--+--- b ---+ 2r ----1- ATENC/ON


II

A:.Apopo
P'. a...:- r= q·t Las leyes de Kepler son
vólldos tonto poro safelltes
& -.... naflrales como pero soIeIltes
L---~----=.:...-------'-----"--------Fi-;::;J;;-g-. -;-;13~.8 atlflcloles

Prob. 1.- La distancia entre /o nerra y la Luna es 60 R(R = radio terrestre). lA qué distancia del
centro de la Tierra un cuerpo colocado en la línea que une la luna y la nerra estaró en
equilibrio? Se sabe además que: masa terrestre = 81 masa lunar.
A)48R B)62R C)54R D)50R E)42R
ResoluciÓn.-
Del gráfico se observa que el cuerpo de TJeml
masa "m debe ser atraído por la Tierra y
H F. F. Luna
por la Luna, con fuerzas iguales para que -----~-------~--- ,
esté en equilibrio.
,, ,,
,, ,,
FT= FL
x ¡ (d-x)-+
MTm MLm
G7 = G(d_x)2 d=60R

=> 9 (d -x) =x

9 9
x= IOd= 10 (60R) => x=54R RPTA.C

Probo 2.- lA qué altura respecto de la superficie terrestre el peso 00 una persona se horó la
cuarta porte? R = radio terrestre.
A)R/2 B)R C)2R D)3R E) 2 R/5
256 Física-Primer Nivel Félix Aucallanchi V.

Resolución.-

Reconociendo que el peso es la fuerza de gravedad que ejerce la Tierra sobre cada uno de nosotros, podemos
decir de acuerdo con los datos que: P = P (4, donde Ps = peso en la superficie terrestre. Y como los pesos
dependen de la gravedad local, concluímos que esta rmsma relación se verificará para las aceleraciones de
la gravedad: g = g/4. Seguidamente, utilizaremos la relación (13.3) para determinar la altura h:

R2 g R2
g=gJ4(R+h)2 :::} -t=gs(R+h)2 :::} (R+W=4R2 :::} h=R RPTA.B

Prob 3.- Suponiendo que el radio de la Luna es 1/6 del radio de la Tierra, y que la densidad de
la Tierra es 1,5 veces m~or que el de la Luna, ¿Cuánto vale la gravedad en la Luna,
si en la Tierra es 9,8 rtüs ? (en m/s2).
A) 1,08 B) 0,96 C) 1,63 O) 9,8 E) 0,18 UNMSM - 80
Resolución.-
Suponiendo que un planeta es esférico de radio R y densidad D, podemos decir que su volumen es V = 41tH3/3,
Y por consiguiente su masa será: M =D. V = D (4ltR3/3).Luego, la gravedad en su superficie la encontraremos
en base a estas deducciones y a la relación (13.2)
M 41tR3D/3 4
g = G R2 = G R2 = "3 ltGRD

Y utilizando esta relación para la Luna y la Tierra, así como los datos del problema, tendremos:

gL .t6LDL (1I6RT)DL gT
g.; =~RTDT = R (l,5D )
T L
:::} gL = ""9 :::} gL = 1,08 mlsl RPTA.A

Probo 4.- Deterrtincr /o aceleración centrípeta en lTI txrto de /o línea ocuatoriol, siel radio terrestre allí
es 6 400 km. AsImismo, encontrcr /o g-o.tedad efecfM:J en dicho /ug::r (gs 9,8 m/s2)
A) 0,065 m/s2 ; 8,754 m/s2 B) 0,042 m/s2; 7,345 m/s2 C) 0,234 m/s2 ; 9,347 mís2
O) 0,034 m/s2 ; 9,766 m/s2 E) 0,342 m/s2 ; 6,343 mls2
Resolución.-
De acuerdo con lo visto en el item (13.5) y en virtud a la Ley de D'Alembert, diremos que un observador
ubicado en el centro de la Tierra (O) apreciará que un punto cualquiera de la superficie terrestre, y en particular
aquel ubicado en la línea ecuatorial, experi-menta dos aceleraciones: La gravedad normal g y el opuesto de la
aceleración centrípeta (-ac) dellugar.Luego, la gravedad efectiva (C estará dada así: ef)
Cef = Cs + (-ac) a:::} ICef I = s, - ac (*) cjg
donde: a = 0)2 R = (21t R'f = 2
41t R {R=6.4.l0 m
6 ·
e T ) T2 T=24h=86400s
2
a = 4(3,14) .6,14.106:::} ac = 0,034 m/s2
e (86400)2

y en (*) : Igef I = 9,8 - 0,034

.. Iger I = 9,766 mls l RPTA.D ~·


··
Gravitación 257

Probo 5.- lA qué altura de la superficie terrestre la velocidad de un satélite es un tercio de la


primera velocidad cósmIca? R = Radio terrestre.
A]5R B] 10R C] 14R O] 8 R E] 6 R
Resolución.-

Utilizando las relaciones del item (13.7) para la velocidad de un satélite y la primera velocidad cósmica
respectivamente, así como la condición del problema, tendremos:

V =
V ra
_1- => 3v = VIra => 3
J (R+h)
GM = JGM
R => h=8R RPTA.D
3

Probo 6.- Unsatélite orbita la Tierracon una velocidad igual a la mitad de la primera velocIdad
cósmica ¿Qué valor tIene la aceleración de la gravedad en dicha órbita? gs =
gravedad en la superficIe terrestre = 10 m/s2.
A] 0,453 m/s2 B] 0,625 m/s2 C] 0,424 m/s2 O] 0,826 m/s2 E] 0,539 m/s2

Resolución.-
Aprovechando la solución del ejercicio anterior encon-
traremos la distancia (el) del satélite al centro de la Tierra: ./---
/------------y~~~·~~·~~~·~············l
Vg """
,,
v=-2-
VIra
=> 2v= VIra => 2
JGM
d =
JGM
R => d=4R
,,
,
d=4R
A continuación calcularemos la gravedad g en la órbita en
base a la relación (13.3):

R2 g
g = gS(4R)2 = l~ => g = 0,625 m/s2 RPTA. B
Observación.- La gravedad g de la órbita desempeña el papel de aceleración centrípeta.

Probo 7.- Un satélite gira en una órbita circular alrededor de la Tierra a una altura donde la
aceleración de la 9ravedad es la cuarta parte de la gravedad en la superficie de la
Tierra. Hallar el penodo de revolución del satélite (considere R el radio de la Tierray g
la aceleración de la gravedad en la superficie terrestre].
A] 41tJR / g B] 21tJR / g 9 41tJg /2R D]41tJ2R/ g E]41tJR / 2g UNI 94 - 2
Resolucién>
En primer lugar calcularemos la distancia r del satélite
hasta el centro de la Tierra, para lo cual utilizaremos
la condición del problema y la relación (13.3):

R2
g' =g2 => => r=2R
r
A continuación, reconocemos que para el movimiento
circunferencial uniforme del satélite, la gravedad g'
que éste experimenta es justamente la aceleración
centrípeta. Luego en base a la obsevación señalada
en el item (6.8) y el resultado anterior, encontraremos
el período T del movimiento.
258 Física-Primer nivel Félix Aucallanchl V.

g 41t2
4" = r2 . (2R).

T=41t
fiK
Vg RPTA.D

Probo 8.- Un satélite gira alrededor de la 7ierra con un período T1 = 16 dios, y un segundo satélite lo
hace con un período T2 = 2 días. Si la velocidad de este último satélite es la mitad de la
primera velocidad cósmica, ¿Cuál es el radio de giro del primer satélite? R =Radio terrestre.

A) 10R 8)8 R C] 16R D) 12 R E) 9 R


Resolución.-

En base al dato de las velocidades diremos que el radio de giro r2 del segundo satélite es: r2 = 4R; ésto en virtud
a lo obtenido en el problema anterior. Luego, de acuerdo con la 3ra Ley de Kepler se debé cumplir que:

16díasj
( 2días
_
-
(íJ
4R => Tt=16R RPTA.C

Probo 9.- Dos satélites S1 y S2 orbltan circularmente alrededor de un mismo planeta. El primero
barre en 144 noras las 2/3 partes del área total de su órbita. El segundo satélit~ tiene
un período igual a 27 horas. Entonces, la razón de los radios de sus órbitas R/R2 es:
A)2 8) 3 C] 4 D) 9 E) 14 UNI 93 - 1
Resolución.-

De acuerdo con los datos podemos reconocer que el primer satélite tiene un período de 216 horas, lo cual se
puede deducir en base a la 2¡!¡¡Ley de Kepler y una regla de tres simples:

J44h
1)
t
S
S
} 1) = 144h.S
t S
= => TI = 216 h

Seguidamente calcularemos la razónRl / R2 entre los radios de giro de los satélites utilizando para ello la 3ra
Ley de Kepler, veamos:

RPTA.C.

Probo 10.- Un satélite gira en torno al planeta M


ubicado en el polo F1' empleando 18
«reses . SI el tiempo para ir de A a 8
es de 1 mes, yde e hasta D es de 3
meses. ¿Qué parte de toda la elipse
es la región sombreado? F2
segundo foco de la elipse.
A) 1/9 D) 3/5
B
8) 3/2 E) 8/5
C] 1/7
Gravitación 259

Resolución."

Sean: S = área de toda la elipse, y S2 = área buscada.


De acuerdo con la igualdad de las regiones AF,B y 6 mes~es,,---=__
CF2D, diremos que sus áreas son iguales a S,. Luego,
utilizando la 2da Ley de Kepler tendremos:

1) Arco AB: S ::> 18 meses A e


S, ::> 1 meses }s, = I~ ...... (1)

2) Arco CD: S ::> 18 meses


S, + S2 ::> 3 meses
}s,+s2=i ...... (2)

y reemplazando (1) en (2), despejamos S2' de modo que : I~ + S2 = t ~ S2 t = RPTA. A

Probo 11." Determinar el tiempo que debería durar el día para que un sujeto colocado en el
ecuador reduzca su peso normal en 25 %

A) 2rtJR / 9 B) 2rtJ3R / 9 C) 4rtJR /g D) 4rtJR / 3g E) 4rtJ3R /g

Resolución ."
De acuerdo al esquema deducimos que: Fe= mg'
Aplicando la relación (9,6) tenemos:

mi» 2R -'- mg ~. W
L.(
- R . (1)

Según la relación (6.7) deducimos que: W = 2; .

Además por condición del problema se deduce que


e' = 3/4
g. Luego, reemplazando en (1):

,.,2 = 3/4g (2rt ]2 3g


VJ R ~ T) = 4R

Luego 2; = !.pj.1 f= ~",:1fIRPTA.


V3g.
D
260 Física-Primer nivel Félíx Aucallanchi V.

I~ AUTOEVALUACION
1.- Indicar verdadero (V) o falso (F) según 5.- Si F es la fuerza de gravedad que ejerce el
corresponda: cascarón M sobre la partícula m, entonces es falso
que:
: ) Gravitación es el fenómeno atractivo entre dos
masas. A) F4 < F3
: ) El peso es la fuerza de gravedad sobre un cuerpo
ejercicio por otro. B) F[ = O
4
( ) Peso y gravedad son dos conceptos iguales. C) F2 = F3 O
m
A) FFV B) FVV C) VFV D) VVF E) VVV D) F[ = F2
2.- Sañalar la afirmación correcta: E) F2 < F3
<\) La Tierra tiene un peso nulo respecto de cualquier
6.- 'Complementar correctamente «La teoría
cuerpo celeste. ____ fué sustentada por »
B) La aceleración de la gravedad no depende del
tamaño del planeta. A) Heleocéntrica, Kepler D) Geocéntrica, T. Brahe.
=) La aceleración de la gravedad solo depende de la B) Geocéntrica, Ptolomeo E) Helicéntrica, Newton
masa del planeta.
D) Al viajar por un túnel hacia el centro de la Tierra, C) Antigua; Ptolomeo
la gravedad aumenta.
7.- En relación al siguiente sistema planetario. los
~) la aceleración de la gravedad en el centro de la tiempos (r) son tales que:
Tierra es nula. 3
A) tu = t34 ~
t- Dadas las siguientes proposiciones:
B) t23 = 2t4[
L El menor valor de la aceleración de la gravedad
en la Tierra se presenta en la Línea Ecuatorial. C) t[2 = t4[ 4 2
[1. En los polos se miden los valores de g sin los D) t23 = t34 2S
efectos de rotación de la Tierra. E) 1[2 = 2t34
1II.Si soltamos una piedra en Lima, la prolongación 1
de la trayectoría pasa por el centro de la Tierra. 8.- En relación a la pregunta anterior, las velocidades
:Y. La variación de g con la latitud se debe básica- (v) son tales que:
mente a que la Tierra es achatada en los Polos y
A)V4>V2
ensanchada en el Ecuador.
Señalar lo correcto: B) v[ = v3

<\) I B) 11 C)III D) IV E) 11 Y III C) v[ < v2

lo Los cuerpos que viajan dentro de una nave espacial 9.- Los radios de la órbitas de dos satélites son tales
!n órbita alrededor de la Tierra están en estado de que: 9R[ = l6R2; luego, la relación de sus períodos
ngravidez porque: T¡lT2 será:
<\) Dentro todos tienen la misma velocidad A) 6/7 B) 9/16 C) 64/27 D) 3/4 E) 4/3
3) La Tierra no ejerce atracción sobre ellos
10.- En el Sistema Solar, el planeta de menor período es:
::) En el espacio no hay aire
A) Jupiter B) Neptuno C) Venus
)) Dentro todos experimentan igual aceleración.
S) La nave no tiene aceleración. D) Marte E) Mercurio
Gravitación 261

PROBLEMAS PROPUESTOS

NIVEL 1 NIVEL 2

01.- ¿Con qué fuerza (en N) se atraen dos asteroides 07.- ¿A qué distancia mínima del centro de la Tierra,
cuyas masas son: 8.108 kg /\ 9. J09 kg, si además la la gravedad es la mitad del que existe en la superficie?
distancia entre ellas es de .16,67 km? Dar la respuesta en función de R = Radio terrestre.

A) 55 B) 64 C)78 D) 72 E) 40 A)R/2 B)R/9 C)R/5 D)R/3

02.- Si una de dos masas se triplica y la distancia 08.- Por mediciones realizadas en la línea ecuatorial,
entre ellas se duplica. ¿En qué razón se encuentran se sabe que la aceleración de la gravedad efectiva es
las fuerzas de atracción gravitacional inicial y final g = 9,78 mls2. Sabiendo que por efectos únicos de
entre dichos cuerpos? gravedad, el valor debería ser go = 9,81 m/sl, ¿Cuál
es el valor de la aceleración centrípetac, (enmls2) de
A) 3:6 B)4:3 C)5:2 D)7:4 E)4:8 los puntos del Ecuador?
03.- Determinar la intensidad del campo gravitatorio
A) 0,05 B) 0,08 C) 0,01 D) 0,02 E) 6,03
en P, si al colocar allí una masa puntual m = 6 kg ;
experimenta una fuerza F = 60 N, tal como se indica.
09.- Si el cuerpo de masam se deja caer en A, ¿Cuál es
A) (-6;-8) N/kg la energía mecánica total del sistema cuando llegue a B?

B) (3;-9) N/kg A) -GMmI7R


C) (12;4) N/kg B) GMm/3R
D)(-9;21) N/kg
C)-GMm/5R
E) (-5;-l)N/kg o X
D) -GMm/2R
04.- Sabiendo que en l!n punto P el campo tiene una
intensidad g = -10 j (N/kg) ¿Qué fuerza (en N) E)GMmlIR
experimentará allí una masa puntual de 6 kg? Dar la
respuesta en función de los vectores unitarios 10.- Un satélite artificial gira alrededor de la Tierra
cartesianos. a una distancia R de su superficie. Encontrar su
rapidez (v) de traslación (enkmls) si: gs = JOm/s2
A) -50] B) 4] C) -60] D) 8] E) -6 ]
/\ R = 6 400 km.
05.- Considerando que la Tierra es una esfera macisa
y homogénea ¿A qué distancia (en megametros) de A) 4/J2 B)2/J2 C)6/J2
su centro, la intensidad del campo gravitatorio será
D) 9/J2 E)3/J2
igual a 6,67 m/s2? Masa de Tierra = 6.1024 kg.
Sugerencia: Utilizar: .J6O '" 7,75 11.- Determinar en qué razón se encuentran los tiern-
A) 5,65 B) 3,89 C) 7,62 D) 4,27 E) 7,75 ~ 3
pos de recorrido ti /\ t2, si: S2 = ¡
06.- Sabiendo que el campo gravitatorio creado por
A y B en P es nulo ¿En qué razón se encuentran x e A) 2/3
y , si mA/mB = 4/9?
B) 3/8
A) 8/2
A B C)3/4
B)2/3
C) 1/2
D) 317
G)---,----------r-----------@-i
I

,
;.-- X
_,

,,
Y ----...;
I

,
D) 5/6
I
E) 217
E) 6/3
262 Física-Primer nivel Félix Aucollonchi V.

NIVEL 3 19.- Sabiendo que: 1( = 9 días ¿Qué tiempo emplea


el satélite indicando para pasar de B hasta C?
12.- En la superficie de un planeta una persona tiene un
peso de 720 N ¿Qué peso (en N) tendría dicha persona A) 18 días <;-

si la masa del planeta se duplicara y su radio se hiciera


el triple? B) 8 días

A) 180 B) 360 C) 140 O) 120 E) 160 C) 6 días A C


SS ;jI
13.- ¿A qué profundidad la gravedad es un tercio de la O) II días ~ 12
existe en la superficie de la Tierra? R = Radio terrestre. I(
E) 15 días B
A) R/3 B) R/4 C) R/6 O) 2R/3 E)R/5
20.- Encontrar en qué relación están los periodos de
14.- Si D es la densidad de un planeta y R es su radio, traslación 1( 1\ 1} de los satélites mostrados.
¿Cuál es la expresión que le corresponde a la gravedad
en su superficie? G = Constante de Gravitación
Universal.

O) 4rr.GDR2 E) 1rr.GDR A) 2/30 B) 27/8 C) 42/1

O) 28/6 E) 5/41
15.- ¿A qué distancia del centro de la Tierra una nave
girará, con la mitad de la primera velocidad cósmica 21.- Sabiendo que el planeta A demora 8 veces lo
alrededor de ella ? R = Radio terrestre. que demora B para dar una vuelta alrededor del Sol.
¿En qué razón se encuentran los radios de giro?
A) 2R B)4R C)3R 0)5R E) 3R/2
,,
A)4 ,
16.- ¿Cuál es la velocidad orbital de un satélite que ,,
gira alrededor de un planeta cuya masa es nueve veces ,,
B)6 ,,
el de la Tierra, y su radio es cuatro veces el terrestre? ,,
(v( = 8 km/s) ,,
C)2 ,
,, ,,
A)8 B) 10 C) 12 O) 15 E) 18 0)8
/~9A
17.- Para el esquema mostrado, ¿Cuál es la expresión .' A ,/
E)3
correcta para la energía cinética del satélite de masa m ?
.... m 22.- Un satélite artificial de la Tierra ha sido lanzado
A) GmMI2R
desde el Ecuador, y se mueve por una órbita circular
B) GmM12R2 en el plano de éste y en el sentido de la rotación de
la Tierra. Si el radio de la órbita es R = 4RT '
C) GM/R

O) GmM/4R
~~) siendo RT = 6 400 km (radio terrestre), ¿Al cabo de
qué tiempo pasará el satélite por primera vez por el
punto de lanzamiento?

E) Gm/R2 A) 18 h, 45 mili, 36 s

B) 21 h, 38 mili, 2 s
18.- Del problema anterior,¿Cuál sería la expresión
correcta para la energía mecánica del sistema? C) 23 h, 40 mili, 48 s
A) -GmM/4R O) GmM/4R O) I día, 2 h , 38 min
B) GmMI2R E) -GmM/3R E) I día, 5 h , 55 mili
C) -GmMI2R
Oscdaciones

OBJETWOS
1.- Reconocer las características principales
de los movimientos osciltorios .
2.- Interpretar y aplicar correctamente las
leyes del M.A.S. y del Péndulo Simple.
3.- Conocer y entender el comportamiento
de las ondas mecánicas y su extensión a
las ondas luminosas.

nuestro alrededor, de manera permanente, se están produ-


ciendo movimientos en donde el móvil va y viene repitiendo
su movimiento de un modo periódico; muchos de ellos son,
además de una característica tal, que por su «suavidad» diríamos que
LOUIS VICTOR DE
son armónicos; entre estos movimientos podemos citar a los vaivenes BROGLlE
de un reloj de péndulo, a la luz intermitente de un faro marino, el bambo-
leo de un bote anclado, los latidos de nuestro corazón, ,etc. En verdad,
oo. (1892 - 1987)
vivimos rodeados de oscilaciones, y por ende de ondas, pues éstas están
íntimamente relacionadas entre sí, Por razones pedagógicas estudiare- Físico francés. Nació el 15-
8-1892 en Dieppe. Fué ¡:xofesor
mos primero a las oscilaciones simples, y a continuación a las ondas. en LaSorbona; en 1924propu-
so dar o codo ixxticua un
11II CONCEIYrOS PRELIMINARES ccrácteronduJatorio{01Cbsde
materia de lo Mecánico
CuónticoJ y con ello funció lo
A) Movimiento Oscilatorio.- Este tipo de movimiento se caracteriza Teoríade los Ondas de Materia
porque el móvil siempre está repitiendo el movimiento, siguiendo (Ondas de Broglie). En ese
la misma trayectoria en ida y vuelta, pasando siempre por un mismo mismo año, Einstein, 01 leer sus
memorias, reconoció inme-
punto de referencia; por ejemplo, un columpio realiza un movi- diatamente el mérito de su
miento oscilatorio, el repique de campanas de una iglesia, etc. trabajo, e incorporó algunos
de sus ideas en sus trabajos de
B) Movimiento Periódico> Es aquel movimiento que se repite de RelotMdad . A su vez, esto afir-
igual forma a intervalos iguales de tiempo. Por ejemplo, el movi- mación hecho por un científICO
de lo tollo de Einsfein tuvo un
miento de rotación de la Tierra respecto de su eje, la visita del efecto profundo en unjó¡en físi-
cometa Halley a la Tierra; el movimiento oscilatorio de un péndulo co austrloco que daría el si-
de reloj, etc. guiente poso gigante : Erwin
Schródinger, entre los años
C) Movimiento Armónico.- Este movimiento solo 10 tienen aquellos 1925 Y 1926 ,período que mor-
có lo llegado de lo concep-
móviles cuya posición viene expresada en términos de seno y/o ción moderno de lo "Mecáni-
coseno. En la práctica todo el movimiento armónico es a la vez un coCuánticauOndulotaia. Por
movimiento períodico. todos estos hechos, el barón
De Broglie obtlNO el Premio
Nobel de Físico de 1929. .
264 Física-Primer Nivel Félix Auca/lanchi V.

PARA RECORDAR IIIIMOVIMIENTO ARMONICO SIMPLE (M.A.S.)


Un móvil que vibra experi- Un movimiento será armónico y simple si en todo momento el
menta una oscilación sim-
ple sirealiza un movimiento móvil realiza oscilaciones en línea recta respecto de un punto llamado
de ida o de regreso pasan- posición de equilibrio (P.E.), de modo que siempre está afectado de
do una vez por la posición una aceleración dirigida hacia dicho lugar, y cuyo valor es directamente
de equilibrio, y desarrolla proporcional a la distancia (x)del móvil hasta el punto P.E.
una oscilación completa si
realiza dos oscilaciones sim-
ples continuadas.

J--==----- ---==----
___ o-o

Fjg 14.1
14.2.A. DESCRIPCION CINEMATICA DEL M.A.S.
Analizando la Fig, 14.2, podemos comprobar que el movimiento
circular de la rueda hace que la proyección (o sombra) de la manivela
m sobre la pared (y) o sobre el piso (x) experimente un movimiento
MUY INTERESANTE
oscilatorio, el mismo que será M.A.S. si la rueda presenta un movimien-
SI el móvil con M.Á.S. no to circular uniforme (M.C.U.).
parte de un extremo, la
elongac/ón (x)deberá incluir
la constante de fase ($) cuyo
significado se aprecia mejor
en el M. e. U.de la figura in-
ferior:

<l--"
M.C.U. A M.A.S.

Fíg.14.2 Fíg.14.3

a) Elongación (x) ." Designamos con este nombre a la distancia que


existe entre el móvil y la posición de equilibrio (O). De la Fig. 14.3
x =A cos (001 + $)
podemos notar que cuando el M.C.U. se inicia en P, el M.A.S. se
inicia en el extremo R;todo ésto sucede en el instante t = O.Si trans-
~ Donde: W/+$ = Angulade fase curre un tiempo t, la elongación viene dada por:
l°) $=0° ~x = A cos (001) I x = A cos (rot) I (14.1)
20) $=90° ~x = A cos (001+90°) Observaciones:
~x = Asen (00/)
1-""""~'-'-DÚn== 0"""1 1\ I x máx= ± A 1 (Amplitud)

·-0'
Oscilaciones 265

b) Velocidad (v).- En el desarrollo del M.A.S., el móvil va presentando ATENCION


distintas velocidades, debido a que tanto su módulo y dirección cam-
bian permanentemente. Esto se puede explicar mejor si observas la' lasSIrelaciones
observas con atención
(14.1) y (14.5)
Fig.14.4, en donde la velocidad v del móvil con M,A.S. no es más concluirás que:
que la proyección de la velocidad tangencial (vt)del M.C.U., y cuando
el movimiento se inicia en un extremo se cumple que: 0= -olx

Lo cual confirma lo esta-


(14.2) (14.3) blecido en el item 14.2

Observaciones: Esto significa que:

Vrnáx = roA (En O) "mrn = O (en los extremos) 1(1 SIx = (+) => a = (-)
e) Aceleración (a) .- De acuerdo con la Fig. 14.5, la aceleración del 2 Slx
Q = (-) => 0=(+)
M.A.S. es igual a la proyección horizontal de la aceleración centrí-
peta del M.C.U. Luego se verifica que:
(14.4) (14.5)
Observaciones:

arnín = roA (En O) 1\ vrnáx =O (en los extremos)


r---------Ve-~--~------------~ ~
V V':\J "~-------'''a
.}&=cot /~~-~ ~ &=cot ,/ !8Ji\~,t
/,' v,' : -.'tI
I
/ ¡-~\,
ac,'-----.

-º~.-\~- -:----6p
I ,

:
I I

\
I--_JL(1
__:0:ia. __l__ ,,' DEBES SABER QUE:
., , I \
,/

-6 p Q- - - - - -- I La ecuación de la eton-
•''.
, 1

• ~ : :,'~ gaclón:
1', "
1 , ,
I
,
1"
,
"
.'.
"
I
I
I
1,'
'.
I
, ' I ., I x = A sen (m t)
,
1

".
I ,1 , I I l' I

,
I I I I
1 ,
1
,
,
,
,
al representarse en un
f I I I
I
I
I
I
I
I
I
I , I I I gráfico x - vs - t origina una
sinusolde similar al que se va
J////////////~%-)))J11t)%)1/. ;r1'~////////////Ji~;Jjy))kA/.
.!-%---f.
grabando en el papel que
se desplaza en la Flg. 14.6.
!-z~
Fzg.14.4 Fzg. 14.5
14.2B. OSCILADORES MECANICOS
Llamamos así a todo aquel sis-
tema físico constituído por un cuerpo
. de masa m y un medio elástico de
constante de elasticidad k, el mismo
que al encontrarse deformado ejerce
sobre el móvil una fuerza recupera-
dora (Fe = -kx), el cual hará que el
cuerpo se mantenga permanente-
mente oscilando, dado que esta fuerza
siempre apunta hacia la posición de
equilibrio del cuerpo. En la Fig. 14.6,
el oscilador se encuentra en P, y es
empujado hacia abajo por el resorte
que está comprimido. '-------------------F-·---14--'6
zg. .
266 Física-Primer Nivel Fé/ix Aucallanchi V.

. OJO a) Frecuencia Angular (00).- En la Fig. 14.2 se probó que el M.A.S.y


el M.C.U. son dos puntos de vista de un mismo movimiento; asi, la
SI revIsas los relacIones frecuencia angular del M.A.S. es la conocida velocidad angular del
[14.6), [14.7) y[14.8) llegarás
o lo sIguIente conclusIón: M.C.U., y que para un oscilador mecánico viene dado así:
-Lo frecuencIa angular, el
período y lo frecuencia, no Ik(Nlm)
dependen de lo amplitud m(radls) =V m(kg) (14.6)
de los oscilaciones>.
b) Período (1').- Es el tiempo empleado por el móvil en el M.A.S. para
realizar una oscilación completa, y en el M.C.U. para efectuar una
revolución. Su valor viene dado así:

t T =~. = 27CJijj (14.7)

e) Frecuencia (j) .-Nos indica el número de oscilaciones completas que


el móvil en el M.A.S , en cada unidad de tiempo. Su medida se
expresa en: Oscilaciones por segundo (ose/s) = Hert: (Hz) .

.1 f= t =in J! I (14.8)

d) Energía Mecánica (E ).- Para un oscilador mecánico, la energía me-


cánica viene dada por ia suma de las energías cinéticas y potencial
elástica, y su valor se conserva si el sistemas se mantiene aislado, es
decir en el vacío. Dicho valor viene dado así:
2
E = 1m1l = 1kA2 j (14.9)
l •
. m 2 lID' 2 .
Observacién» La energía mecánica de un oscilador es independiente de la
gravedad; por ello no incluye a la energía potencial gravitatoria.

14.2C. ACOPLAMIENTO DE RESORTES


Cuando reunimos dos o más
resortes en un todo, el conjunto
adquiere un comportamiento idén- 1
MUY IMPORTANTE tico al que tendría si todos son sus- •
tituídos por un único resorte lla- k.¡
12En un acoplamiento de mado equivalente. El valor (kt;) de
resortesen serie, lo fuerzo la constante del sistema equivalente (a) En Serie
interno en los resortes es dependerá del tipo de conexión:
lo mismo poro todos
ellos, e igualo lo fuerzo 1) En serie.- Los resortes se unen
que experimento el re- j_ 1\ 1\ A 1\ Á 1\ L
sorte equivalente: unos a continuación de otros: ...
······YVVVVVv----
1
Feq = F1 = F2 =FJ

22 Cuando los resortes se


acoplan en paralelo se
verifico que lo fuerzo en
el resorte equivalente vie-
I ke-I = k1-1 + k2-1 + k3-1 1 (14.10)'
2) En paralelo.- Los resortes tie-
nen sus extremos unidos entre sí: '
I
.~
~

(b) En Paralelo
ne dado así:
(14.11)
Fig 14.7
Oscilaciones 267

11II PENDULO SIMPLE PARA RECORDAR

Una cuerda inelástica, fija por ¡;==~~,"",",,=r-=~"""""~=~==-J Dooooue la amplitud an-


uno de sus extremos y por el otro su- , gular (e) del movimiento
pendular es muy pequeña,
jetando a una lenteja pesada, consti- ¡ D
se puede decir que la ten-
tuyen un Péndulo Simple, el mismo .,' sión(T]de la cuerda es prac-
que como todo oscilador mecánico ticamente igual al peso (P)
tiene la característica de retornar per- g~ de la masa pendular:
manentemente a su posición de equi- T = P2 = P cose: cos e =1
librio. Observando la Fig, 14.8 pode-
mos apreciar que el péndulo separa- :. T =P
do de su posición vertical (B) tiene
siempre una componente P l del peso,
que desarrolla el papel de tuerza re-
cuperadora, la cual apunta siempre
hacia la posición "B", de manera que
al regresar a dicho lugar pasa y x
continúa por inercia hasta llegar al
otro extremo (A o C) en donde Pies
máximo, lo cual obliga al cuerpo a
repetir su movimiento de manera
periódica y armónica.
Fig. 14.8
14.3A. ELEMENTOS DEL MOVIMIENTO PENDULAR
a) Longitud Pendular (1 ).- Es la longitud de la cuerda, o propiamente
del péndulo. En la Fig. 14.8, I es también el radio del arco AC.
b) Masa PeduJar (m).- Llamamos así a la masa del cuerpo que expe-
rimenta el movimiento pendular. Lo recomendable es que el cuerpo
tenga forma de lenteja para minimizar los efectos del rozamiento:
e) Gravedad Local (g) .- Sin duda, el movimiento pendular no se
produciría si no existiese gravedad, pues ella es la que origina las
fuerzas recuperadoras del movimiento armónico.

d) Oscilación.- Llamaremos así al movimiento de ida y vuelta realizado


por el péndulo de la Fig. 14.8: AC + CA.
e) Amplitud Angular (9m).- Es el máximo ángulo de desviación del ATENC/ON!
péndulo con relación a su posición de equilibrio. Para fines prácticos Enla Flg. 14.8 la recta DO
consideramos como razonablemente aceptables que9m ~ 10°, pues define la posición de equi-
solo así el movimiento pendular se hace más rectilíneo. librio del movimiento pen-
dular, la misma que tiene
f) Amplitud Lineal (A).- Es la elongación (x) máxima del movimiento orientación vertical. Esta
característica no es más
pendular realizado sobre la recta X. que una consecuencia de
la gravedad local. Estoper-
143B.PERJODOYFRECUENCIADELMOVIMIENTOPENDUL1\R mite establecer la siguiente
conclusión.
Analizando la dinámica del movimiento pendular mostrada en la
Fig. 14.8, diremos que la fuerza recuperadora o de Hooke en la posición -ta posición de equilibrio
A, está dada así: del movimiento pendular es
paralelo a la gravedad lo-
- la = -P sen 9 ; donde P = mg /\ sen 9 = f cal».
268 Física-Primer Nivel Félíx Aucallanchí V.

DEBES SABER QUE:

Elplano de oscilación del


Luego: k.x = mg. f ~
movImIento pendular es in-
variable, vale decir que es y reemplazando esta expresión de k en las relaciones (14.7) y (14_8)
el mismo a través del para el período y frecuencia respectivamente de un oscilador mecánico.
tiempo. tendremos:

k= If I
r----""'-'-"",

(14.12) T=21t.{f ; y; 2~ (14.13)

14.3C. LEYES DEL MOVIMIENTO PENDULAR


Ira Ley.- Se le llama también Ley del Isocronismo,
y establece que «El
movimiento pendular tiene un período independiente de la amplitud
angular», siempre que ésta no exceda de los 10° (Fig. 14.9).
2daLey.- De acuerdo con esta Ley se establece que: «El período de un
péndulo es independiente de la masa pendular». De la Fig. 17.9
notamos que m < M; a pesar de ello se cumple que: Tm = TM'
3Il1 Ley.- Esta ley establece que: «El período es directamente
proporcional con la raíz cuadrada de la longitud pendular» De la
Fig. 14.9 se verifica que:

T U -Ii

4ͪ Ley.- Según esta ley se cumple que: «El período de un péndulo es
inversamente proporcional con la raíz cuadrada de la gravedad local».
De la Fig. 14.11 se establece que:

1
Tu -
¡g

IMPORTANTISIMO

SI un péndulo estuviera
dentro de un sistema ace-
lerado, entonces en las rela-
ciones (14.12) y (14.13) la
cantidad g representa a la
t!
:f.~~
IN Y2· Ley 3N Ley 4"Ley
gravedad efectiva. Fig. 14.9 Fig. 14.10 Fig. 14.11
14.3D. APLICACIONES DEL PENDULO SIMPLE
a) Péndulo que bate segundos.- Se dice que un péndulo bate segundos
cuando cada oscilación simple la realiza en un segundo, de modo
que al desarrollar una oscilación completa, emplea en total dos (2)
segundos. Su longitud viene dada así:

1I = [!;- I (14.14)
Oscilaciones 269

b) Péndulo de Faucault.- En vista de que el péndulo desarrolla sus


oscilaciones en un mismo plano de manera invariable, Jean L.Faucault
utilizó esta propiedad para demostrar que la tierra rotaba alrededor de
su eje Norte-Sur, comprobando que el plano de oscilación de su péndulo
giraba 11°15' en cada hora en el sentido de Norte a Este (en París).
e) Péndulos en la Geo1ogía.- Dado que el período pendular varía con la
gravedad local, y ésta varía con la estructura del terreno, es que el péndulo
simple es utilizado en la búsqueda de yacimientos de minerales o
depósitos de petróleo, pues una pequeña variación de "g" por causa de
aquellos originará una sensible variación en el período (1) del péndulo.

flROBLEMAS RESUELTOS (FA PARTE)

Probo 1.- La ecuación de movimiento de una masa de 0,75 kg que oscila en un resorte es:
x = 0,25 cos (8 t), donde x se expresa en metros y t en segundos.¿Cuál es la energía
total de la masa? Dar la respuesta en joules.
A] 2 B] 2,5 C] 3 D] 1,5 E] 1
Reso}ución.-

En base a la relación (14.1) para la elongaciónx de un cuerpo con M.A.S., y comparándola con la que se nos
dá en los datos, podemos establecer que:
A COS (ffit) = 0,25 cos (8 t) ~ A = 0,25 m y (¡) = 8 radls

Luego al velocidad máxima de la masa será: vmáx = roA = (8 radls) (0,25 m) ~ vmáx = 2mls
Finalmente, la energía mecánica total de la masa la encontraremos a partir de la relación (14.9), considerando
que ésta al pasar por la posición de equilibrio sólo tiene energía cinética:

RPTA.D

Probo 2.- Se sabe que la aceleración máxima de un M.A.S. es 60 cm/s2, Ysu velocidad máxima
es 30 cttüs. Se desea saber a qué distancia de la posición de equilibrio la velocidad
del móvil será 24 cmls.
A] 10 cm B]8cm C]6cm D] 9cm E] 1 cm
Resolución.-
Utlizando las relaciones para la aceleración y velocidad máximas, obtendremos (¡) y la amplitud A :
amá x (¡)2 A 60cm/ s2
v má x = roA ~ 30cm / s = (¡) (¡) = 2 radls

~ roA = 30 A = 15 cm

Luego, de la fórmula (14.3) para la velocidad, tendremos:

v =lPJ A
2
-x 2 24 cm/s = (2 radls) J (I5cm)2 - x2

x=9cm RPTA.D
270 Física-Primer Nivel Fé/ix Aucallanchi V.

Probo 3.- En la figuras 1,2 y 3 se muestra un resorte, el cual en reposo tiene un largo de 1,2 m, y
con fuerza adicional F tiene 1,5 m de largo. Si la fuerza F cesa (se quita) abruptamente,
la misma tendrá un movimiento cuya ecuación será:
A) 0,5 sen (oof); k (0,2 m) = 10 N
B) 0,3 sen (oof); k (0,3 m) = 10 N
C) 0,5 sen (oof); k(O,3 m) = 10 N
O) 0,5 sen (oof); k (0,2 m) = 10 N
E) N.A. UNI 78 Fig.2
Fig.3
Resolución.-

En base a la Fig. 2 podemos reconocer la posición de equilibrio del sistema Bloque + Resorte, de modo que
en base a la Ley de Hooke se puede establecer que la deformación del resorte es: x = 1.20 - 1,00 = 0,20 m.
Luego:
Fresorte = Peso => kx = mg => k (0,2 m) = ION

Ahora, de la Fig.3 se reconoce que el sistema Bloque + Resorte, al ser liberado por F, tendrá oscilaciones cuya am-
plitud A estará dada por :A = 1,50 - 1,20 = 0,30 m. Luego, la ecuación del movimiento estará dada por la relación
(14.1), en donde como se sabe podemos usar la función seno o coseno, dependiendo de las condiciones iniciales
del movimiento. Sin embargo, de acuerdo con las alternativas vemos que será preferible utilizar al primero:

x = A senúor) x = 0,3 sen (rot) RPTA.D

Probo 4.- Un cuerpocue/g:Jdelexfremo de un rescxte y oscila verticalmente con el ceoxx: de 2s./lJau-


menta la masa del cuerpo e'11 kg el nuevo r::aiocb es de 4s. ¿()"Jéj es ellICJIcf00 la masa iniciar?

A) 1/2 kg B) 1/3 kg C) 1/4 kg O) 1/6 kg E) 1 kg UNI88


Resolución.-

De acuerdo con los datos vemos que será recomendable utilizar la relación (14,7):

Estado Inicial: TI = 21t.J!i => 2 S = 21t.J!i (1)

Estado final: T = 21tJ}


2 => 4 S = 21t J "':,k
g
(2)
A continuación dividimos (2) + (1) miembro a miembro y despejamos "m":

4m=m + 1 => m=lkg RPTA.B

Probo 5.- Un bloque es suspendido de un resorte ideal


logrando estirarse 16 cm ¿Con qué fre-
cuencia oscilará el bloque si a partir de su
posición de equilíbrio es impulsado para
efectuar un M.A.S. vertical? (g = 1t2 m/s2).
r':
A) 0,2 O) 2,5 ¡:¡
B) 0,8 E) 4,0 J----------
C) 1,25)
Oscilaciones 271

Resolución.-
Analizando al bloque en su posición de equilibrio podemos establecer
que la fuerza recuperadora (F) del resorte y el peso: P = mg son
iguales, entonces:

F=P ::::} ko= mg ::::} s. = g


m [;
(*)

Luego, utilizando la relación (14.7) para la frecuencia tendremos que:

I fk I ¡g I ("";2 1t
f= 21t Vm = 21t V[; = 21t V0J6 = 21t(0,4)

l
.. f= 0,8 osc/s ::::} f = 1,25 ose/e RPTA. C

Probo 6.- Determinar el oencoo de las oscilaciones


del bloque m, si todos los resortes tienen la
misma constante de elasticidad (k).

A) 21tJm/k D) 1tJm/3k

B) 1tJ2m/k E) 21tJm/3k

C) 21tJ3m/k

Resolución.-

Considerando que el sistema Bloque + Resorte inicia sus


oscilaciones desde su posición de equilibrio, su análisis se
vuelve más simple al punto de poder prescindir de la fuerza
de gravedad, dado que las oscilaciones solo lo producen las
fuerzas recuperadoras de los resortes. Asimismo, reduciendo
los dos resortes superiores que están en paralelo: k'= k + k =
2k, observamos que para cualquier desplazamiento vertical x
del bloque, los resortes k y k' se deforman la misma longitud,
y ello solo se presenta cuando éstos están en paralelo (ver
item 14.2.c). Luego, la constante de elasticidad del sistema
será: k~q = k + k' = 3k. Finalmente, el período de las
oscilaciones estará dada por la relación (14.7):

T =2 1t ¡¡ RPTA. E

Probo 7.- Dos masas iguales m, y m2 oscilan con igual


amplitud A. Después de transcurrido un
tiempo 1. cuando m, está en x = A/2
(subiendo), la posición de m2 es x = r./3/2)
A (bajando). El ángulo de fase de m2 con
respecto a m, es entonces:
A) 1t/4 D) 51t/ó

B) 1t/ó E) 11t/ó

E) 1t/3
272 Física-Primer Nivel Félix Aucollonchí V.

Resolución.-
De acuerdo con lo explicado en la página
N° 222 en relación a la procedencia de la
fórmula (14.1) para la posición x, se indicó
que los ángulos de fase <1> estaban vincula-
dos con la circunferencia donde se desarro-
lla el M.C.U., que proyectando origina al mI
M.A.S. Así pues, ubicando a los móviles
PI y P" correspondientes amI y m2 tal como
se indica en el gráfico adjunto podemos
deducir que el ángulo de fase (<1» de m2
respecto de mI viene dado así:
<1> = e2 - el····· (*)
Pero, de acuerdo con la figura:
el = 30° y e2 = 270° - 30° = 240°
Luego, en (*) :
<1> =240° - 30° = 210° => • = '; Tad RPTA.E

Probo 8.- Un péndulo oscílo con un período de J1fj s. ¿Cuál será su nuevo período en segundos
sí su longitud se incremento en un 60"lo?
A)4S B) 8 C)2 O) 10 E) 5
Resolución.-

Originalmente se tiene: r¿ = 21tl-f = .JlOs ..... (1)

Y por condiICIió n de l probl ema: T -_ 21t VIL+60Lll


f g 00 -_ 21t 'f76 VfI
g
-,¡ 1,0

Luego, reemplazando (1) en (2) : t; = (10) (Jl,6) = .JlO.l6 =J16

RPTA.A

Probo 9.- Un reloj de péndulo hecho en la fierro es llevado a un planeta X donde la gavedad es 4 ve-
ces ma¡or que la de la fierro. Después de 1hora en la fierro, el reloj en el planeta X marcará.
A)2h B)1/2h C)4h O) 15mín E)1h UNI79
Resolución.-
Para resolver este ejercicio sería conveniente partir de la siguiente reflexión: «Si dos relojes de péndulo
iguales experimentan la msma gravedad para un número igual de oscilaciones (n) en ambos, los intervalos
de tiempo transcurridos serán iguales».
Así pues, en el caso analizado dejemos que en ambos relojes se efectuen igual número (n) de oscilaciones,
para así comparar los tiempos transcurridos para cada uno, Luego:

- Reloj ubicado en la Tierra: (21tH)


- Reloj Ubicado en el planeta X.; l1t2 = nT2 = n
(21tJEJ
Oscilaciones 273

fll2 _ (g
Luego: RPTA.B
lh - v4i

Probo 10.- Un péndulo de longitud L se hace


oscilar de dos modos.' Fig. 1 Y Fig.2.
¿En qué relación se encontrarán sus
S/9L ~g
periódos de oscilación T2 / T1?·
,o"'--"clavo
A) 6/5 O) 5/6 ~g ,

B) 2/3 E) 5/9

C)4/9 Fig(l) Fig(2)

Resolución.-

Debemos reconocer que el péndulo de la Fig. 2 es en realidad un péndulo especial, constituído por dos pén-
dulos simles de longitudesLy 4U9, que se desarrollan de modo que cada uno contribuye con su semiperiodo
para el periodo del sistema. Luego:

T = Ta + Tb = 1t
2 2 2
(l:
Vg
+ 1t J 4L /9 g

y dividiendo miembro a miembro :

y Tl 5
Tl = '6 RPTA.D

Probo 11.- Si la longitud de un péndulo simple aumentase en 2 m su periodo se triplicaría. La


longitud del péndulo en metros es .'

A) 0,25 B) 2,00 C) 2,25 0)4,00 E) 4,25 UNI89


Resolución.-

Atendiendo las condiciones del problema y utilizando la relación (14.12) para el periodo de un péndulo
simple, tendremos:

Al inicio: Dividiendo miembro a miembro :

Al final:
37) _
7) -
J 1+2111
1 ~ 9
1+2111
I

y despejando "1" encontramos: 1 = 0,25 m RPTA. A


274 Física-Primer Nivel Félix Aucallanchi V.

Probo 12.- Un péndulo simple de 10 kg Y de 2m de longitud es retirada 10 cm de su posición de


equilibrio y dejada en libertad. ¿Cuál es la energía cinética del péndulo al pasar por
su posición más baja? (g= 10m/s2).
A)1.2J B)OAJ C)O,2SJ D)2,Sj E)1SJ

Resolución.-
Reconociendo que la amp!itudA = 1Ocm de las oscilacio-
nes son bastante pequeñas con relación a la longitud del
péndulo, diremos que el movimiento desarrollado es
armónico y simple. Luego, la energía cinética en la posi-
ción de equilibrio coincidirá con el valor de la energía
mecánica total del sistema.

-E
Ec- m-
- l.m 2
2 vmax ..... (vmáx=W/'t
,.d)

::::}
Ec =~ mm2A2 pero m=H
2
l gA l (10)(10)(0,1)2
::::} E =-m-- ::::} E=-
e 2 l e 2 2

.. Ee = O,25J RPTA.C

Probo 13.- Se ha construído un túnel que pasa por el centro de la Tierra a lo largo de todo un
diámetro. Si soltamos un objeto en la superficie terrestre de modo que ingrese por el
túnel, se observará que él experimentará un movimiento oscilaforio. ¿Cuál es el perio-
do de dichas oscilaciones (en segundos), si consideramos R = 64000 km, yg= 10 m/s2
en la superficie? Despreciar los efectos de rotación de la tierra y el rozamiento en el
túnel.
A) 11001t B) 10001t C) BOO1t D)SOO1t E) 1 6001t
Resolución.-

Cuando el objeto se encuentra a una distancia x del centro de +--R-t


la Tierra, experimentará una fuerza F cuyo valor podrá deter- ,,
,,
minarse en base a la Fig. 13.4, en donde notamos que la gra- ,,
vedad (g) en el interior de la Tierra varía !inealmente, tal que: ,,
,,

Luego: F = mg ::::} F = mgs· (1) (1 s )x ....


= g (*)

y como vemos, la fuerza de gravedad (F) varía linealmente


con la posición "x'' del móvil con relación al centro de la
Tierra. Como se recordará, esta forma de dependencia sólo
se presenta en el movimiento armónico simple, donde: F =
kx. Y comparando con (*), encontramos:

kx=(7fgs) x::::} k=7fgs

A continuación utilizamos la relación (14.7) para el período. x R


Oscilaciones 275

T= 21t Gi
Vk = 21t~ m
m.g s l R
T= 16001ts RPTA.E

Probo 14.- Unpéndulo simple de longitud L se encuentra oscilando con un período Tsuspendido
del techo de un ascensor que se encuentra en reposo con respecto a la Tierra. Siel
ascensor empieza a moverse hacia arriba con una aceleración constante m, el
cetiooo del péndulo:
A) Es igual a T . B) Esmayor que T C] Esmenor que T
O) Esigual a (T- T/m) E) N.A UNFV 86
Resolución.-

Tal como se explicó en el Capítulo 9, la gravedad efectiva dentro de un ascensor que acelera con a = m y
hacia arriba está dada por: gel = g + m . Luego, el período de oscilación del péndulo vendrá dado así:

1) Ascensor en reposo: T = 21t J 1


gel
..... (1)

2) Ascensor acelerado hacia arriba: T = 21t J 1


gif
=> T = 21t J 1
g+m
..... (2)

Luego dividiendo las ecuaciones (2) 7 (1) encontramos que: TT'=J 1 <l =>, T' < T RPTA.C
g+m

Probo 15.- Encontrar la frecuencia de las oscilaciones de un péndulo simple de longitud L = 50


cm, si viaja colgado del techo de un bus que viaja horizontalmente con una
aceleración 0= 7,5 m/s2 (g = 10 m/s2) .

A) 5/1t B) 5/21t C] 21t O) 2/n E) 4

Resolución.-

En base al gráfico adjunto podemos reconocer


que la posición de equilibrio del péndulo se
ha desviado de la vertical el ángulo e. Este
ángulo es el mismo que forma la gravedad
efec,riva ( g ef) en el interior del vagón con la
vertical, e
A continuación, del triángulo vectorial encon-
tramos:

=> gcf = 12,5 mls2

Finalmente encontraremos la frecuenciafdel


péndulo utilizando la relación (14.13):

=-.l.Jgef _ -.l./12,5mls2
f 2n L - 2n V 0,5 nI => f = ~ »sc/s RPTA.B
276 Física-Primer Nivel Félix Aucallanchi V.

DEBES SABER QUE:


IIIIMOVIMIENTO ONDULATORIO
En lo Fig. 14.12, los mo-
léculas de agua solo suben Todo lo visto y estudiado de la Mecánica hasta aquí nos permite
y bajan verticalmente res- hacer la siguiente pregunta:¿ Existe algún ente distinto de una partícula
pecto de su posición de e-
quilibrio (horizontal] cuando o conjunto de partículas, capaz de desplazarse de un punto a otro? La
los ondas posan horizontal- respuesta es Sí, y ellos son las ondas.
mente por el lugar que

Jh
ocupan.

(a) tÓ

.:=1)'j)l'J PH
. ,,} J!) .1':
,~

CUIDADO! (b)

Losondas mecánicos solo


se transmiten si los medios Fig.14.12 Fig.14.13
son elásticos, homogéneos
e Isotrópicos.
14.4.A. CONCEPTO DE ONDA
Un medio será Isotróplco si
suspropiedades físicos son
los mismos en todo direc- Llamamos así a toda pertubación producida en un medio sólido,
ción. líquido, gaseoso e incluso en el vacío, y que se transmite por vibraciones
de un lugar a otro transportando energía sin el movimiento mismo del
medio. De acuerdo con su naturaleza pueden ser:
Al) Ondas Mecánicas.- Son aquellas que se generan en los medios só-
lidos, líquidos o gaseosos, en donde las perturbaciones se transmiten
por vibraciones de las moléculas. Ejm: El sube y baja de un bote
ante el paso de las olas, el ondeo de una bandera, el sonido, ..... ,etc.
A2) Ondas Electromagnéticas.- Son las que se producen en el' vacío
por causa de estímulos eléctricos y magnéticos. Son los únicos
que no necesitan de un medio mecánico para propagarse.Ejm: La
luz, las ondas de radio, de televisión, los rayos X, ... , etc.
A3) Ondas de Materia.- Son aquellas que acompañan de manera inhe-
rente a todos los cuerpos, y que se hacen más evidentes cuando la
velocidad con que ellos se mueven es relativamente grande.
OJO!
14.4B. TIPO DE ONDA
Enlo Fig. 14. 13, 01golpear
lo joven lo base de lo loto, B1) Ondas transversales> Estas se caracterizan porque las moléculas
ésto vibrará haciendo que del medio oscilan con respecto a su posición de equilibrio de manera
los moléculas del aire
encerrado también oscilen, perpendicular a la dirección en que se propagan las ondas. En la
de manero que estos Fig .14.14, la cinta blanca del muelle sube y baja hasta su posición
vibraciones 01 llegar o lo de equilibrio.imientras la onda termina de pasar por ella. ¿Sabes
llamo lo apagarán. hacia dónde viaja la onda?
Oscilaciones 277

B2) Ondas Longitudinales.- En este tipo de ondas las partículas del MUY IMPORTANTE
medio oscilan en la misma dirección en que se propagan las ondas.
En el ejemplo de la Fig. 14.15, por medio de un lápiz, el resorte es Unpulso de onda es solo
una perturbación que ex-
obligado a oscilar, notándose que el pulso generado hace que las perimenta un medio.
espiras vibren en la misma dirección en que éste se propaga. Una sucesión continua de
pulsos se constituyen en una
•• onda.

ATENCIÓN

La velocidad de onda (v)


representa la rapidez con
que la onda cambia de
posición en un medio de-
terminado. Esconstante si el
medio es uniforme, homo-
géneo e Isotrópico.

I v =~=Af 1 (14.5)

Fig.14.14 Fig.14.15
l4.l4C. ELEMENTOS DE UNA ONDA

Cl) Ciclo.- Llamamos así a la oscilación completa que realiza una


parte del medio cuando pasa una onda por el lugar que ella ocupa.
En una onda transversal el ciclo es la silueta móvil que vemos.

C2) Período (T).- Es el tiempo que emplea un ciclo en pasar por un


punto del medio. Es también el tiempo que utiliza una parte del
medio en efectuar una oscilación completa.

C3) Frecuencia (j).- Representa el número de ciclos que atraviesan


un plano de referencia en cada unidad de tiempo. Se expresan en
ciclos/segundo (c.p.s) o en hert; (Hz).

C4) Amplitud (A).- Llamamos así a la máxima elongación lineal que


experimenta una parte del medio cuando por ella pasa una onda.
--
C5) Cresta.- Son los puntos del ciclo de máxima amplitud positiva.
En las onda transversales, son los puntos más altos.

C6) Valle.- Son los lugares del ciclo de máxima amplitud negativa.
En una onda transversal, son los puntos más bajos.

C7) Longitud de Onda (A.).- Es la distancia que recorre la onda en


un tiempo igual al período. Es también la distancia entre dos crestas
o valles consecutivos.
278 Física-Primer Nivel Félix Auca/lanchi V.

ATENC/ON!
Enlasondas senoldales, el
número de onda (k) y la fre-
cuencia angular (w) permi-
ten encontrar la velocidad
(v) de los ondas. Observa:

211:
le ~ w
v= T= 211: =? v = k
w

Fig 14.16
D)ONDASSENOIDALES
Cuando las distintas partes de un medio oscilan con M.A.S. ante
el paso de las ondas, se dice que éstas son ondas senoidales , como el
mostrado en laFig.14.16. Así, un punto cualquiera del medio de coor-
denadas (x;y) debido al movimiento ondulatorio oscilará de modo que:

~~A~e.u ~:tt'Dt) l (14.16)


INTERESANTE donde: k = 21t/A (número de onda), ro = 21t/T, Yt = instante de tiempo.
Después de concluir una El signo (+) o (-) se empleará si la onda se mueve respectivamente
superposición de ondas, ca- hacia la izquierda o hacia la derecha del eje x.
da una continúa su propa-
gación r;nanteniendo Intac- E) VELOCIDAD DE ONDA EN UNA CUERDA TENSA
tas todas suscaracterísticas.
Cuando se producen ondas en una
cuerda tensa, estas se propagan con una
velocidad cuyo viene dado por la si-
guiente relación:

r::~q (14.17)

donde: T = Tensión en la cuerda;


11= masa / longitud
PARA RECORDAR
F) COMPORTAMIENTO DE LAS ONDAS
Dado que en un M.A.S. se
incluye de fase $, en el Fl) Superposición de Ondas- Cuando dos o más ondas concurren en
movimiento ondulatorio esta un mismo lugar de un medio, se puede apreciar que las partes del
característica aparecerá en medio vibran de manera que su desplazamiento viene dado por la
la ecuación general de suma de los desplazamientos que cada onda produce de manera
dicho movimiento:
independiente.
y= x Asen (kx ± rot + $) F2) Interferencia de Ondas> Denominamos así al fenómeno de su-
perposición de dos o más ondas que poseen igual frecuencia.Si
Así dos puntos distanciados durante la superposición los valles o las crestas coinciden , la
21tA, vibrará con Igual fase
o en fase y dos puntos interferencia es constructiva, y se dice que las ondas están en
distanciados (21t + 1)fJ2 vi- fase. Pero si las crestas coinciden con los valles, la interferencia es
bran desfasados 90°). destructiva, y se dice que las ondas están desfasadas.

Oscilaciones 279

~- .
DEBES SABER QUE:
~ Durante el fenómeno de
retraccIón las ondas com-
~ bIan los valores de su veloci-
dad y longitud de onda, sIn
embargo la frecuencIa per-
--------~~~---- manece constante.

~
Interferencia Constructiva
Interferencia Destructivo
------------------~~--~~----~----------------~
Fig 14.19 Fig 14.20
F3) Reflexión y Refracción de Ondas.- Cuando las ondas inciden sobre
una superficie u obstáculo, ellas cambian la dirección de su movi-
miento, retornando al medio original de propagación; a ésto llama-
remos Reflexión. Si las ondas pasan de un medio a otro diferente,
su dirección de propagación se desvía, observándose además un
cambio en el valor de la velocidad y en la longitud de onda, mas
no en la frecuencia; a todo ello llamaremos Refracción.

Fig 14.21 Fig 14.22


F5) Difracción de Ondas.- Llamamos así al fenómeno que experimen-
tan las ondas cuando pasanjunto a los bordes de un obstáculo o de
OJO
una abertura, y que consiste en un cambio en la dirección de
propagación de las ondas. Enel tetxxreao de polariza-
con, parte de la energb que
F6) Polarización de Ondas.- Cuando una onda es obligada a pasar se transmite se pierde. de
por una ranura, parte de ella dejará de vibrar y sólo la parte que es rrxxxi cue las mcbs que con-
paralela a la ranura continuará oscilando con igual frecuencia, am- trvxn 9J popagacim b tu-
C€() con una meror energb.
plitud y longitud, aunque su intensidad habrá disrninuído; a todos
estos efectos llamaremos polarización .
•l~~~~ , >

~~ >,
0'_

.
I1,

Fig 14.23
280 Física-Primer Nivel Fé/ix Aucallanchi V.

VELOCIDADES DEL G)SONIDO


SONIDO
Llamaremos sonido a toda aquella perturbación que sea percibida
(o OOC) por el oído humano, lo cual solo sucederá si la frecuencia de éstas vi-
Medio mis
braciones está comprendida entre 20 y ~O 000 n«
Estas vibraciones
pueden propagarse por cualquier medio mecánico sólido, líquido o
Acero 5100 gas. Se sabe que las ondas sonoras están constituídas por ondas longitu-
Agua 1450 dinales, en donde las moléculas vibran con relación a sus posiciones
Aire (200C) 340
de equilibrio en forma paralela a la propagación del sonido.
Granito 6000
H) EFECTO DOPPLER
Hidrógeno 1270
Hierro 5100 Es conocido el hecho de sentir más intenso el sonido del motor
Vidrio 5500 de un automóvil ciuando se acerca a nosotros que cuando se aleja, en
Oxígeno 317 este caso se dice que la fuente sonora se mueve. Si el automóvil estuviera
detenido y nosotros caminaramos hacia él, el sonido nos parecerá más
intenso cuando nos aproximamos que cuando nos alejamos. Todo ésto
ECO!
se debe a que durante el movimiento del foco emisor o del oyente, las
SllIl sonido de corto dura-
ción se reflejo yvuelote olobser-
ondas que percibimos están, más o, menos concentradas. Este efecto
vador 01 cabo de un tiempo fue descubierto por Christian Doppler en 1842.
no inferior o 1110de segll1-do,
dicIxJ observrx:Jor percibe uno rereteccrones
repetición del sonido original:
Un eco. /. ",\
i"'t.
'"
Poro q..¡ep.;edo hd::>erecolo
st.perf/c/e reflectoro debe esicr ;j
oIejocJo (XJr/o menos 17 m.
y
~~
ccndensacicnes

~
'- Diapason
Vibrando
Fig 14.25 Fig 14.26

JlRD8LEMAS RESUELTOS (21M NRTE)


Probo 16.- Se emffe en el aire un sonido con frecuencia de 800 c.p. s que luego penetra en el agua. Sien-
do la ve/lxidad del sonido en el aire de 340 mis yen el agua de 1 450 mis. entonces la
relación: (Lmgitud de onda en el agua) 1(Lmgitud de onda en el aire) es aproximadamente:
A)0,23 B) 0,62 C) 2,82 D) 3,14 E) 4,26 UNI 83 - v
Resolución>
En base a la relación (14.1) para la velocidad de una sola onda, y teniendo en cuenta que la frecuencia/de
ésta no se altera al pasar de un medio a otro, tendremos:

RPTA.E

Probo 17.- Un observador determinó que había 2.5 m de separación entre un valle y una cresta
adyacente de las olas superficiales de un largo, y contó 33 crestas que pasaban en
35 segundos ¿Cuál es la magnitud de la velocidad de las olas superficiales? (en mis)
A) 1117 B) 22;7 C] 3317 D) 4417 E)5517 UNI 90
Oscilaciones 281

Resolución.-
Bajo el supuesto de que el número de crestas coincide con el número de ondas completa, tendremos que la
frecuencia de éstas está dada por:

f= N° ~e ondas =:) I= 33 hertz


Tiempo 35
Asimismo podemos decir que de acuerdo con los PLANO DE
datos, la longitud de onda viene dada por:
A = 2.d = 2 (2,5 m) =:) A = 5m
-/-- OBSERVACION

TRES ONDAS
Por último, la velocidad de las olas estará dada por la ,
relación (14.15) : ~
. ! 1-
v=l.f=5m·
35
33
Hz =:) v mis = 3i
RPTA. C
T -'-1-
d'd
Observación.- En rigor al número de crestas contadas desde un plano de observación determinado.éste
coincide siempre con el número de ondas completas menos uno.

Probo 18.- En una cuerda tensa se producen ondas con una longffud de onda de 5 centimetros. Si la
onda recorre 100 centímetros en 5 segundos, su frecuencia en ciclos por segundo (Hertz) es:
A) 1 B) 4 C] 2 O) 3 E) 5 UNMS 93
Resolución.-
De acuerdo con los datos podemos determinar la velocidad v de las ondas en base a la relación (4.1):

v =!!... = l005cm =:) v = 20 mis


t s
A continuación,utilizando la relación (14.15) para determinar la frecuencia, dado que por datos: A = 5cn!..

Luego: v = A .f =:) 20~n!. =(5cm).f f= 4hertz RPTA.B


s

Probo 19.- Se mantiene tensa una cuerda flexible de 30 m de longitud y 10 kg de masa entre dos
postes con una tensión de 2 700 N . Si se golpea transversalmente la cuerda en uno de
susextremos, hallar el tiempo se segundos que tardará la onda transversal producida en
alcanzar el otro extremo.
A) 1 B) 1/2 C] 1/3 O) 1/4 E) 1/6 UNI 84 - I
Resolución.-

Según los datos, podemos reconocer que: L = 30 m ,


m = 10 kg, T = 2 700 N .Luego, por la relación (14.17)
.. - l.
-
,/

encontraremos la velocidad v de los pulsos de onda:

v= fI = JTL = J2 700.30
Vil m ro
=:) v = 90 mis
Seguidamente, encontraremos el tiempo t que emplea
un pulso en recorrer la distancia e = L; para ello usaremos ' .... ,
la relación (4.1) vista en M.R.U.

e 30m
t = -;;-= 90mJs RPTA.C
282 Física-Primer Nivel FélixAucallanchi V.

Probo 20.- La figura muestra una onda producida en la superficie del agua de una piscina. La
velocidad de propagación es 1 rrils. Halle el período de la onda en segundos.
A) 0,2 D) 10,0 y (cm)
B) 1.0 E) 0,5

C)0,1

UNI 93 - 2
Resolución.-

Atendiendo el gráfico de las ondas en el plano x - y, podemos afirmar que éstas tienen una longitud: A. = 20
cm. y como la velocidad es v = I m= 100 cm/s, encontraremos el período T de las oscilaciones en base a la
relación (14.15).

v=y A =>
T= 1=
v 20 cm
100 cml s => T= 0,2 s RPTA.A

Probo 21.- Cuál es la velocidad de las ondas transversales en una cuerda de 2 m de longitud y
100 g de masa, sometido a una tensión de 80 N?

A) 10 rtüs B) 30 rrfs C) 20 trüs D) 50 trüs E)40 mls

Resolución.-
pulso
Extrayendo los datos, tendremos:

m = 0,1 kg ; L = 2m ; T = 80 N
Luego, la densidad lineal de masa se obtendrá de
la fórmula (14.17) encontramos la velocidad de
las ondas:
Jl = mIL = 0,1 kg/2m = 0,05 kglm
y recurriendo a la fórmula (14. ), encontramos -"~---------L----------~
la velocidad de las ondas:

v
rf I 80N
= V ¡; = V 0,5 kg iz > v = 40 mis RPTA. E

Probo 22.- La ecuación de una onda transversal que se propaga en una cuerda es:

y = 4 sen 2n (;0 - O:1)


Donde x e y están en centímetros, y t en segundos. ¿Cuál es la velocidad de
propagación de las ondas?

A) 1 ttüs B) 3 rnts C) 2 ttüs D) 5 rnls E) 4 mls


Resolución,-

Haciendo una comparación entre la ecuación dada con la fórmula (14. ), encontraremos los parámetros
físicos: k y oi, veamos:
Oscilaciones 283

y = Asen (kx - últ) = 4 sen (2n


20 2n)
x-O,1t

k= 2ncm·l úl = 2n rad
20 0,1 s

y por la fórmula (14. ) encontramos la velocidad de propagación de las ondas dadas:

úl 2n/0,1 rad l s
v=-=--- --- v=200 cm/s
k 2n/20 -1
cm

v = 2m/s RPTA. C

Probo 23.- La elongación de una onda en función de la posición y tiempo está dada por: '1 = 8
sen(3x - 1020t) con x en metros y t en segundos. Luego, la velocidad de propagación
de la onda én mls es igual a :
A) 1 020 B) 510 C]340 O) 3060 E) 24 UNI - 92
Resolución.-

Comparando la ecuación dada con la relación (14.16) para ondas senoidales, tendremos que:
Asen (kx - últ) = 8 sen (3x - l 020t)

:=) k = 3 radlm. y: O) = 1 020 rad/s

Seguidamente encontraremos la velocidad v de las ondas en base a la relación (14.18)

v = .!Q = 1 020rad/s
k 3 rad/s

:=) v = 340 mis RPTA.C

Probo 24.- Elhecho de que una persona se mueva a 3 mts Ydetecte una frecuencia de 444 Hz
de una fuente emisora de 440 Hz se explica por:
A) El efecto DOPPLER.
B) Unproceso animado de detencción de 4Hz.
C] Que la fuente emisora se mueva con una velocidad menor de 3 mis.
O) Que la fuente emisora se encuentre a 3m.
E)Que la distancia de separación es de 12 m.
UNMSM84
Resolución.-

La única razón para que un oyente recepcione ondas de sonido con distinta frecuencia es el hecho de que él
y/o la fuente experimenten un movimiento relativo, lo cual fué estudiado por el científico Chistian DOP-
PLER, por lo cual a este fenómeno se le denomina Efecto Doppler.
RPTA.A
284 Física-Primer Nivel Félix Auca/lanchi V.

1.- o es una característiea del M.A.S.: A) 1


A) Es un movimiento periódico. B) 2
B) Es un movimiento vibratorio. C) 3
C) Es un movimiento armónico. D) 4
D) Es un movimiento rectilíneo.
E) Todas son
E) Es un movimiento curvilíneo. incorrectas
2.- Un móvil con M.A.S. al pasar por su posición
de equilibrio:
( ) Tiene una velocidad máxima.
( ) Posee una aceleración nula. 6.- Dados los siguientes osciladores mecánicos,
éstos tendrán períodos (7) tales que:
( ) Presenta su máxima elongación.
Indicar verdadero (V) o falso (F):
(1) m
A) FVV B) FVF C) VVF D) FFV E) VVV
3.- En el M.A.S. de amplitud 2m, es falso que:

Q
o Q Q
R. S T
-O-- m --& m ~-m-G--m o-

(v = velocidad; a = aceleración; 1= tiempo)


~ ~>~>~ ~ ~=~=~ C) ~<~<~
A) I(PQ) = I(QR) ~ ~<~<~ m ~>~>~
B) IvQ I = IvSi < IvR I 7.- Si duplicamos la amplitud de las oscilaciones
C) lIQ =-lIS del sistema mostrado, es falso que:

D) I(QR) = (R5)
k m
E) IlIT I > IlIS I > IlIRI ---'lWO~
4.- En todo M.A.S.: A __ .? . _ A
( ) La velocidad es variable.
( ) La aceleración es constante.
( ) Existe una fuerza recuperadora. A) El período se conserva.
Señalar con verdadero (V) y falso(F). B) La energía mecánica aumenta.
C) La posición de equilibrio (O) es la misma.
A) VVF B) FVV C) FFV D) VFF E)VFV D) La frecuencia se incrementa.
5.- En relación al sistema físico mostrado en E) La energía potencial se cuadruplica.
donde no existe rozamiento, se dan las ggui~ntes .
afirmaciones (g = 10 m/s2): . ....:~ _. 8.- Elige l.as palabr~~ que completen correcta-
- -; ••• J!-.:.. r_ mente la siguiente oracion:
1) Es un movimiento oscilatorio. «La de un oscilador mecánico corn-
Il) Es un movimiento períodico. puesto por un resorte y una masa, es independiente
III) Es un M A S de la ------------------- ».
• . .. . .• A) velocidad máxima; aceleración.
IV) El período de oscilación es 8 s. B) energía mecánica; frecuencia .
...>
Indicar cuántas de las afirmaciones son correctas. C) aceleración máxima; masa.
D) amplitud; velocidad.
E) frecuencia; amplitud.
Oscilaciones 285

9.- Los períodos (7) de los péndulos mostrados 13.- Del ejercicio anterior, si mantenemos la mis-
son tales que: ma frecuencia pero aumentamos la tensión en la
A) T,=T2=T3 cuerda:
( ) Aumentará la longitud de onda.
B) T,<T3<T2
( ) Disminuye el período de las oscilacione dc P.
C) T2<T3<T, ( ) Aumenta la velocidad de las ondas.
D) T2=T,<T3
Marcar verdadero (V) o falso (F)
E) T,>T3>T2 A) VVV B) VFV C) FFF D) VFF E) FFV

14.-Cuando un tren parte del reposo, lo que se trans-


mite desde la locomotora hasta el último vagón es:
I} Velocidad 1Il) Aceleración
1I) Energía IV) Un pulso
10.- Si un péndulo es llevado a una altura igual al
radio terrestre: Señala la(s) correcta(s):

1) Su período aumenta A) Il Y 1II B) I Y III C) III D) III Y IV E)ll Y IV


11) Su amplitud angular disminuye 15.- Las ondas generadas de la Fig. 14.12 se en-
1lI) Su frecuencia disminuye cuentran con un obstáculo P. Indique el comporta-
miento posterior de las ondas:
IV) Su energía mecánica aumenta
Indicar la(s) proposición (es) correcta(s)
A) IylIl B)IIyIV C)I D)llI E) IV B) I~

11.- Un astronauta lleva un reloj de péndulo a la


Luna. Entonces en dicho lugar: t" • ~>

( ) El período del péndulo disminuirá.


(
(
)
)
La frecuencia del péndulo aumentará.
El reloj se atrasará.
,
C) ll-_ ~ D)
( ) Para poner a tiempo el reloj, será necesario dis-
minuir la longitud del péndulo. ~ El
Señalar verdadero (V) y falso (F) >1
A) FVVV B) FVFV C) FFVV D) VVVF E) FVVF
12.- Si la mano de la persona aumenta la frecuen-
cia de sus oscilaciones, será cierto que: 16.- Sabiendo que las líneas circtrtares. represen-
tan las crestas de ondas en el instante mostrado, se
afirma que: Ondas
Cuerda - En C hay interfe-
tensa
rencia constructi-
v,a.
- En B hay un valle
- En A hay interfe-
A) La velocidad de las ondas aumenta. rencia destructivo
B) La longitud de onda diminuye.
C1 La amplitud de las oscilaciones aumenta.
D) El tiempo que emplea P en dar una oscilación
aumenta Señalar verdadero o falso según corresponda
E) El número de ciclos visibles en la cuerda dis-
minuirá. A) FFF B) VVV C) VVF D) FVF E)VVF

• ,
286 Física-Primer Nivel Félíx Aucallanchí V.

PROBLEMAS flRO'UESTOS

NIVEL! 07.- Si un péndulo simple de 8 oscilaciones err..


32 segundos, ¿Cuál es la longitud (en m) de dicho
01.- Una partícula experimenta 60 oscilaciones en péndulo? (g = n2 mls2)
12 segundos. ¿Cuál es es su frecuencia en Hz y su
período en segundos? A)3 B)2 C)8 D)4 E) 9

A) 5; 0,2 B) 3 C) 2 ; 5 D) 4 E) 1 ; 1,3 08.- Indica cuál de los péndulos, al ser liberados


desde las posiciones indicadas, llegará primero a la
02.- La partícula mostrada gira con M.C.U. y veloci- posición de equilibrio.

Tir C)T,
dad angular ú) = la re vis en una circunferencia de
radio r = 14cm. Con relación al M.A.S. de la sombra
proyectada en la pared vertical, se pide la rapidez
máxima (en mis). Considerar: (rr '" 2217) b)~ 4°
L
a) i
A) 2,6
B)6
C) 8,8
2m : :
L
m I~m
A)a B)b C)c D) a y b E) Todos
D) 3,2
E)4 09.- En la figura se muestra un pulso que emplea
0,4 segundos en ir desde A hasta B, ¿Cuál es la
03.- En el oscilador mecánico mostrado, se sabe velocidad de las ondas (en mis) de la cuerda?
que: k= lOONlm;m=4kg ;A= 10cm;calcularla
aceleración máxima (en mls2) del movimiento. A) 20
~
A) 1,3 B)30
A B
B) 2,6 C)40 ,,
F ,
C)2,8 D)50 lE
, 16m
,
D) 2,5 E) 60

E) 1,2 10.- Determinar la rapidez (enm/s) de las ondas que


se producen en la cuerda mostrada.
04.- Determinar la constante elástica del sistema de
resortes mostrado. A)2
A) 1 k
B) 2 k
3k - 6k
B)3
C)4
SON 45kg~1
~'"";'E--- 9m :
C) 4 k D)5
D)7k
E) 6
E)3k
11.- En una cuerda se producen ondas senoidales de
OS.- ¿Cuál es el período (en s) de un péndulo si su modo que su ecuación de onda (en el S.I.) es :
longitud es de 4m y g '" n2 mls2?
A)3 B)6 C)2 D)5 E)4 y = 0,1 sen (-Ix - 1-t)
06.- ¿Cuál es la frecuencia en hert: de un péndulo de 9m Determinar la velocidad de las ondas.
de longitud en un lugar donde la gravedad es de 4n2 mls2?
E) 3/4 A) 1 B) 1,2 C) 1,3 D) 1,4 E) 1,5
A) 1/3 B)2/1 C)2/6 D) 6/4
Oscilaciones 2Kl

NIVEL 2 3k

12.· La posición (x) de una partícula con M.A.S.


viene dada por: x = 0,4 cos (51). Encontrar una ecua- } 7mII11J~~\ ~m
ción para la velocidad y la aceleración en función del
tiempo 1.
::k
__ WJlWW ~

~
A) v = -2 scn 51 ; a = -10 cos 51 A) 7rrJ20s B) 20rrJ7s C) 18rrJ5s

B) v = 2 cos 51 ; a = 2 sen 5/ D) 15rrJ6s E) N.A

C) v = -5 sen 21 ; a = -13 cos 51 18.- Un mismo péndulo es colocado en la Tierra y en


un planeta donde la gravedad es nueve veces más
D) v = -2 cos 31 ; a = 7 sen 51 intensa. ¿En qué relación se encontrarán sus períodos
respectivamente?
E) N.A.
A) 3 : I B) I : 3 C) 4 : 2 D) 4 : I E) 2 : 3
13.- Un cuerpo con M.A.S. oscila con una frecuencia
angularw = 5 rad/s y una amplitud A = 10 m. ¿Cuál 19.- Si dos péndulos tienen sus longitudes en la ra-
es el valor de la velocidad y la aceleración para la zón L,: L2 = 9: 4 ¿En qué relación deberán encon-
posición x = 6m? trarse sus correspondientes períodos?
A) v = -40 mis; a = 150 mls2 A) 3 : I B) 3 : 2 C) 3 : 3 D) 4 : I E) 2 : 3
B) v = 40 mis; a = 150 mls2 20.- Si los períodos de dos péndulos están en la
razón T,: T, = I : 2 ¿En qué relación se encontrarán
C) v = 150 mis; a = 40 m/s' sus longitudes L,: L2?
D) v = 50 mis; a = 100 mls2 A) 4 : 2 B) 3 : I C) I : 4 D) 4 : I E) 3 : "2
E) v = -100 mis; a = 150 mls2 21.- ¿Cuál es la longitud de un péndulo que bate
segundos? g '" n2 mls2
14.- Se sabe que la aguja de una máquina de coser
oscila con una amplitud de 2cm y experimenta una A) I m B)2m C)3 m
aceleración máxima de 50 m/s". ¿Cuáles es su fre-
cuencia angular? D)4m E) 5 m
A) 10 rad/s B) 30 rad/s C) 50 rad/s 22.- En una cuerda tensa se producen ondas
senoidales de frecuencia angular w = íSradls'j nú-
D) 30 rad/s E) 60 rad/s mero de onda k = 3m-' ¿Cuál es la velocidad de pro-
pagación de las ondas (en mis)?
15.- Si la velocidad máxima de un móvil con M.A.S.
es de 36cmls y su frecuencia es de 2rrJ Hz ¿Cuál es la A) I B) 3 C)5 D)2 E)6
amplitud de las oscilaciones (en cm)?
23.- Indicar cuál es la amplitud de la oscilación en el
A)8 B) 10 C)5 D)9 E) N.A. lugar donde se produce la superposición.
16.- Si para estirar I cm a un resorte se necesita una
fuerza de 4 N. ¿Con qué período oscilará una masa
de 9 kg colgada de él?

A) 3 ns B) I ns C) 0,5 ns
A) ICln B) 3 cm C) 4 cm
D) 2ns E) 0,3 rts
D)2cm E) N.A.
17.- Determinar el período de las oscilaciones del
sistema mostrado m = 49 kg , k = 50 Nlm.

H
288 Física-Primer nivel Félix Aucallanchi V.

NIVEL 3 29.- Un péndulo simple se encuentra dentro de un


ascensor, observándose que las oscilaciones duran
24.- Si la velocidad máxima de un móvil con M.A.S. 3,14 s. Si la longitud del péndulo es de 2m, ¿Cuál es
es de 36 cm/s, y su frecuencia es de 2/n Hz, ¿Cuál es el valor y dirección -le la aceleración del ascensor?
la amplitud de las oscilaciones? (g = 10 m/s2)

A)3cm B) 12 cm C) 9cm A) cero B) 2m/s2 (i)

D)6cm E)5cm D) 2 m/s? (-1-) E) 1m/s2 (-1-)

25.- Un platillo cargado se suspende de un resorte 30.- ¿A qué altura sobre la superficie terrestre será
estirándolo 25 cm. Se retiran 360 g del platillo, que- necesario llevar un péndulo simple para que su pe-
dando en ella 640 g. ¿Cuál es la ecuación del movi- ríodo se duplique? R = radio terrestre.
miento que se genera? g '" n2 m/s2
A) 0,5 R B) 1,5 R C) 3R
A) x = 25 cos (5m) D) x = 0,25 cos (2,5m)
D)2R E)R
B) x = 15 cos (2m) E) x = 2,5 cos (m)
31.- Dos pulsos de onda generados en una cuerda
C) x = 1,5 cos (m) tensa, se mueven como se indica en la figura ¿Cuán-
to tiempo tardarán en pasar uno sobre el otro a par-
26.- Cuando un bloque es colgado suavemente de un tir del instante indicado en el gráfico?
resorte, logra estirarlo 16 cm.Si en lugar de esto el
bloque sujeto al resorte es dejado caer, oscilará de v=2cm/s v=2 cmls
""---c> -<J----I'-
manera que la ecuación de su movimiento será ....
(g '" n2 m/s2)

A) x = 0,08 cos (2,5 m) +2m+······8cm·······~·2m·~·

B) x = 0,08 cos (5 m) A) 2 s B) 3 s C) 4 s D) 6 s E) 9 s

C) x = 0,16 cos (2,5 m) 32.- La ecuación de una onda transversal en una cuer-
daes:
D)x= 0,16 cos (5 m) y = 0,15 cos (3x - 6t)
E) x = 0,32 cos (2,5 m) Calcular la aceleración máxima (en l/1/s2) transversal
de la cuerda en m/s? (Los datos están en el S.L)
27.- Un cuerpo oscila con M.A.S. y amplitud
A = 20 cm ¿A qué distancia de la posición de equili- A) 1,2 B) 2, 7 C) 3,6
brio, tendrá una energía cinética que será el triple de
su energía potencial? D) 5,0 E) 5,4

A) 10 cm B) 15 cm C)5cm 33.- Si las ondas del ejercicio anterior se producen


en una cuerda tensa de 10 n metros de longitud.
D)8cm E) 12 cm ¿Cuántas ondas completas se podrá apreciar en ella?
28.- ¿Cuál es el período de las oscilaciones del siste- A)8 B) 12 C) 14
ma mostrado? k = 121 N/m y m = 0,98 kg (n = 2217)
D) 15 E) 10

A) 0,1 s B) 0,5 s C) 0,8 s

D) 0,4 s E) 0,2 s
F(uidós
OBJETIVOS
1.- Entender los conceptos de densidad,
peso específico y presión.
2.- Establecer las principales propiedades
mecánicas de los líquidos y los gases a
partir de sus características moleculares.
3.- Conocer y aplicar los principios de Pas-
cal y Arquímides, tanto para líquidos
como para gases.

1estudio de la materia bajo la forma de sólidos, líquidos y


gases nos conduce inevitablemente al reconocimiento de
que ellas están constituídas por moléculas, las mismas que
según su orden y configuración física les concede determinadas propie-
dades. En esta parte del curso nos dedicaremos a describir y formular
las leyes que gobiernan el comportamiento de los líquidos y de los
gases, a los que indistintamente denominaremosjluidos, porque tienen
ARQUlMEDES
la propiedad de fluir, es decir, que se pueden escurrir con facilidad.
En la página 69 habíamos establecido que una rama de la Mecáni- (287-212 a.de C.)
ca estudia a los fluidos, y se llama Mecánica de los Fluidos, la misma Este gran sabio e inventor
que se divide en : griego nació en suocuso.
colonia griega ubicada en
A) Hidromecánica.- Estudia a los líquidos, y a su vez se divide en: Slcllia (sur de Italia). Además
Al) Hidrostática : Líquidos en reposo de descubrir la Ley de la
Palanca y dar un sinnúmero
A2) Hidrodinámica : Líquidos en movimiento de aplicaciones como la
polea, los aparejos. las co-
B) Neumomecánica.- Estudia las propiedades mecánicas de los ga- tapultas. el tornillo o espiral
ses y se divide en: de Arquímedes para la
extracción de agua •..., etc.
Bl) Neumostática : Gases en reposo .EI problema de la corona
B2) Neumodinámica : Gases en movimiento. del Rey Hleron» es sin duda
una de las historias más
11II ANALOGIAS y DIFERENCIAS ENTRE conocidas de Arquímedes,
11III LIQUIDOS y GASES
pues ella le condujo al des-
cubrimiento de la Ley del
Empuje. que fué publicada
Los fluidos en general carecen de forma propia; sin embargo, en en su libro «Sobre los cuer-
el caso de los líquidos, ellos poseen volumen definido, adoptando la pos flotantes>. Sus inventos
forma del recipiente que lo contiene, y si su volumen es menor presenta impidieron por cerca de tres
una superficie libre que será plana si la presión externa es uniforme, y años a los romanos poder
invadir Siracusa. muriendo
será horizontal si el líquido está en equilibrio o acelerando verticalmente. por manos de los hombres
Tanto en los líquidos como en los gases las moléculas se encuentran en del General romano Maree-
un permanente movimiento caótico llamado «Movimiento browniano». lo el año 212 a de C.
290 Física-Primer Nivel Fé/ix Aucallanchi V.

DEBES SABER QUE: Asimismo, dado que en los líquidos las moléculas se encuentran relati-
vamente cerca unas de otras, ellos se vuelven incompresibles; ésto no
Los valores de las densida-
des expresadas en kg/m3
sucede con los gases. Sin embargo, en ambos se manifiestan propiedades
son 1 000 veces mayores elásticas.
que las que se dan en g/cm3. Lú¡vü:los SlIpf!I'jicie Libre Plana Y
Por eJemplo:d
Horizontal
OHp = 1000 kg/m3 = 1 g/cm3

TABLA DE DENSIDADES
Sustancia

Materia Nuclear 2.1014


Centro del sol 100
Platino 21.4
Oro 19.3 Los I/quidos IOn

Mercurio 13.6
elárticos,pw.s
m:upert1lJ su forma
Plomo . 11.3 original
Cobre 8.9
Hierro 7.8
La Tierra 5.0 Fig 15.1
Aluminio 2.7
McxJera de écxro 1. 12 11II DEFINICIONES PREVIAS
Agua 1.0 a) Densidad (D).- Es aquella magnitud escalar que nos indica la canti-
Hielo 0.92 dad de masa que tiene un cuerpo por cada unidad de volumen. Cada
Kerosene 0.80 sustancia (sólida, líquida o gaseosa) tiene su propia densidad.
McxJera de roble0.80
Madera de balsa 0.1 3 Densidad = Masa ~ I D- m 1 (15.1)
Volumen - V
Oxígeno 0.001 43
Aire 0.001 29 En el S.l. la densidad se expresa en kg/m3.
Nitrógeno 0.001 25 b) Peso Específico (p ).- Denominamos así a la magnitud física escalar
Hello 0.000 18 que nos informa el peso que posee una sustancia por cada unidad
Hidrógeno 0.000
Mejor vacío logado
Espacio Interestelar
Espacio Intergalóctico
09
10.19
10.24
10-29
de volumen.
Peso Específico = Peso
Volumen
~ EIJ p= V (15.2)

En el S.l. el peso específico se da en N/m3.


Observaciones.«
OJO! mg
1) Dado que: P = mg ~ p= ~ I p=D.g 1 (15.3)
Cuando la gravedad del
V
lugar es g = 9.8 m/s2• se
verifica que p y O tienen el
2) De la relación (15.1): m=DV ~ I P=DgV I (15.4)
mismo valor numérico si el
primero se expresa en: e) Presión (p ).- Cuando un cuerpo entra en contacto con otro se pre-
senta entre ellos fuerzas de acción y reacción, las que no se concen-
gr f kgf
-- ,0,-- tran en un punto, sino mas bien lo hacen en una superficie. La pre-
cm3 m3 sión viene a ser la magnitud física tensorial que nos indica la forma
y el segundo en g/cm3 o kg/ cómo una fuerza se distribuye perpendicularmente sobre una su-
m3 respectivamente. perficie. Su valor medio se determina así:
Fluidos 291

Fuerza Normal ~ DEBES SABER QUE:


Presión = Area :=} ~ (15.5)
*] Si disminuímos el área de
En el S.l. la presión se expresa en pascal (Pa): lPa = lN/m2. aplicación de la fuerzo,
la presión aumentará.
Esto es lo que buscamos
Presión F.
/i,, / F=8(SON)= 400N con las agujas, clavos,
cuchillos, ..., etc.
, 2
A=4m.2m =8m
,,
,, F~ *] Si aumentamos el área
"~
- .
~A"'" ,.~':.':-'.:.~..:.',,'
" ~ SON SON 'SON
=:>p=F =~
A 8m de aplicación de la fuer-
zo, la presión disminuirá.
sDj;{-~t:r~-"'~-- .• p=SOPa Esto se utilizo para cami-
R-F+F. 1/" / / .fi' 2m nar sobre la nieve, en los
1m2} 4m asientos, en la base de
F=Fuerza Normal las construcciones, ....etc.
Fig 15.2

11II PRESION HIDROSTATICA (Ph) OJO!

Cuando nos sumergimos en el agua notamos por propia experien- Una clara prueba de que
la presión hidrostótica eJer-
cia que a mayor profundidad la presión que experimentamos es cada ce fuerzas normales sobre
vez mayor cuanto más nos sumergimos. Esto se debe fundamentalménte las paredes Internas es el
al peso del líquido que se ubica por encima de nosotros. Cuando el lí- hecho de que el chorro de
quido está en reposo, el valor de la presión que ejerce un líquido de líquido de la bolsa salga
formando 9C!'con ella.
densidad DL a una profundidad h viene dado por la relación:

Presiá Húh tótit:a- (Densidad del ) (G .J~.l\ (Altura del )


n 'OS - Líquüo raveaaa¡ Líquido

:=} IPb =DLgh I (15.6)


Observaciones» Cuando queremos encontrar la presión total en UII punto
interior de un líquido, debemos saber qué presión Po experimenta la superficie
libre del mismo; de este modo la presión total p vendrá dada por:

. I P =r; +DLgh I (15.7)


INTERESANTE

Unex¡:;eriTlentosencilb aie
prueba la fTXJltidireccionali-
ckxt á3 b {Xesíónconsisteen
surrergir una pequeña gota
de acene En el SEnO de 0/00-
hal diluído, observándose la
farrx.r;iéf¡ á3 lila g:;ta esféri-
ca, cue sob se explicaría si la
presión es b rrisrro en toda
Su superficie, y asimisrro en
toda dirección y senticb.

Fig 15.3

11II PRESION ATMOSFERICA (Po)


Como bien sabemos, la Tierra está envuelta por una capa gaseosa
llamada atmósfera, la misma que está compuesta por una mezcla de gases: Experimento de Plateau
Nitrógeno, oxígeno, argón, anhidrido carbónico, hidrógeno, ... , etc.
292 Física-Primer Nivel Félíx Aucal/anchi V.

Ya todo ello llamamos aire. Este inmenso océano de aire tiene


peso, y por lo tanto ejerce presión sobre cualquier punto ubicado en su
interior, de modo que es al ni vel del mar donde esta presión es máxima,
y va disminuyendo a medida que aumentamos la altura respecto de
aquel (Fig. 15.4).

EXPERIMENTO DE TORRICELLI
En 1 644 Evangelista Torricelli ideó un mecanismo al que llamó
barómetro, para medir la presión atmosférica (Fig. 15.5), comprobando
que ésta era capaz de equilibrar el peso de una columna de mercurio de
76 cm de altura cuando el barómetro se situaba al nivel del mar. Ahora,
por la relación (15.6) tendremos:
Po = DHg·gh = (13 600 kg/m3)(9,8 mls2)(0,76 m) ~ Po = 1,OLl05 Pa
EVANGELISTA TORRICELU
(160B-1647)
Científico italiano. Fué dlscí-
pu/odel gm Gol/leo, paquien
llegó a saber que el aire tenía
peso. TiNO serios cuestlona-
mientas a la popular creen- 111I
cia de lo fIIosofla Aristotélica
de que: -La naturaleza le tie-
ne horror al vacío». Por aque-
lla época se sabía que una
bomba de vacío no podía
-elevar. agua por encima de
1 Hg -----------
los 10m, Y para él ésto ocu- Fig 15.4 Fig 15.5
rría simplemente porque el
aire o atmósfera es el que por
su peso puede equilibrar una
columna de agua de hasta
11II MANOMETROS
10 m, Y si ésta fuera de mer- En la práctica es difícil medir la
curio, de hasta 76 cm, por ser presión absoluta de un gas o de un líqui-
de mayor densidad. Luego, do, pues en todo momento nos vemos
los líquidos suben por los afectados de la presión atmosférica. Los
sorbetes cuando succ/ona-
mas aire, porque dlsminuí- manómetros son dispositivos físicos
mas la presión en esa zona, que permiten medir directamente la di-
y no porque el líquido preten- ferencia que existe entre la presión ab-
da llenar la parte vacía. Torri- soluta y la presión atmosférica. Para el
celll concluyó que la ,altura
de la atmósfera es aproxima- caso mostrado en laFig. 15.6, la presión
damente de 500 km, y su manométrica viene dada por la altura Hg
peso es de 10 N por cada de la columna de mercurio:
ctn? al nivel del mar; en
donde la presión es móxima.
Fig 15.6

DEBES SABER QUE: . TEOREMA FUNDAMENTAL DE LA


Se llamó atmósfera (atm) a • HIDROSTATICA
la unidad de presión que era
igual a la que ejerce la atmó- Cuando dos puntos están dentro de una masa líquidaen reposo, se verifica-
fera al nivel del mar y a CJ'C. rá que: «La diferencia de presiones entre ambos puntos depende directamente
1atm = 76 cm Hg de la diferencia de sus profundidades». Así, de la Fig. 15.7 se cumplirá que:
= 1,033 kgf/crr¡2
1atm =1,01.1OSPo z 1,01 Bar
*1 Bar = 100 kPo = 105 Po
I Pz - Pt = DL g('z -'1) I (15.9)
Fluidos 293

Corolario> «Todos los puntos pertenecientes a una masa líquida en reposo y


ubicados en una horizontal soportan la misma presión total».

11II VASOS COMUNICANTES


Es el conjunto de recipientes unidos entre sí, en los que al vertir un
líquido, éste alcanzará el mismo nivel horizontal en todos los recipien-
tes, lo cual se explica por el corolario del teorema anterior. (Fig. 15.8).

BLAlSE PASCAL

(1623-1662)
Este científico francés fué
un niño prodigio que vivió
solo 3?,años, l:' susúltimos 16
Fig 15.7 Fig 15.8 anosjlos paso en una pro-

11I1 PRINCIPIO DE PASCAL


funcjb con templaclón reli-
giosa. Fué filósofo, físico y
geÓmetra. En 1648 publicó
en el libro «tratté de L'
Tanto los líquidos como los gases tienen la propiedad de transmitir Equilibre des Uouers», el
únicamente presiones, verificándose que: «Toda variación de presión en principio que hoy lleva su
un punto de unfluido se transmite íntegramente por igual y en toda direc- nombre. Antes de cumplir
ción a todos los otros puntos del mismo». Ver ejemplo de la Fig. 15.9. los 18 años escribió un libro
de Geometría, e Inventó la
11IIII PRENSA HIDRAULICA
primera máquina calcula-
dora. Más tarde, en estre-
cha colaboración con
Una de las principales y más importantes aplicaciones del princi- Fermat estableció los funda-
pio anterior es sin duda la prensa hidráulica, dispositivo físico que se mentos de la teoría moder-
muestra en la Fig. 15.10, en donde una pequeña fuerza (Fl) se convierte na de las probabilidades. Al
en una fuerza mayor (F2) gracias a la relación existente entre las áreas enterarse de la experiencia
de Torrlcelli envió a su
(A2/A 1) de los pistones, tal que: cuñado FlorinPerier la cúspi-
de de la montaña Puy de

(15.10) F'l. = (~ )Ft 1\ ez = (4;}1 (15.11)


Dome (1000 m), observan-
do un descenso de 80 mm
en la columna de mercurio.
Hizo lo mismo en la Iglesia
=;;Unhombre puede empujar un de Saint Jacques de la
I
Boucherle (50 m), compro-
I pequeño pistón y equilibrar la bando que por cada 10,5
luerza de varios hombres" m la altura de mercurio
descendía 1 mm.
1

I~ <.A¡~.·.~
--c> . ...':).:;:(.
...•...
i.:.,.
. .•.•....
'.
~ CUIDADO!
. l:1p.
Las relaciones (15.10) y
(15.11) se han establecido
suponiendo que la prensa
es 100"10 eficiente, es decir,
1 en la transmisión de energía
no ha existido pérdidas.
Fig 15.9 Fig 15.10
294 Física-Primer Nivel Félix Auca/lanchi \1.

JlROBLEMAS RESUELTOS (f1lA MRTE)


Probo 1.- Una botella pesa 0,22 kg. Cuando se le llena de agua pesa 0,38 kg, Y cuando se le
llena de kerosene su peso es de 0,351 kg. ¿Cuál es la densidad del kerosene?
A) 0,819 g/cm3 B) 0,980 g/cm3 C) 0.1 g/cm3
O) 0,522 g/cm3 E) 0,688 g/cm3 UNFV 83
Resolución.-
Expresando todos los pesos en gramos fuerza, y recordando que un gramo masa es equivalente a un gramo
fuerza, tendremos que:
a) Botella con agua.- La masa de agua que hay dentro de la botella será:
PH 0
2
= Ptot(l) - P botella = 380 grf - 220 grf =:} 'lnH20 = 160 g
(*) Recordando que un gramo de agua ocupa un volumen de un centímetro cúbico, tenemos que el volumen
interno de la botella es: V = 160 cm',
b) Botella de kerosene.- La masa de kerosene que hay dentro de la botella es:
Pkerosene = Ptot(2) - P botella = 351 grf - 220 glf mkcroscne = 1'31 g

/
=:}

y finalmente encontraremos la densidad del kerosene por medio de la relación (15.1):


D _ mkerosene _ 131 g
=:} DK = 0,819 g/cm3 RPTA.A
K - V - 160 cm3

Probo 2.- Tres cuerdas de 6 m de longitud cada D


una presionan verticalmente a un poste
de 100 N de peso y 15,10-4 m2 de sección
transversal, como se muestra en la figura.
La tensión de cada una de las cuerdas es
e -_
de 50/Jó N. Escoja la presión correcta
en N/M que el poste ejerce sobre el suelo.
\
\
\
---
\
A) 1(J5 O) 15.106 r----,- B

B) 15,1(J5 E) 10.4
\
\
\
\
--
C) 106 A

UNI84 -1
Resolución.-
En base a la vista superior del sistema y al ángulo e
que forman las cuerdas, podemos deducir que la . base
triangular ABC es equilátera, de modo que:
x = L.fi/3

A continuación se puede notar que las tres cuerdas L


forman con la vertical (poste) el mismo ángulo e, cuyo
coseno se puede determinar en base al triángulo som-
breado:

AL-------------------~R
L
Fluidos 295
D

~ cose = -16
3 .
. Seguidamente reconocemos que la fuerza F J.. que ejer- B _
ce el poste sobre el piso es la suma de su peso mas la
\ L
contribución de las componentes de las tres tensiones \
en las cuerdas (T). Luego, utilizando la relación
(15.5) tendremos que:
\
\
...----- J H-A--
- e
--
\ Area r" »>

r: P+3Ty P+3Tcose \
\
P=A= A A
\

A
100+3(50/16) (16/3) 2
~ P= 15.10-4 ~ P =.. lO:" Nlm RPTA.A

Probo 3.- La presión atmosférica sobre la syperficie


A indicada en la figura es 1.033 kg /crri2.
Si la densickxJ del líquido del recipiente y
del tubo es 13,6 g/cm3, la altura h es:) h
A
A) 14,0 cm D) 7,59 cm

B) 3,54 cm E) 35,4 c'1

C) 75,9cm UNMSM86
Resolución.-
Reconociendo que el esquema dado concuerda con el experimento de Torricelli (item 15.5) debemos establecer
la igualdad de las presiones en base al teorema fundamental de la hidrostática, utilizando asimismo la relación
(15.6) para la presión hidrostática: (Deg = PL)

P tubo -- P A ~ P h- p ~ 13,6 J.!.....h = 1,033.!5.L ~ 13,6J.!.....h = 1 033 ..E..


líquido - A 3 2
cm cm cm3 cm2
h = 1 033 cm .~ h '" 75,9 cm RPTA.C
13,6

Probo 2.- La base del émbolo de una bomba impelente es un círculo de "D" cm de diámetro.
¿Qué fuerza en nevvtons es preciso ejercer sobre dicho émbolo para elevar el agua a
una altura de."H" metros? (g = 10 m/s2).

Al 0,25 1t 02H B) 250 7t02H C) 1t02H UNFV82


Resolución>
En base al esquema podemos reconocer que la 1'0
presión ejercida en x se debe a la fuerza F que
buscamos. Ahora, si el diámetro del émbolo es D
cm, expresado en metros será: DII OO.A continua-
ción, el área del émbolo estará dada por: F
A

A = ¡ r= D
( 100)
1tD2
4.104 (m
2
)
:: x =::

Ahora, aprovechando el Teorema Fundamental de


la Hidrostática diremos que los puntos x e y expe-
rimentan la misma presión total. Y utilizando las
296 Física-Primer Nivel Félix AucalJanchi V.

relaciones es (15.6) y (15.7) para la presión, tendremos:

F
Px = Py:::} A+ Po = Phidrostática + Po

Reemplazando datos y despejando F:

:::} F=A.IOJ.IO.H= (1tD:J.


4.10
IQ4H :::} ~PTA.A

Prob. 5.- Dos líquidos 1 y 2 que no se mezclan


Vado
están en equilibrio en un tubo de vidrio
en forma de U, como se muestra en la
figura. La relación entre las presiones
en los puntos A y B, PAlPa es:

A) 1/3 O) 4/3
B) 2/3 E) 2
C)1

UNI83-2
Resolución.-
En base a la relación (15.6) podemos reconocer que
los puntos A y B experimentan presiones debido al
líquido que se encuentra por encima de ellos, luego:
-r-r-

= D2 D,
PA =_D2·g·h }
PB- D ¡.g. h
PA
PB DI
(1) + ..
h
A
,--
h h

Trazando una horizontal por la interfase que separa a


B • +
h
h
los líquidos l y 2 podemos establecer la igualdad de .....1..... ___ H!!rJZ!!fJ:!qj _ _ _ _ • ~
x
presiones entre x e y:
lnterfase y

DI
P« = Py

PA
-=-
2
y de (2) en (1):
PB 3
RPTA.B

Probo 6.- ¿Cuál es la presión sobre el fondo del cilindro


mostrado, que tiene dos líquidos de den-
sidades 01. y O2, con 2 01 = O2 = 1 200
kg/m3 y 1,L b = a = 0,6 m (g = Y,a m/s2).
=~=~=~=~~~~~=~=~~~=
TapaI :::::::: DI::::::::
-Ta
A) 0,94 N/cm2 O) 9,6 N/cm2 :::::::::::::::::::::::::::::
---t,
B) 9,4 N/crn'2 E) 1,29 N/cm2 D2 b

C) 9,6 N/crn'2 '---- __ ----L - - - L


UNI89
Fluidos 297

Resolución.-
A pesar que no se indica qué lugar ocupan los líquidos
Tapa
en el recipiente, se puede asegurar que el menos denso
(DI) se ubica arriba. Luego, la presión total que se
ejerce en el fondo viene dado por la presión que ellí-
quido 2 aplica directamente en el fondo, mas la presión
que ejerce el líquido 1 en la interfase de los líquidos, de
modo que la masa del líquido 2 sirve para transmitir
esta presión. Hasta el fondo. Luego:

Pr = Plfq(l) + Plfq(2) = DIga +D2gb

Pt = [600.0,6 + 1200.1,2]. 9,8 ~ Pr = 9408 N/m2 , ó , PT "" 0,94 Nlcm2 RPTA.A


Nota aclaratoria.- En la presión total que se ha calculado no se incluyó a la presión atmosférica debido a
la placa que se colocó encima del recipiente, el cual sella herméticamente al sistema.

11III PRINCIPIO DE ARQUIMEDES EUREKA!

El rey Hieron había dado


Cuando vamos a la playa y llevamos una pelota , casi siempre uno determinado masa de
intentamos sumergirla en el agua; al hacerlo lentamente notamos cómo oro o un orfebre para lo ela-
el agua se opone a nuestro intento, durante esta experiencia sentimos boración de uno corono.
que el agua intenta sacar la pelota fuera de ella, empujándola hacia arriba Cuando fué terminado y
entregado se sospechaba
. Asi mismo es una experiencia conocida el ver flot~ un barco o un que habia sido adulterado
globo aerostático;en todos estos casos está siempre pr~~~nte una fuerza con plato. Se encargó lo so-
que proviene del líquido o de un gas, es decir de un fluid~~n general.La luclon de este coso o Arquí-
medes, pero sin que dañara
explicación de este fenómeno y en particular de la fuerza.,nvolucrada , lo corono. Se cuento que
se debe al sabio Arquímedes , quien estableció que: «Todo; cuerpo total uno vez 01Ingresar o los ba-
o parcialmente sumergido en un fluido en equilibrio, exp~rimenta por ños públicos, Arquímedes
parte de éste una fuerza neta vertical de abajo hacia arriba, a la que notó que el agua tebotsaoo
llamaremos empuje y cuya recta de acción pasa por el centroide del o medido que Introducía su
cuerpo en el agua, lo que
fluido desalojado, ». le permitiría luego llegar o
Para determinar el valor del empuje (E) utilizaremos el ejemplo lo solución del problema.
de la Fig.I5.ll.EI cuerpo suspendido y el recipiente vacío pesan ION Emocionado y olvidando
y IN respectivamente. Al sumergir el cuerpo en el agua, su peso se vestirse, salió corriendo por
los calles gritando EUREKA,
reduce a 6ft{, observándose que ahora la balanza marca 5N.Estas nuevas que en griego significo iLO
lecturas nos permiten deducir que :EI cuerpo recibe un empuje hacia DESCUBR/!.
arriba de 4N y el líquido desalojado pesa igualmente 4N.
En conclusión:

1) E = Preal -Paparente

2) E = PESO Iíq, desalo'


~.

y de la relación (15.4)
OJO
i E = PL Vs.1 (15.12) En lo Flg. 15. 11 se puede
apreciar que el volumen del
líquido desalojado es IglKJl01
lE = Dvg·Vs 1 (15.13 ~~~===::::::::.~_-=::::::::===~ voIunen sumergicbdelcuerpo:
VD = Vs
298 Física-Primer Nivel Félix Aucallanchi V.

Debemos reconocer que las relaciones (15.12) y (15.13) , se han


deducido a partir de la relación (2), en donde el peso del líquido
ATENC/ON !!
desalojado se ha sustituido por una expresión en función de su peso
Los relaciones (15.12) esjecífico (PI) y de su volumen, el mismo que coincide con el volumen
y(15.13) , se han deducido sumergido del cuerpo (Vs) .Del mismo modo, en la relación (15.13),se
poro el coso de un líquido, ha reemplazado PL por su equivalente visto en la relación (15.3), es
sin emborr;¡o estos relaciones decir en función de la densidad del líquido (DL)y de la aceleración de
son tomblen aplicables poro
el coso de un gas la gravedad local (g).

11II FLOTACION
Cuando introducimos un cuerpo en un líquido o en un gas, éste
puede adoptar diferentes posiciones, los cuales dependerán del empuje
recibido por parte del fluido. Estas posiciones pueden ser tres:
12 Cuerpo flotando fuera del fluido
En este caso se verificará que el peso del cuerpo es equilibrado
por el empuje del fluido. Asimismo se observará que el volumen del
cuerpo es mayor que el volumen de su parte sumergida..
Peso = Empuje 1\ Ve> Vs (Fig. 15.12a)

22 Cuerpo sumergido en equilibrio


En estos casos se comprueba que el peso del cuerpo es equilibrado
por el empuje del fluido. Del mismomodo, es evidente que el volumen
del cuerpo coincide con el volumen sumergido.

Peso = Empuje 1\ Ve= Vs (Fig.15.12b)

iCUlDADO! J2 Cuerpo en el fondo


En el coso (c) de IoFIg. 15.m Cuando el peso del cuerpo es mayor que la fuerza de empuje, se
solo existirá empuje si existe comprobará que el cuerpo se dirige hacia el fondo del recipiente que
líquido en lo base del blo- contiene al fluido.De igual modo se logra apreciar una coincidencia
que; de no ser así: E = O
entre los volumenes del cuerpo y de su parte sumergida.

(Fig.15. 12c)

Observaci6n.- Si mezclamos varios líquidos no miscibles en un recipiente,


al cabo de un tiempo éstos se separarán, de modo que el más denso se coloca
en elfondo, y sobre él en orden de densidades decrecientes los otros líquidos
(Fig. 15.13).

DEBES SABER QUE:


Cuando un globo floto en (a) ~
el aire, llamaremos fuerzo
d
oscensionol (F o lo expresión: (b)
Fa= E - P (e)
donde: E = Empuje
P = Peso del globo Fig 15.12 Fig 15.13
Fluidos 299

PROBLEMAS RESUELTOS (21M NR7f)

Probo 7.- Unapiedra pesa en el aire 80 N Ysumergida completamente en el agua 35 N. Entonces,


la densidad de la piedra es:
A] 0,8 g/cm3 B] 8,0 g/cm3 C] 7,8 g/cm3 O] 3,5 g/cm3 E] 2.4 g/cm3
UNMSM90
Resolución.-
De acuerdo con los datos se tiene que el peso real de la piedra es: P Real = 60 N, lo que significa que su masa
es m = 6 kg. Asimismo, se reconoce que debido al empuje (E) del agua la piedra experimenta una aparente
pérdida de peso tal que:

E = P Real - P Aparente = 60 N - 35 N ~ E = 25 N
Y de acuerdo con la relación (15.13) para el empuje, encontraremos el volumen (V) de la piedra:
t 1 1
••

DHO.g.V=E ~ 103.1O.V=25 ~. V=25.1O-4m3


2

A continuación calculamos la densidad (D) de la piedra en base a la relación (15.1) y a los datos deducidos
hasta aquí:

m 6kg 2400
D =V= 25.10-4 m3 = 2 400 kg/m3 ~ D = l 000 g/cm3 ~ 7 D = 2.4 g/cm3 RPTA.E

Probo 8.- Un bloque de metal pesa 20 9 en el aire, 18 9 en el agua y 16 9 en un f/uído des-


conocido. ¿Cuál es la densidad del f/uído desconocido en g/cm3?
A] 0,50 B] 0,75 CJ 1,00 O] 1,25 E] 1,50 UNI93 -1
Resolución.-
Procediendo de un modo similar como se hizo en el problema anterior, primero calcularemos el volumen (V)
del bloque, y a continuación la densidad (DL) del líquido desconocido. Veamos:
a} En el agua.- De acuerdo con los datos, trabajaremos con gramos fuerza (gr), de manera que el empuje
estará dado por la relación (15.12 ): .

~ l ~ .V = 26 gr - 18 gr ~ V = 8 cm3
cm3
b) En el líquido desconocido.- Procediendo de la misma forma tendremos:

E2=PR-PA2 ~ PL.V=PR-PA2 ~ PL(8cm3)=26gr-16gr ~ PL=l,25gr/cm3

Luego, la densidad del líquido será: D~= 1,25 glcm3 RPTA.D

Probo 9.- El 15"10del volumen de un tronco que está flotando en agua se encuentra encima de
la superficie del nivel del líquido. ¿Cuál es el peso en kg-f del metro cúbico de la
madera que constituye el tronco?
A] 750 B] 650 C] 950 O] 850 E] N.A. UNFV 88-1
300 Física-Primer Nivel FélixAucollonchi V.

ResoIución.-
Recordando que el peso por unidad de volumen de
un cuerpo es su peso específico (p), y que el peso de
=
todo cuerpo está dado así: P P V según la relación
(15.2), utilizando la condición de equilibrio del
tronco y el D.C.L. adjunto, tendremos:

~ PT· V= ( -foo kg 85
1000 m3 100 V
E
RPI'A.D

Prob. 10.- Siun submarino tiene un peso totol de 398 toneladas, el volumen sumergi90 cualdo
floto en lo superficie del mor (densidad del agua del mor = 1,026 g/cm3) es en f'T'r3:

A) 398 B) 408,3 C) 1 026 D) 257 E) 387,9 UNMSM87


Resolución.-
Reconociendo que la densidad del agua y el peso
específico expresados en g/cm3 respectivamente
son numéricamente iguales, tendremos que:

p.gua = 1,026 grlcm3 = 1,026.10' k:glrnl.


Asimismo, debemos recordar que: 1 1 = 10' kg .
Luego, procediendo como se hizo en los dos últi-
mos ejercicios, tendremos que por equilibrio:
E =P ~ Pagua. Vs = 3981

398.103 kg
~ v: - ----=--=-"---_:_
S - 1,026.103 kg 1m3 AGUA DE
MAR
~ VS'" 387., m3 RPI'A. E

Prob.ll.- Los esferas homogéneos A ya, que tienen el mismo volumen y están pegados por
medio de un pegamento, se mantienen en equilibrio inmersos en el agua. Cuando
los esferas se despegan. lo esfera A sube y floto con lo mitad de su volumen fuero del
agua, y lo esfera a se hunde hasta el fondo del recipiente. Determinar lo densidad
en g/cm3 de los esferas A ya respectivamente (densidad del agua = 1 g/cm3)
A)0,50 ; 1,50 a) 1,50; 0,50 C) 0,50; 1,0 D) 0,05 ; 1,50 El 0,05; 15,0
UNI94-1
Resolución.-
Debemos reconocer que si los volumen es (V) de las esferas son iguales, el empuje (E) que ambos experimentan
son iguales, y sus pesos estarán dados por la relación (15.4): P A = DA g V, Y P B = DB g V.
Fluidos 301

Asimismo indicaremos que la densidad del


agua está representado por Do = l g/cm3.
A continuación, por condición de equi-
librio tendremos:
Fig (1): P A + PB = E + E
~ DAgC + DBgV = 2 D"gV
~ DA+DB=2Do (1)
Fig(2): PA=EA ~ DAgV=Do.g(VIl)

Do Fig. (2)
~ DA =2········ (2)
Finalmente, de (1) Y (2) encontramos que: Fig. (1)
DA = 0,5 g/cm3 y D. = 1,5 g/cm3 RPTA.A

Probo 12.- Considerar un bloque homogéneo sumergido en un líquido, como se indica en la figura,
y T es la tensión en la cuerda. La tabla muestra los datos obtenidos pora dos líquidos
diferentes. Hallar el volumen del cuerpo en crrf3.
(tomar g = 10 m/s2)
T
D{g/cm3) T{N) ~ =::==:=:==== :=:=:=:===:
~~~~~~~~~::::
:::::::::::
1.6 2 ,"---------
::--------- ------D'
1.2 4 .:.------------------------- :.

A)200 B) 300 C) 400 D) 500 E) Faltan datos


UNI 94-1
Resolución.-
De acuerdo con el D.C.L. del bloque, deducimos que por
equilibrio se cumple en los dos casos que: P = T + E. Luego,
se puede establecer que en los dos casos se verificará que:
TI + El = T2 + E2 => El - E2 = T2 - TI "'----'I!1l.
Y expresando los empujes por la relación: E = P V, Ylos valores --------- ~.....--lLl..
de tensión en kilogramos fuerza: 1N <> 100 gr, se establece-
rá en (*) que: P1 V - P2V= 400 gr - 200 gr

gr gr ) _ 200
~ ( 1,6 -3 - 1,2 -3 V= 200 gr ~ V = -cm 3 ~ V=500cm3 RPTA.D
cm cm 0,4

Probo 13.- Si las masas son iguales y el sistema está en equilibrio, ¿Qué sucede si se retira el
vaso?
O" < e < 90"
A) m2 jola a la masa m 1
B) rti, jala a la masa m2
C) No se mueve
D) Depende del líquido
E) N.A. 8
PUCP93
302 Física-Primer Nivel FélixAucallanchi V.

Resolución.-
Al analizar el equilibrio de cada blo-
que cuando aún no se retira el vaso,
encontramos que:
Bloque (1): T = Pt sena
Bloque (2): P2 = E + T (E = empuje)
~ P2 = E + P, sena (*)
Luego, al retirar el vaso desaparece el
empuje (E) en el bloque (2); ello pro-
vocará que (1) ascienda y (2) des-
cienda, dado que- en (*) notamos que:

RPTA.A

Probo 14.- Se mantiene un cuerpo de 100 g de


masa y 0,8 g/cm3 de densidad dentro
del agua a una profundidad de 20 m, ----------
--
- T

Y
... ":".
=::::=:=: :.;....
tal como lo indica la figura. Sise libera
al cuerpo, ¿En qué tiempo llega a la \: :_~~~~~:_): 20 In
superficie del agua?
Densidad del agua: 1 g/cm3
Aceleración de la gravedad: 10 m/s2 ___
~,~~~_~
1
A) 2 s D) 8 s
B) 4 s E) 4,5 s
C) 1,5 s UNFV93
ResoIución.-
Desde que la densidad del cuerpo es menor que la del
agua, podemos asegurar que se verá obligado a salir
del reposo y subir aceleradamente (E > P). Luego,
aplicando la 211i Ley de Newton podemos calcular la
aceleración (a) del movimiento:

ma = LF - LF ~ (Dc' V) a = E-P
a favor en contra
de "a" de "a"

~ DcVa=DogV-DcgV ~ a=(~:-l)g

y reemplazando datos: a = 2,5 m/s".


Seguidamente encontraremos el tiempo que le toma al cuerpo recorrer el espacio e = 20 m con Vi = 0, para
lo cual utilizaremos la relación (4.8) del M.R.U.Y.:

l
20 = '2 - 2,5 t2 ~ 1= 4s RPTA.B

Nota aclaratoria.: En este tipo de ejercicios suele despreciarse lafricción del cuerpo con el líquido. salvo
que se dé alguna característica de su viscocidad en los enunciados de los mismos.
Fluidos 303

Prob.15.- Unbloque de madera se encuentra flotando en un recipiente con agua. Siel recipiente
es elevado con una aceleración a = g/5, ¿Qué fracción del volumen del bloque se
sumerge?
Densidad de la madera = 0.1 g/cm3, Densidad del agua = 1,0 g/cm3
A) OJO B)0.12 C) 0.17 D) 0,80 UNFV92
Resolución>
Analizando el equilibrio en la Fig. (1), tendremos que:
El =P m ~ DogVS1 = DmgV
Dm
~ VS1=D·V (l)
O

Ahora, reconociendo que la fuerza de empuje que


un líquido ejerce depende de la gravedad local:
=
E D .gV, luego de analizar la Fig. (2) debemos
indicar' que el empuje E2 depende de la gravedad
efectiva del interior: g = g + a. Así, aplicando la 211i
Ley de Newton en el'D.C.L. del bloque, tendremos:
mil=E2-Pm ~ (DmV)a=Do(g+a)VS2-Dm·gV
~ Dm' Va + DmgV = Do (g + a) VS2 Fig (1) Fig (2)
~ Dm (g + a) V = Do (g + a) VS2
Dm
VS2 =D'V .....(2)
o
Finalmente comparando los resultados (1) Y (2) concluímos que el volumen sumergido en ambos casos es el
mismo, e igual a:
RPTA.A

Probo 16.- Un globo aerostática de 1800 kg-f de peso se halla en reposo en el aire. ¿Qué masa
de lastre se debe arrojar para ascender con una aceleración constante de 2,8 m/s2
(se considera 9 = 9,8 m/s2)?
A) 2,2 kg B) 400 kg C) 492.4 kg D) 980 kg E) 220 kg UNMSM 80
Resolución>
De acuerdo con laFig (1) notamos que el empuje
neumostático (E) logra equilibrar el peso (P) del
globo. Luego: E = P. Asimismo, de la Fig. (2)
puede apreciarse que el nuevo peso del globo es:
P - P11 donde: P L = Peso del lastre; pero conside-
rando que el volumen (V) del globo no ha
cambiado, diremos que el empuje neumostático
que recibe del aire es el mismo. Luego, por la 211i
Ley de Newton tendremos:

m'a = E - (P - PL) ~ (P~PL J a = P - (P - P L)

~ P _P =.f P ~ P, ~ _P_ = 1 800 kg


L a L 1+ g/a 1+9,8/2,8 Fig (1)
L
E Fig (2)

:. P L = 400 kg ~ mL = 400 kg RPTA.B


304 Física-Primer Nivel Félix Aucallanchi V.

: ~ ISTA AUTOEVALIJACION

1.- En la figura se muestran tres barómetros con ()Pt > P2


distintos líquidos y al nivel del mar. Indicar lo
incorrecto. () En un principio se hace
el vacío en el tubo largo.

T () El peso de líquido en

76c
tJJlS2cm
-l-
el tubo largo es mayor
que en el corto.
Señalar verdadero (V) o falso (F)
A)VVVFV B)VVVVV C)VVVFF

A B D)FFVVV E)FVVVF

A) "A" es mercurio S.- En relación a una prensa hidráulica:


B)DB =2DA 1) La menor fuerza se presenta en el émbolo de
C) De = DB menor área.
11) El menor desplazamiento se dá en el émbolo de
2.- Considerando el sistema mostrado, completar la
oración: «El agua del vaso debido mayor área.
a que la presión que ejerce sobre el papel es _ no El trabajo realizado en el desplazamiento de un
__________que el ejercido por la atmósfera». émbolo es siempre igual al que se realiza en el
otro.
A) Caerá, menor Indicar lo correcto:
B) Caerá, mayor
A) I B) II C) III D) 1 y III E) 1 y 11
C) No caerá, mayor
6.- Si extraemos el aire del recipiente mostrado, la
D) No caerá, menor esfera que flotaba en equilibrio sobre el agua:
E) No caerá, igual
A) Permanece igual. ~
B) Se hunde. Aire
3.- En relación a los manómetros mostrados se afirma
que: (p = presión del gas; Po = presión atmosférica). C) Sobresale más. ol)))
D) Vibra verticalmente.
1) 2) 3) E) Girará.
7.- En relación al esquema mostrado, es cierto que
la lectura del dinamómetro:
T A) No cambia si ellí-
h
quido fuera cam-
1 biado.
B) Aumenta si cambia-
mos el líquido por
1) r, > Po Il) P2 = Po III) P3 > Po IV) r, < P3 otro más denso.
C) Disminuye si cam- ~L
Señalar lo incorrecto: biamos el líquido
A) I B) Il y III C) III y IV D) I y II E) IV por otro más denso.
D) Aumenta si el líqui-
4.- El siguiente es el esquema de un sifón, en donde


do cubre todo el
el líquido puede pasar de un recipiente A hacia otro bloque.
B. Al respecto se propone que:
E) No cambia si el
( ) r, = Po - DL·ght bloque se retira
( ) P2 = Po - DL·gh2 del líquido.
Fluidos 305

PROBLEMAS PROPUESTOS
NIVEL} OS.- Una esfera se encuentra sumergida hasta la
mitad en agua. Hallar la densidad (englcII/3) del ma-
01.- En el gráfico mostrado ¿En qué posición 1; 2 ó 3 terial de la esfera.
se experimenta mayor presión. A) 0,5
A) En l B) 0,6
C) 0,7
B) En 2
D) 0,4
C) En 3 E) 0,8
D) N.A 06.- Un tronco de pino en forma de cilindro recto
flota en agua con 1/4 de su volumen fuera de ella.
E) Igual en los tres puntos ¿Cuánto vale la densidad de dicho tronco?
02.- Una esfera de plomo A) 0,25 g/cm3
se suelta sobre la superficie
del agua. Con relación al B) O,75g/cm3
empuje sobre la esfera
podemos decir que: C) 0,8 g/cm3

D) 2 g/cm3
A) Es mayor en "A"
B) Es mayor en "B" que en "C" E) 0,62 g/cm3

C) Es mayor en "C" que en "D" 07.- Un trozo de metal de 2kg y 5 glc1l/3 de densidad
D) Es mayor en "D" que en "C" se sumerge ocmpletamente en agua atado a una cuerda
E) Es igual en B,C y D que lo sostiene. Calcular la tensión de la cuerda para
el equilibrio (g = 10m/s2).
03. - Señale verdadero (V) ó falso (F) respecto a dos
cuerpos de igual volumen. A) 12N

1) Soportan el mismo enpuje cuando están totalmente B) 16N


sumergidos en un mismo líquido. C) 18 N
11) Si lo anterior es cierto ambos tienen igual densidad. D) 14 N
11I) Tienen igual masa. E) 20N

A) VVV B) VFF C) VVF D)FVV E) FFF


08.- La densidad del hielo es aproximadamente
04.- Los líquidos A Y B 0,9 g/cm3 y la del agua del mar Ig/cm3 si "V" es el
están contenidos en un volumen total de un témpano de hielo que flota en
mismo tubo, entonces po- agua de mar. Entonces el volumen sumergido del
demos afirmar que: B témpano de hielo es:

A) La densidad de "B" es A A) 0,1 V B) 0,2 V C) 0,9 V D) 0,7 V E) O,8V


mayor que en "A"
09.- Encuentre la densidad de un cuerpo (en kg/m3)
B) La densidad de "A" es mayor que "B" que flota en agua con el 40% de su volumen fuera
de ella.
C) La densidad de "A" es el doble que" B"
D) La densidad de "B" es el doble que "A" A) 400 B) 300 C) 800

E) N.A. .D) 600 E) 900


n Física-Primer nivel Fé/ix Aucallanchí V.

NIVEL 2 14 - La esfera hueca mostrada de 20 kg y 0,02 11I3


está atada al fondo de un tanque que contiene un
líquido de densidad I 500 kg/1II3. Hallar la tensión
10.- Determine la presión hidrostática (en kPa) que del cable. (g = 10 /II/s2)
soporta el buzo a 20 In de profundidad en el agua.
A) 100 N
A) 220
B) 2 OON
B)280
h C) 1 000 N
C) 196
D) 10 N
D) 250
\
E) 320 E) 2 OOON

11- La figura muestra un reci~iente cilíndrico provisto 15- La figura muestra una prensa hidráulica provista
de un émbolo móvil de 1/2 m-de área, si la presión en de dos émbolos móviles de pesos despreciables cuyas
el fondo del recipiente es de 26 000 Pa. Calcular la áreas son: Al = I 11I2;A, = 2 m2. Hallar FI (en kN)
fuerza que ejerce el émbolo sobre la superficie libre necesaria para mantener al bloque de masa In = 4 000 kg
del líquido. en reposo.( g = 10 m/s2 ).

A) 2 000 N ¡F
B) 3 000 N

C)4000N

D) 5000 N

E) 6000 N
20}fl p=80000N/m3
Al

~O
I~:~_~
____ m

A2

12 - La esfera mostrada pesa 8 000 N ; tiene un volu-


men de 0,5 11/3 Y está totalmente sumergida en agua. A) 12 B)13 C) 14 D) 15 E) 8
Hallar la tensión del cable para el equilibrio (g = IOm/s2).
16.- La figura muestra dos esferas de volúmenes
A) 3 000 N iguales y densidades 900 y 1700 kg/m), Determinar
la densidad del líquido que establece el equilibrio de
B) 4 000 N los cuerpos.

C) 5 000 N A) 1 200 kg/m3


3
D) 8500 N B) l 300 kg/m ~l
C) 1 800 kg/m3
E) 300 N
D) 4 000 kg/m3
13.- Un cubo de madera cuyas aristas miden 40 cm
pesa 400 N Y está en equilibrio en la posición E) 2 000kg/m3
mostrada. Hallar "x"
17.- Un cuerpo pesa 100 N en el aire y 80 N en el
A) 0,211/ agua. ¿Cuál es el volumen (en 11I3) de dicho cuerpo?
(g = 10 m/s2)
B) 0,311/
A) 0,003 B) 0,002 C) 0,004
C) 0,25 m
D) 0,0005 E) O, 007
D) 0,2811/
18.- Del ejercicio anterior ¿Cuál es la densidad
(en g/cm3) de dicho cuerpo? (g = 10 m/s2)
E) 0,5 11/

A)3 B) 2 C)4 D) 5 E) 7
Fluidos ?IJ7

NIVEL 3 A) Asciende
1,3)Desciende
19.- ¿eón que aceleración desciende un cuerpo de
densidad De soltado desde el borde de un recipiente c'h·,¡o cambia
que contiene un fluido de densidad DL = Dc/4
D) Falta conocer el volumen de la piedra
A) 3/2 g
E) Falta conocer la densidad de la piedra.
B) 3/4 g
11111 23.- La figura muestra dos bloques (1) y (2) de pe-
e) 3/5 g
O~a sos 10 y 40 N respectivamente y volúmenes iguales
en equilibrio. Hallar el empuje sobre el bloque (2).
D) 1/2 g (No hay rozamiento).
E) 5/2 g A) 30 N
20.- Se mantiene un corcho de 0,8 g/cm3
de densidad,
dentro del agua a una velocidad de 20m. Tal como se
B)20 N
indica. Si se libera el corcho. ¿En qué tiempo llegará
e) 50 N
a la superficie del agua? (g = 10 mls2)
D)40 N

r
A) 4 s
E) ION
B) 5 s

e) 7 s 20m .24.- Un ascensor sube con una aceleración de 5 m/s"


en su interior lleva un líquido de 2g/cm2 de densidad.
D)6s Hallar la diferencia de presiones entre los puntos A

E) 2 s
,----------1 y B (g = 10 mls2); h = 80 cm.

21.- Un tanque cilíndrico, cuya base tiene 2 m2 de


A) 25 kPa t a
v-.
área contiene agua hasta una altura de 2 m. En cierto
B) 22 kPa
v-.
instante el vigilante nota que el nivel del agua está 1'--' r-
disminuyendo y marca la correspondiente altura, e) 20 kPa Ir- A. Ir-
luego repite las marcas cada media hora como se
muestra. La cantidad de litros de agua que se han
perdido hasta el instante en que el vigilante hizo la
D) 28 kPa
r--

Ir-
B.I h
r-

Ir-
quinta marca o!s. 1'- 1'-
E) 24 kPa

h (m) 25.- Se tiene un cilindro formado por una madera y


un metal como se indica; hallar la altura «x» que debe
-------¡--------,---- --- -r- --- ----,
I , , t tener el metal para que el conjunto flote en equilibrio.
I I I 1
3
1,5 -- ---~--------~--------~--------¡ DMADERA = 0,7 g/cm3; DMETAL = 9 g/cm
_______
I
J___
, I
_ __ .1
, I
i
,I
,
, I I L
,1 :
"
'
5 cm
0,5 -------¡---- ----'1-------:-:--::-,;-'""-----1, I

, , ,
'-----'-' ----'-' -~'~-~ t (h)
O 2 3 4
A) 0,5 b) 3 e) 0,5.103 D) 1,5

22.- En un vaso con H20 flota un pedazo de hielo en


cuyo interior se encuentra una piedra. ¿Qué pasa
con el nivel del H20 cuando el hielo se derrite
A) 5 cm B) 6 cm C) 7 cm D) 2 cm E) 8 cn('
I completamente?

I
I
Física-Primer nivel Fé/ix Aucallanchi V.

26.- Determinar el volumen sumergido del bloque 31.- Se libera un cuerpo de densidad D = 2 glem3
(D = 0,75 glem3) sumergido en un líquido «A» si el desde el fondo de un recipiente de' 24 m de
sistema está en equilibrio (Volumen del bloque 32cm3). profundidad, y que está lleno de un líquido de densi-
Dacei!e = 0,8 glem3 dad DL = 2,6 g/cm3. ¿Cuánto tiempo empleará en
llegar a la superficie libre del líquido?
bloque
A) 10 cm3
A) 0,2 s B) 0,5 s C) I s D) 3 s E) 4 s
5 cm
B) 12 cm3
32.- Un cubo de 2 m de arista cuyo peso es 30 N
5cm flota tal como se muestra en la figura. La esfera tiene
C) 14 cm3
la mitad de su volumen en el a~ua, y su peso es
90 kN. ¿Cuál es su volumen en m ?
D) 15 cm3
A
A) 12
E) 20 cm'
B) 10
27.- En la fi~ura ¿Qué empuje experimenta el bloque?
a = 2,2 mis ; V s = 200 cm3 C) 8

A) 24 N D)6

B) 2,4 N E)4

C) 3,2 N 33.- La esfera mostrada se encuentra sumergida en


un 60% en el líquido "2". Si las densidades de los
D) 8,2 N líquidos sonD,= 5glem3 y D,= 15glem3, ¿Cuál es
lizO la densidad de a esfera? -
E) 2,5 N
A) IOgle",3
28.- La figura muestra dos bloques (1) Y (2) de igual
volumen y pesos 10 Y 40 N ; hallar el empuje que B) 13glem3
experimenta cada bloque para el equilibrio. (No hay
rozamiento). C) 12glem3

A) 25 N
D) I1 glem3

B) 30 N
E) 8glem3 ~-----------------
C) 40N 34.- Dos esferas de igual volumen V = 3 m3 y pesos
p¡ = 12 kN Y P = 8 kN se encuen-tran en equilibrio
D) 60 N en un líquido desconocido. Se pide hallar la fuerza
de tensión (en kN) en la cuerda que las une.
E) ION
A) I
29.- Calcule la densidad que tiene un cuerpo que
flota en un líquido cuya densidad es 8 000 kgll1l3, B) 3
sabiendo que lo hace con e125% de su volumen fuera
del líquido (dar la respuesta en glem3). C) 2

A) 6 B) 8 C) 10 D) 12 E) 14
D)4
30.- Un cuerpo pesa tres veces menos en el agua que
fuera de él. ¿Qué aceleración experimentará al ser E) 5
totalmente sumergido en el agua?

A) g/2 B) g/4 C) g/3 D) 2g/3 E) 3g/2


Termometria
y Di{atación
ORlETfl.VS
1.- Analizar el concepto de Temperatura y
el importante papel que desempeña en
el desarrollo de los fenómenos térmicos.
2.- Conocer el principio de funcionamiento
de los termómetros y las principales
escalas termométricas.
3.- Interpretar y aplicar las principales leyes
que explican la dilatación de sólidos y
líquidos.

uando nos hablan de verano o invierno, inmediatamente lo


asociamos a nuestro conocimiento de lo caliente y de lo
frío. Estas palabras se ven muchas veces acompañadas de
calor y temperatura, dos cosas distintas, pero que se encuentran muy
vinculadas entre sí. Muchos fenómenos térmicos se deben al calor, y
todos ellos serán explicados a partir de este capítulo. Sin embargo,
iniciaremos nuestro estudio con el análisis de la temperatura.
11II S~NSACIONES TERMICAS
Mediante nuestro sentido del ,.~~~' WILLlAM THOMSON
tacto y otras circunstancias fisiológicas
experimentamos ciertas sensaciones (1824-1901)
por las que afirmamos que un cuerpo Físicode un poder creador
está frío o caliente. Larnentablemen- enorme. Nace en Belfast
te, por su carácter cualitativo y sub- (Inglaterra) en 26-6-1824.
jetivo, no podemos distinguir si una Desde 1846 fué profesor en
sensación es doble o triple de otra Glasgow. Tiempo después
sería nombrado Lord Kelv/n
sensación similar que hayamos por la reIna VIctoria por sus
experimentado antes. La experiencia brillantes trabajos (más de
del filósofo inglés John Locke (1632- 600) en diferentes campos
1704) 1 F 16 1 de la ciencia, destacando
, que se muestra en a 19. ., entre ellos: el tendido del
plantea la pregunta: ¿El agua que sale primer cable submarino
del caño esta fría o caliente? Esto nos para telecomunicación en
demuestra que nuestras experiencias el Océano Atlántico: con
sensoriales no son buena base para Joule estableció lo que en la
. . actualidad se conoce como
1a Ffsica: Sin embargo, debemos
reconocer que el mismo estímulo
térmico que produce en nosotros las
I el efecto Joule-Thomsonen
Termodinám/ca:elestablec/-
miento del Cero Absoluto, y
sensaciones de frío o caliente produce por ende la creación de la
di f . AGUA DEL escala absoluta de tempe-
en o t ros cuerpos mo I icaciones cAÑo rotura que lleva su nombre.
observables, como por ejemplo: La ~--'-'~~------=-~~
dilatación. Fig 16.1
310 Física-Primer Nivel Félix Auca/lanchi V.

CALOR
En la Flg.16.2 el calor es la
11II CONCEPTO DE TEMPERATURA
energía que se transmffe del Cuando las moléculas de un
fósforo hacIa el hIelo. En la cuerpo se agitan en promedio con
FifJ.1Ó.3.el calor es la ener-
gla que fluye del bloque A gran rapidez, decimos que su
al bloque B cuando el/os temperatura es alta, y si la agitación
mantienen una diferencia es lenta diremos que su temperatu-
de temperaturas. ra es baja. Así pues, la temperatura
es una magnitud tensorial que nos
indica el grado de agitación molecular
que en promedio tiene un cuerpo.
Obviamente no tiene sentido hablar
de la temperatura del vacío.
Fig16.2
DEBES SABER QUE: 11II EQUILffiRIO TERMICO
Llamaremos equilibrIo Si dos cuerpos a diferentes tem-
térmIco a aquel estado peraturas se ponen en contacto, al
particular en el que las
moléculas de dos o mós cabo de cierto tiempo ellos adquirirán
cuerpos en contacto vibran una temperatura de equilibrio, cuyo
en promedio con la misma valor estará comprendido entre la
velocidad. En este estado alta y la baja. Este comportamiento

r=~~:;~\t.~
~~~:~:
«'~'i~H,AI,,~~
las temperaturas se igualan.

tTB~T&~TA 1 - Fig16.3

OJO!
11ITERMOMETROSYESCAIA51ERMOMETRICAS
El termómetro es un cuerpo de pequeña masa, que al ponerse en
Llamamos fusión al contacto con otro cuerpo mayor, alcanza el equilibrio térmico, de modo
proceso mediante el cual que la temperatura del mayor permanece sensiblemente fija. Este
un sólido se convierte en principio de funcionamiento ha sido utilizado para elaborar diferentes
líquido, y denominamos
ebullición a aquel proceso escalas de medida, las cuales a su vez han considerado dos fenómenos
en donde un líquido se naturales que se producen siempre del mismo modo (al nivel del mar y
convIerte en vapor. a 45° de latitud), siendo éstos: El punto de fusión y el punto de ebullición
del agua. Estas escalas son:
A) Escala Celsius.- Aquí el agua se funde a O°C y hierve a 100°C.
Entre dichos puntos se han hecho 100 divisiones: 1 división = 1°C.
B) Escala Kelvin.- Es una escala absoluta cuyas divisiones son
iguales a las de la escala Celsius. Aquí el cero absoluto está 273
divisiones por debajo del punto de fusión del agua: 1división = IK.
CERO ABSOLUTO C) Escala Fahrenheit.- En esta escala existen 180 divisiones entre
el punto de fusión y el punto de ebullición del agua, los cuales
Esel estado hipotético en
el que las moléculas de un están a 32°F y 212°F respectivamente.
cuerpo dejan de vibrar. En *) Sean e, K y F las lecturas de una misma temperatura en las tres
la próctica la temperatura
más l:x:j:Jq..e se ha rxnsegul- escalas, las mismas que en la Fig. 16.4 aparecen en una misma
do es de 10 -e keMn. horizontal. Luego, por la proporcionalidad de los segmentos
(Teorema de Thales) se establece que:
Termometría y Dilatación 311

e K-273 F-32 TERMOMETRO CUNiCO

:=¿l::.:::F L ----
100 100 180 Presentan un estrangule·
mIento muy cerco del bulbo

I loor
K-273 F-32
----
metálico que contIene el
~ mercurio. de manera que
S 9 continúo indicando lo tem-
0-........ 27::r'" 32..·· P.F peratura de lo persono o
pesar de estor separado de
o él.

[F == ~c + 32j (16.2) 43°C


-.P.J 9 -460 Cero
........ AlMolldo
42°
Observaciones: 41°
40°
*) 1div (0C) = 1div (OK) 39°
P.E. = Pamlo de Ebullición del .4gua
P.F.= PJDJIO de Fusión del .Agua *) 1div (0C) = 1,8div (OF) 38°
37°
Fig 16.4 36°

I)IU TÁCI()~ [)~ §()LI[)()§ ." LIl2UI[)()§

Como sabemos, los cuerpos están constituídos por moléculas,


los que en el caso de sólidos y líquidos guardan entre sí distancias más
o menos fijas. Si calentamos o enfriamos un cuerpo, observaremos
que ellos se dilatan o se contraen respectivamente; ésto se explica
porque a nivel molecular el cuerpo a alta temperatura aumenta su
agitación y por ende las distancias intermoleculares, a baja temperatura
ocurre lo contrario. Los ejemplos de la Fig. 16.5 confirman un hecho:

(a) Experimento de
Gravesaode
INTERES/WTE

(1)~IIÍiIIII".

Fig 16.5
312 Física-Primer Nivel Félix Auca/lanchi V.

COEFICIENTES a DE
,
SOLlDOS DILATACION LINEAL
Si calentamos una varilla o
+--- L¡ ----9-
10.5 re»¡ alambre como el de la Fig. 16.6,
Sustancia Ti (/71111117717717711111117777177~
comprobaremos que sufre una di-
latación (M), cuyo valor depen-
Aluminio 2.3 derá de la longitud inicial (L¡) y del +--M:-t
cambio de temperatura (f..1) por el TWIlIliIli177227Z11/77171l7llIli/7Il/ÚD
Bronce 1.8
coeficiente de dilatación lineal (a). ~ Le ----4

I AL= L¡. a.!J.T I


Zinc 2.9 (16.3) t:.L=Lr-L¡
. . llT=Tr-T¡
Cobre 1.7
Unidod(a) = °C",oF", K"
Acero 1.2
~
I'.Le= L¡ (1 + a!J.I) 1 (16.4) Fig 16.6
Latón 1.9
• DILATACION SUPERFICIAL
Oro 1.4
Cuando calentamos una lá-
Plata 0.9 mina o placa como la mostrada en
la Fig. 16.7, comprobamos que su
Plomo 2.9 superficie experimenta una dila-
tación (M), cuyo valor viene dado
Vidrio 0.9
por: M =ArA;
Pyrex 0.3 1M =
A¡P!J.T I
(16.5) Y ~.2a
~ = Coeficiente de dilatacián
~ I Ae= A¡ (l+P!J.1) I (16.6) su¡ierficial
Fig 16.7

• DILATACION VOLUMETRICA
Es indudable que al calentar ,.--------------,
o enfriar un cuerpo, todas sus c· b~cr
dimensiones: largo, ancho y altura, b~. '. f .;

COEFICIENTES Y DE
LlQUIDOS
~~i~~~e::aennC~~~i~~~~~ee~~o J~ a¡ V¡ .~.::a
r
Vf:"

dilataci?n. realmente se produce :.: .:.:.'.>


.10·4¡C·l una variación en el volumen (f..V),
Sustancia
cuyo valor estará dado por: T¡ T.
t

Aceite 6 I!J.V = VI "(!J.T I (16.7) y::~~ldilatociónvolumétrica

Alcohol 7.5 ~ ! Ve = Vi (1+ "(!J.I) I (16.8) Fig 16.8


Agua (10·2(J'C) 1.5
• APLICACIONES DE LA DILATACIO~
Gasolina 10
A) Listones bimetálicos.- Una buena cantidad de dispositivos que
Glicerina 5 funcionan automáticamente lo hacen utilizando un listón extendido
o enrollado, compuesto por dos metales de diferente coeficiente
Kerosene 10 a, de manera que al sufrir un cambio en su temperatura se dobla,
se enrolla más o se desenrolla. Esto se explica por la diferente
Mercurio 1.8
dilatación que cada componente experimenta. En la Fig. 16.9a, el
Petróleo 10 listón a la temperatura TI' presenta una orientación vertical, dado
que cada componente de listón posee la misma longitud.
Termometría y Dilatación 313

B)Dilatación de Agujeros.- En el experimento de Gravesande (Fig. DEBES SABER QUE:


16.5a), la esfera podrá pasar por el aro si ésta también se ha
SI llevamos o un gráfico
calentado. Esto significa que los agujeros en los sólidos se dilatan t-vs-r lo ecuación (16.4]. se
como si estuvieran llenos del material que los rodea. Lo mismo le obtiene uno línea recto
sucede al interior de las vasijas cuando las calentamos (Fig. 16.9c). cuyo pendiente (tg9] viene
dado por:
C) En las construcciones> Cuando se construye una vía de ferrocarril, se
deja un espacio entre riel y riel por los cambios de temperatura ambiental.
Por esta misma razón se adicionan rodillos en los extremos de los puentes.
!tge= L¡.a

L
(a)
T,a---.¡t ----------------
i
@
(b) ~f

T>1j .•••• ,' D~

(e) í
..--- -- J.
Fria
- --- ~ T
¡,
)

~U
Calientej 11 , , Ti Tf
al> <lz

FigJ6.9
.' IADENSIDAD DEPENDE DE IA TEMPERATURA
Es evidente que si calentamos un cuerpo su volumen aumenta, pero
como su masa es prácticamente la misma, conc1uímos que su densidad
disminuye, dado que ésta es inversamente proporcional con el volumen.
Esto explicaría que los vientos se producen por causa de que el aire
caliente es menos denso, y por ello es empujado hacia arriba, y el aire
frío que es de mayor densidad, baja a ocupar su lugar. En general, la
densidad Df de un cuerpo a la temperatura T¡ viene dada por: INTERESANTE
Cuando un lago se
congelo. bajo la capa de
(16.9) hielo se encuentra el agua
líquIdo a O"C. y más abajo
el agua está más caliente
_ COMPORTAMIENTO ANOMALO DEL AGUA (4°C]. Esto se explica por el
comportamIento anómalo
Se sabe que el agua es una de las del agua.
pocas sustancias que al calentarse desde
O°C hasta 4°C en vez de dilatarse se
contrae, como lo indica el gráfico
volumen-vs- Temperatura de 1g de agua
en la Fig. 16.10. Esto explica a su vez 1,002
que el agua alcanza su máxima densidad
de 1 glcm3 a 4°C, que es cuando su
/v
t"---- ../
volumen es mínimo. Por encima de esta 1,000
temperatura el volumen aumenta, y el
comportamiento del agua se normaliza.
¿Por qué se rompen las botellas de ga- O
2 6 10
seosa cuando las dejamos por un L-~~ ~-=:""--:--::-=-=,
buen tiempo dentro de la nevera? Fig 16.10
314 Física-Primer Nivel FélixAucallanchi V.

1ROBLEMAS RESUELTOS

Probo 1.- Untermómetro centesimal marca 28°. Enun termómetro Fahrenheit dicha temperatura
es:
Al 50AoF UNMSM87
Resolución.-
Utilizando directamente la relación (16.2) para el paso de una escala a otra, tendremos:

F = ~ e + 32; donde por dato: e = 28° ::::) F =~ (28) + 32 ::::) F= 82,4° RPTA.C

Probo 2.- Si dos termómetros graduados en las escalas Fahrenheit y Celsius respectivamente
señalan el mismo valor, un termómetro graduado en la escala Kelvinmarcará:
B) - 200 C] 32 ° E) 233 ° UNMSM83
Resolución>
Seax el valor de la temperatura en las escalas Centígrada y Fahrenheit establecida por condición del problema.
Luego, utilizando la relación (16.2) tendremos:

x-.2. x+ 32
-5 x - .2. x = 32 ::::) x = - 40
5
Esto significa que la temperatura a la cual coinciden las lecturas en las escalas Celsius y Fahrenheit es - 40°C
y-40° F. Seguidamente encontraremos el valor de esta temperatura en la escala Kelvin, utilizando para ello la
relación (16.1):

K = e + 273 ; donde e = - 40° ::::) K = - 40 + 273 ::::) K = 233° RPTA.E

Probo 3.- Sidefinimos una nueva escala termométrica 0N, en la cual el punto de ebullición del
agua es 500° N Y el punto de fusión del hielo es 100° N, la relación entre esta nueva
escala tN Y la celsius te es:
A) tN = (3.te + 100) W O) tN = (5 te + 100) W
B) tN = (400 te + 100) W E) tN = (te + 400) W
C] tN = (4 te + 100) °N UNMSM94
Resolución.-

Haciendo un esquema similar al elaborado en la


Fig. 16.4, indicándose en él los datos del proble-
ma, podemos utilizar el Teorema de Thales para 500 --------------------- 100
los segmentos señalados allí:

(tN -100) te
400 = 100 ::::) tN-loo=4te
t tc
400 N 100

tN = (4 te + 100) ; ó:

RPTA.C 100 --------------------- O


Termometría y Dilatación 315

Probo 4.- Un termómetro de mercurio tiene una escala que marca OOX cuando la temperatura
es de - 20°C, Y marca 240 X para 100° C. ¿Cuántos grados X corresponden a la
0

temperatura humana de 3rc?


A)3rX UNI93 -1

Resolución .-
°X °C
Procediendo del mismo modo como se hizo
en el problema anterior, elaboramos un gráfico
para las dos escalas señaladas de modo que 240 """""""""""""""""""" 00 ""1'
en él estén indicados todos los datos del
ejercicio, siendo tx la temperatura solicitada
pues empleando
tendremos que:
el Teorema de Thales
240 ti --------------------- 37 120

~=XL tx O 57
240 120
O -------------------- -- -20--
RPTA.C

Probo 5.- Untermómetro está graduado en esca- -


la de °C y otro en grados °X tal que
guardan la relación de la figura. (A qué
temperatura ambos termómetros
marcarán la misma temperatura?
A) -15° O) -18°

8) 15° E) 20°
C) 18°

Resolución.-
Según el esquema dado, estableceremos la relación entre la escala Celsius y la nueva escala "X,

o X -30 °c-o o X -30 °c o X =30


C + 30 .... (*)
330-30 100-0 300 100

Sea TIa temperatura común (en ambas escalas) que buscamos. Luego, en (*):

T= 3 T+ 20 2 T= - 30

T= _150 RPTA.A

Probo 6.- Calcular las longitudes en cm de una varilla de latón y de una varilla de hierro para
que tengan una diferencia de longitud constante de 5 cm a todas las temperaturas.
Los coeficientes de dilatacJón lineal del latón y del hIerro son 0,000 018 por °C y
0,000012 por °C respectivamente.
A)20:15 8)10:15 C)5:10 0)25:20 E)20:5
UNI 83 - 2
316 Física-Primer Nivel FélixAucallanchl V.

Resolución-
Para que las varillas tengan siempre la misma diferen-
cia de longitudes a cualquier temperatura, deberán 4.(latón)
experimentar ambos igual cambio en sus longitudes,
es decir, si ambos aumentan o disminuyen su longitud
en la misma medida, la diferencia de sus longitudes ···
será siempre la misma. Luego, empleando la relación
(16.3): +
·· -.-scm-r
~ = ~ => L¡¡. uH .1l.T = ~.a,..IlT
..
=> L¡¡.I,2.1O-5=~.I,8.1O-5 6
L¡¡(hierro )
3
=> L¡¡ = "2 ~ ..... (L¡¡ »i; )
y por condición del problema: L¡¡ - ~ = 5 cm

3
=> "2 ~ - ~ = 5 cm => LL = 10 cm y Lu = 15 cm RPTA.B

Probo 7.- Se doblo un alambre de 2 m de lon-


gitud en formo circular; notóndose
que quedan 2 cm poro completar
lo circunferencia, siendo lo tem-
peratura de 20°C. Si se caliento el
alambre así doblado hasta 80 °C,
¿Cuónto faltaró para completar lo
circunferencia?
(uo1ombre = 5.10-5 0C-1]

A] 2,006 cm B] 2,01 cm
C] 2,014 cm D] 2,02 cm
E]2,024 cm
Resolución.-
Calculemos el ángulo que subtiende la longitud inicial de arco faltante :

2 l 0
9= 200 = 100 rad '" 0,57

Dado que este ángulo es bastante pequeño,podemos considerar que la dilatación de dichos arcos va de
acuerdo con una dilatación lineal. Así entonces, aplicando la relación (16.4):

Ir = li (1 + u 111) => Ir = 2 [1 + 5 . 10-5 (80 - 20)]

=> Ir = 2,006 cm RPTA. A

Probo 8.- ¿Cuónta agua se derramo de uno vasija de pyrex lleno con un litro hasta el borde o
20°C, si se caliento hasta 90 0C? Loscoeficientes de dilatación cúbico del pyrex y del
agua son 9.10-6 0C-1 y 2, 1.10-4 °C-1 respectivamente.

A] 4,83 cm3 B] 48,3 cm3 C] 8A cm3 D] 14,07 cm3 E] N.A.


Termometría y Dilatación 317

Resolución.-
Al calcular el volumen final de cada sustancia tenemos:
Pyrex: Vf(P) = Vi(p)(1 + rp~T) = 1 000 cm3[ 1 + 9.10-6(90 - 20)] = 1 000,63 cm3
Agua: Vf(H20) = Vi(H20 P+ rH20~T) = 1 000 cm3[l + 2,l.l0-4(90 - 20)] = 1 014,7 cm3

Así, el volumen de agua que se habrá derramado será:

~V = 1014,7 - 1 000,63 RPTA.D

Prob. 9.- Unvolumen de 960 cm3 de Hg (coeficiente de dilatación cúbica = 0,000 18 0C-1)está
dentro de un recipiente de 1 000 cm3 de capacidad (exacta), fabricado de un
coeficiente de dilatación cúbica = 0,000 012 8 0C-1.Se eleva su temperatura desdé
OOCa 2500C. Entonces:
A) El Hg ocupa exactamente la capacidad del recipiente.
B) Se derrama 3,2 cm3 de Hg.
C} Se derrama más de 3,2 cm3 de Hg.
O) Esnecesario añadir 3,2 cm3 de Hg para llenar totalmente el recipiente.
E)Queda aún un volumen vacío en el recipiente.
UNI84-l
ResoluciÓn.-
En el gráfico se han indicado los datos del problema,
y reconociendo que la variación de temperatura del
sistema es: ~T =
250°C, utilizaremos la relación
(16.8) para determinar los volúmenes finales, tanto R
del mercurio (Hg) como del recipiente (R), y así
compararlos. Luego:

Hg: Vf = Vi [1+ rHg.~l1 = 960 cm3 [1 + (1,8.10-4) (250)]

~ Vr = 1 003,2 cm3

R: Vf = Vi [1 + rR • ~11= 1 000 cm3 [1 + (1,28.10-5) (250)] 960cm


3

-- Hg
~ Vr = 1 003,2 cm3 -------------
s-» --- •••••.••

y comparando los resultados, concluímos que el mercurio


ocupa finalmente toda la capacidad del recipiente.

RPTA.A

Probo 10.- Unmatraz de vidrio de 250 cm3 de capacidad se llena completamente con mercurio
a 200C. ¿Cuánto mercurio se derramará al calentar el conjunto hasta 1000C? Los
coeficientes de dilatación cúbica son:
rvldrlo = 1.2 x 1O-51C}-1 rmera.rlo = 18 x 1o-5lCj1

A) 3,36 cm3 B) 3,60 cm3 C} 2,88 cm3 O) 2,69 cm3 E)0,192 cm3 UNI92
318 Física-Primer Nivel Félix Aucaflanchi V.

~}_ -->:
Resolución.-
Para determinar el volumen de mercurio (Hg) :'-
., :
Vdcmmado
derramado, bastará con determinar las varia- ,
! ~ ,
I

ciones de volumen que experime. tan el mer- '/ -"',1

curio y el-vidrio (V) para el cambio de tem- V¡=250cm


3

peratura óT = 100 - 20 = 80° C. Luego,


empleando la relación (16_7) tendremos para
cada uno:
Hg: ÓVHg=V¡yHg.ÓT =250 cm3(18.1O-5)(80)

MERCURIO ~RlO
:::} ÓVHg = 3,6 cm3
V ..-/
v.. óVv.=V¡. Yv' óT= 250cm3 (1,2.10-5)(80)

:::} ó Vv' = 0,24 cm3


Finalmente el volumen derramado será:

Vderr = 3,6 - O,24:::} Vderr = 3,36 cm3 RPTA.A

Probo 11.- El mercurio tiene una densidad de 13,6 g/cm3 a 0° C. ¿Cuál será la densidad de éste
a 3000C, si consideramos que el coeficiente de dilatación cúbica del mercurio es
constante y que su valor medio es 1,85. 1O-4°C-1.

A) 12.7 g/cm3 B) 12,9 g/cm3 C) 13,1 g/cm3 O) 13,3 g/cm3 E) 13,5 g/cm3

UNFV92
Resolución.-
De acuerdo con los datos del problema podemos reconocer que éstos son: D. = 13,6 g/cm3, y g = 1,85.10-4
( l/0C), Y óT == 300°C. Seguidamente, haremos uso de la relación (16.9) para determinar la ~ensidad final
del mercurio:

Reemplazando datos tendremos:

13,6 g/cm3
D, = 12, 885 g/cm3
Df = 1+(1,85.10-4 )(300)

D, •• 12,9 gJcm3 . RPTA.B


Termometría y Dilatació~ 319

1.- Elige las palabras que completen mejor la si- A) -370°C


guiente oración: «La temperatura es _
proporcional con la movilidad molecular, e B) _4°F
independiente de la de los cuerpos».
C) -2K
KJ Directamente , masa
B) Inversamente , masa 6.- Acerca de los siguientes hechos, señala lo
incorrecto:
C) Directamente , forma
1) Los líquidos se dilatan más que los sólidos.
D) Inversamente , densidad
11) Si con una flama calentamos el bulbo de un
E) Inversamente , presión termómetro, el nivel deHg primero baja y luego
2.- Para medir la temperatura de una hormiga: sube.
IlI) Si calentamos agua desde O°C hasta 4°C, el
1) Reunir 100 de ellos y echarlas en un vaso.
nivel en el recipiente sube.
11) Introducir un termómetro en el vaso.
A) I B)II C) III D) I Y 11 E) II Y III
I1I) Dividir la temperatura encontrada entre 100.
IV) Multiplicar la temperatura encontrada por 100. 7.- Cuando enfriamos el listón bimetálico, donde al
> a2, la esferita inicialmente en reposo:
Señalar los pasos que consideras necesarios:
A) Cae
A) 1, II Y III B) I Y II C) 1, 11 Y IV
B) Oscila
D) Ninguno E) Todos
C) Va hacia la pared
3.- Dos cuerpos A y- B con temperatura diferentes
TA >TB se ponen en contacto. Luego: D) Salta
( ) El frío se calienta. E) Queda quieto
( ) La temperatura de equilibrio es más próxima a
la del cuerpo de mayor masa. 8.- Si calentamos el aro mostrado:
( ) La temperatura de equilibrio puede coincidir ( ) R: Aumenta
con TA.
)d: disminuye -1-
Indicar verdadero (V) y falso (F) según corresponda:
)9: no varía d
A) VFV B) VFF C) VVV D) FVV E) VVF
Señalar verdadero
- -+
4.- Un médico midió la temperatura de una persona (V) y falso (F)
y encontró el valor 86. Luego:
1) La persona está sana. A) VVF B) VFV C) FVV D) FFV E) VVV
11) La escala utilizada es la Fahrenheit.
I1I) La persona está muerta. 9.- Se tiene un trozo de hielo flotando en agua, y
todo en equilibrio. Cuando el hielo se derrite:
IV) La escala utilizada es la Celsius.
A) h aumenta
Señalar lo correcto.
B) h disminuye
A) I B) 1, III C) 11, IV D) I1, III E) I1I, IV
C) h no cambia
5.- En un laboratorio de investigaciones un científico
midió la temperatura a la cual cierto gas se convierte D) faltan datos
en líquido (liquefacción). ¿Cuál de los siguientes E) N.A.
valores pudo obtener el científico?
320 Física-Primer Nivel FélixAucallanchl V.

PROBLEMAS PROPUESTOS

NIVEL 1 A) 1 014 cm3 B) I 018 cm3 C) I 020 cm3

01.- Un cubo de latón de arista 10cm se dilata 6cm3 D) I 042 cm3 E) 1050 cm3
¿Cuál es el aumento de temperatura?
07.- Una placa de metal tiene las dimensiones de
(aLATÓN = 20.10-6 °C-I)
10 x 10m cuando su temperatura es de 10°C. Se
A) 200°C observa que cada lado se incrementa en 20 mm cuan-
do se calienta hasta 110 "C. ¿Cuál es su coeficiente
D) 500°C de dilatación superficial?

02.- Se tiene un anillo de bronce (a = 19.1O-6°C-1) de A) 3.1O-6°C-1 B) 4.10-6 °C-I C) 5.10-6 °C-I
20 cm de diámetro interior. Si debe tener un diámetro
D) 2. 10-6 0C-I E) 4.10-5 °C-I
de 20,076 cm ¿Cuál debe ser el incremento de tem-
peratura al que se le debe someter?
08.- Un reloj de péndulo marca las horas exactamen-
A) 50°C B) 100°C te a 20°C si su estructura es metálica. i.Qué sucederá
con las horas marcadas por él cuando la temperatura
D) 200°C E) 300°C del ambiente aumenta en 30°C?

03.- Un rnatráz de vidrio (av = 9,2.10-5 0C-1) con un A) Se atrasa B) Se adelanta C) No varían
volumen de 1 litro se llena totalmente de mercurio
(aHg = 18,2.10-5 0C-1) a 20°C. ¿Cuál es el volumen de
D) F.D. E) N.A.
mercurio que se derramará cuando la temperatura
sube hasta 50°C.? 09.- Una varilla de metal de coeficiente a = 1O-50C-1
experimenta una elevación de temperatura de 50°C
A) 15,55 cm? B) 7,81 cm3 C) 2,7 cm3 ¿En qué porcentaje incrementó su sección recta?

D) 15,9 cm3 E) 14,62 cm3 A) 0,1 % B) 0,3 % C) 0,4 %

D) 0,5 % E) 0,6 %
04.- Se tiene un alambre de' cobre de 100 m de longi-
tud a O "C ¿Qué longitud poseerá a 100°C?
10.- Hallar el aumento de volumen que experimenta
(acu= 16.1O-60C-1) 100 cm3 de mercurio cuando su temperatura se eleva
de 10 "C a 35°C. (YHg = 18.10-5 °C-))
A) 100,18 m B) 100,16 m C) 100,22 III

A) 0,5 cm3 B) 0,45 cm3 C) 0,25 cm3


D) 100,25 m E) 100,32 m
D) 0,35 cm' E) 0,2 cm'
05.- Una pieza de metal de coeficiente [3 = 2.10-4 °C-I
a la cual se le ha sustraído un círculo de Icm de radio 11.- A partir del siguiente gráfico, determinar el coe-
se pretende hacer pasar un esfera de 1,02 cm de ra- ficiente de dilatación del material empleado en °C-I.
dio. ¿En cuánto debe incrementarse la temperatura
de la lámina tal que la esfera logre pasar por dicho A) 0,2.10-3 0C-I
orificio? l(cm)
B) 7,5.10-5 0C-I
A) 100 -c B)250°C C) 50°C
30 -- ~J~ _
C) 0,1.10-3 0c-I
D) 200°C E) 300°C
D) 0,3.10-1 0C-I
06.- Un líquido presenta un volumen de 1 000 cm'
cuando su temperatura es O"C ¿Qué volumen posee- E) 0,5.10-2 0C-I T(k'C)
rá cuando su temperatura sea de 200°C?
(YUQUIDO = 7.10-5 0C-1)
Termometría y Dilatación 321

NIVEL 2 16.20.- Un recipiente de vidrio de 2,2 litros contie-


ne 2 litros de un líquido cuyo YL = 13,8.10-4 °C-I,
12.- Un cuerpo se encuentra a 20° e, y experimenta todo a O "C. ¿Hasta qué temperatura deberá calen-
un aumento de 50 K. Si luego la temperatura dismi- tarse el sistema sin que el líquido se derrame?
nuye en 36° F, ¿Cuál es la temperatura final del cuer- Y = 8.10-4 °C-I
L
po en "Ct
A) 190°C B) 160°C C) 150°C
A) 500e B) 100e C) 45°e D) 300e E) 55°e
D) 180°C E) 200°C
13.- La temperatura de un cuerpo disminuye en 54° F,
ya continuación aumenta en 60°C. Si la temperatura 20.- Un péndulo que bate segundos es metálico, y
final del cuerpo fué 300 K, ¿Cuál fué su temperatura funciona correctamente a la temperatura ambiente.
inicial en "Ct Si al calentar el péndulo el período se incrementa en
0,1 s, ¿En qué porcentaje se dilató el péndulo?
A) _9°e B) -1 °e C) -3°e D) _5°e E) -r c
A) 0,39% B) 0,60% C) 0,46%
14.- Se tiene un termómetro en °e mal calibrado, en
donde las temperaturas de fusión es 5°C. Cuando D) 0,53% E) 12,5%
este termómetro marque 25°e, ¿Cuál es la tempera-
tura verdadera en ° F? 21.- En el interior de un cubo de metal existe una
cavidad esférica. Al calentarse el cubo se observa
A) 35°F B) 68°F C) 12°F D) 20°F E) 18°F que el diámetro de la esfera se dilató 1 mm. ¿En
cuántos cm2 se dilataron las caras del cubo?
15.- Se ha construído una escala absoluta (A) donde
el agua se solidifica a la temperatura de 210° A. ¿Cuál A) 30,07
es la temperatura en "C cuando en esta escala la mar- B) 56;04
ca sea 280° A?
C) 10,06
A)810e B)400e C)95°e D)91°e E)51°e
D) 40,01
16.- En una vasija de vidrio de I 000 cm3 de capaci- E) 60,05
dad se tiene 980 de mercurio ¿Hasta qué temperatu-
ra habrá que calentar el conjunto para que el mercu- 22.- Al calentar una placa de metal se observó que
rio esté a punto de derramarse? 8H = 1,810-4 ; v = e su superficie se incrementó en 0,6%. ¿En qué por-
0,8.10-6 °e ; TI = 16° C. g
centaje se dilató su perímetro?
A) l300e B) 150 e 0
C) 1000e
A) O, I %B) 0,3% C) 0,2% D) 0,5% E) 0,9%
D) 120 e 0
E) 1400e
23.- Un alambre tiene un coeficiente de dilatación
17.- Una regla de aluminio es exacta a 20°C. Si hace- a, y a ooe su longitud es Lo' Si el alambre experi-
mos una medida con ella a 700e y obtenemos 100 menta unavariación I1Ten su temperatura, tal que:
cm, ¿Cuál es el error cometido? I1T = -l/a. ¿Cuál será su longitud final? Si tu re-
sultado es impensado, ¿Qué reflexión harías?
A) 0,002 3 cm B) 0,004 5 cm C) 0,008 7 cm
A)6 B)4 C) Imposible D) 5 E)O
D) 0,115 cm E) 0,004 4cm
24.- Se tiene una placa triangular, tal como se muestra.
18.- La gráfica muestra una longitud de dos varillas Si su coeficiente de dilatación superficial es 2.10-4 0C-l
en función de la temperatura si sus longitudes inicia- ¿Cuál será el incremento de temperatura que deberá
les están en la relación de 3 es a 2 y sus coeficientes experimentar para que la placa toque el techo.
son: al = 1,2.1O-6°C-I /\
a2= 1,8.1O-60C-I L A) 2.1O-5"C
Hallar: tg O / tg <p B) 180°C

A) J212 D) 1 C) 360°C

B) 1/2 ' E) .J3/2 D) 100°C


E) 500°C
C) 1/4
'-------T
322 Física-Primer nivel Félix Aucallanchi V.

NIVEL 3 A) 4,04 B) 2,04 e) 4,96 O) 5,02 E) 4,50

30.- ¿Qué fracción del volumen de un depósito de


25.- En 1:>•.gura, ¿En cuántos "C se debe incrementar vidrio debe llenarse con mercurio a O°C para que el
la temperatura de las barras para que sus extremos se volumen de la parte vacía permanezca constante
junten? aA = 15.10.3 °e·1 1\ aB = 1O-30el cuando el conjunto sea sometido a un calentamiento?
a (vidrio) = 2,5.1O-5/OC, y (Hg) = 1,8.1O-4¡OC.
A) 40°C
A) 5/12 B) 1/9 C) 5/18 O) 5/36 E) 1/2
B)300e
B
e) 20°C
A") r 31.- Un alambre de coeficiente de dilatación
a = 2.1O-5/oC tiene la forma mostrada en la fi-
gura cuando T = O°C. ¿Qué distancia (en cm)
60 cm 6cm 30 cm
O) 50°C existirá entre R
y B cuando la temperatura sea
T¡= 100°C?
E) 100°C
-t20cmt
A) 100,5
26.- UIi alambre de 60 cm de longitud se dobla en B
forma circular. Supóngase una separación entre los B) 100,4
extremos de I cm. Si se eleva la temperatura hasta
100°C la separación aumenta hasta 1,002 cm ¿Cuál
es el coeficiente de dilatación lineal del alambre?
e) 100,3 fA
40cm
IOOcm

O) 100,2
A) 4.10-5 °e-I

O) 1O-5°e-1
B) 3.10-5 De-I

E) 6. I 0-5 °el
C) 2.10-5 Del
E) 100,1
L IOOcm

27.- El sistema está compuesto por dos alambres 32.- Un cierto metal a cierta temperatura tiene una
del mismo material inicialmente a 20 De ¿A qué densidad D. Cuando aumenta su temperatura en t::..T,
temperatura se observa que los alambre y la barra su densidad varía en -t::..D. Entonces, su coeficiente
forman un ángulo de 74°? aALAMBRE = 5.10-4 "C'! de dilatación lineal es:
aBARRA = 5,88.10-
3 °e-I
J\
A) sono»: B) 3t::..D/Dt::..T e) 3Dt::..TN t::..D
o
A) 120°C O)t::..D/3D.t::..T E) Faltan datos
...J
L
B) 100°C
I 33.- En el interior de un trozo de hielo se ha coloca-
do un pedazo de corcho. Si el hielo se derrite, el
e) 200°C L
nivel de agua:
O) 400 °0

E) 126°C
1 f--- L----1

28.- Una wincha metálica de 5 cm de longitud es


exacta a 15°C, un día en que la temperatura del am-
biente es 35 "C se mide un terreno obteniéndose 100
m de longitud ¿Cuál es l a verdadera longitud del
terreno? aMETAL = 4. I0-4 Del

A) 100,8 m B) 100,6 m C) 100,5 m


A) Sube
O) 100,7 m E) 102 m B) No cambia
29.- Un líquido de densidad Do = 5 glcm3
a O°C es e) Baja
calentado hasta 100°C. Si su coeficiente de dilatación
volumétrica esy= 8.10-5 0C-I, su densidad (en glcm3) O) Depende del volumen del corcho
a dicha temperatura será: E) Falta información
Caiorimetría
OBJETJl{)S
1.- Conocer e interpretar correctamente
el concepto de calor, así como sus prin-
cipales formas de propagación.

2.- Analizar las leyes que permiten ex-


plicar el comportamiento térmico de
los cuerpos cuando ellos se ven afec-
tados por el calor, y principalmente
los cambios de fase.

n nuestra vida es muy común hablar de calor y de cambios


de «estado», y como la lógica lo impone, es una necesidad
el hacer mediciones de aquella forma de energía llamada
energía calorífica, o simplemente calor. Saber en qué medida el calor
determina el comportamiento térmico de los cuerpos es uno de los ob-
jetivos principales de este capítulo. Sin embargo, es necesario reconocer
que ha sido muy arduo y prolongado el trabajo de los científicos para
llegar a descubrir las leyes que permiten explicar todos estos fenómenos.

DII¿QUEESELCALOR?
BENJAMIN THOMSON
Hablar de calor es hablar de un tipo especial de energía que solo
aparece o existe en tránsito; jamás se le puede aislar, dado que es una (1753-1814)
energía que se transmite de un cuerpo a otro debido a la diferencia de
Ingeniero norteamericano.
temperaturas que estos presentan. Así pues, concluímos que el calor es Emigró a Europa durante la
una energía no almacenable, y sólo existe mientras exista una diferencia Independencia de losE.E.U.U.
de temperaturas. Entró al servicio del príncipe
de Bavlera, quien lo hizo ge-
• PROPAGACION DEL CALOR neral de su ejército, y le
confirió el título de Conde
A) Por conducción.- El calor puede viajar dentro de un cuerpo o de de Rumford. Estando a car-
un cuerpo a otro en contacto con él por medio de la agitación de las go del arsenal de Münlch
comprobó que al taladrar
moléculas, de una zona de alta temperatura hacia otra de baja tempe- los cañones, éstos genera-
ratura. Esto se da principalmente en los sólidos, siendo los metales ban tal cantidad de calor
los que mejor lo conducen, y en orden decreciente: la plata, el cobr-, que podían hacer hervir
mas de 50 litros de agua en
el oro, el aluminio, ... , etc. Entre los malos conductores de calor pc- cada uno de ellos. Estosre-
demos citar: el aire, la lana, la madera, el agua, etc. sultados lo condujeron en
B) Por convección.- Debido a que una elevación de temperatura dis- 1798 a admitir como única
explicación de los mismos
minuye la densidad, especialmente de líquidos y gases, entonces que: -el calor es sólo una
las masas calientes suben y las frías bajan, generándose un movi- manifestación del movi-
miento cíclico, que llamaremos convección. Este efecto se aprecia miento de las moléculas de
al hervir el agua, y en nuestra atmósfera es la causa de los vientos. los cuerpos».
324 Física-Primer Nivel Fé/ix Aucallanchl V.

DEBES SABER QUE: C) Por radiación.- Por experiencia sabemos que al acercamos a una
fogata sentimos el calor que proviene del fuego; algo similar sucede
1(1) Al tocar un martillo no-
taremos que el mango
con el calor que nos llega desde el Sol cruzando el espacio vacío.
de madera nos parece Así, el calor puede viajar por radiación de ondas electromagnéticas
menos frío que la cabe- y en el vacío. Se comprueba que los cuerpos mejores emisores de
za de fierro. Esto se de- energía radiante son también los mejores absorbentes, y el mejor de
be a que la madera
conduce con menos ra-
ellos es el cuerpo negro. El mejor reflector es el blanco.
pidez el calor con re-
lación al fierro. a) Por Conducción
b) Por CorrveccIón
2(1) Las aves abren susplu-
mas para hacer Ingre-
sar aire entre ellas, y
evitar así el enfriamiento
de su cuerpo. Estose ex-
plica porque el aire es
mal conductor del calor.
Agua
Agua caliente (900C)
3(1) Las neveras de los refri-
geradores están siem-
pre en la parte más alta
para que el aire frío, que Tterra
es mas denso, baje en-
friando los alimentos de Fig 17.1
la parte Inferior.

4(1) LaropaOSCtIaes/J1g'CJ1 _ UNIDADES DE CALOR


absorbente de energía
caloríflca.
a) Caloría (cal).- Representa la
cantidad de calor que se debe su- Termómetro
ministrar a Ig de agua para ele-
CONTENIDOCALORICODE var su temperatura en 1°C (de
ALGUNASSUSTANCIAS
14,5°C a 15,5°C). Ver Fig. 17.2.
. .
DIGER/BLES
" ,
SUSTANCIA kcal b) Kilocaloría (kcal).- Se le lla-
ma también gran caloría, y es
1 bola de heladc 145
de vainilla la cantidad de calor que se debe
Pan con merme- 225 suministrar a 1 kg de agua para
lada elevar su temperatura en 1°C. Se
verifica que 1 kcal = 103 cal. L-----' __ -:=::---:-=-::!
Vino tinto seco 75
(1 vaso)
whisky(1 sorbo) 105 e) B.T.U. (British Thermal ,--------------,
Torta de choco- 140 Unit).- Es la cantidad de calor
late (1 tajada) que debemos suministrar a 1 lb
de agua para elevar su tempera-
tura en 1°F (de 32°F a 33°F).
Se verifica que:
1 BTU = 252 cal
1 Libra
IMPORTANTISIMO d) Joule (J).- Es la unidad de deAglltl
medida del calor en el S.1. En
Dado que el calor es una el Cap. 19 se demostrará que:
energía no almocenable, la
medimos por los efectos
que producen.
1J = 0,24 cal
Fig 17.3
Calorimetría 325

• DEFINICIONES CALORIMETRICAS Colores Específicos

a) Capacidad Calorífiea (K).- Cuando proporcionamos calor a un Sustancio caUg.oC


cuerpo y éste varía su temperatura, decimos que él posee capacidad
calorífica, la cual se define como aquella magnitud escalar propia Aluminio 0,211
para cada cuerpo, y que representa Cobre 0,091
la cantidad de calor que debe ganar Q Hielo 0,501
o perder para elevar o disminuir su
Hierro, Acero 0,110
temperatura en un grado.
Latón 0.091
K= Calor Q
Modero 0,648
Variación de Temperatura
Oro 0,030
Plato 0,060
(17.1)
Plomo 0,029
. cal kcal BTU J T Vidrio 0,202
Unidades (K)=-, --, --,- Fig.17.4
°C °C °F K Aceite 0,504
Agua 1,000
b) Calor específico (Ce).- Se le llama también capacidad calorífica especí-
Kerosene 0,514
fica, y viene a ser la magnitud escalar propia para cada sustancia, y
cuyo valor nos indica la cantidad de calor que debe ganar o perder Mercurio 0,030
la unidad de masa para elevar o disminuir su temperatura en un grado. Aire 0,240
\q:x.Y 00 IQ.D 0,528
(17.2) M'
.~" "'~ . m

CALOR/METROS
Observacién.« Para un mismo cuerpo se verifica
que: Son dispositivos físicos que
Ig;'m.c.1 (17.3) Fig 17.5
permiten medir el color es·
pecíflco de uno sustancio
determinado. Básicamente
e) Calor sensible (Qs ).- Si al proporcionar o quitar calor a un cuerpo está constituído por un reci-
éste logra variar su temperatura exclusivamente por esta razón, se piente metálico que es pro-
piamente el calorímetro, un
dice que ha sido afectado por un calor sensible. De ésto diremos líquido conocido, (general-
que el calor sensible es aquel que solo produce cambios en la tempe- mente agua), un termóme-
ratura de los cuerpos, y cuyo valor viene dado así: tro y uno cubierto térmlca-
mente aislante (tecnopor).
=
-Si Tf > T¡ => Qs (+ ):Calor Ganado
{ - Si Tf < T¡ => Qs = (-) :Calor Perdido
T Colorimetro
d) Equivalente en agua de un ealorÍmetro (M H 0).- Denominare-
mos así a aquella masa de agua que para efectos l1e cálculo tiene la
propiedad de sustituir a la masa de un calorímetro, incluyendo sus
accesorios. Si el calorímetro tiene masa me y calor específico Cee'
entonces su equivalente en agua se obtendrá así:

(17.5)
326 Física-Primer Nivel Félix Aucallanchl V.

PARA NO OLVIDAR:
Elproceso de vaporIzacIón
puede realizarse de dos for- Es muy frecuente el uso del término estado o estados de la mate-
mas equivalentes: ria, y sobre todo en los niveles iniciales de nuestra educación. Pues
bien, ha llegado el momento de hacer aquí una reflexión simple que
1Q) Por Ebulliclón.- En este nos permita hacer un correcto uso de los términos estado y fase de
proceso el líquido elevo
sutemperatura, y desde aquí en adelante, para lo cual utilizaré el siguiente ejemplo: Dos bebedo-
su Interior se va convir- res Ay B acuerdan festejar su encuentro tomando cerveza: A tomó dos
tiendo en vapor. y B cinco botellas. Sin duda los dos estarán ebrios (están en la misma
2Q) Por evaporacIón. - Por fase), pero asimismo reconocerás que B está más ebrio que A (están en
este medIo y sólo en lo diferente estado) ... , entendiste?
superficIe, un líquido se
transformo en vapor, Llamaremos fase a aquella composición física homogénea que
provocando un enfria- presenta una sustancia en un determinado rango de presiones y
miento en los copos temperaturas. Designaremos estado a aquella situación particular de un
Inferiores del líquido.
cuerpo definida por su presión, volumen y temperatura. En nuestra
naturaleza una misma sustancia puede existir en tres fases: fase sólida,
fase líquida y fase gaseosa (vapor).Cuando un cuerpo pasa de una fase a
otra se llama cambio de fase, y ésto solo sucede cuando los cuerpos
CALORES LATENTESDE experimentan una ganancia o pérdida de calor, siempre a una presión y
FUSION- temperatura determinadas.
(A lo presión'OO 1 atm)

Sustancio Punto de (collg] Condensación


FusiónrCJ

N -210 6,1
Hg -39 2,8
HP O 80
S 113 13,2
Al 659 91
Ag 960 21
Cu 1083 43
Sublimación Directa
Acero 1400 50
Pt 1500 27 Fig 17.6
Fe 1530 65
.CALORLATENTE <2L)
Se denomina también calor de transformación, y es la cantidad
ATENC/ON de calor que debe ganar o perder un cuerpo para cambiar de fase sin al-
terar su temperatura. Su valor depende del tipo de proceso, del tipo de
*) Lo sublimaclón directo es sustancia y de la cantidad de masa a transformar.
el proceso por el cual un
cuerpo sólido poso o lo
fase gaseoso sIn posar • CALOR LATENTE ESPECIFICO (L)
por lo fase líquido. Lo
naftallna y el hIelo seco se Es la magnitud escalar propia para cada sustancia y tipo de trans-
subliman directamente. formación; representa la cantidad de calor que debe ganar o perder la
*) Lo subllmac/ón regresivo unidad de masa para cambiar de fase sin cambiar su temperatura.
le permite o un vapor con-
vertIrse en sólido. El resi-
duo de los chimeneas se
generon por sublimac/ón
regresivo.
(17.6) I L= ~ I Unidades: (L) = cal/g; kcal/kg; BTU/lb; J/kg.
Calorimetría 327

Observación.- Cada sustancia tiene un L correspondiente para cada tipo de CUIDADO!!


cambio de fase.
El aumento de la presión
sobre la mayoría de los cu-
• CAMBIO DE FASE DEL AGUA erpos ocasiona un aumento
en su temperatura de fusión
a) Fusión-Solidificación.- La temperatura de fusión y la de solidificación y/o de ebullición. Esto no
son iguales, y dependen en general de la presión que experimente la ocurre con el agua, pues
masa a transformar. Cuando esta presión es igual a la atmosférica nor- como se sabe el hielo se
=
mal se cumple que: Tf= Ts O°c. Asimismo, el calor latente de fusión funde a O °C. Silo presión es
mayor, se funde a menos de
(Lr:) y de solidificacion (Ls) son iguales: Lr: = Ls = 80 cal/g. O °C, y silo presión es menor,

:. I QF = Qs = 3O.m I (17.7)
se funde a más de O °C.

b) Ebullición-Condensación.- El agua hierve o el vapor de agua se


condensa siempre a la misma temperatura, y el valor de éstas depende
de la presión. Si la presión es la atmosférica normal, se verifica que:
TE = Te = 100°C. Además, el calor latente de vaporización (Ly) y Calores Latentes de
el de condensación (Le) son iguales. Vaporización
Lv = Le = 540.m (A la presión de 1 otm)

.. IQv = Qc = 540.ml (17.8)


SUstanciO
Punto
de Ebufli· L(oollg)

1&1 PUNTO TRIPLE


clóniCJ

Cuando hacemos variar la pre- N -197 48


P (mmHg)
sión sobre una muestra cualquiera Aire -192 50
F V
podemos hacer que ésta cambie su O -183 51
fase a diferentes temperaturas. Si 8r 59 44
graficamos lo que sucede con la pre-
AJcohJI 78 204
sión y la temperatura del agua (Fig.
17.7) encontramos un punto en don- Vapor HP 100 540
de el agua coexiste bajo la forma de Hg 357 65
hielo, líquido y vapor. A ésto llama- 0,01
remos punto triple (T).
Fig.17.7

IJ TEOREMA FUNDAMENTAL
CALORIMETRIA
DE LA

«Cuando mezclamos dos o más cuerpos a diferentes temperatu-


ras, ocurre que el calor que pierden los cuerpos calientes lo ganan los IMPORTANTE
cuerpos fríos». Esto no es más que una aplicación de la Ley de Conser-
De acuerdo con la Flg.
vación de la Energía. 17.7 se puede reconocer
que:
:EQ = :EQ
Ganan Plerdea 1) OT es la CUNa de sub/l-
cuerpos fríos . :' CU4erpos calicata moción.

Observacián» Se recomienda que el cálculo de los calores sensibles se realice 2) TFes la CUNa de fusión.
de modo quesu valor final sea positivo. para ello las variaciones de tempe- 3)
ratura se deberán encontrar así: TV es la CUNa de vooo-

I1T = T mayor - T menor


328 Física-Primer Nivel FélixAucallanchi V.

PROBLEMAS RESUELTOS
Probo 1.- ¿Qué valor tiene el calor específico de un material cuya masa es de 20 g, si para
elevar su temperatura en 3(J'C se necesita 60 calorías de energía caloríficd? (en cal/g 0C].
A) 0,1 B) 0,011 C] 0,025 O) 40 E) 10 UNMSM92
Resolución.-
De acuerdo con los datos podemos reconocer que: m 20 g; tJ.T = = 30°C YQ = 60 cal. Luego, utilizando la
relación (17.2) para el calor específico tendremos que:
60 cal
Ce = 0,1 cal/g °C RPTA.A
Ce = 20g. 300C

Probo 2.- Se dan dos recipientes uno a 800Fy el otro a 205°F.¿Cuánta agua se debe tomar de
cada uno para tener 200 lb de agua a 1500F?
A) 80 lb, 120 lb B)88 lb, 112 lb C] 50 lb, 150 lb 0)75 lb, 125 lb E)N.A.
UNFV 88-1
Resolución.-

Sean mi y m~ las masas de agua (en libras) que


buscamos, y las temperaturas: TI = 80°F Y T =
205°F respectivamente. Luego, por condición ael
problema, la temperatura de equilibrio de la mez-
cla de estos debe ser: TE = 150°F, Yademás la su-
ma de las masas deberá ser tal que:
mi + m2 = 200 lb (1)
Pues bien, en base al Teorema Fundamental de la
Calorimetría y a la relación (17.4) para el calor
sensible se podrá establecer que:
Calor que gana = Calor que pierde
(mi) (m2)
=? mi .Ce (TE - TI) = m2Ce (T2 - TE)
11
=? mi (150°F - 80°F) = m2 (205°F - 150°F) => mi = 14 m2 ....• (2)

y resolviendo (1) y (2) encontramos: m~ = 88 lb Y mz = U2lb RPTA. B


Observación.- Para el cálculo del calor (Q2) perdido'por m2 se ha utilizado la diferencia de temperaturas
(T2 - TE) dado que T2 > TE' Y así el resultado es positivo.

Probo 3.- El gráfico representa la tempe-


ratura T en función del calor rec,
absorbido por 20 g de un líquido 120 :
Inicialmente a OOC. ¿Cuánto vale
el calor específico de la fase
gaseosa en cal/g.oC? 80 .

A) 1,25 8)0,63 C]6,30


-O-l'--I-'OOO'-----3"""'OOO----4-000.:..---Q(coI) .
O) 12,50 E) 1,00 UNI9l
Calorimetría 329

Resolución.- T("C)

Analizando el gráfico podemos reconocer que:


1) La fase líquida se inicia en "O" y se prolonga
hasta "A".
120 ·························'".~~~·K.···z··················· 'f}
2) En "A" se inicia el proceso de ebullición y ter-
: 40" e
mina en "B", todo a la temperatura constante 80
A B .0
:
de 80°e. ~
lOOOcaI ;
3) La fase de vapor se inicia en B y se prolonga
hasta C, observándose que en dicho proceso Q(cal)
el vapor absorbe el calor: Q = 1 000 cal y eleva
su temperatura en: IlT = 40°e. Luego, por la o 3000 4000
relación (17.2) del calor específico, tendremos:
1000 cal
~ Ce = 1,25 callg °C RPTA. A
Ce = 20 g. 400C

Probo 3.- ¿Qué cantidad de agua se puede llevar al punto de ebul/íclón (a presión atmosférica)
consumiendo 3 kW-h de energía? La temperatura Inicial del agua es de 1Q°C. Se
desprecian las pérdidas de calor.
Al 28,8 Kg Bl 286 Kg C] 28,6 9 O) 57,2 9 El 572 9 UNI88
Resolución.-
Recordando que: IkW - h = 3,6.106 J, Y 1 J = 0,24 cal,
Q
deducimos que el calor disponible para la operación es:
6 6 ~bulliCi6n
Q= 3 (3,6 . 10 J. 0,~4/al} 2,592. 10 cal
10·e roe-e
y utilizando la relación (17.4) para el calor sensible 'en-
contramos que:
Q = (m Ce llT)agua ~ 2,592 • 106 cal = m. l ;~~ • (100° C - 10°C)

.~ m = 28,8.103 g ~ m = 28,8 kg RPTA. A

Probo 5.- La cantidad de calor expresada en kilocalorías que se requiere para cambiar de fase
un kilogramo de hielo desde su punto de fusión hasta su total vaporización es:
A)273 B) 610 C] 720 O) 870 E) 480 UNMSM 89
Resolución.-
En el esquema se indican todos los procesos por los cuales pasó el agua proveniente del hielo.

c5
T = roe-e ~
-l b
l=OOC ~l=ooe ~3=OOC ~'-'--
~~-,-,:.,:-'..:::_~
--,'
m- "oS • (' --''\.. •.••
.-; Fase líquida ~ l'- : --' ( .. ~(
::(f,FUSIO: ~:;f/:-~-~~~•• ~1:=:;}~:~:EBULL~ION -!~~!-}i1)
HIELO ! v ! ! v ! ! v ! VAPOR
QI Ql Q3

1) Cálculo del calor defusión: Q) = L F .m = (80 k~;l) (1 kg) ~ Q) = 80 kcal


330 Física-Primer Nivel FélixAucal/anchi V.

2) Cálculo del calor para calentar el agua: Q2 = (mCel1T)H 0= Ikg.l kkc~~(lOO°C-OoC) => Q2 =IOOkcal

3) Cálculo del calor de vaporización: Q3 = t; . m = (540 ~Z;l ) (1 kg~' => Q3 = 540 kcal

Finalmente, el calor empleado de principio a fin será: QT = Q l + Q2 + Q3 => QT = 720kcal RPTA. e


Probo 6.- ¿Qué cantidad de.hle/o a O°C se requiere mezclar con un kilogramo de agua para
bajar su temperatura desde 800C a 40°C? (el calor específico del agua es 1kcal/kgOC,
y el calor de fusión del hielo es 80 cal/g).
A) 0,50 kg B)2,00 kg C] 2,50 kg O) 4,0 kg E) 1/3 kg UNMSM 86
Resolución.-
De acuerdo con la condición del pro- Fusión Calentamiento
blema, el sistema inicalmente formado ~
por: hielo + agua debe quedar a la
temperatura final: T. = 40°C. Luego,
en base al esquema a~junto y al Teore-
ma fundamental de la Calorimetría
tendremos:
o ,80
Hielo : Agua
Qgana = Qpierde
'hielo agua
(masa: m) Q3 i (masa:l kg)
~
::::} QI + Enfriamiento
'---v---'
latente

1
::::} (80 kcal)m+m(1 kcal )(4QOC- OOC) = l kg(1 kcal )(800C- 4QOC) => m=-kg RPTA.E
~ ~~C ~~C 3

Observación.- 80 cal = 80 kcal; l cal - l kcal


g kg Y g.oC- kg.oC

Probo 7.- ¿Cuántosgramos de hielo a temperatura t = -BoCse fundirán en 1,05 kg de agua a tem-
peratura ÓOOC?Calor específico del hielo = 0,5: Calor de fusión del hielo = BOkcal/kg.
A) 150 B)400 C] 500 O) 750 E) 900 UNI 82 - 2
Resolución.-
Bajo el supuesto que la masa de agua caliente Calentamiento Fusión
1,05 kg = l 050 g . solo es suficiente para ~
fundir al hielo, diremos que la temperatura ,,
,
de equilibrio se establece en O°c. Luego, en ¡ QI 1 Q2 t
base al esquema adjunto y al Teorema Fun-
damental de la Calorimetría, tendremos que:

Qgana = Qpierde -8~ ~60


hielo agua Hielo : : Agua
(masa: m) ! Q3 ! (masa:l 050 g)
::::} QI + Q2 Q3
'----v---" '-".----J '-".----J , ...• '
sensible latente sensible Enfriamiento

=> (mCel11)hiclo + 4:. m = (m.Ce·l11)agua


Calorimetría 331

=> m.(o,5 cal )[O"C- (-80C)] + (80 cal)m= 1050g(1 cal )(600C- O"C)
g.oC g g.oC

.• m = 750 g RPTA. D
Observación.- En los datos no aparecen las unidades del calor específcio del hielo; pero ello no es necesario,

dado que por teoría se sabe que: Ce = 0,5 = 0,5 cal = 0,5 kcal
g.oe kg.oe

Probo 8.- Se desea fundir un bloque de hielo de 10 kg que se encuentro o - 15°C de temperatura,
¿Cuál será lo menor cantidad de agua o 86, <rC requerido poro fundir el bloque de
hielo? Color de fusiónde hielo = 79,7 cal/g: Color específico del hielo = 0,48 cal/ifC:
Color específico del agua = 1,0 cal/ifC
A} 7,2 kg B} 10 kg C} 15 kg O} 150 kg E}869 kg UNI 83 - 2
Resolución.-
Desde que se nos pide encontrar la QI Q2
mínima cantidad de agua, suponemos
que ésta solo logra fundir el hielo y ~ T(°C)
10 deja a O°C, lo que vendría a ser la -15 ~86,9oe C>-
temperatura de equilibrio del sistema.
A continuación procederemos como Hielo Agua
se hizo en el problema anterior: (masa: 10 kg) Q3 (masa: m)

QI + Q2 = Q3 => (mCe~1)hielo + 4:. m = (mCe~1)agUa

=> lOkg(0,48 k:~~~}00C- (-15°C)] + (79,7 k~;l)(lOkg) =m. 1 k:~~~' (86,9°C- O°C) ~

.. m = lOkg RPTA. B

Probo 9.- Un cubo de hielo cuyo masa es de 50 gYcuyo temperatura es de - 10"C se coloco en
uno estanque de agua, el cual se encuentro o DoC. ¿Qué cantidad de agua se :Soli-
dificará? Color latente del hielo = 80 cal/g: Color específico del hielo = 0,5 cal/g.oC
A}24 g B}3,125 g C} 50,0 g O} 80,2 El 80,0 g UNI ~'- 2
Resolución.-
Debemos reconocer que para el hielo (TI
= - 10°C) el agua del estanque se encuen-
tra caliente; por tanto al ponerse en con-
tacto con él, absorberá calor (Q,), lo cual
le permitirá solo elevar su temperatura
hasta T2 = O°c. En relación al agua del Calentamiento
estanque, éste se encuentra en un estado ~
bastante predispuesto para solidificarse,
pues su temperatura es T3 = O°c. Luego ! QI i LIQUIDO
cualquier pérdida de calor le provocará
un inmediato cambio de fase, formándose :0 T(°C)
una masa mA. de hielo alrededor del cubo
de hielo dado, Así pues, aplicando el Hielo ¡ Agua •
(masa: 50g) , Q2 : (masa: m,.)
Teorema fundamental de la Calorimetría ~
tendremos que: Solidificación
332 Física-Primer Nivel FélixAucallanchl V.

Qgana = Qpierde => Q¡ Q2


hielo agua ~ '-.,-------'
sensible latente

=> (mCe ót)hielo = mA . t-; => 50g (0,5 g~~IC J [O°C - (-10°C)]= m A • 80 C;l
=:} . mA =3,12Sg RPTA. B

Probo 10.- Un calorímetro cuyo equivalente en agua es de 50 g contiene 300 g de agua a la


temperatura de 28°C. Si se Introducen 20 g de hielo a O°C, ccuát será
aproximadamente la temperatura final de equilibrio?
A) 18°C B) 22°C C) 24°C O) 28°C E) 30°C UNI92
Resolución.-
En lo que se refiere al calorímetro, éste logra perder
un calor Q.l que vendrá dado por las relaciónes
(17.4) y (11.5):

Q3 =mca¡CecaIÓT= MH20 .CH20 .sr , siendo


28
M H O su equivalente en agua. Y ahora, em-
plelndo el Teorema Fundamental de la
Calorimetría y el gráfico adjunto tendremos que: AGUA
Q4 m.=300g
Qgana = Qpierc!e + Qpierde
hielo agua calorfmetro

Q¡ + Q2 = Q3 + Q4 => L¡!nH + (mH· Ce. ó1)H 0= MH O'


222
CeH O' st + mA· CeH
2
O' ÓTA

=> 80.20 + 20.1. (TE - O) = 50.1. (28 - TE) + 300. 1. (28 - TE)

RPTA.B

Probo 11.- Hallar la temperatura resultante de la mezcla de 992 g de hielo a O°Cy 160 g de
vapor de agua a 1000C. Ce (agua) = 1 cal/g.oC; L(hielOJ = 80 cal/g; L(vaporJ = 540 cal/g.
A) 40°C B) 30°C C) 20°C O) 10 °C E) 5 °C
Resolución.-
m;» 160 g (VAPOR)

:c..L..-..::.=:..=::.:"--+-=::.::='-=-=+--===:::....-......,...,==--"------r=->=----r:::==- T (DC)
100

HIELO: mH = 992 g

Según el esquema mostrado en la página anterior, estamos suponiendo que el hielo se funde, y el agua obte-
nida de él se calienta hasta T",--además que el vapor se condensa y el agua obtenida de él se enfría hata TE'
Luego, aplicando el Teorema Fundamental de la Calorimetría tendremos:
Q¡ + Q2 = Q3 + ~ Q4
'---V--'~ ~
latente sensible latente sensible
=> 80 mH + mH . CeH o : (TE - O°C) = 540 my + my CeH o . (100°C - TE)
2 • . 2
.. TE=20°C RPTA. e
Calorimetría 333

17'M AUTOEVAI.UACIÓN

1.- Elige las palabras que completen mejor la oración: A) El blanco no se hundió.
«El calor es una forma de , y la tempera-
B) El negro no se hundió.
tura es la magnitud que mide el grado de _
___molecular». C) Se hundió más el blanco.
A) Energía, agitación. D) Se hundió más el negro.
B) Movimiento, calma. E) Los dos se hundieron por igual.
C) Fuerza, vibración. 6.- Dadas las siguientes afirmaciones:
D) Velocidad, aceleración.
E) Movimiento, reposo. 1) En invierno usamos abrigo porque ellos impi-
den el paso del frío.
2.- En el experimento
11) Las refrigeradoras funcionarían mejor si tuvie-
mostrado, al calentar el
triángulo metálico, la ran las neveras abajo. •
moneda: I1I) Las paredes interiores de un termo son «platea-
das» para evitar pérdida de calor por conduc-
( ) Cae ción.

( ) No cae Señalar verdadero (V) o falso (F).

( ) Queda más ajustada A) VVF B) VVV C) FVV D) FFV E) FFF


Señalar verdadero 7.- Se asegura que si dos cuerpos A y B caliente y
(V) o Falso (F). frío respectivamente se ponen en contacto, ellos
alcanzan una temperatura de equilibrio. Al respecto
A)VFF B) VVF C) FVV D) FVV D) FFV E) N.A.
se afirma que:
3.- En la parte inferior de una barra metálica se han 1) "A" tiene calor y parte de él lo cede a •.B •...
pegado con cera unas tachuelas, que como se mues-
tran en la figura ya van cayendo. ¿En qué dirección 1I) La energía que "B" recibe de "A" se llama calor.
se propaga el calor? I1I) El calor que almacenan A y B en el equilibrio
A) ~ B) t C) t- D) J. E) N.A. térmico son iguales.
Señalar lo correcto:

A) I B) 11 C) III D) I y 11 E) 11y III

8.- Suponga dos bloques A y B de zinc, cuyas masas


son M A Y MB' tal que M A > MB" Luego:
( ) KA < KB
( ) CeA> CeB
4.- Si en tu habitación existe una estufa encendida y ( ) Si QA = QB ~ sr,» st;
abres la puerta con una vela encendida en tus manos,
¿Hacia qué lado se orientará la llama de la vela? Indicar verdadero (\1) y falso (F) según corresponda:
A) FFV B) FVV C) VVV D) VVF E) VFF
A) Hacia arriba D) Hacia afuera
B) Hacia abajo E) Hacia adentro 9.- Tres cuerpos A, B Y C de igual masa tienen un
comportamiento térmico que viene dado por el
C) No sucede nada
gráfico Q-vs-T. Luego, es cierto que:
5.- En un día soleado tendemos sobre la nieve dos
pañuelos, uno blanco y el otro negro. Al cabo de un A) CeA> CeB > Cee
tiempo se comprobará que:
334 Física-Primer Nivel Félix Aucallanchi V.

A)IyIV B) 11Y V C) 1 y II

ºt e
D) 1, I1I, V E) I1, IV Y V

13.- Se tiene una olla de aluminio con agua calien-


te, y tapada con una placa también de aluminio. Se
dispone de bloques de hielo a O°c. Entonces, para
enfriar más rápidamente al agua:
T 1) Colocamos los bloques en la base de la olla.
1I) Colocamos los bloques en los costados de la olla.
10.- Dados los siguientes fenómenos, se pide se- I1I) Colocamos los bloques sobre la tapa de la olla.
ñalar el nombre que no corresponde:
A) Sólo 1 es verdadero.
1) Un pedazo de hielo se derrite al sacarlo del con-
B) Sólo 11es verdadero.
gelador. •
11) La ropa mojada se seca cuando se pone al sol. C) Sólo III es verdadero

I1I) Un trozo de naftalina "desaparece" en el interior D) Falta más información para decidir.
de un armario.
E) Ninguna de las anteriores.
IV) La superficie externa de una botella de cerveza
muy fría se cubre de gotitas de agua. 14.- Los animales en general pierden calor a travéz
de su piel. Los elefantes pierden gran cantidad de
A) I-Fusión D) Il-Liquefacción calor porque su piel es grande debido a :
B) IV-Condensación E) Ill-Sublimación Directa A) Sus grandes patas
C) 11-Vaporización B) Sus pequeños ojos
11.- Asumiendo que la presión es constante du- C) Sus grandes orejas
rante los siguientes fenómenos, indicar la afirmación
incorrecta: D) Su gran peso
A) Durante la solidificación de un cuerpo, éste pierde E) Su pequeño cerebro
calor.
B) Si un cuerpo aumenta su temperatura, no se pro-
duce un cambio de fase.
C) Durante la condensación el vapor pierde calor,
D) En el proceso de evaporación el líquido se enfría.
E) Siempre que una sustancia absorbe calor eleva
su temperatura.

12.- Cuando aumentamos la presión sobre un


cuerpo, es cierto que:

1) El calor absorbido o cedido disminuye.


ll) La temperatura de solidificación del cuerpo au-
menta.
IlI) La capacidad calorffica del cuerpo permanece
igual.
IV) La temperatura de condensación del cuerpo dis-
minuye.
V) El calor latente de fusión aumenta.
Calorimetría 335

I'ROBLEMAS I'RO'UESTOS

NIVEL 1 específicas de modo que Ce A > CeBo Se toman


muestras de igual masa de cada uno y se les
proporciona la misma cantidad de calor. Luego
01.- Un cuerpo de 200 g Ycalor específico Ce = 0,2
cal/g. °C se encuentra a I00° e. ¿Qué cantidad de calor ( ) A elevará más su temperatura
(en cal) almacena a dicha temperatura?
.( ) B elevará más su temperatura
A) 20 B)30 C) 40 D) 50 E) O
( ) Ambos elevarán por igual su temperatura
02. Si mezclamos dos masas líquidas iguales de agua
a 10°C y 70°C de temperatura, la temperatura de Señalar verdadero (V) o falso (F) según corresponda:
equilibrio ( en °C ) será _
A) VVV B) VFV C) VFF D) FVF E) N.A.
A) 40 B)30 C) 20 D) 10 E) 1 11.- Se tiene 2 g de hielo a 0° e. ¿A qué temperatura
03.- Un cuerpo al ganar 120 cal eleva su temperatura (en 0C) quedará si se le proporciona 160 cal?
en 30°C ¿Cuál es su capacidad calorífica (encal¡OC)? A)O B) 12 C) 80 D)4 E) 16
A) 1 B) 2 C)3 D) 4 E) 5 12.- Tenemos 5 g de agua a O°e. ¿Qué cantidad de
04.- Del ejercicio anterior, ¿En cuántos fC disminuirá calor (en cal) se le debe extraer para convertirlo en
la temperatura del cuerpo si pierde 8 cal? hielo a O°C?

A) 10 B) 12 C)2 D)4 E) 15 A)80 B) 120 C) 800 D) 400 E) 160

05.- ¿Cuál es calor específico de un material, si por 13.- Si a I g de agua a 100°C se le entrega 540 c~l a
cada gramo necesita 2cal para elevar su temperatura la presión atmosférica normal, ¿Cuál será su tempe-
en 10°C? ratura final (en °C )?

A) 0,1 B) 0,2 C) 0,3 D) 0,4 E) 0,5 A)O B) 10 C) 100 D) 400 E) 50

06.- Se tiene un cuerpo de 100 g que eleva su tem- 14.- A 3 g de vapor de agua a 100°C se le extraen
l 080 cal. su temperatura (en °C) final será - _
peratura en 5°e. ¿Cuántas calorías debió ganar duran-
te el proceso? (Ce = 0,6). A) 10 B) 20 C) 80 D) 100 E) 60
A) 100 B) 200 C) 350 D) 400 E) 300 15.- Del problema anterior, ¿Cuántos gramos de agua
07.- Del problema anterior, ¿Qué cantidad de calor se condensarían si se hubieran extraido 1 620 cal a
(en kcal) habrá perdido el cuerpo si su temperatura partir de la temperatura indicada?
disminuyó en 20°C? A) 1 B)2 C)3 D) 0,4 E) 0,5
A) -1 B) -2 C) -3,5 D) -4 E) -1/2 16.- Un cuerpo de 15 g se encuentra en su tempera-
08.- Un calorímetro de cobre de 300 g posee un tura de fusión y recibe 600 cal que logran fundirlo
Ce = 0,19 cal/g. 0e. ¿Cuál es su equivalente en agua Sin levar su temperatura ¿Cuál es su calor latente de
(eng)? fusión ( en cal/g ) ?

A) 57 B) 20 C) 35 D) 40 E) 30 A) 40 B) 20 C) 30 D) 15 E)5

09.- Se tiene un calorímetro que posee un equivalente 17.- Se sabe que l:~calor latente de vaporización de
en agua de 224g. Si su calor específico es 0,28cal/g. °C. una sustancia es de 150 cal/g . Si además se sabe que
¿Cuál es la verdadera masa (en kg) del calorímetro? se encuentra en su temperatura de ebullición ¿Cuántas
kcal debe suministrársele para provocar la ebullición
A) 0,5 B) 0,2 C) 0,35 D) 0,4 E) 0,8 completa de 80 g de dicha sustancia ?

10.- Dos cuerpos A y B tienen capacidades caloríficas A) 14 B) 12 C) 11 D)1O E)9


336 Física-Primer nivel Fé/ix ,ll.ucallanchi V.

NIVEL 2 A) 55 B)30 C) 60 D) 67 E) 87

18.- Si un cuerpo al ganar 60 cal eleva su temperatura 26.- Se tiene 100 g de agua a 20°C, ¿Qué cantidad de
en l5·C ¿Cuál es su capacidad calorífica? kilocalorías se le debe suministrar para vaporizarla
totalmente?
A) 4 cal B) 5 cal C) 6 cal
·e ·e . ·e A) 54 B)58 C) 62 D) 68 E) 70

27.- En un calorímetro de equivalente en H20 igual a


D) 7 cal E) 8 cal 50 g se tiene 60 g de agua a 25°C si se introduce 90 g
·e ·e de agua a 45 ·C. Determinar la temperatura de equi-
librio.
19.- En un recipiente de capacidad calorífica despre-
ciable se vierten 300 g de agua a 20 ·C y 700 g de A) 34°C B) 31,8 °C
agua a 90·C. ¿Cuál es la temperatura final de equili-
brio? D)40°C E) 41°C
A) 10·C C)48·C 28.- El calor que reciben 10 g de un líquido hace que
D) 69·C
su temperatura cambie del modo que se indica en el •
gráfico Q-vs-T. Se pide encontrar el valor de la
capacidad calorífica en cal/'C y el de su calor
20.- Un calorímetro de 300 g Y ce
= 0,08 cal/g ·C
específico en cal/g. 0e.
contiene 50 g de agua a 20·C. Se introduce una pieza de
metal de 100 g a 140·C dentro de dicho calorímetro. Q(ca/)
Hallar la temperatura final de equilibrio (en ·C).
= 0,37 cal/g ·C.
CeMETAL

A) 60 B)70 C) 80 D) 90 E) 50

21.- Se tiene un cubito de lO g de hielo que se en- ~----~..¿n:"C)


cuentra a O ·C y se dispone de una fuente de calor
que puede entregar 900 cal hasta agotar su combus- A) K = 2,5 callg °e; Ce = 0,25 callg·e
tible. ¿Cuál sería la temperatura (en rC) final?
B) K = 3,5 callg °e; Ce = 0,35 callg "C
A) 20 B) 10 C)5 D)8 E) 50
C) K = 0,25 callg °e; Ce = 2,5 cal/g "C
22.- Si a 3 g de vapor de agua a lOO·C se le extraen I
620 cal, su temperatura (en .C) final será: D) K = 2 cal/g ·e ; Ce = 0,20 cal/g °e

A) 100 B)50 C) 60 D)70 E) 80 E) N.A

23.- Calcular la cantidad de calorías que se le debe 29.- Una cacerola tiene una capacidad calorífica de
entregar a 10 g de hielo que se encuentra a -10 ·C 60 cal/rC . Cuando su temperatura era 20°C, reci-
para transformarlo en agua a IO·C. bió 240 cal. ¿Cuál será su temperatura final?
A)21·e B)24°e C)22°e D)23°e E)N.A
A) 800 B) 900 C) 950 D) I 000 E) 9 820
30.- Un trozo de metal de calor específico Ce = 0,6
24.- Se tiene un calorímetro de cobre de 300g ¿Cuál cai/g.tC y masa m = 400 g recibe 3 600 cal, de
es el equivalente en agua (eng) de dicho calorímetro? manera que su temperatura aumenta hasta lOO°e.
.(Ceeu = 0,19 cal/g·C) ¿Cuál era la temperatura inicial del metal?

A) 40 B) 19 C) 57 D) 60 E) 28 A) 85°C B) 86°C C) 87·e D) 88°C E) 89 °e

25.- En un recipiente de capacidad calorífica des- 31.- Se introducen 80 g de agua a 1000e en un calorímetro
preciable se mezclan 300 g de agua a 25 ·C con de masa m = 400g Y Ce = 0,05 cal/g. "C a 0° e. ¿Cuál
700 g de agua a 85 ·C. Hallar la temperatura de es la temperatura de equilibrio de este sistema?
equilibrio (en rC) A) 40°C B) 50·e C) 60·e D) 70°C E) 80 °e
Calorimetria 337

NIVEL 3 A) 80g B) 150g C) 107g D) 782g E)322g

32.- Un bloque de hielo de 2kg se lanza verticalmen- 39.- Tres kilogramos de hielo a -20°C se vierten
te hacia abajo con V = 1Om/s desde una altura de 1011/. sobre un depósito de capacidad calorífica desprecia-
Suponiendo que al~mpactar convierte toda su ener- ble que contiene 12kg de agua a 80°e. ¿Cuál será la
gía en calor. ¿Cuántas calorías absorve el hielo? energía calorífica intercambiada entre el hielo y el
(g = 10 //l/S2) agua? (en kcal).

A) 48 B) 64 C) 72 D) 96 E) 100 A) 340 B) 408 C) 562 D) 616 E) 725

33.- Una resistencia disipa 500 joules por segundo 40.- En un recipiente de capacidad calorífica
si se le pone en contacto con un bloque de hielo de despreciable se tiene un bloque de hielo de 2,2 kg a
720 g a O· e ¿Cuánto tiempo demorará el hielo en O°e. Calcular a qué temperatura debe ponerse en
fundirse íntegramente? contacto con el hielo una bola de fierro de 8 kg de
masa para lograr derretir al hielo en forma exacta.
A) 8 min B) 34 min C) 20 min

D) 5 mil! E) 16 mil!

34.- En un calorímetro ideal se mezclan "m" g de
agua a 20°C con "m" g de agua a 90°C, y luego que 41.- Un recipiente térmicamente aislado contiene
ellos alcanzan su equilibrio térmico, se mezclan con agua a 20°C, y se introduce en él 60 g de hielo a ooe,
"11/" g de agua a 10°e. ¿Cuál será la temperatura final y se observa que no todo el hielo se funde. ¿Cuántos
del sistema? (en 0C). gramos de agua líquida había inicialmente en el
recipiente?
A) 15 B) 20 C) 30 D) 40 E) 50
A) Exactamente 200 D) Más de 300
35.- Una pieza metálica de 10kg y Ce = 0,2kcal/kg. "C
está a la temperatura de 20°e. ¿Cuántos kJ se B) Exactamente 400 E) Menos de 240
requieren invertir para calentarla hasta 120°C? Si el
proceso duró 40 s ¿Qué potencia se desarrolló en kW? C) Menos de 100

A) 750; 40,5 B) 636 ; 25,5 . C) 836 ; 20,9 42.- Se tienen tres líquidos diferentes a las tem-
peraturas de 20°C, 30°C Y 40°C respectivamente,
D) 520; 32,5 E) 950; 15,2 y todos ellos de igual masa. Cuando se mezcla el
primero y el segundo, la temperatura final es de
36.- Se realiza una mezcla de leche con avena: la 27,5°e, y cuando se mezclan el segundo y el tercero
masa de la avena es el doble de la leche. Inicialmente la temperatura final es de 37,5°e. ¿Cuál será la tem-
la temperatura de la leche y de la avena son 10°C y peratura final si se mezclan el primero y el tercero?
70°C respectivamente. Al obtener el equilibrio (en 0c).
térmico la temperatura es 20°e. Entonces, la relación
entre los calores específicos de la leche (el) y de la A) 25 B) 28 C) 32 D) 34 E) 38
avena (e2) será:
43.- Si le suministramos 530 cal a l O g de hielo a-
A) 1,5 B) l/lO C) 5 D) 1/2 E) 10 10°C, ¿Cuál será el estado final del agua?

37.- ¿Qué cantidad de calor en kcal se requiere para A) 4 g de hielo y 6 g de agua a ooe
fundir 10 kg de plata inicialmente a 10°C, sabiendo
que su Ce = 0,05 kcal/kg.iC, &. = 21 kcal/kg, y su B) 6 g de hielo y 4 g de agua a ooe
temperatura de fusión es 960°C?
C) LOg de hielo a ooe
A) 520 B) 480 C) 685 D) 782 E) 322
D) 10gdeaguaaOOe
38.- Un vaso de masa despreciable contiene 500 g de
agua a 80°e. ¿Cuál debe ser la cantidad de hielo a- E) 10 g de agua a 48°C
20°C que se debe colocar en el agua para que Ja
temperatura final sea 50°C? (aproximar).
338 Física-Primer Nivel Félíx Aucallanchi V.

Lectura N° 3

La sublimación es el paso de una sustancia directamente de la fase sólida a la fase gaseosa.


La transformación inversa o paso de la fase gaseosa a la sólida se llama sublimación regresiva. El
yodo a presión atmosférica se sublima calentándolo; rápidamente se desprenden vapores violáceos
que pueden condensarse en fase sólida sobre cualquier superficie fresca. El anhidrido carbónico
sólido (hielo seco), empleado en la conservación y transporte de helados, también se sublima a
presión atmosférica, desprendiendo vapores blancos muy semejantes al vapor de agua. Otros cuerpos
que se subliman a presión atmosférica son el alcanfor, el ácido benzoico y la naftalina. Los sólidos
sublimables se distinguen por el olor penetrante que producen a causa de los vapores que emiten.

La sublimación obedece a leyes análogas a las de la fusión:

1) Para cada presión los cuerpos sólidos tienen una temperatura fija a la cual se subliman.
2) Durante la sublimación la temperatura permanece invariable.

3) Durante la sublimación el cuerpo absorbe cierta cantidad de calor. El calor absorbido por la uni-
dad de masa al sublimarse se denomina calor de sublimación.
,-~=-~~~~--~~~
p
La temperatura de sublimación aumenta con la presión ex-
terior. La curva de sublimación es la expresión gráfica de la rela-
ción entre la temperatura de sublimación y la presión. Los puntos
de la curva representan los estados de equilibrio de un sólido y p
SOLIDO
su vapor, o sea, aquellos estados en los cuales un sólido y su
vapor pueden coexistir. Los puntos por encima de la curva repre- VAPOR
sentan los estados en los que la sustancia sólo puede encontrarse
~f------=T
en la fase vapor. La curva termina bruscamente en un punto P,
indicándonos que para temperaturas y presiones correspondien- '-----------n-:----i
tes a P es imposible el equilibrio entre el sólido y su vapor. Fig. l

Si ahora dibujamos en un mismo diagrama la curva de fu-


sión, vaporización y de sublimación para una misma sustancia
.encontraremos que los extremos que los extremos de las 3 curvas P
se cortan en un punto denominado punto triple. (Punto P de la
fig. 2):ta región comprendida entre la curva de sublimación
(fe) y la de fusión (PA) contiene los.estados en los cuales la sus-
'tancia puede existir sólo en-la fase sólida. La región cómprendida .
entre Iacurva de fusión (PA~y de vaporización (PBD) corresponde , ~'
a:la fase-liquida, y la región comprendida entre la cm'Va de vapori-' ..
zación y de sublimación corresponde a la fase gaseosa. .e

. eaaa una de las curvesrepresema los estades-en.los que


es posisle el eqeilibrio o existencia d~ las fases colindantes. Fi-
~m~nte;,6}'pt,l1ft~ tripJ~representa'el estadoe~,~l cual ésposible
él-eqúil.ihl'Ío o coe.iisrenciade una sustancia en sus fases sólida,
liE¡ütday gaseosa. La exrstenciadel punto trip Téfué sugerida por, '-'---:-"~'--"_-"'-4-,-....L.O~~-::!"
. Jante's.:r\<mtson., . f •

~ ..
'"
."
" ,
..
o Teoría Cinética
de (os Gases

OBJETlYOS
1.- Conocer las principales leyes que ex-
plican el comportamiento de los Gases.

2.- Analizar e interpretar la Teoría Cinética


de los Gases.

urante el desarrollo del Capítulo. 16, en donde se estudió la di-


I latación de los cuerpos, no se incluyó allí a los gases. Pues bien,
en el presente capítulo intentaremos explicar la dilatación de
los gases, para lo cual será necesario reconocer que estos fenómenos se di-
ferencian sustancialmente de los sólidos y líquidos porque se ven
sensiblemente afectados por la presión. Un estudio experimental sobre el
comportamiento de los gases nos permitiría demostrar que los cambios de
presión producen considerables variaciones en el volumen y la temperatura
del gas. Asimismo, este análisis nos conducirá a encontrar qué tal com-
portamiento puede expresarse con sencillas relaciones matemáticas entre
la presión (p), volumen (V) y temperatura (1). Las leyes que encontraremos DANIEL BERNOUW
serán aproximadamente válidas para los gases reales (aire, 02' H2, N2,
...,etc), y si el gas verifica exactamente estas leyes, se llamará gas ideal. (1700-1782)

11II PROCESO ISOTÉRMICO Este afamado y honorable


científico nacIó en suzo. y
a) Descripción.- Cuando vamos llenando de mercurio (Hg) el tubo de provino de una familia dedI-
cada a las ctenctos. Fué
la Fig. 18.1 comprobaremos que el aire encerrado soporta en los casos matemático y físico, e Im-
(2) y (3) una presión cada vez mayor, y si observamos su volumen no- partió susenseñanzas en la
taremos que el volumen se ha ido reduciendo. Si ahora hicieramos lo con- Academia de Ciencias de
trario, es decir, vamos reducien- .--~~~~---~-------, Rusia. Entre sus principales
contribuciones a la Física
do la columna deHg, lo que sig- está la ley que lleva su
nifica reducir la presión, diremos nombre: Elprincipio de Ber-
que el volumen de aire encerra- noulll, poro explicar el
movimiento de los fluídos.
do irá aumentando. Cuando de-
sarrollamos esta experiencia sin
alterar la temperatura del siste- \.
I
Publicó un tratado acerca
de las mareas, yen particu-
lar sentó las bases de la
ma, decimos que se está efec- TeoríaCinética de los Gases,
tuando un proceso isotérmico. introduciendo en la Física y
en forma definitiva los
Observación.- Isos <> igual, conceptos de átomos y
Thermos <> Temperatura" . Fig 18.1 moléculas.
340 Física - Primer Nivel Félix Aucallanchí V

OJO! b) Ley de Boyle-Mariotte.- El físico inglés Robert Boyle en 1662, y


el francés Edme Mariotte en 1676, por procedimientos similares al
El aumento de la presión
ocasiona una disminución
descrito en el paso (a) concluyeron que:
del volumen de un gas, y ,A temperatura constante, el volumen(V) de una masa gaseosa es
ésto a su vez produce un inversamente proporcional con la presión (p) que experimenta».
aumento de su densidad.
Luego:
Si T = constante ~ Va ~ ; ó: I p. V = Constante I (18.1)

Observación.- Si graficamos p-vs- V, obtendremos una hipérbola llamada


Isoterma.

" A mayor Presión, P (Pa)


menor Yolronen" p-vs-v
600
,,
SOO ,
,, •
300 ---t---
,: ,
---T----,..---
200
:, ,: :,
100 ---t----~---!-------
,, ,, ',
~~'~~~~~----6~~m)

Fig 18.2

DIEI PROCESOISOBARJCO
a) Descripción.- En el experimento de la Fig. 18.3 comprobarás que
el volumen del aire encerrado en el caso (1) aumentó cuando instalamos
el mechero encendido, es decir, la masa de aire se dilató debido al au-
mento de temperatura. Si hicieramos todo 10 contrario, es decir, retira-
mos el mechero, ocurrirá que el aire encerrado se irá enfriando; ésto a
su vez significa que la temperatura disminuye, y asimismo veremos
que el volumen del aire se reduce. Como en todo momento la presión
del sistema se mantuvo constante, diremos que se ha producido un
proceso isobárico.
b) Dilatación de Gases.- El (1) (2)
científico Gay Lussac descu-
brió que: «Todos los gases se ~~!rn==*=P $=P=I~
dilatan igualmente porque
todos tienen el mismo coefi-
ciente de dilatación volumé- Po
DEBES SABER QUE: frica». Hgp
~f.- J. Aire
El fenómeno de la dilata-
ción de los gases fué es-
¡ Vf = Vi (1 + yT) I (18.2)
-..7

tudiado Inicialmente por


Gay tussoc. pero la ley que donde: y = 1/273 0C-1
gobierna los procesos
isobáricos se atribuye a y T = Temperatura en °C
Jacques Charles, aunque Observación.- Isos<>igual,
muchos se lo atribuyen al
primero. baros<>presión Fig 18.3
Teoría Cinética de los Gases 341

e) Ley de Charles> El físico francés Jacques Charles en 1785, por ATENC/ON


procedimientos experimentales similares al descrito en el paso (a), des-
En un gróflco V-vs·Tla peno
cubrió que: <<A presión constante el volumen (V) de una masa gaseosa diente (k) de la recta estó en
es directamente proporcional con su temperatura absoluta (T) ». proporción Inversa con la
presión correspondiente.
Si p = constante => Va T t f = Co~tante 1 (18.3)
Esto significa que a mayor
Inclinación corresponde
una menor presión.
".Amayor Temperatwa. mayor Volumen" V (111')
T2

PARA RECORDAR

Al graflcar la relación (18.2) •


encontraremos que la oto-
300 320 340 longaclón de la recta con-
duce a la temperatura del
Fig 18.4 cero absoluto: ·273° C.

11II PROCESO ISOVOLUMETRICO


a) Descripción del proceso> Si analizamos el experimento de la Fig.
18.5a podemos establecer que el gas se encuentra confinado en un re-
cipiente de volumen fijo. Al calentar el gas se observa que la tem-
peratura aumentó de T¡ a T2' y la presión indicada en los manómetros
muestran un notable aumento dep¡ aP2' Del mismo modo, si dismi-
nuímos la flama del mechero, el gas se enfriará y la presión disminui-
rá. Este proceso se denomina mas abreviadamente: isométrico,

b) Ley de Gay-Lussac.- El físico francés JosephLoui Gay Lussac en


J 802 propuso que: <<A volumen constante la presión (p) de una masa
de gas es directamente proporcional con su temperatura absoluta (T)>>.
OJO!!
Si V = constante => paT P- Constante (18.4)
r- Al aumentar la temcero-
tura de un gas su volumen
también lo hace, y ello eco-
".Amayor Temperatura,moyor Pruión " slona una dismInucIón de su
(b) densIdad. Luego:
p (Pa)
1
Da-
T
l000 ~.~ !

100 i
600
,//.
/;
:
¡ ¡
:
ATENC/ON!

La pendiente de la recta
,/ ¡ :
,/: : en el grófico o-vs-t es ltwer-
I': : samente porporcional con
(a) 300 3S0 400 7{K> el volumen correspondlen·
te. Esto es, a mayor tocnno-
Fig 18.5 clón, menor volumen.
342 Física - Primer Nivel Félix Aucallanchi V.

DEBESRECORDAR QUE:
Unmal de una sustancIa es
11II LEY DE AVOGADRO
una masa de ésta, en gra- El físico italiano Amadeo Avogadro en 1811 estableció que: «Vo-
mos, numérIcamente igual lúmenes iguales de diferentes gases a la misma temperatura y presión
a la masa molecular de la contienen el mismo número de moléculas»,
misma, la cual se represen-
Hubieron desde entonces ,-----,,---------,
tapor M.
innumerables experimentos que N¡=N2
han confirmado esta ley, que en
un principio fué propuesta como
hipótesis, En la actualidad se sabe O o
que un mol de cualquier gas posee V< o
un número determinado de molé- {
o
culas llamado Número de Avoga-
dro (NA):
NA = 6,023.1023 moléculas/mal Fig 18.6

_- ECUACION DE ESTADO D~ U~ GASIDEAL


OJO!! Como habrás podido comprobar hasta aquí la densidad de un gas
(D) es directamente proporcional con la presión (P) que experimenta y
Volumenes Iguales de dos con su masa molecular ( M), pero inversamente proporcional con su
cuerpos de diferente masa temperatura (T). En términos aritméticos, escribiríamos así:
molecular
MA y MB, (MA > MB) D (J. PTM ~ ~ (J.
T
P M ; ó; pV (J. (~ ) T ... (*)
tendrán masas diferentes: donde como sabemos: mI M =n (número de moles). Y agregando en (*)
mA > mB, Ello permite ase-
gurar que las densidades una constante de proporcionalidad R obtendríamos la siguiente igualdad:
son también diferentes:
DA > DB• Luego: I;V~~Tn 1 (18.5)
Observacién;» La relación (18.5) es conocida como la ecuación de estado de
un gas ideal. Yen ella R se conoce como la Constante Universal de los Gases,
y su valor en el S.I. es: R = 8,31 joule/mol.kelvin .

• ECUACIÓN DE PROCESOS
Si durante un proceso la
masa de un gas permanece cons- P
tante, entonces el término "n" de
la relación (18.5) originaría una
1'2 ~) r=:
:Tl
nueva constante Rn, tal que: ¡
V
p T = constante
¡
f
Cuando un gas pasa del es- ¡ (2)
p¡ -------T-------------------- cfT
tado I l j definidopor i p., V¡yT¡ ,,
f
'
'
2
MNEMOTECNlA al estado (2) definido por P2' V2 , ', .
, '
y T2 se verifica que: ! :
Lo relación (18.5) se re-
cuerda fácilmente silo rela- IY
cionamos con: PtVl. p;Vz
.Pavo Igual ratón-
T=¡;;- (18.6)
Fig 18.7
Teoría Cinética de los Gases 343

PARA RECORDAR:
• MODELO MOLECULAR DE UN GAS
EnFísica el término: Condi-
De acuerdo con la teoría de Daniel Bernoulli, el modelo molecu- ciones Normales [C.N.)
lar de un gas presenta las siguientes características: Los gases son muy s/r;nlfica suponer una pre-
sion Igual a 1atm y una tem-
compresibles, son expansibles, son elásticos, presentan escasa cohesión peratura de O"C. Así,un mol
y no poseen superficie libre. Asimismo, de acuerdo con esta teoría: de cualquier gas en C.N.
Las moléculas del gas son todas idénticas, se mueven caóticamente, su ocupa un volumen de 22.4
número es muy grande, el volumen de cada uno es insignificante y no I/tros. Con estos datos se
determina R en la relación
existen fuerzas entre ellas, salvo las que se producen durante los cho- [18.5).
ques, los mismos que son elásticos y de cortísima duración .

' INTERPRETACION CINETICA DE LA


• •• TEMPERATURA
Según la teoría cinética, las moléculas de un gas tienen una energía
cinética promedio (Ecmp) directamente proporcional con la temperatura:

[ Ecmp =i kT 1 (18.7)
N
(MoUctila8) T

donde k =Constante de Boltzmann


jf:n -1)
k =1,38.10-23 joule/molécula • K c:E
y: T =Temperatura absoluta del gas ~
Si ¡; es la velocidad cuadrática media ~ ~
de las moléculas del gas, entonces al
~,W ~
sustituir 1/2 mm(~)2 en la relación
(18.7), siendomm la masa de una molécula, 1 2
1:2m.v
obtendremos:
Ecmp= N
INTERESANTE
(18.8) Fzg 18.8
Las N moléculas c/u de
masa m de un gas que en
en donde se puede apreciar que la temperatura de un gas es directamente promedio presentan una
proporcional con el cuadrado de la velocidad de las moléculas. Luego: velocidad v y que ocupan
un volumen V, ejercen una
«La temperatura nos indica el grado de agitación con presión media [p) sobre las
paredes del recipiente que
que se mueven las moléculas de un cuerpo» los contiene. y que viene
dado así:
Observacián.:
En la relación 18.8, el término v que representa a la velocidad
cuadrática media de un gas, viene a ser el promedio calculado a partir
de los cuadrados de las velocidades de las molé¡ulas de un gas a u=a
temperatura determinada. Conocidas estas velocidades, se procede, á

así:

2 2 2
=2 Vt+V2+ •••••+V
v = n
n
344 Física - Primer Nivel Félíx Aucallanchi V

PROBLEMAS RESUELTOS
Probo 1.- En un balón de paredes rígidas se tiene nitrógeno a la presión de 8,31.105 Po a la
temperatura de re. Si la capacidad del balón es 40 1, calcular la masa de N2 que
contiene el balón (M N = 28 g/mol).
2
A) 0,08 kg B) 0,16 kg C) 0,24 kg O) 0,32 kg E)DAD kg
Resolución.-

Aplicando la relación (lS.5) tenemos:


pV S,3 1.105 N/m2• (40.10-3) m3 100
n=-= n=-males
RT S,31 J lmol. K.2S0 K 7
m 100
Luego, de: n = !Vi m=nM m= 7 mal (28 g/mal) = 400 g .. m =O,4kg RPTA. E

Probo 2.- Ungas ideal que se encuentra a 2rC es calentado y comprimido de tal manera que su
volumen se reduce a la mitad y supresión se triplica. La temperatura final del gas en "C es:
A) 450 B) 386 C) 243 O) 177 E) 100
Resolución.-

De acuerdo a los datos establecemos lo siguiente:

Estado Inicial: TI = 300 k; PI ; VI Estado Final: T2 = ? ; P2 = 3 PI ; V2 = 1/2 VI

PI- V¡ _ 3 PI.I/2 VI
Aplicando la relación (18.6) tenemos: 300K - ~
t; = 2'3 (300) = 450 K

Finalmente: T2 (Oe) = 450 - 273 RPTA.D

Probo 3.- Una lata cilíndrica de 8 cm de altura y 3 cm2 de sección contiene aire en condiciones
normales. Sientonces se la cierra con un piston de 4 N de peso, y que baja lentamente
sin cambiar la temperatura, iA qué altura "h" del fondo de la lata se detendrá el
pistón? (presión atmosférica: Po = 100 kPa).
A) 3,6 cm B) 5.4 cm C) 6,2 cm O) 7 cm E) 7,9 cm
Resolución.-
Como el proceso se realiza isotérmicamente, aplicare-
mos la relación (lS.l)
= (PI + PpiSlón)Ah2 P=4N
PI VI = P2 V2 ~ PIAhl ••••• (1)
-Peso:
Según la relación (15.5 ), Ppislón= ~.

Reemplazando en (1) : P1h1 =(Pl +~}2 (2)

Sustituyendo: PI = 100 kPa = 10 Nlcm2, en (2):

IO.S=(1O+1}2 ~ hz = 7cm RPTA. D


Teoría Cinética de los Gases 345

Prob. 4.- Poro la figura mostrada se conocen lossiguientes p(kI'a)


datos de un gas Ideal:
Proceso 1-2: Isotérmico: T1 = 800 K; P1 = 4 kPa
Proceso 2-3: Isobárico: P2 = 1 kPo P, ~
Proceso 3-1: Isovolumétrlco: V3 = 0,5 m3
Calcular la temperatura en el estado 3 y en el
volumen en el estado 2.
P3 ···········3, ,2
A} 3 200 K; 2m3 B] 200 K; 2 m3 C] 600 K; 1 m3 .: .:
D] 200 K; 1m3 E] 600 K; 2m3 ¡ j

Resolución-
Entre los estados 3 y 1 aplicamos la Ley de Gay Lussac (relación 18.4).

~ =~ ~ 13= 1)(~~ ) ..... (P3 = /J2)

Luego: 13 = 1)(~ ) ~ 13 = 80<{ ¡) .. T3 = 200 K


Entre los estados 1 y 2 aplicamos la Ley de Boyle - Mariotte (relación 18.1).

PIVI = P2V2 ~ V2 =,,{:~) (V1 =V3)

Luego: V2 = V3(:~) ~ V2 = 0,5( T) .. V1 = 2m


3
RPTA. B

Probo 5.- Con la más moderna bomba de vacío se puede lograr (en un recipiente) vacío hasta
alcanzar la presión de 1,333.1 (}Ó Po. A la temperatura de 27 ac, ¿Cuántas moléculas
por cm3 habrá en tal recipiente (aproximadamente]?
A] 3,22.1014 B] 5,35.1013 C] 3,22.1()B D] 3,22.107 E] 5,35.1{)Ó
Resolución.-

Aplicando la ecuación (18.5) tenemos: PV = RTn ~ V = f.t (~) ..... (1)


Sabemos que 1 mol de cualquier gas tiene un número de moléculas numéricamente igual al número de Avo-gadro
= =
(NA 6,023.1()23 moléculas), y 1 m3 1()6cm3• Luego, en (1), el número de moléculas porcm3 vendrá dado por:
3
n. _ --L6,023.102 (mOlé CUlaS)
V - RT' 106 cm3
n 17 1,333.l0-{j Pa
Reemplazando: V = 6,023.10 8,31 J /molécula. K. 300 K

~ v= 3,22. lOS ( mO!C3 ulas) RPTA.C

Probo 6.- En un balón rígido habian 10 kg de un gas a la presión de 5.105 Po. Después de una
extracción a temperatura constante la presión en el balón se redujo hato la mitad.
Calcular la masa de gas extraída del balón.
A} Falta conocer el tipo de gas B} 3 kg C] 4 kg D} 5 kg E}8 kg

346 Física - Primer Nivel Félix Aucallanchi V.

Resolución.-

Aplicando la ecuación (18.5) tenemos:


p.V ~~
Estado Inicial: ni = ~/ (1) Estado Final: nf = RT (2)

Dividiendo (1) entre (2), recordando que V¡ = Vf, tenemos:

~=.!l => mi M =.!l


nf Pf ms M Pf
5
Con los datos obtenemos: m = (112.5.10 ) 10 => mf= 5kg
f 5.105
Finalmente, se extrajeron: 10-5= 5kg RPTA.D

Probo 7.- Calcular la energía cinética promedio de las moléculas dé cualquier gas 0127 "C,
A) 5,52.10'21J/molécula B)8,28.10.21J/molécula C) 10.20J/molécula
O)1.152.10-21Jlmo/écula E)Falta conocer la masa molecular del gas.
Resolución.-

Aplicando la relación (18.7), teniendo presente que: T = 127 + 273 = 400 K, tendremos:

3 -23 joule 21
E =-2.1,38.10 /' 1 K .400K =>. Ecmp =8,28.10- joule/molécula RPTA. B
cmp moiecu a.

Probo 8.- Unhaz eJeátomos eJevaxx eJemercurio es disparCJCb sobre un discopequeño. Silo rapi-dez
eJelosátomos es4ClJmls,yseadhieren al discocuancJo chocan con el{unamasade 2.10-9kg
choca contra el disco ccx:Jasegundo).La fuerzaque ejercen losátomos sobre el disco es:
A) 8.10-4N B) 8.10.7 N C] 2.10.4 N E)2.10-7 N E)N.A.
Resolución.-
Al chocar cada masa con el disco, cada una de éstas ejerce
una fuerza que se puede calcular aplicando la relación (12.2)

7.ilt=m(vf - Vi) (v¡=0;vf=400m/s; ilt = Is; m = 2.10.9 kg)

_ mv _ 210-9 AOO átomo


=> F -ilt- l deHg

F= 8.10.7 N RPTA.B •
Probo 9.- Enun día caluroso la temperatura ambiental es 37°C. Calcular la velocidad cuadrática
media eJelasmoléculas eJeaire del ambiente.{masa de una molécula de aire: 4,8.10-26 kg)
A) 360 tnt: B)444 rrfs C] 517 rtüs O) 590 rrüs E) 618 mts
Resolución.-

Mediante la ecuación (18.8) despejamos ~, obteniendo: ~= J~


3.1,38.10-23.310
Reemplazando: v= I'::':=':='::'--;;':;=-::'-=-
4,8.10-26
v=517m/s RPTA.D

Teoría Cinética de los Gases 347

" ,,'" ;¿;.n", ~

....:
I,vA AUTOEVÁI.tIACION - .
o' > ~;~ '" , "" ¿

1.- Dadas las proposiciones, indicar lo correcto. a) altas, altas e) bajas, bajas e) N.A.
b) bajas, altas d) altas, bajas
1) Durante una expansión isotérmica la presión
disminuye. 6.- Dado el siguiente
11) Cuando comprimimos isobáricamente un gas gráfico, es falso que: P
la temperatura disminuye. B
IlI) Si calentamos isométricamente un gas, la pre- 1) A Isóbara
sión aumenta. I1) B Isoterma
1II) C Isócora A
A) I B) II C)III D) I Y Il E) Todas IV) D Isóbara
D
2.- De acuerdo con el A) I D) III
gráfico p-vs- V. la re- p B) II Y IV E) IV
lación correcta entre las C) II y III
v
temperaturas es:
A) TA > TB = Te 7.- Los gases ideales no son:

B) TA = T = Te
B
A) Compresibles D) Muy cohesivos
B) Expansibles E) N.A.
C) TA < TB < Te
C) Elásticos
D) TA = TB > Te
o v 8.- No es una característica de las moléculas de un
E) TA > TB > Te gas ideal:
3.- En la figura se v A) Todas son idénticas
muestran tres isóba-
B) Tienen movimiento ordenado
ras, cuyas presiones
son tales que: A C) Su número es grande
D) Solo los choques producen fuerzas entre ellas
A) PA < PB < Pe B E) Los choques entre ellas son brevísimos
B) PA = PB < Pc
9.- Si duplicamos la velocidad de las moléculas de
C)PA>PB>Pe
un gas ideal:
D) PA > PB = Pe
) Su energía cinética molecular promedio se du-
E) PA =PB =r« plica
4.- La relación correc- ) La temperatura se cuadriplica
ta de los volúmenes P ) La presión se duplica
para las isócoras mos-
tradas es: B
Indicar: verdadero (V) o falso (F)
A) VVV B) VVF C) VFF D) FFF E) FVF

A) VB < VA < Ve A
B) VB = VA > Ve 10.- A la misma presión y temperatura, dos volu-
C) VB = VA = Ve
e menesiguales de dos gases diferentes tendrán:

D) VB < VA = Ve 1) Igual número de moles


11) Diferente masa
E) V B > VA> Ve T III) Igual número de moléculas
5.- Elige las palabras para completar correctamente IV) Diferente energía cinética molecular promedio
la siguiente oración: «Los gases reales sometidos a , Indicar lo incorrecto
_____________presiones y temperaturas, se
comportan como un gas ideal». Al I Y 11 Bl 11 Y 1II Cl 11I D) I El 11 Y IV
348 Física-Primer nivel Félix Aucallanchi V.

I'ROBLEMAS 1'R000UESTOS

NIVEL 1 A) I 250

::
p (Pa)

Nota.- En todos los gráficos las unidades de presión, B) 1 300


volumen y temperatura se dan en: Pa, 11I3 y K. C) I 350
01.- Dado el gráfico p- V de un gas ideal, encontrar la D) 1400
•••••.•••~
temperatura (en K) en 2, si en 1 es de 300 K
E) I 450 3 5
A) 900
p
2 06.- Encontrar la presión (en Pa) en "2", si en "1"
B)800 fué de 400 Pa.
V
C) 700 A) 100
4 .---------~
D) 600 B) 150
v
E) 500 5 15
C) 200
D) 250
2 i~T .,.
2.- Sabiendo que la temperatura en " I " es de 600 K, E) 300 300 400
¿Cuál es la temperatura (en K) en "2"?
07.- Determinar el volumen (en m3) en "2", si en
A) 700 P " 1" es: VI = 61/1]
B) 710 A)20
p,"
600 . -- ----12 .... 1
B)30 500
C) 720

D) 730 500 - - - .TI V


C) 10
400 - _. ::::~2
O) 15
T
E)740
E) N.A. 600 800
03.- Para el proceso mostrado determinar el volumen 08.- Utilizando los datos del ejercicio anterior, encon-
(en 11I3) en 2. trar el número de moles que se utilizó.
A) 1
p R = Constante Universal de los gases

B)2
C)3
0)4
E) 5
:::~I
~----~----~10~~

04.- Del ejercicio anterior se sabe que el número de


V
A)3IR B)5IR

A) 6,21.10-21

O) 6,51.10-21
C)21R

B) 6,12.10-21

E) N.A.
0)41R E)6IR

09.- Encontrar la energía cinética media (enJlmo/écllla)


de las moléculas de oxígeno a la temperatura de 27° C.

C) 5,21.10-21 •
moles empleado fué 1/ = 0,5 moles, encontrar la 10.- Sabiendo que la masa de una molécula (mo) de
temperatura (en kelvin¡ del gas: cualquier sustancia se obtiene por la relación:
11I0 = Ni IN , donde Ni es la masa molecular de la
A) 997,3 B) 979.3 C) 999,5 sustancia y"NA el número de Avogadro, te pido que

O) 995,3 E)481 encuentres la velocidad cuadrática media (v) en mis


de las mol~culas de oxígeno a 27° C.
05.- Dado el siguiente proceso determinar la tempe- A) 583,7 B) 653,3 C) 681,5
ratura (en K) en 2, si en 1: TI = 600 K.
D) 683,7 E) N_A.
Teoría Cinética de los Gases 349

NIVEL 2 A) 20,100.1023 D) 35,15.1023


B)25,110.10 23 E) 40, 1 11. 1023
11.- Una masa de gas se encuentra aumentando su
volumen a temperatura constante. ¿Qué sucederá con C) 30, 115.1023
su presión?
19.- ¿Cuántas moles, se encontrarán en 170g del gas
A) Aumentará O) Faltan datos NH3?P.A.(N)= 14, P.A. (If)= l.
B) Disminuirá E) N.A.
A) 5 moles B) 10 moles C) 15 moles
C) Permanecerá constante
O) 20 moles E) 25 moles
12.- En un proceso de compresión isotérmica el volu-
men de un gas disminuye a su tercera parte. Si la pre- 20.- Del problema anterior, ¿Cuántas moléculas exis-
sión inicial era de 10 kPa, ¿Cuál será su presión final? ten en la muestra dada del gas?

A) 30 kPa B) 40 kPa C) 50 kPa A) 6,023.1024 B) 6,032.1024 C) 6,003.1024

O) 60 kPa E) 70 kPa 0)6,203.1024 E) N.A.

13.- En un proceso isobárico la temperatura de un 21.- ¿Cuántas moles habrán en una muestra que con-
gas se duplica. ¿Qué volumen tendrá al final del pro- tiene 90,345.1023 moléculas?
ceso, si al inicio era de 4 ¡n.l? A)15 B)IO C)20 0)30 E)25
A)5m3 B)9m3 C)811l3 0)7m3 E) N.A.
22.- Un gas ocupa un volumen de 5 m3, y soporta
14.- Un gas experimenta un proceso isométrico (vo- una presión de 8 310 Pa. Si en tales condiciones su
lumen constante), de tal modo que la presión dis- temperatura es de 400 K, ¿Cuántas moles contiene
minuye a su cuarta parte. Si la temperatura al inicio el gas en el recipiente?
era de 1 200 K, ¿Cuál será su temperatura final? A) 12 B) 13 C) 12,5 O) 13,5 E) 14
A) 350 K B) 300 K C) 150 K
23.- ¿Qué presión ejercen las moléculas de un gas
O) 200 K E) 250 K que ocupa un volumen de 4 m3, siendo la masa de
cada molécula igual a 3.10-28 kg, Y el número de
15.- Un gas ideal presenta un volumen de 5 m': Si moléculas igual a 4.1 025, siendo la velocidad cuadrá-
mediante un proceso duplica su temperatura y dis- tica media de ellas igual a l 000 mis?
minuye su presión en un 50%, ¿Cuál será su nuevo
volumen? A) 30'k Pa B) 40 k Pa C) 50 k Pa

O) 60 k Pa E) 70 k Pa

16.- Determinado gas ocupa un volumen de 211I3 a la 24.- Si un gas experimenta una presión de 60kPa, y
temperatura de 27° C. Si la presión se triplicase el un volumen de 2 m3 es ocupado por N = l 026 molé-
nuevo volumen sería de 1m3, ¿Cuál es la tempe- culas, ¿Cuál es el valor de laenergíacinética molecular
ratura al final (en K) de este proceso? promedio?
A) 1,8.10-21 j B) 1,6.10-21 j C) 1,5.10-21 j
A) 400 B) 450 C) 500 O) 550

17.- Cuando un gas es calentado su temperatura se


E) 600
O) 1,3.10-21 j E) N.A. •
duplica y su volumen se hace 50% mayor. Sabiendo 25.- ¿Cuál es la energía cinética molecular que en
que la presión inicial era de 800 Pa, ¿Cuál será la promedio tienen las moléculas de cualquier gas a la
presión al final del proceso? temperatura absoluta de 600 K?
A) 1,5 kPa B) 1,2 kPa C) 1,7 kPa A) 1,20.120-21 j O) 1,20.150-21 j
D) 1,1 kPa E) N.A. B) 1,24.120-20j E) N.A.

18.- Encontrar el número de moléculas que contiene C) 1,~5.110-2Ij


un gas cuya masa (11I)y masa molecular es :
26.- ¿Cuál es la velocidad que poseerán las moléculas
m = 200 g 1\ M = 40 g/mol del problema anterior, si su masa es: 24,84.10-27 kg?
350 Física-Primer lúv~l réux Auca/lnnchi V.

B) 203 mis C) 153 mis A) 300 ; 400

E) 253 mis B) 400 ; 300

27.- ¿Cuál es la presión que las moléculas de un gas C) 300; 200


a I 600 K, si a 400 K la presión es de 2 atm'l
0)600; 400
A) 8 atm B) 5 atm C) 4 atm O) 9 atm E) 7 atm
E) 400 ; 600 o 4 8
28.- Un gas ideal a 27° Cpresenta un volumen de 3m3.
¿Qué volumen presentará a la temperatura de 127" C: 36.- Un gas ideal ocupa un volumen de O'Surl.siendo
su masa total de 300 g. Si la presión que se ejerce
A) 2 m3 B) 8 m3 C) 4,4 m3 O) 9 m3 E) 6m3 sobre las paredes del recipiente es en promedio igual
a 288 kPa, ¿Cuál es la velocidad cuadrática media de
NIVEL 3 las moléculas del gas? (en km/s).
A) I B) 0,8 C) 1,5 O) 1,2 E) 2
29.- Un gas ideal ocupa un volumen de 3 m3 y sopor-
ta una presión de 8 kPa. A continuación su volumen 37.- ¿Cuál es la masa de una molécula de argón, si su
se incrementa en 5 m3, sin variar su temperatura. masa molar es de: 4.10-2 kg/mol? (en kg).
¿Cuál será-el valor de la presión final? (en kPa).
A) 4,1.10-27 B) 5.10-26 C) 6,64.10-26
A)3 B} 4 C) 5 O) 6 E) 7
O) 3.10-27 E)N.A.
30.- Se tiene un gas a la temperatura de 300 K, Y
manteniendo la presión constante incrementa su tem- 38.- ¿Cuál es la velocidad cuadrática media que po-
peratura en 100 K, adquiriendo entonces un volumen seen las moléculas de Argón a la temperatura de 664 K?
de 4 m': ¿En cuánto varió el volumen del gas? (en m3). (en mis).
A) 540 B) 643 C) 560 O) 650 E) N.A.
A) 5 B) 4 C) 3 O) 2 E) 1

31.- Un gas ideal se encuentra en un recipiente cerra- 39.- La rama corta y cerrada de un tubo de Mariotte
do. La presión del gas se incrementa en un 0,4% al está lleno de aire en una longitud de 18 CIIJ. ¿Qué
calentar el gas en IK. ¿Cuál fué la temperatura inicial altura de mercurio habrá que echar en la rama abierta
del gas? (en K). si se quiere reducir a 2/3 el volumen de aire en la
rama corta? (en cm).
A) 290 B) 280 C) 250 O) 300 E) 350 A)72
32.- Un gas se encuentra encerrado en un recipiente
de 8,31 m3 soportando una presión de 1,3 kPa y una
B) 80 T
76 cm
¡-
C)84 18 cm
temperatura de 400 K ¿Cuántas moles del gas se
encuentran en el recipiente? 0)86 1 J-
A)2 B) 3 C) 5 O) 6 E) I E) 88

33.- Del problema anterior, se sabe que parte del gas 40.- Un tubo de vidrio de I cm2 de sección transver-
abandona el recipiente, de modo que su presión des-
ciende a 500 Pa y su temperatura llegó a ser 250 K.
¿Cuántas moles salieron del recipiente?
sal tiene cerrado uno de sus extremos. Se le llena
totalmente de mercurio y se le inyecta 1,39.10-4 moles •
de gas ideal, como se indica en la figura adjunta. Se
observa que a T = 300 K Y presión de 1 atmósfera
A) 1 B) 2 C) 1,5 O) 2,5 E) 3
(76 cm /-Ig), h , = 10 cm. ¿Cuál es el valor de h2?
34.- Al elevar en I K la temperatura de un gas a (en cm).
volumen constante, la presión aumentó en 0,2%. ¿A A) 50 Gas
qué temperatura inicial se encontraba el gas? (en K). ideal
B) 45
A) 600 B) 550 C) 490 O) 450 E) 500
C) 40
35.- Un gas perfecto realiza el proceso 1-2-3 de ma-
nera que T, = 600 K. Se pide encontrar las tempera- O) 35
turas en los-estados I y 3 respectivamente: (en kelvin) E) N.A.
Termodinámica

OBlETlKJS

. 1.- Conocer la equivalencia entre el


trabajo y el calor a partir del experi-
mento de Ioule.

2.- Interpretar y aplicar las leyes de


la Termodinámica.

uego de muchísimas discusiones realizadas hasta el siglo


pasado se vinieron abajo las creencias de que el calor era
un fluido que vivía en los cuerpos calientes; ésto sucedió a
partir de los excelentes resultados experimentales logrados por Joule,
comprobándose así que el calor es una forma de energía que se puede
convertir en trabajo. Desde ese momento se ponía las bases experimenta-
les de una nueva rama de la Física que se dedicaría a estudiar las distintas
transformaciones de energía en calor, ésta sería bautizada con el nombre
de Termodinámica.
SADICARNOT
IDI EQUIVALENTE MECÁNICO DEL CALOR (1796-1832]

El inglés James Prescott Joule el año 1878 pudo comprobar me- IngenIero militar. físIco y
matemático francés. Duran-
diante un simple experimento que una cantidad de trabajo realizado te su breve existencia (murió
(cualquiera sea su naturaleza) produce siempre una cantidad definida a los 36 años] realizó genia-
de calor. Así, del ejemplo de la figura les Investigaciones en Ter-
se tiene que el bloque de masa m = modinámica, por lo que es
20,9 kg colocado a una altura h = 0,5 m considerado el padre de
esta Importante rama de la
al ser liberado, por efecto de la caída Física. Sus investigaciones
convierte su energía potencial en tra- fueron publicadas en su
bajo que hacen las paletas en el inte- obra «Reflexiones sobre )0
Fuerza Motríz del Calor. en

rior del cilindro, el cual contiene m' = 1824. Estos trabajos con-
100 g de agua, y que por efecto de la duJeron al descubrimiento
agitación eleva su temperatura en !:iT de la Segunda Leyde la Ter-
= 0,25° C. A continuación dividimos: modinámica y al principio
de que el máximo rendi-
W mgh (20,9. 10.0,5) J miento de una máquina
= térmica depende solamen-
Q - m' Ce!:iT (l00. 1.0,25) cal agua te de las temperaturas de
trabajo. Descubrió también
el. ciclo más eficiente, el
mismo que hoy lleva su
Fig 19.1 nombre: Ciclo de Carnot.
352 Física - Primer Nivel Félix Aucollonchl V.

CUIDADO
• DEFINICIONES PREVIAS
1'9 SIQ es el calor en calo-
rías,entonces el trabajo a) Sistema Termodinámico.- Denominamos así al sistema físico sobre
W en jaules equivalente el cual fijamos nuestra atención y estudio. Sus límites pueden ser
es-taró dado por: fijos o móviles (Fig. 19.2a).
W=4,18. Q b) Sustancia de Trabajo> Es la sustancia líquida o gaseosa que recorre
internamente el sistema. En él podemos almacenar o extraer energía.
2'2) SI W es el trabajo en e)
joules y Q es el calor Estado Termodinámico.- Es aquella situación particular de una
equivalente en calorías, sustancia, cuya existencia está definida por las propiedades
se verlflcaró que: termodinámicas : Presión, Volumen, Temperatura, Densidad, etc.
Q = 0,24. W d) Proceso Termodinámico.- Llamamos así al fenómeno por el cual una
sustancia pasa de un estado (1) a un estado distinto (2) a través de
una sucesión ininterrumpida de estados intermedios. (Fig. 19.2b).
e) Ciclo Termodinámico.- Viene a ser el fenómeno por el cual una
sustancia, partiendo de un estado, desarrolla varios procesos, al fi-
nal de los cuales retorna al estado inicial (Fig. 19.2c).
SistemadeLimite
MÚYiI

p p

RECOMENDAC/ON a) r "\""'.J
1"') l l
b) e)
Dado que en el 5.1. tanto ~ ", (Ga.J) : ~ :
el calor como el trabajo se
..•._ 1 ,- •.• "\ J"'.!
expresan en joules, se re-
comienda hacer las con- Sustanciade Trabajo Y .Y
versiones de calorías a
jaules antes de plantear las Fig 19.2
ecuaclones durante el de-
sarrollo de un problema.
11II TRABAJO REALIZADO POR UN GAS IDEAL
En el ejemplo de la Fig. 19.2a vemos que el émbolo del cilíndro
se está desplazando hacia arriba, y estamos seguros que ésto está ocu-
rriendo porque el gas desde adentro debe estar aplicando fuerza hacia
arriba. Todo ésto no hace más que confirmar que durante una expansión
los gases efectúan trabajo. Igual opinión tendríamos si el émbolo des-
ciende, aunque aquí el signo del trabajo del gas sería distinto: Fuerza
del gas sobre el émbolo hacia arriba y desplazamiento hacia abajo. Se
logra comprobar que en particular los gases ideales hacen un trabajo
ya sea en una expansión o en una compresión, y su valor dependerá

IMPORTANTISIMO siempre del tipo de proceso realizado, tal como veremos a continuación.
Diremos que un proceso o A) Proceso Isobárico.- En la Fig. 19.3a, el gas encerrado experi-
ciclo es reversible cuando. menta una expansión isobárica, en donde el gas inicialmente ocupa el
deten/éndolos en cualquier
punto de su desarrollo
volumen V1 y finalmente el volumen V2, de manera que la fuerza
pueden retornar al estado F (= pA) aplicada sobre el émbolo hace que éste se desplace la distancia
Inicial o Invertir el ciclo de Ax; así pues, el trabajo (W) del gas vendrá dado por:
manera natural sin con-
sumo adicional de energía. W = F. Ax = pAAx ; donde: A.Ax = V2 - V¡
Estomós que un con-cepto
es una definición. .. W=p(V2-V¡) ;0: [W=p.&VI (19.1)
Termodinámica 353

Observación.- En el gráfico Presián-vs- Volumen, el trabajo de cualquier DEBES SABER QUE:


proceso viene dado por el área bajo la curva.
12) Un proceso de expan-
B) Proceso Isotérmico.- En este tipo de proceso, el trabajo realizado sión isobárlca se gráfica
así:
depende de los estados inicial y final del gas, y asimismo de la constante
(C) obtenida de la Ley de Boyle: e = Pt V¡= P2 V2, de manera que:
p
V2
W=C.ln (y-) (19.2)
1 2

C) Proceso Isovolumétrico.- Dado que aquí el gas está impedido


de cambiar su volumen se concluye que el trabajo es nulo.
v
p
22) Un proceso de compre-
sión Isobárica se grafica
b) así:

V
p

W=(-) I
o) P':2 1'0:';;:;;':""0
p = F/A => F=p.A p¡ ·················t1 W¡2 =0
V=cte
Fig.19.3

11II ENERGIA INTERNA DE UNA GAS IDEAL (U)


PARA NO OLVIDAR
Al hacer un minucioso estudio del movimiento molecular de un
gas, reconoceríamos que éstos, además de tener movimiento de trasla- Un proceso adlabátlco es
ción, tienen un movimiento de vibración y también de rotación. Claro aquel durante el cual el
sistema no absorbe ni cede
está que todos los gases tienen sus moléculas con estos movimientos; calor al exterior. Una rápida
sin embargo, comprobaremos que la suma de todas las energías debidas compresión o descompre-
a éstos, además de las energías potenciales, nos dan un resultado lIamado slón po-dría dar origen a un
Energía Interna del Gas, cuyo valor depende exclusivamente de la proceso adiabático. •
temperatura del gas. Para el caso de gases monoatómicos constituídos
por N moléculas, esta energía viene dada así: p

U =LEcm = N (~kT) => [~U~==~~~R~l1~n~,.;""~~p-v-'1


(19.3)

Observaciones.-
3
1) Las variaciones de energía interna están dadas por: ó'U = - Rnó'T
2
2) En un proceso isotérmico la energía interna (U) se mantiene constante;
v
por lo tanto: ó'U = O
354 Física - Primer Nivel Félix Aucallanchi V.


ATENC/ON

SI un sistema absorbe o
11II PRIMERA LEY D~ LA TERMODINÁMICA
cede color. y asimismo, re- En la Fig. 19.4a, el rneche-
clbe o desarrollo trabajo, ro suministra calor al sistema, T i f
éstosdeberán asociarse con 1 d . 1 1 r ~~
un signo, el cual se eliglrá as pare es transmiten e ea or v\""\"" v L-
se9ún lo siguiente conven- al gas, el cual además de aumen- a) r\ '"' r r> .•....•
1"""'\

L, v J \......
\......
clon:· - tar su temperatura y por consi-

1
r\ '"' r- r- I'""'\v !:J.V
-guiente su energía interna, tam- l..- v Jl..- l..-J

bién puede realizar un trabajo al


desplazar el émbolo de la posición
1 a la posición 2. Todo ello nos
confirma que: «La. energía calo-
rífica (Q) que se suministra a un
! Q !~
sistema, parte se convierte. en b)_m~
trabajo (W) y parte se utiliza en
variar su energía interna (!:J.U)>>. . .~

(19.4) . Fig.19.4
a) Proceso Isotérmico.- El gas mantiene su temperatura de manera
MUY IMPORTANTE que !:J.U = 0, y así todo el calor se convierte en trabajo: Q = W.
El área encerrado por el b) Proceso Isovolumétrico.- En este caso el gas no realiza trabajo:
ciclo en un gráfico p-vs-V W = 0, y así todo el calor sirve para calentar o enfriar al gas: Q = !:J.U.
nos dá el trabajo neto (W ]
realizado. SIeste ciclo es ¿fe e) Proceso Adiabático.- Dado que aquí: Q = °~
W = -!:J.U=U¡-Uj;
sentido horario, el trabajo ésto significa que si el gas efectúa trabajo (expansión), ello lo hace a
neto será positivo, y si es de costa de disminuir su energía interna, es decir, enfriándose. Si por el
sentido antlhorarlo será
negativo. contrario el gas recibe trabajo (compresión) produce un aumento en la
energía interna, es decir, el gas se calienta .

p • MAQUINA TERMICA

8
+-'------:=:--v
La idea de aprovechar el calor para producir movimiento es un
anhelo que desveló a muchos hombres. Se conoce de un rudimentario
artefacto inventado por Hieron (siglo 1de N.E.) que convertía calor en
movimiento; pero sin duda es la máquina de vapor de James Watt la
primera máquina térmica construída con la exprofesa misión de conver-
tir calor en trabajo mecánico.
Principio básico de una máquina térmica Ciclo Termodinámico
OJO!

En uno máquina térmico y
en un ciclo determinado se
establece que:

QA = Calor absorbido en el
foco caliente.
Liqaddo~
QB = Calor perdido o ce-
dido 01foco frío.
WN= Trabajo neto del ciclo.
Bomba
Se observa también que:
WN = QA - QB = QN Fig 19.5
Termodinámica 355

Observación.- Toda máquina térmica presenta dos focos de temperatura: DEBES SABER QUE:
Caldero (TA) y Condensador (TB).

Eficiencia de una máquina Térmica: [11 =t =l-t-I (19.5)


Una máquina refrlgerado-
ro absorbe calor de un foco
frío aprovechando un tro-
txüo. y esta energía se la
entrega a un foco cal/ente.
• SEGUNDA LEY DE LA TERMODINAMICA
Si la Primera Ley de la Termodinámica se basa en el principio de
Conservación de Energía, la Segunda Ley se sustenta en la manera na-
tural que tiene el calor para propagarse, de las zonas de alta temperatura
hacia las de baja temperatura. La Segunda Ley aplicada a máquinas
térmicas se puede enunciar así:
1º En una máquina térmica es imposible que durante un ciclo todo el
calor suministrado se convierta íntegramente en trabajo
2º No es posible construir una máquina o dispositivo físico que opere
contínuamente en un ciclo, recibiendo calor de una sola fuente y
produciendo una cantidad equivalente de trabajo.
3º Es imposible que exista una máquina 100 % eficiente . IMPORTANTE

• CICLO DE CARNOT El gráfico 19.6b es un es-


quema simplificado de una
El francés Sadi Carnot ideó un ciclo reversible constituido por máquina térmica, en donde
T". YT 8 son las temperaturas
cuatro procesos: Dos de expansión y dos de compresión. Asimismo, cuto y baja {de la máquina]

~\n
este ciclo posee dos procesos isoténnicos y dos adiabáticos, según co- respectivamente. El calor
mo se muestra en la Fig. 19.6a. que se absorbe del foco
Máquina Reversib1e cal/ente es QA' yel que se
Carnot definió la máquina re- cede al foco frío es Qp-
versible como aquella que desarro- 'ÁdiabátiC08 Nótese que WN es el trabaja
P " neto que ofrece la máquina
Hael ciclo reversible ideado por él, 1\ 1
al exterior.
y propuso los siguientes teoremas: • \

Teorema 1.- De todas las máqui-


,, , \
\

nas térmicas que trabajan entre dos , \

\
temperaturas la que tiene eficiencia a) \
\

máxima es la máquina reversible.


Teorema 11.- Todas /as máquinas
reversibles que trabajan entre /as dos
mismas temperaturas poseen iguales y
eficiencias, independientemente de
la sustancia con que trabajen.
De ésto se desprende que la eficien-

cia llc de una máquina que desarrolla
el ciclo de Carnot solo depende de b)
las temperaturas absolutas de trabajo:

1~=1·~1 (19.6)
PARA NO OLVIDAR!

Relación
de Kelvin:
!QA
T
A
= QB
T
B
I (19.7) Condensador
'-----------,P,=¡:-g-::1-=9-='.6
La relación de Ke/vln solo
se deberá emplear para
máquinas reversibles, sean
estas máquinas térmicas o
máquinas refrigeradoras.
356 Física - Primer Nivel Féllx Aucallanchi V.

PRtIILEMU RESUELTOS
Probo 1.- Un automóvil de 1 tonelada de masa desplaza a 20 mis por una pista de hielo. ¿Qué
cantidad de hielo a OOC se derrite desde el Instante que el auto empieza a detenerse?
A) 500 9 B) 600 9 C] 700 9 O) 800 9 E) 1 kg
Resolución.-

Sea Q el calor que se genera al frenar el automóvil y que fundirá una masa de hielo (m ), que como sabemos
viene a ser el calor latente o de transformación para fundir el hielo, vistos en el item 1~.5. Este calor no será
otra cosa que la conversión del trabajo para detener el auto (W) a costa de variar su energía cinética. Luego,
aplicando la relación Q = 0,24 W dada en la columna de la página N° 352, tenemos:

mH ~ = 0,24 (1/2 mAv2) mH .80 = 0,24 (1/2.1 (00)

Despejando: mH = 600 g RPTA. B

Prob. 2.- Un resorte de acero de 0,6 kg de masa y constante elástica k = 11 KNlm es comprimido
15 cm. Calcular cuánto se calienta el resorte.
A) 0,2°C B) 0,35°C C] OA5 °C
Resolución.-
Aplicamos la relación: Q = 0,24 W. Q

Al comprimir el resorte se realiza un trabajo igual a la energía


potencial elástica del resorte. Luego:
í í i\
2
2 O,12k.x
(mCet1T)res. =O,24[1/2k.x] ~ t1T= m Ce (1)
res. res. F
F
De la tabla de la pag N° 325 : Ce = 0,11 cal/g.iC; k = 11 000 N/m
2 sr -m
En (1): t1T= 0,12 .~~~~,15) ~ M=O,45°C RPTA. e
Prob. 3.- En el proceso indicado, las temperaturas en p (!Pa)

o
el estado 1 y 2 son Iguales. SI el calor trans- p, 1
terldo en el proceso es 90 kJ, calcular la
presión en el estado 1, sabiendo que la pre-
sión en el estado 2 es 40 kPa.
A) 60 kPa O) 90 kPa 40 -----t------~2
I

B) 70kPa E) 100 kPa , I

C]80 kPa OL---~O~5----------~2~----3


, V(m)
Resolución.-
Como en un proceso isotérmico t1V = O, concluimos según la expresión (19.4), que el calor transferido en el
proceso es igual al trabajo realizado en él. Por lo tanto, al calcular el área del trapecio que encierra el
proceso 1-2 tenemos:

J
-2-e, kPa.(2-0,5)m
W¡_2= (40+ 3
~ 90kJ= (40+ Y) .1,5kJ Pt =80kPa RPTA.C

/'
Termodinámica 357

Probo 4.- En la figura se muestra el cIclo p (kPa)


termodinámico realizado por - -- - 1
una máquina térmica. SI en
cada ciclo se realiza 1 kJ de
trabajo, calcular el trabajo
realizado en el proceso tso-
térmico.
A) 0,5 kJ O) 2,5 kJ
B) 1.5 kJ E) 0,25 kJ

C)2kJ

ResoluciÓn.-

Como en el proceso 2-3 el gas se comprime, el trabajo será negativo, y en el proceso 3-1 el trabajo es nulo.
Luego:
~ WI_2 + [1 (0,1 - 0,6) ] kPa.m3 + °= 1 kJ

WI_2 - 0,5 kJ = 1kJ w•.2= l,5kJ RP'Í'A.B

Probo 5.- Para un gas perfecto monoatóm/co se conocen los siguientes valores:
Estado Inicial: P1 = 4 kPa; V1 = 2 m3 ; T1 = 400 K Estado Final: V2 = 5 m3
SI el gas se expande isobárlcamente, calcular el trabajo realizado por el gas y la
cantidad de calor trasnferido.
A) 12 kJ; 24 kJ B) 12 kJ; 30 kJ C) 6 kJ; 24 kJ O) 6 kJ; 30 kJ E) 18 kJ; 28 kJ
Resolución-
Por la relación (19.1): W = P (V2 - VI) = 4 kPa (5 - 2) m3

W= 12kJ

Por la relación (19.3): su: 23 P2V2 -23 PI"í


3 3
llU = -2, P(V2 - VI), = -(12) = 18kJ
W
2 •
Luego: Q = llU +W = 18 kJ + 12 kJ = 30 kJ RPTA.B

Probo 6.- En un proceso termodinámIco, un gas monoatóm/co trlpllcó su volumen Inicial de


1m3 al expandirse Isobárlcamente. SIel calor transferido en este proceso fué 400 kJ,
calcular la presión del proceso.
A) 80 kPa B) 70 kPa C)60 kPa O) 50 kPa E)45 kPa
358 Física - Primer Nivel Félix Aucallanchi V.

Resolución.-

Aplicamos primero la relación (19.3) :

3 3 3 3
!!.U = 2" P2V2 -2 PIVI = 2 p(V2 -VI) = 2 p(3VI- V¡)

=> !!.U=3pVI······(1)

Para el cálculo del trabajo aplicamos la relación (19.1) :

=> W = 2pVI (2)

Utilizando la relación (19.4), y lo obtenido en (1) y (2), tendremos:


Q=!!.U+W=3pVI+2pVI => Q=5pVI

y reemplazando el dato de VI ' se obtiene:

=> 400=5p.l p = 80kPa RPTA. A

Probo 7.- Enuna máquina térmica que funciona según el Ciclo de Carnot, el calor rechazado
por el foco fria equNale a la cuarta parte del calor que absorbe la máquina. Si la
temperatura del foco frío es rc, calcular la temperatura del foco caliente.
C) 819°C O] 847 -c E] 1 OOO°C
Resolución.-

Según los datos: QB =¡QA' y TB = 280 K. Ahora, aplicando la relación (19.7) tenemos:

TA =1120K

TA eC) = 847°C RPTA.D

Probo 8.- Lastemperaturas de los focos frío y caliente de una máquina térmica que opera
según el Ciclo de Carnot son 7°C y 427°C respectNamente. Calcular la eficiencia
térmica de la máquina y el calor rechazado por el foco fria, silo máquina realiza un
trabajo de 12 kJpor ciclo.
A]60% ; 8 kJ B] 60% ; 10 kJ C) 50% ; 8 kJ O] 50 %; 10 kJ E] N.A.
Resolución.-

Reconociendo que TB = 280 K Y TA = 700 K, aplicamos la relación (19.6)

TI =1_280 => TI=O,6=60%


e 700
Ahora aplicamos la relación (19.5)
W 12
TIc= QA => QA = 0,6
Finalmente: QB = QA - W = 20 - 12 RPTA.A

/
Termodinámica 359

19NA AUTOEVALUACIÓN

1.- Dadas las siguientes proposiciones, señalar Señalar lo incorrecto. P


verdadero (V) o falso (F) según corresponda:
A) WI2 = (+)
( ) I kcal = 4,18 kJ.
B) W23 =O
( ) El trabajo siempre se convierte en calor.
( ) Solo el trabajo mecánico puede convertirse en C) W31 = (-) 2
calor. D) Wciclo = (+)
A) VFF B) VVV C) FFV D) FFF E) FVF E) T3> TI 3
v
2.- Elige las palabras que completen mejor la 7.- Del problema anterior se afirma que:
siguiente oración: «La variación de energía _
_________________
de un gas es independiente del tipo ( ) IWI21 < IW311
de ».
( ) QI2 = (+)
A) Potencial , gas ( ) Q23 = (+)
B) Interna , proceso
Indicar vedadero (V) o falso (F)
C) Calorífica, estado
D) Cinética ,ciclo A) FFV B) VVF C) FVF D) VFV E) FVV
E) Interna , estado
8.- Sabiendo que: 1-2= Adiabática; 2-3 = Isóba-ra;
3.- Cuando frotamos dos bloques de hielo a -10°C: 1-3 = Isoterma. Se afirma que:
1) El trabajo mecánico se convierte en calor. () WI2 > W23 P
II) Los bloques aumentan su energía interna.
( ) Qcic/o = (+) 2y
III) La temperatura de los bloques aumenta necesa- I

riamente. ( ) IW311 > IW231 I


I
I
I

Señala lo correcto: Señalar verdadero I


I
I
(V) o falso (F): I
A) I B) Il C)III D) I Y III E) Todas I
I
I
I

A) FFV D) VVF Y ----~-----i---- II


4.- En un proceso de compresión isobárica es falso I I I

que: B) VFF E) VVV X IXI X I

C)VFV v
A) La temperatura disminuye.
B) El trabajo es negativo. 9.- En relación al problema anterior, es falso que:
C) El calor es negativo. A) UI - U2=(-) D) T2 >TI
D) El volumen disminuye. B) WI2 = U2 - UI E) Q31 = (+) •
E) La energía interna aumenta. C) U3 - U2 = (-)
5.- En una expansión iso térmica es cierto que: 10.- Dadas las proposiciones, indique la(s) correc-
ta(s):
A) La presión aumenta.
B) La energía interna aumenta. 1) Es imposible que el calor fluya de un foco de
baja temperatura a otro de mayor temperatura.
C) El trabajo es negativo.
D) Todo el calor se convierte en trabajo. Il) La eficiencia de la máquina térmica de Carnot
es 100%.
E) El calor es negativo.
III) La eficiencia de una máquina de Camot solo
6.- En el gráfico se muestra un ciclo constituído por depende de las temperaturas absolutas de tra-
los procesos 1-2 = Isoterma, 2-3 = Isócora, 3-1 = bajo.
Adiabática. A) 1 B) II C) III D) 1 y 11 E) 11y III
360 Física - Primer Nivel Félix Aucallanchi V.

NIVELl A) 2/3

D
B)2
Nota- En todos los gráficos la ~resión, volumen y tem-
peratura se expresan en Pa, m y K respectivamente. C) 5/3
01.- Calcular el trabajo realizado (en k.J) por el gas en D)3
el proceso 1-2 mostrado.
T=300K
P E) 3/5
A) 2,4
600 mj :> 2
07.- A continuación se muestra un proceso termodi-
B)2,1
námico, en el que se desea averiguar el cambio que
C) 3,4 experimentó la energía interna (en 1) del sistema, si
se sabe que se pierden 400 1 de calor y se reciben
D) 4,2 V 3001 de trabajo.
E) 4,3 6 10
A) -450 B) 500 C) -600 D) 650 E) -700

02.- Sabiendo que en el ejercicio anterior las ener- 08.- Para el caso mostrado se pide calcular el trabajo
gías internas en los estados I y 2 son: UI = 300 1, neto (en 1) que se realiza en cada ciclo.
U2 = 500 1. ¿Qué calor (en k.J) absorbió el sistema
durante el proceso realizado? A) 100 P
A) 2,4 B)2,6

en el proceso 1-2 indicado.


C) 3,3 D) 4,3

03.- Determinar el trabajo (en k1) que realiza un gas


E) 1,5
B) 200

C) 300
200

100
·····~D
·
:
··
.
..
:

A) 1,75 P D)400 V
1 5 8
B) 1,70 400
E) 500

C) -1,75 09.- Si el ciclo mostrado en el problema anterior lo

D) -1,70
300 •••~ V
realiza una máquina térmica, y además se sabe que el
calor que absorbe en cada ciclo es de I 2001, ¿Cuál
es la eficiencia de dicha máquina?
E) 1,65
7 12
A) 25 % B) 30 % C) 35 % D) 40 % E) 45 %
04.- Si en el ejercicio anterior, las energías internas en
1 y 2 son: UI = 600 1 Y U2 = 4001, ¿Cuánto calor 10.- Conociendo los valores del calor absorbido (QA)
(en k.J) perdió el sistema en el proceso señalado? y perdido (QB) por cada ciclo de una máquina, se
pide encontrar el trabajo neto (en 1) del ciclo y la
A) -1,90 B) 1,92 C) -1,93 D) 1,94 E) -1,95 eficiencia correspondiente: QA = 6001, QB = 3001

05.- Se sabe que el calor absorbido en el proceso 1-2 A) 100; 60 % B) 200 ; 80 % C) 300 ; 50 %
mostrado en la figura del Probo 01, es Q = 15001,
¿Cuál es el cambio que experimentó la energía interna D) 150; 50 % E) 350 ; 10 %
(enjaules) del sistema?
11.- Encontrar la eficiencia de una máquina de Carnot
A) 150 B) -250 C) 350 D) -900 E) 550 si sus temperaturas de trabajo son: TA = 227° e,
TB = 127°C
06.- Si en el balón de gas mostrado la energía interna
es igual a 900R (R= Constante Universal de los gases), A) 20 % B) 30 % C) 40 % D) 50 % E) 60 %
¿Cuál es el número de moles que posee?

l'
Termodinámica 361

NIVEL 2 18.- Un gas ideal ocupa un volumen inicial de 10m3,


ya continuación es comprimido isobáricamente hasta
12.- ¿Qué cantidad de trabajo en kJ se requiere rea- adquirir un volumen de 7 m3. Si la presión fué de 300 Pa,
lizar para calentar m = 200 g de agua desde 10°C ¿Qué trabajo realizó el gas durante la compresión?
hasta 60°C?
A) 100 B)300 C) -900 D)-752 E)-734
A)41,8 B)50,8 C)41,4 D)45,1 E) 11,3
19.- Cuando se lleva un sistema del estado" i" al
13.- ¿Cuánto trabajo se debe efectuar para fundir estado "/' siguiendo la trayectoria iaf se encuentra
exactamente 20 g de hielo a O°C? (en kJ) que Q = 50 J y W = 201. Siguiendo el recorrido ibJ,
Q = 36 J. Calcular el trabajo siguiendo dicha tra-
A) 5,608 B) 6,632 C) 6,668 D) 2,328 E) N.A.
yectoria.

aO
p
14.- Suponiendo que todos los cuerpos al detenerse A) l J
convierten toda su energía mecánica en calor, ¿Cuántas I
calorías produce el choque del bloque? (g = 10m/52). B) 5J

A) 15 C) 3J

D) 7 J i b
B) 63

C)36 E) 6J v
D) 72 20.- Del problema anterior, averiguar la energía in-
terna inicial del gas (enjaules), si al final del proceso
E) 34 fué de 700J.
15.- Sabiendo que el resorte mostrado, tiene una cons- A) 140 B) 670 C) 546 D) 760 E) 734
tante elástica k = 25 Nlcm, y en la posición mostrada
se encuentra estirado 4 cm, ¿Cuántas calorías podrá 21.- Si una máquina térmica trabaja entre 27°C y
generar el sistema cuando se haya detenido por cau- 227°C absorbiendo 600J de calor y cediendo al ex-
sa del rozamiento? terior 420 J de calor, ¿Cuál es su eficiencia?
A) 1,5 A) 30% B) 25% C) 19% D) 20% E) 17%
B) 6,3
22.- Considera que una máquina térmica siguiera un
C) 3,6 ciclo de Carnot, trabajando entre las temperaturas
D) 7,2 del problema anterior. ¿Cuál sería su eficiencia?
E) 4,8 A) 10 % B) 30 % C) 20 % D) 40 % E) 34 %

16.- Se tiene un bloque de hielo de 100 g a 2 m de 23.- ¿Será posible que una máquina térmica que tra-
altura, y se le deja caer libremente en un lugar donde bajase entre las mismas temperaturas del Probo 22
g = 10m/52. ¿Qué cantidad de calor ganará el hielo absorva 500J y entregue un trabajo neto de 225J en
por efecto del choque con el piso, si ésta proviene cada ciclo?
del 80% de la energía mecánica? (en calorías).
A) Si es posible
A)384 B) 163 C)336 D) 752 E) 734
B) Es imposible
17.- Determinar en el trabajo (enkJ) realizado porel . C) Sería posible si el trabajo neto es mayor
gas en cada caso, al pasar del estado l al estado 3,
siguiendo los procesos indicados. D) Faltan datos

A) 8,4 E) N.A.

B) 6,3 24.- ¿Cuál es la temperatura del foco frío de una


máquina de Camot cuya temperatura alta es de 400
C) 3,3 K y su eficiencia del 50%?
D) 1,4 A) 100 K B) 300 K C) 200 K
E) 7,3 D) 400 K E) N.A.
5 12
362 Física-Primer nivel Félix Aucallanchi V.

IVEL3 A) 150 B) 200 C)250 D) 300 E) N.A.

25.- La diferencia de temperaturas entre las aguas de 30.- Un gas ideal efectúa un ciclo como el indicado
arriba y las aguas de abajo de una catarata de 84m de en la figura. Sabiendo que el calor neto que recibe
altura es en cC . durante cada ciclo es de 360J, ¿Qué trabajo se desa-
rrolla en la expansión isotérmica 3-1?
(Ce(agua)=4200Jlkg.oC)

A) 100 J p (Pa)
A) I B) 84 C) 10 D) 5 E) 0,2

26.- Una resistencia eléctrica recibe de una fuente B) 300 J


eléctrica una potencia de 50 W El bloque de hielo en
donde se encuentra el resistor es de 720 g Y está a C) 120 J
O°C, ¿Cuántos minutos demorará el hielo en fundir-
400 ••••••••••
se íntegramente? ~

A) 80 B) 84 C) 10 D) 5 E) 0,5
D) 480 J
2: 1:
E) 40 J 0,5 0,8
27.- Suponiendo que un bloque metálico está en su
punto de fusión y tiene un calor latente de fusión 31.- Una máquina de Carnot absorbe 75 k J Y reali-
L (joule/kg), ¿De qué altura se le debe soltar para que za 50 k J de trabajo; la temperatura dcl foco caliente
se funda íntegramente? g = aceleración de la grave- es de 900 K. ¿Cuál será la correspondiente tempera-
dad (1IIIs2) ? tura del foco frío? (en K).

A)2Ug B)Ug C) gl2L D) giL E) Lg A) 240 B) 300 C) 320 D) 400 E) 500

28.- Un gas monoatómico realiza un proceso de ex- 32.- Una máquina térmica que opera según el ciclo
pansión isobárica 1-2. ¿Cuál es el incremento cn su de Carnot recibe 500 k J durante la expansión
energía interna en kJ, si recibió 5 kJ de calor? isotérmica. Si su eficiencia es del 66%, calcular el
calor rechazado durante la compresión isotérmica
A) 2,0 P (Pa) (enkJ).
2
B) 3,5 800 ...... ) > A) 170 B) 190 C) 210 D) 350 E) 400

C) 5,4 33.- Una máquina de Carnot trabaja con su foco


caliente a 227°C ¿En cuántos grados Celsius deberá
D).6,0 3 disminuir la temperatura del foco frío para aumentar
E) 0,2 6 12
..
V (m )
la eficiencia en I%?

A) 20 B) 50 C) 15 D) 10 E) 5
29.- Sabiendo que el sistema mostrado recibe 350J,
y con ello logró comprimir el resorte indicado en 34.- Una máquina térmica absorbe 700J de calor de
20 cm. ¿Cuál fué el cambio producido en su energía un foco a 400 K Y rechaza 200 J de calor hacia una
interna ea joule ? k = 50 N/cm. foco de 300 K durante cada ciclo que dura 25 segun-
dos. Luego podemos afirmar que:
A) El trabajo neto por ciclo es de 4001.

B) La potencia de la máquina es de 20 watts.

C) La eficiencia de la máquina es del 50%.

D) La eficiencia de la máquina es del 25%.


20cm ¡
E) Esta máquina tiene un inaceptable desarrollo.

,-
E Iearostát) ea

ORlE7TVOS
1.- Describir y fundamentar las distintas
interacciones entre cuerpos electriza-
dos.

2.- Conocer los conceptos de carga, campo


y potencial eléctrico.

uien haya presenciado alguna vez la aparición de un rayo at-


mosférico, no tengo la menor duda de que habrá quedado -.
muy impresionado por la magnitud del fenómeno natural.
Tales sucesos son la expresión máxima de la naturaleza en cuanto a fe-
nómenos eléctricos se refiere. En este capítulo y en el siguiente estudia-
remos todas aquellas situaciones en las que las cargas eléctricas se en- - . CHARLES AGUSTlN
cuentran en estado de reposo, y a esta rama de la Física que las estudia COULOMB
la llamamos Electrostática. En el Capítulo 22 estudiaremos los fenóme-
nos en donde las cargas eléctricas se encuentran en movimiento, denomi- (1736-1806)
nándose a esta parte de la Física: Electrodinámica.
Este notable científico
nació en Angulema, Frar»
_ ELECTRIZACION clo. y se le recuerda princi-
palmente por la ley física
Cuando frotamos un peine o regla de plástico, ellos adquierenla que lleva su nombre, y que
propiedad de atraer cuerpos ligeros. Así, los cuerpos con esta propiedad explica la interacción entre
dos cuerpos cargados
se dice que se encuentran electrizados, descubrimiento hecho por Thales e/éctriccmente. Nacido en
de Mileto (siglo V a. de C.) al observar que un trozo deámbar(sustancia una familia de elevada po-
resinosa que en griego se llama elektron) frotado con piel de animal sición. fué influenciado por
podía atraer pequeños trozos de paja o semilla. las itIeas liberales de

1m ¿POR QUE SE ELECTRIZA UN CUERPO?


Voltaire y Rousseau. En su
juventud fué ingeniero
militar, trabajando como
tal en la India. Al regresar a
La teoría atómica actual nos ha permitido descubrir que cuando Francia se interesó más por
frotamos dos cuerpos entre sí, uno de ellos pierde electrones y el otro la experimentacIón cientí-
los gana. Se aprecia que estos cuerpos manifiestan propiedades eléctri- fica. inventado la "Balanza
Electrostática" para medir
cas, aunque éstas no son iguales. Si por algún medio podemos regresar las fuerzas e/ectrostáticas.
los electrones a sus antiguos dueños, en cada cuerpo desaparecerían las lo cual le permitió esta-
propiedades eléctricas; ésto se explica porque ahora en los átomos de btecer su célebre ley. Tam-
cada uno el número de electrones es igual al número de protones, y en bién' estudió la fricción en
máquInas, la elasticidad de
tal estado los cuerpos son neutros. De todo ésto concluímos que: «Un los metales, de fibras de
cuerpo se electriza simplemente si alteramos el número de sus electrones». seda, etc.
364 Física - Primer Nivel Félix Aucallanchl V.

INTERESANTE
_ CONCEPTO DE CARGA ELECTRICA
1) CUerdofrotcTnos in trozo
de vidrio con uno tela Designamos con este nombre a aquella propiedad que adquieren los cuer-
de seda, el primero pos cuando en ellos existe un exceso o defecto de electrones. y que 10 mani-
adquiere uno cargo fiestan mediante los efectos de atracción o repulsión sobre los demás cuerpos.
positivo.
«Un cuerpo posee carga positiva si tiene un defecto de electrones.
2) Cuando frotamos un pe- y carga negativa si presenta un exceso de electrones».
dazo de plástico con
uno tela de lona, el Se dice que la carga eléctrica se encuentra cuantizada; ésto signifi-
primero adquiere uno ca que en la naturaleza la carga más pequeña la posee el electrón. y toda
cargo negativo. carga deberá ser un múltiplo de ella.
Estosfenómenos son con- Carga de un cuerpo o ~
siderados los patrones de , N Entero -e = N (20.1)
comparación poro deter- Carga del Electron
L-~~~~ __ ~~~~ ~
mInar los signos de los
cargos.
.PRINCIPIODECONSERVACIONDEIACARGA
Este principio se basa en el he- (1) (2)
cho de que al frotar dos cuerpos entre ~' ~
sí, la carga que uno de ellos pierde, .Antu ~ ~
el otro lo adquiere, conservándose así
la carga del sistema; ésto es: «La Durante 050-
carga no se crea ni se destruye. solo

ATENC/ON
se transporta». En el ejemplo de la
Fig. 20.1 se establece que:
Despuú
@ (1)
-25
~
~
(2)

(l:q)antts = (l:q) después 1 (20.2) .


En /o naturaleza hay cuer- I. . FIg20.1
pos que boja determinados
condiciones se comportan
como buenos o malos con-
r:r.n
ala LOS CUERPOS
ELECTRICAS
SEGUN SUS PROPIEDADES
ductores de lo electricidad.
A este grupo se les llamo
sem/conductores, y entre a) Conductores.- Son aquellos que
ellos se pueden citar 01 permiten el paso de las cargas por el in-
SlIJcloy 01Germanlo. terior de su masa sin alterar sus
• propiedades químicas, Entre ellos están

!~
los metales, el aire húmedo, el cuerpo
de los animales, el agua acidulada, ...,etc.
En la Fig. 20.2, el cuerpo metálico está
cargado positivamente (le faltan
electrones), y es conectado a TIerra por
medio de un conductor, el cual transporta Ablonte
desde ésta los electrones que le faltan al ",.,...,--1L1L-rrrm
cuerpo metálico, quedando finalmente ~ ~Electrones
descargado. La Tierra actúa siempre e
PARA NO OLVIDAR: de este modo: «Le da electrones al
Mediante uno cuidadoso que lefalta y recibe del que le sobra».
frotación podemos llegar o b) Aislantes.- Se les llama también .
cargar un aislante. Sin em-
bargo, esta cargo quedo dieléctricos o malos conductores; y se FIg 20.2
confinado solo en lo región caracterizan por ofrecer gran resistencia al paso de las cargas por el
frotada, es decir, no se dis- interior de su masa. Sin embargo, se electrizan fácilmente por frotación.
tribuye 01resto del aislante.
Entre ellos tenemos a los plásticos, la madera, el vidrio, el aire seco, ...,etc.

-'
Electrostática 365

ELECTROSCOPIOS
• ELECTRIZACION DE ~OS CUERPOS
El electroscoplo es un dis-
1) Por frotación.- Uno de los cuerpos que se frota pierde electrones positivo estacionario que
y se carga positivamente, el otro gana los electrones y se carga negativamente. permite comprobar si un
2) Por contacto.- Cuando ponemos en contacto un conductor carga- cuerpo está o no electri-
zado. SI el cuerpo lo está,
do con otro sin carga, existirá entre ellos un flujo de electrones que du- las laminillas del electros-
ra hasta que se equilibren electrostáticamente. copla se cargan por induc-
3) Por inducción.- Cuando acercamos un cuerpo cargado llamado ción, y por ello se sepa-
rarán.
inductor a un conductor llamado inducido, las cargas atómicas de éste
se reacomodan de manera que las de signo contrario al del inductor se-
sitúan lo mas próximo a él.
1)Por Frotación 3) Por Inducción
Conductor

~
2) Por Contacto
.,
...."
+++
++

Inducido
Aislante

Fig20.3

.POLARlZACION DE UN AISLANTE DEBES SABER QUE:


La diferencia eléctrica entre un con-
ductor y un aislante estriba en que el pri- Cuando iluminamos una
placa de zinc con una ra-
mero posee electrones libres que permi- diación de rayos ultravlo-
ten el desplazamiento de carga a través tetas. ésta emite electro-
de ellos, mientras que el segundo no los nes,ypor ello adquiere una
tiene. Sin embargo, los aislantes tienen carga positiva. Estemodo
especial de cargar un
la propiedad de que al estar cerca de un cuerpo se denomina efec-
inductor, sus moléculas se polarizan, es to fotoeléctrico.
decir, el centro de las cargas positivas no
coincide con el centro de las cargas nega-
tivas. Así pues, el trozo de papel de la
Fig.20.4 ha ordenado sus moléculas pola-
rizadas, de manera que el sector A se ha
cargado negativamente, y el sector B po-
sitivamente. Debido a este fenómeno el Atracción del Dieléctrico
trozo de papel saltará hacia el inductor.
Fig 20.4

_ MEDICION DE LA CARGAELECTRICA
La carga de un cuerpo se mide por el número de electrones que po-
see en exceso o por defecto. Esta carga se expresará por las letras: q ó Q.
En el S.I. la unidad de carga se llama coulomb CC), y se define del
siguiente modo: «Un cuerpo tiene una caiga.de le Ú perdió-o ganó
6,25.1018 electrones». .
366 Física - Primer Nivel Félix Aucal/anchi V

CARGA PUNTUAL
_INTERACCIONES ELECTROSTATICAS
Utilizaremos este nombre
para designaraquella carga A) Ley Cualitativa.- Esta ley se extrae de la misma experiencia, y es-
que posee un cuerpo cuyas tal lec e que: «Dos cuerpos con cargas de la misma naturaleza (o signo)
dimensiones geométricas se repelen, y de naturaleza diferente (signos diferentes) se atraen».
soninsignificanteso despre-
ciables frente al resto de los B) Ley Cuantitativa.- La intensidad de la atracción o repulsión fué
cuerpos. descubierta por Charles A. Coulomb en 1780, y establece que: «Dos
cargas puntuales se atraen o se repelen con fuerzas de igual intensidad,
en la misma recta de acción y sentidos opuestos, cuyo valor es directa-
mente porporcional con el producto de las cargas e inversamente pro-
porcional con el cuadrado de la distancia que los separa».
Para el ejemplo de la Fig.20.5, se verifica que:

¡ F = ~e ~ I (20.3) r-

q
-¡---------q-.2

CUIDADO!! en donde k tiene un valor que depen- F ._. ~


de del medio que separa a los cuer-
Al reemplazar los datos de pos cargados. Si el medio fuera el
las cargas en las relaciones vacío se verifica que: -+--~- d

¡
!
(20.3) y (20.4), éstas no
/ e
deberán Incluir los signos, ke =9.109 N.m2 2
pues se trata de averiguar Dos Cargas
el módulo de la fuerza o del En q¡ ,q2 = coulomb (e)
campo, y como sabemos el S.I 'd=metrot mr
los módulos son siempre ~ F=newton(N) Fig20.5
positivos.
_ CAMPO ELECTRICO
1) Concepto de Campo Eléctrico.- Toda carga eléctrica altera las
propiedades del espacio que la rodea, el mismo que adquiere una «sensi-
bilidad eléctrica» que se pone de manifiesto cuando otra carga ingresa
a esta región. Así, llamamos campo eléctrico a aquella región de espacio
que rodea a toda carga eléctrica, y es a través de ella que se llevan a cabo
INTERESANTE las interacciones eléctricas.
2) Intensidad de Campo Eléctrico (E).- La existencia de un campo
La constante electrostá-
tica de Coulomb (ke) tiene eléctrico se manifiesta por las fuerzas que ella ejerce sobre toda otra
un valor que en general de- carga colocada en él. Se define la intensidad del campo en un punto 4e
pende del medio que se- él como la fuerza que recibiría la unidad de carga puntual y positiva
para a las cargas, el cual colocada en dicho punto. Por ejemplo, si en la Fig. 20.6 la intensidad del
viene dado por la siguiente
relación: campo creado por la larga puntual Q
en el punto P es 200 N/e, ello significa
k =--
1
que el campo ejerce una fuerza de 200 -_._
P E _~
e 41lE,.€o
N a toda carga de 1 e colocada en q: F
donde E, y Eo son respecti- dicho punto. La intensidad del campo
vamente la permltlvidad creada por una carga puntual viene dada
eléctrica relativa del medio
y la permltividad eléctrica
por la siguiente relación:
d~
del vacío. En el S.I.setiene:
Eo = 8,85.10-12 C2.m2fN [IEI=ke fz- I (20.4) Esfera - Punto
y para el vacío
La unidad de E en el S.1. es el:
e, = 1
newton/coulomb = N/e. Fig20.6

l'
Electrostática 367

3) Fuerza del Campo (F).- PARA NO OLVIDAR


Aprovechando el ejemplo del item anterior podemos establecer Una carga de prueba es
que: Una ca~a puntual q colocada en unpunto del campo donde la in- por definición de signo po-
tensidad es E experimentará una fuerza F que vendrá dada así: sitivo; luego. frente a una
carga positiva deberá ote-
PiiE' ~ q=(+) jarse. y frente a una nega·
(20.5) {Pi -l-E' ~ tiva deberá acercársele.
q=(-) Este es el criterio para di·
bujar los vectores campo:
4) Principio de superposición de campos.-
De acuerdo con este principio se r-------:-----...,.-.-., 1) Sedibujan alejándose si
la carga que lo genera
establece que: «La intensidad del campo -q2 () +q¡ es positiva.
eléctrico que producen varias cargas O -. -
en un mismo punto viene dada por la .... _~1<'." "'" P s,
suma vectorial de las intensidades de ...-.z --1' (~) ~
campo que cada una produce de ma- +qb·········· .. ····_···· El E¡. Es
nera independiente sobre dicho lugar»,
Del ejemplo de la Fig, 20.7: -q4/ E4

I ET =!: E I (20.6) O Fig 20.7 2) Se dibujan apuntando


hacia la carga que lo
5) Campo creado por una esfera conductora cargada.- produce si ésta es ne-
Cuando cargamos una esfera metá- gativa.
lica o un conductor en general, se veri- .•. + .•. Q
+
fica todo un movimiento electrónico
interno que dura un lapso muy corto, .•. + E
---- - ---- ------ c---t::>-
observándose que todas las cargas se : .•. P:
+
ubican en la superficie externa del con- .•. .•.
.•.
ductor, de manera que en su interior el
campo es nulo, y éste existe solo desde
la superficie externa hacia afuera. Tal ----d ---4
es la característica del campo y de las
cargas en un conductor eléctricamente
en equilibrio. Para el caso de la esfera
conductora, el campo externo se deter-
mina como si toda la carga se ubicara
en el centro de la esfera. Así pues:

(20.7) I E =ke ~ I~ d ~ R o
E=(J .
R
•..
d
Fig20.8
6) Líneas de Fuerza.-
El concepto de línea de fuerza fué introducido por Michael Faraday
el siglo pasado para representar gráficamente a un campo. Estas líneas'
se trazan de manera que en cada pun- E DEBES SABER QUE:
to el vector E sea tangente a ella. Las . 33
líneas de fuerza se dibujan saliendo de
.. d I
2 J
tnea ae
¿L' Las líneas de fuerza se
concentran más en las
Ias cargas P?sltJvas y.entran o a as Fuerza
cargas negativas. En cierto modo una El 1 regionesdonde el campo es
más intenso. Además. dos
línea de fuerza es la trayectoria que se- líneas de fuerza solo se
guiría una carga puntual positiva de- cortan en los puntos en
jada en libertad dentro del campo. '----------F=:-ig-;;;-20"".-;:!9 donde el campo es nulo.
368 Física - Primer Nivel Félix Aucallanchi V

MUY INTERESANTE

La propiedad que tienen


los conductores de distribuir
las cargas por su superficie
hace que éstas se concen-
tren más en las puntas o
zonas agudas, y menos en
los l/anos o hendiduras. El
campo en laspuntas es ver-
daderamente muy Intenso
que, en ocasiones produce
chispazos eléctricos de
*
*
descarga.

Fig20.10
7) Campo Eléctrico Uniforme y Estacionario.-
Son aquellos en los que la in- '(-+-) --------- •••
-:-: -(_-,)
tensidad del campóE es la misma en A~ q(J;¡----=-F
todos los puntos del espacio que ocu- (+») ') h' (-)
pa, y que no cambia a través del tiem-
po. Se representa por líneas de fuerza (+)-);;'--""'F:---<!a ••• 11 (-)
paralelas, del mismo sentido, e igual- B
mente distanciadas entre sí. Del ejem- (+) • ••• /¡' (-)
plo de la Fig. 20.11: F c--cffc
(+») ~q
"' /{ (-)

Fig20.11
8) Blindaje Electrostático.-
El hecho de que el campo sea nulo a) ~

rn i1áJJ
6~ ~b
en el interior de un conductor en equili-
brio eléctrico ha permitido investigar
y experimentar otros casos como el de
laFig. 20.12a, en donde una esfera metá- --- - -
lica cargada, al tocar el interior de la
RIGIDEZ DIELECTRICA caja metálica, queda completamente
descargada, de manera que toda su car-
Cuando un dieléctrlco ga queda en la superficie externa de la
(aislante) se ve afeétado b)
por un campo eléctrico caja, provocando asimismo que el cam-
que progresivamente va en po en su interior sea nulo.
aumento, existe un valor
para cada material a partir Así pues, se descubrió que una
del cual un campo mayor cavidad en todo cuerpo conductor-es
lo convertirá en un cuerpo una región eléctricamente aislada, es
conductor. Para el caso del decir, no será perturbada por los efec-
aire, el valor de la rigidez
dieléctrica es de 3.1[J6N/G. os eléctricos externos al conductor. A
Así, un conductor cargado este efecto de aislamiento se le llama
que produce un campo su- «Blindaje electrostático» o jaula de
perior a dicha rigidez se
descargará mediante un Faraday, dado que él pudo experimen- "Las laminillas del electros-
chispazo, dado que el aire tarlo sometiéndose a una gran desear-
copio no se separan"
se habrá convertido en un ga eléctrica exterior que no logró al- '-- ~~_~""""':=_:_"-=-::_:_='
conductor. canzarlo. Fig 20.12
Electrostática 369

TlROBLEMAS RESUELTOS (fRA MRTE)

Probo 1.- Se tienen 2 cuerpos cargados con 2. 10-4 Y 4. 10-4 coulombs, respectivamente. Si la
fuerza de repulsión desarrollada es de 80 N en el aire, la distancia entre dichos cuerpos
es:

A) 2,5m BJ4;Om C) 3,Om O) 3,5 m E) N.A. UNFV 87


Resolución.-

De acuerdo con los datos tenemos: q I = 2.10-4 e, q2 = 4.10-4 e y F = 80 N; Y por tratarse de un medio como
el aire, emplearemos la Ley de Coulomb dada por la relación (20.3), con kc = 9 .109 N.m2/C2. Luego:
__ k . q,·q2 (2.10-4)(4.10-4) 11 __ 9.8.10
F e d2 ~ 80=9.109 d2 ~ a- 80 ~ d=3m RPTA. C


Probo 2.- En las figuras mostradas o, y G2 son
cargas puntuales, y F1 ' F2' F3 son las fuerzas res-
pectivas que una de ellas ejerce sobre la otra en cada sItuación.

o----------------~
q¡. b ~q¡
O----------~
q2~ e--t!2

2
Si se cumple que: F1 = F2 .F3, la relación entre a, b yc debe ser:
A) ab =d B) ea =~ C) be =d O) be = (q/q~ E)ab = (q/q.,J d UNI88-1
Resolución.-
Sin tener en cuenta los fenómenos de atracción y/o repulsión de las cargas, obviaremos los signos de las
cargas para solo concentramos en los módulos de las fuerzas entre ellos, obteniéndose:

F =k q¡·q2 q? qi
I e a2 F2 = ke b2 y F3 = kc ~

be =a2 RPTA.C

Probo 3.- Las dos esferitas de ó gramos de


masa cada una, penden de hilos
de seda de 130 cm de longitud.
Si tienen Igual carga ¿Cuánto es
dicha carga en coulomb?(g = 10
m/s2).

A) 25/9.10-12 O) 25/9. 10-4

B) 5/3.10 -11 E) N.A.

C) 5/3.10.(¡ . .
i---- 1 m ---:
r
370 Física - Primer Nivel Félix Aucallanchi V.

Resolución.-

Reconocemos los siguientes datos:


m = 6 .10-3 kg ~ P = 6 .10-2 ,V (peso)
d = lm y q, = q2 = q (incognita)
Fuerza eléctrica
Luego de completar los datos geométricos en repulsiva
la figura, reconocemos que el triángulo som-
breado es Pitagórico; a continuación hacemos ~F

¡Jp
el D.C.L. de la esferilla de la izquierda, y por
semejanza de triángulos: Triángulo vectorial
y Triángulo sombreado, tendremos la siguien-
te porporcionalidad:
F P F _ í.P _ _:!.1.. í. -2
50cm = 120 cm ~ - 12 ~ kc - d2 . 12 (6.10 )
F

~ 9. lcP.iL
(1)2
= 25.10-3 q = "35 • 10-6 e RPTA.B

Probo 4.- El sistema se mantiene en equilibrio, ¿Cuál es el valor del peso W?,
Al k q1·q2
d2
B)2kQ1.Q2
cf2

C) 4kQ~Q2

O) k Q1·Q2
LA q2
~-----------
2 d2
~d~
E) N.A. UNFV 89-1

Resolución.-

Debemos reconocer que en las alternativas, la T T


constante de Coulomb (k ) que conocemos se
ha indicado por la letra "kf,. A continuación po-
demos notar que la tensión en
de la cuerda que
sostiene a la carga positiva (+ q2) es de acuerdo 2T=W
con el D.C.L. del sistema Bloque + Polea:

T= WI2
Luego, del D.C.L. de + q2' vemos que por
equilibrio en el eje "X" se cumple que:

W ~----------
~ 4=F ~ x
-q\
RPTA.D
Electrostática 371

Probo 5.- Se tiene un triángulo equilátero de lado "a". En cada uno de los vértices de este tri-
ángulo se ubica una carga puntual - q. Si queremos que el sistema permanezca
estático, que carga (magnitud y signo) se debe colocar en el centroide del triángulo.

A) ~ a. positiva O) ~ a. negativa

8)/3 o, positiva E) 3/3 q

C] 3 e; positiva UNI82-2
Resolución.-

Resulta evidente que cada carga - q recibe -q


,
fuerzas F de igual magnitud y de repulsión ,
por parte de las otras dos, las que se ubican ,,
,,
en la prolongación de los lados del triángulo
,,

equilátero, y que por lo tanto forman entre sí
el mismo ángulo: 60°. Asimismo, para evitar x
que cada uno salga despedida en forma radial \a
,
y hacia afuera, se necesita colocar una carga
Q de signo opuesto (positiva) que se encargue ,
de atraerlas a todas por igual con una fuerza ,
F' que equilibre a la resultanteR de las fuerzas ,,
F. Luego, por teoóa de Vectores se sabe que: x

R= JF 2 + F2 +2 FF cos 600 ~ R = J3 F -q

Y por equilibrio: F' = R ~ F' = J3 F


Ahora, reemplazando en (*) cada fuerza por la
relación (20.3), tendremos:

RPTA.A

Probo 6.- Cuatro cargas positivas y negativas,


pero de igual ma9nitud, están
ubicadas en los vertices de un +qQ- ----- -.1 - - - - - - - O+q
cuadrado con centro en el punto 4. ,
,
¿En cuál de los puntos, entre los ,
,,
señalados con los números 1, 2, 3, 4
ó 5, deberíamos colocar una carga 5: '3
+ o. siqueremos que la fuerza sobre 4 ,
,
esta carga tenga la mayor mag-
,
nitud. ,
,
A) Sólo en 1 O) En 3 ó en 5
-qO------i --------O-q
8) Sólo en 2 E)En4
C] En 1 ó en 2
UNI 94-1
372 Ftsica- Primer Nivel Félix Aucallanchi V.

Resolución.-

Analizando el esquema nos damos cuenta que los ___


.l!._... J
+q . - - - . fl..... +q
puntos 1 y 2 quedan descartados, dado que las
fuerzas de repulsión de parte de las cargas laterales ,
,
, ,
se anulan entre sí, ello por la simetría del sistema. , ,
, ,
Luego, la fuerza máxima solo puede presentarse , ,
entre los puntos 3; 4 ó 5. Podemos reconocer que , ,
, ,
los puntos 3 ó 5 son equivalentes por simetría; ,
5: ,
sin embargo, al pretender que la fuerza resultante 3
sea máxima, siendo las cargas las mismas para
cualquier posición elegida, deducimos que ésta
lo será cuando la distancia desde dicho punto a
todas las cargas sea lo mínimo posible, pues como
se sabe la Ley de Coulomb establece que: F (l 1IJ2;
por tanto, entre los puntos 3 ó 4 el que satisface
-q ----------- -q
esta condición es el punto 4. 2

RPTA.E

Observación ..- Se puede demostrar que al colocar la carga + q en 3 Ó en 5, la fuerza resultante sería:
F == 2,18 k q2fa2 .

Probo 7.- Se tiene una barra cubierta con cierto material aislante de la que cuelgan 2 hilos me-
tálicos. De las siguientes alternativas cuál es correcta.
A) Si se acerca una carga Q
positiva, se acercan.
Material aislante
B) SIse acerca una carga Q
positiva, se separan.
C) No se mueven.
Hilos metálicos
D) Se separan sea cual sea el Q
signo de la carga.
E) N.A. PUCP93
Resolución>

Por un fenómeno de inducción, los extremos libres de los hilos se cargan con cargas de signos iguales y
opuestos a Q. Así se justifica una separación entre ellos, cualquiera sea el signo de Q.

RPTA.D
Electrostática 373

Probo 8.- Determinar la distancia x en me- -25J.1C Ü+9J.1C p


tros,si se sabe que el campo eléc-
trico resultante en P es nulo.
0
..
- ------ - - ------ --- -- --- - -- -
...
- - - -- - - - -- - - --~

A) 5 D) 2
B) 4 E) 1 o x ------;0:3-· -- 3m ~
C)3
Resolución.-

De acuer~o co~la regla para dibujar los vectores q¡ =-25 J.1C q2=+9 J.1C
campo (E¡ y E2), procederemos a representar- O---------------------------O--------~
¡ ¡ El : E2
los según como aparecen en el gráfico adjunto. A
continuación establecemos que el campo eléctrico
. .
será nulo en P si y solo sí los vectores campo allí . .
sean de igual módulo y de direcciones opuestas, ~~=3m~
tal como se ha indicado en el gráfico; luego, •
empleando la relación (20.4) tendremos: -€$>-------,--- di = (x+ 3) m~~---~$
- - q¡ q2 25 !lC 9 !lC
fE¡f=fE2f => ke dt =ke di => (x+3)2 =31

x=2m RPTA.D

Probo 9.- Dado el siguiente sistema de cargas, se


pide encontrar la intensidad del campo
(10 ------------------ Q (2)
,
resultante en el punto P (en N/C), sa- , ,
,,
biendo además que la figura es un cua- ,
drado de 3 m de lado, y las cargas son: ,,
a, = -1.4010-8 C, q2 = 1,2.[2.10-8 C, y ,,
,
q3 = -1,2.1o-B C. ,
,
,
A) 10 E) 18 ,
,
,
,
B) 12 D) 16
C) 14
pi ----------~-------- 6 (3)
Resolución.-
y
y

E¡=14

6
E,= 12
6

s; ~"-,::-4S_°c>- -~-q3
E3
(1) (2) (3)
374 Física - Primer Nivel Fé/ix Aucallanchi V. .

Según los datos, procedemos a configurar los vectores campo E¡, E2 YE3' los cuales tendrán módulos que
calcularemos utilizando la relación (20.4):

9 (I,4.IO-s) N 9 (1,2 J2.IO-s) r::: N 9 (1,2. lO-s) N


E¡ = 9.10 2 = 14- ; E2=9.10 c:» =6,,2-; E3=9.10 2 =12-
3 C (3,,2) C 3 C

En el gráfico (2) están representados los vectores campo, y en el cual luego de descomponer a E2 encontramos
las componentes Ex y Ey del campo resultante.

Ex=12-6=6N/C ,y: Ey=14-6=8N/C => ER=J62+82 => ER=lON/C RPTA.A

Prob.l0.-0eterminarladistanciax(enmetros) •. ..
para que la carga "2" se encuen-
tre en la posición mostrada, siendo
o, = - 6 .10-7 c. •
A) OA
B) 0,3

C)5,0
0)4,0

E) 3,0
= N- ----;-------~
(1)

:
'~

.;.--- x ---~
. IEI = 600 N/C

:
'
(2) :
~

ResoluciÓn.-

Como te habrás dado cuenta, en los datos no aparece


el valor ni signo de la carga q2; sin embargo ello no Y
es necesario conocer. Consideremos que su signo es
positivo; entonces, q¡ que es negativo, lo atraerá con T
una fuerza F, y el campo lo rechazará con una fuerza:
q2E, visto en el item 20.9. Sucederá exactamente todo
lo contrario si q2 es de signo negativo. Luego, en base
al D.C.L. indicado, en donde por razones didácticas
F
hemos considerado a q2 como positivo, tendremos
por equilibrio que:
q¡q2
LF = O => F = q2E => kc -2- = q2E P
x

=> 9.109 (6.1~-7) = 600 => x = 3m RPTA. E


x

Probo 11.- Un resorte de material aislante tiene una cons-


tante de elasticidad k = 20 N/cm. Sabiendo
que la esfera se encuentra en equilibrio y
cargada con q = 5.10-4 e, se pide encontrar
la deformación del resorte (en cm), siendo : 30°
el campo uniforme de intensidad E = 6.104
N/e. ----~-~~~---~7
A) 1 0)4 --~-~1'7----7E

B) 2 E) 5
C)3
Electrostática 375

Resolución.-

Al hacer el D.C.L. de la esfera reconocemos que: qE = fuerza


del campo, y kx = fuerza en el resorte; y por equilibrio se
debe cumplir que:
kxfl
2
~ =qE ~ 1(20::) X=(5.1O-4C)(6.104~) qE
x
~ (20::)X=2(5.1O-4C) (6.104~)
p
x=3cm RPTA.C

Probo 12.- Una gota de aceite cargada, con una masa de 104 gramos, se halla estacionaria en
un campo eléctrico
(g = 10 m/s2).
vertical que tiene 200 N/C de intensidad. La carga de la gota es: •
A)5,0.1o-aC B) 4,9. 10.9 C C] 9,B.1[J8 C O) 5,0 .1(J9 C E) Ninguna UNI93 - 2
Resolución.-

De acuerdo con las condiciones del problema podemos ela- qE


borar el esquema adjunto, en donde reconocemos que: qE =
Fuerza del campo, y mg = Fuerza de gravedad (peso), las
mismas que por equilibrio deberán ser iguales. Luego:
q m
(10-7 kg)(lOmls2)
qE=mg q = 200 N le
mg
RPTA.D

Probo 13.- Un cverpo de 2 kg Ycarga q= +500 J1C es dej<:x:kJen liOOrtcxfen un lugar cbnde g = 10 m/s2,
y el campo eléctrico es uniforme, vertical hacia arriba y de intensidad F==6.1Q4 N/e.
¿Cuál es la aceleración en m/s2 que experimenta dicho cuerpo?

A)O B) 2 (Í ) C]5{Í) O) 5 (.1,) E) 6 (.1.)

Resolución.-

Procediendo de un modo similar al problema anterior, podemos


reconocer que la fuerza del campo es: F

~ F = qE = (500.10-6 C) (6 .10-4 N/C) ~ F = 30 N

yel peso: P == mg = (2 kg) (lO mls2) ~ P= 20N

Luego, siendo F > P, existirá una aceleración hacia arriba,


cuyo valor encontraremos aplicando la 2¡!¡¡Ley de Newton:

ma = F-P ~ 2 a = 30 - 20
p

~ a = 5 mls2 (i) RPTA. e


376 Física - Primer Nivel Félíx Aucallanchi V.

BID POTENCIAL ELECT;RICO



a) Concepto de Potencial Eléctrico.- Cuando transportamos 'una
carga por el interior de un campo eléctrico ,desarrollamos un trabajo contra
las fuerzas electrostáticas.Como se recordará del tema deenergía , se sabe
que si un cuerpo recibe trabajo, gana energía .por tal razón es entendible
que al hacer trabajo sobre una carga dentro de un campo, ello se convertirá
en energía .la misma que quedará almacenada poda carga y el campo en
el punto donde ésta se estacione. De este modo se puede reconocer que
cada punto del campo posee ul!a propiedad energética que llamaremos
potencial eléctrico, el cual por su naturaleza escalar permite describir dicho
campo sin recurrir a sus originales aspectos ectoriales.
PARA NO OLVIDAR
b) Potencial Eléctrico Absoluto> El potencial de un punto expresa
En la relacIón (20.8), al la energía que presenta la' unidad de carga puntual y positiva colocada
reem-plazar datos debe en dicho punto. Analicemos el siguiente ejemplo: Si el punto P de la
Incluirseel signo de Q.
Fig. 20.13 tiene un potencial de 50 voltios (= 50 J/C), ello tiene dos
interpretaciones principales:
1) Un agente externo deberá realizar un trabajo de 50 J por cada
coulomb que transporte desde el infinito hasta el punto P.
2) - El campo-eléctrico.desarrollará
un trabajo de 50 J por cada coulomb
cuando lo transporte desde P hasta el
infinito.
El potencial creado por una carga pun-
tual Q a una distancia d viene dado por:

(20.8)

Lá unidad de potencial en el SJ. es el


voltio (V): 1V = 1 J/C. Fig20.13
DEBES SABER QUE:
e) Traslación de una carga dentro de un campo.-Cada vez que
El potencial eléctrico nos enfrentamos al problema de mover una carga dentro de un campo
absoluto de un punto P de eléctrico, debemos saber reconocer cómo se presentan las fuerzas que
un campo Jamblen se de-
fine como el trabajo que participan en el movimiento.Para ello es ilustrativo describir los casos
debe realizar un agente que se muestran en la Fig .20.14, en todos estos ellos se observará que la
externo por cada unidad fuerza que ejerce el agente externo: Fext ,actúa siempre a favor del
de carga puntual y positiva
que debe trasladar desde movimiento, en cambio.todo lo contrario ocurre con la fuerza que ejerce
el InfinIto hasta dIcho el campo: Fcarn o.En todos estos casos se puede apreciar que el trabajo
punto.De este modo la que desarrolla el agente externo es positivo, y el que realiza el campo es
relación que permite su
có/culo viene dada así: negativo.

Caso 1.- Una carga positiva es obligada a acercarse a otra carga positiva

Caso 2.- Una carga negativa es obligada a alejarse de una carga positiva.

Caso 3.- Una carga positiva es obligada a alejarse de una carga negativa

Caso 4.- Una carga negativa es obligada a acercarse a otra carga negativa
Electrostática 377

I ¡ ENERGIA POTENCIAL
ELECTROSTATlCA
(1)
.r., •
Desdeque eltratxJjo realizc::x:b
Q q F
0-------- ---~---~-
P Fccmrpo
por el agente exteroo quedo
almacenado como energía,
llamaremos energía potencial
electrostática, a aquella que
posee un sistema de cargas
como consecuencia de
haberse realizado un trabajo
durante su formación. Su
valor se puede deducir así:
U =w ••Ag.Ext_
Fig 20.14 ~p

Si ahora analizamos los casos mostrados en la Fig .20.15


comprobaremos que en todos ellos la fuerza que ejerce el agente externo: d on cie:
e. W ..~p
Ag.Ext. - V
- p.q
FexI' se aplica en contra del movimiento de la carga, todo lo contrario
y de la relación (20. B), para
ocurre con la fuerza que ejerce el campo: F campo .Por esta razón ,en todos
estos casos, el trabajo que realiza el agente externo es negativo y el trabajo Vp' tendremos:
del campo es positivo.
W Ag.Ext. = k Qq
I •• ~p e d
(a)
~ ~ (U=ke']
Q q F
0-------; ---~--!!I-.
Fcampo

Q (b) ~_ Q (d) ~

0-------; ----~-!~ 0-------· ----~!!--


p Fel11ltpo
campo
TRABAJO DEL AGENTE
Fig 20.15 EXTERNO

d) Trabajo eléctrico. Cuando el traslado de una carga q se hace con Observando todos los ca-
velocidad constante, entonces la fuerza que aplica el agente externo es sosde las Fig. 20.14 Y 20.15,
concluiremos que siempre
igual, pero opuesta a la fuerza que el campo ejerce sobre la misma que transportemos una carga
carga. De este modo podemos asegurar que el trabajo realizado por q dentro de un campo y con
ambos son siempre iguales, pero de signos contrarios. Para efectos de velocidad constante .se cum-
nuestro estudio, el trabajo del campo Wc es el que más nos plirá que el trabajo rea-
lizadopor el campo eléctrico
ínteresa,verficándose que ella depende del potencial eléctrico Vp que y el realizada por el agente
posee el punto P desde donde parte la carga q hacia el infinito, o hacia externo son siempre del mis-
donde llega la carga traída desde el infinito.De este modo el valor del mo valor pero de signos con-
trabajo realizado por el campo viene dado por la siguiente relación: trarios.Luego :

Iwc =q.~p 1 (20.9)


W Ag.Ext.
•• ~p
=-W
e
y de la relación (20.9),

El signo del trabajo Wc ,puede obtenerse a partir del diagrama de fuerzas tendtemos
que participan en el movimiento, o simplemente a partir del resultado
de sustituir los signos de la carga trasladada(q), y del potencial (V p) en W Ag.Ext. =- V
••~p q p
la relación (20.9) .

.•.
378 Física - Primer Nivel Félíx Aucallanchí V.

SUPERFICIES d) Principio de Superposición de Potenciales


EQUIPOTENCIALES
Por el mismo hecho que los campos de varias cargas se superpo-
Son aquellas regiones de nen, se establece que: «El potencial electrostático creado por varias
es-pac/o ubicados en el In- cargas en un punto del campo está dado por la suma escalar de los po-
teriorde un campo eléctrico tenciales creados por cada carga en dicho lugar y de manera indepen-
en donde todos los puntos
que lo conforman poseen el
diente». Del ejemplo de la Fig_ 20.7 se establece que:
mismo potencial eléctrico.
Estassuperficies resultan ser
perpendiculares a las líneas
de fuerza del campo eléc-
I Vt~ =l: V =vt+V 2
P
+ .•. ! .---_(_2_0_.1_0_) --._-, •
trico. e) Tensión Eléctrica
Cuando liberamos una carga pun-
tual q en el interior de un campo pa-
sando del punto A donde el potencial
es VA a otro punto B de potencial V B'
se verifica que el campo habrá realiza-
do un trabajo W;-4B' que vendrá dado
así:
(-)

Fig20.16
En el caso de un campo Y llamamos tensión eléctrica a la diferencia de potencial: VA - VB = VAB.
uniforme, estas superficies Cuando compramos una bateria, o una pila, lo que estamos adquinendo de
sonplanas y paralelas entre ellas es su tensión eléctríca, la misma que se expresa en voltios.
sí.
Observacián.: Resulta evidente que un agente externo para transportar la misma
carga q desde B hasta A efectuará un trabajo WAC->Bidéntico al que realiza el
campo para trasladar la misma carga pero desde A hasta B. Luego:

¡ W:--+A =q(VA -VB) 1 (20.12)

f) Potencial creado por una esfera conductora electrizada


Como ya hemos explicado ante-
riormente, para cálculos exteriores a la
esfera podemos considerar que toda la
carga se encuentra ubicada en su cen-
tro, de modo que el potencial en punto
P de la Fig. 20.17 estará dado por:
INTERESANTE

Al liberar una carga pun-


tual dentro de un campo
(20.13) ¡ Vp =ke ~ 1 <=> d ~ R v
etécmco. ella se moverá de Cuando analizamos el interior de la k"Q
las zonas de alto potencIal R
hacia las zonas de boja po- esfera ~nc-ºntramos que el campo es
tencial. Este es el prIncipio nulo (E = O), y por lo tanto el trabajo
físico que justifica el movi- que hacen las fuerzas eléctricas para
miento de cargas en un
alambre conectado a una trasladar cualquier carga entre do.s pun-
pila eléctrica: "Las cargas
van del polo posItivo (+) tos A y B es cero: W;--+B= O. Luego, '-- R_-=:-:--=d-:-:::!
hacia e/ p% negativo (-)". porla relación (20.11) encontramos que Fig 20.17
Electrostática 379

la diferencia de potencial entre los puntos A y B es también nulo, o 10 ATENeION!!


que es lo mismo: VA =VB" Esto demuestra que el interior de todo con-
Reuniendo las relaciones
ductor es un volumen equipotencial. . (20.4] y (20.13] se puede
demostrar que para un
g) Relación entre campo y potencial campo (E] y potencial (V]
en un punto P del campo
Si nos fijamos bien en el cam- creados por una carga
po uniforme de la Fig, 20.18, podemos puntual se cumple que:
reconocer que la intensidad de campo
E y la distanciad entre las superficies ~ ~
u
I
o •
I
equipotenciales V A Y VB (VA> Ve) es-
tán relacionadas entre sí del siguiente
.- tJ tJ •
Al donde "d' es la distanciade
.- o •..
I

modo: o la carga hasta el punto P.


I lB
l,....v~A~.""".V~.B--,~="""E~.d-"'.!
(20.14) • P P .-
• e!l
I
o
d--é-
o •
I

E
donde A y B no están necesariamente I I

en una misma línea de fuerza. Fig20.18

PROBLEMAS RESUELTOS (21M NR7t)

Probo 14.- En un cuadrante de un círculo se ubican:

q1 = + 80 1tEo e;
q2 = - 20 1tEo e;
q3 = - 40 nEo e ,
,,
como indica la figura adjunta. Calcular el ,,
po-tencial eléctrico en el punto P.(Eo es la ,
,
,
constante dieléctrica).

A) 1 V/m B) 2 V/m C) 3 V/m


p r-'~\ ~5_o_- - - - - - - - - - - - - -6 q3
6 Sm 6
0)4 V/m E) 5 V/m

UNI92
Resolución.-
TaLcomo se indicó en el item 20.8, la constante de Coulomb está dado por la expresión general:

l
ke = 41tE (para el vacío)
o
Luego, utilizando los datos y la relación (20.8) para el potencial creado por una carga puntual y de acuerdo
con el Principio de Superposición de Potenciales, se tendrá que el potencial total solicitado en P está dado
por:

T 1 2
q
VP=VP+VP+VP=k...!.+k..1.+k..l=k
3 eR e
q
R e
q
R e
[q 123+ q + q
R
1
380 Física - Primer Nivel Félix Aucal/anchi V.

=> => v: = 1 V/m RPTA.A

Probo 15.- Trescargos puntuales positNas {+ q] y tres


cargos puntuales negatNas (- q) se ubican
en los vértices de un hexágono regular de
lodo o, como se indica en lo figuro. ¿Qué
trabajo es necesario realizar pora traer
uno cargo - 2q desde un punto muy dis- -q +q
tante y coiocona en el centro del hexá-
gono? k = q2/4 n€o .
A]-2J3 k B]2./2 k C]-k

O] 2 k E] O
UNI91
Resolución.-

De acuerdo con las observaciones hechas en el item 20.10, el trabajo solicitado para traer a la carga (- 2q)
desde el infinito y colocarla en el centro del hexágono estará dado así:

w;~ e = (- 2q) Ve ; donde Ve = Potencial total en el centro (C) del hexágono.

y de acuerdo con el Principio de Superposición de Potenciales encontraremos el potencial Ve

=> V
e
= l:V=+3k !L-3k
eR
!L
eR

Por ello: W;~e = (- 2q) (O) => w~~c = O RPTA.E

Probo 16.- Dos cargos puntuales de 5 microcoulomb y 2 microcoulomb respectivamente, están se-
parados uno distancio de 5 cm. Lo energía electrostática de este sistema es:

_1_ = 9 x 109 NrrfC-2


4n€o
A] 1,8 J B] 0,9J C] 3,6J O] 4,5J E] N.A.

Resolución.-

Debemos recordar que todo sistema físico conservativo al recibir trabajo lo almacena como energía; éste es
el caso de los sistemas formados por cargas electrostáticas. Pues bien, la energía que almacenan dos cargas
q¡ y q2 será igual al trabajo que se debe realizar para trasladar, digamos a q2 y sin acelerarlo, hasta una dis-
tancia d de la carga fija q¡, donde ella genera un potencial Vp' Luego, de acuerdo con la relación 20.9 ten-
dremos:

:P
=> +---d---4 ao

u= 1,8} RPTA.A

/
Electrostática 381

Probo 17.- En cierto campo eléctrico el punto


A está a un potencial de 50 von. yel
punto a está 075 volt. Para mover
una carga de 10-3 coulombios de A
hasta a se requiere un trabajo de: .•
E
;··
..
l B

A) 50 ergios ~ ---::-- 3 m

a) 50joules -----AT .----7--...1.


C) 25.10-3 joules
D) 50.10-3 joules
E) 75.10-3 joules UNMSM94

Resolución.-

De acuerdo con lo explicado en el item 20. I Oe, el trabajo realizado por una agente externo para trasladar una
carga q dentro de un campo eléctrico y sin acelerarlo depende solo de los potenciales de partida y de llegada,
y ello estará dado por la relación 20.12.

RPTA.C
"

Probo 18.- La figura representa algunas figuras equpotenciales de un campo electrostáticb, y los
valores de los potenciales correspondientes. Determine el trabajo realizado para
llevar una carga negativa q = - 2.10-6 e del punto A al punto a.

A) 6.10-5 J +20Y +10Y -lOY -20Y


'O
a) - 6.10-5 J A', , '\ ,
,, ,, r ,

C) Depende del camino seguido.


,
, , . ,
,
,
r
r
'

,
,, ,,, ,,
, '
'
'

D) 4.10-5 J
r
, ,
, ,
,
.
, '
, ''
,, '"-
E) -4.10-5 J ,
/
r
r
, ,
",,,,' .;,1
\B'"
UNI94-2

Resolución.-

De acuerdo con el esquema dado podemos reconocer los potenciales de A y de B: VA = + 20 V, Y v: = - 10 V.


Luego, el trabajo que debe realizar un agente externo para trasladar una carga q = - 2.10-6 desJe A hasta e
B, y sin acelerarlo, está dado por la relación (20.12).

W:~B =q(VB-VA)= (-2.1O-6C) [(-IOV)-(+20V)] ~ W:~B=+6.10-SJ

RPTA.A

Probo 19.- La figura muestra tresplacas conductoras paralelas con susrespectivos poten-ciales.
Luego, con respecto a los campos eléctricos El y EII' medidos en vcittm. es correcto
afirmar que:
382 Física - Primer Nivel Félix Aucallanchi V.

A) El = 1; Eu = 2 SV 8V 12V

B) El = 200; Eu = 100

C) El = 100; Eu = 200 ~ ~ TI
O) EI= Eu = 115
~2cm~3cm--4
E) El = Eu = 100 UNI 93 - 2
Resolución.-

De los datos y de la explicación dada en el item


10.IOg podemos decir que los campos eléctricos
en las regiones 1y 11son uniformes, y sus módulos
estarán vinculados con las superficies equipoten- e B A
ciales por medio de la relación (20.14). Así pues:

Región 1: El' di = (VB - Ve)


~ El' (3 .10-2 m) = (8 V - 5 V)

=> El = 100 V/m


Región 11: Eu' du = (VA - VB) Eu' (2.10-2 m) = (12 V - 8 V)

~ En=200V/m RPTA. e
Probo 20.- Una esfera conductora de 20 cm de radio posee una carga de 106. 10-7 C. A 304,8 m de
distancia se coloca una estera conductora de 15 cm de radio y descargada. SIse conectan
por medio de un alambre conductor ¿Cuál será la nueva carga de la primera esfera?
A) 61.10-7C B) 60.1O-7C C) 45.1O-7C O) 58.10-7C E) 63.10-7C
Resolución.-
+--- d -----t
Sea q = 106.1O-7C, la carga inicial del sistema la misma que
solo lo posee la primera esfera. Sean q F Y q las cargas que
finalmente tendrán las esfereas (1) y (21, venl1cándose que la
carga total antes y después del suceso es la msma : .
--o 2
:,.. interruptor
q = qlF + q2F (1)
A continuación, diremos que luego de cerrar el interruptor la cargas eléctricas se
redistribuyen hasta que ambas esferas alcancen el mismo potencial eléctrico, así: F F
VI = V2 (*)

Pero debemos tener en cuenta que cada potencial mencionado se determina en base a la siguiente relación:
Potencial de un conductor en (X) = Potencial propio del conductor + Potencial del lugar (X)
F F
Vesf.(1) + Vinducido = Vesf.( 2) + V inducido
(2) en (1) (1) en (2)

y utilizando las fórmulas (33.9) y (33.20 tendremos:

qlF q2F q2F qlF qlF rl (d+rl)


kc -;:;- +k¿ (d+rl+r2) =ke' -;:; +k¿ (d+rl+r2) ......... (2)
q2F = r2 (d+r2)

Finalmente de (1) y (2) : RPTA.A


Electrostática 383

20MA AUTOEVALUACIÓN
~ '" v "

1.- Indicar la proposición incorrecta: Indicar verdadero (V) o falso (F).


A) En un átomo neutro, el número de electrones es A) VVF B) FVF C) FFV D) VVV E) FFF
igual al de protones.
B) La carga más pequeña del universo lo tiene el 5.- En base al ejercicio anterior y a la disposición
electrón. mostrada en la figura, procedemos a cerrar el inte-
rruptor S. Entonces:
C) Los conductores presentan electrones libres.
D) El aire seco es un aislante. 1) Bajan electrones hacia Tierra.
E) Dos cuerpos que se rechazan eléctricamente ne- Il) El conductor M queda con carga positiva.
cesariamente tienen cargas positivas. UI) Si alejamos el inductor, M se descarga.
2.- En relación a la Ley de Coulomb se afirma que: Señalar la(s) correcta(s):
( ) Solo se cumple en el vacío. A) 1 C) III D) 1 Y III E) Todas
B) "
( ) Es válido para todo cuerpo cargado.
6.- Se tienen cinco pequeñas esferas conductoras
( ) La c~mstante k. tiene un valor que depende del iguales y descargadas. Una de ellas se carga eléctri-
medio, camente con una carga q; luego, el resto de esferas
Indicar verdadero (V) o falso (F). se ponen en contacto de una en una con la primera.
Entonces, la carga eléctrica final de la primera esfera
A) FFV B) FVF C) FVV D) VVV E) FFF será ....
3.- Una esfera conductora cargada positivamente se A) q!2 B) q/4 C) q/8 D) q/16 E) q/32
conecta a Tierra mediante un cable metálico. En-
tonces: 7.- En un experimento se obtienen las siguientes
medidas:
A) La Tierra gana electrones. ql = 8.10.19 e, q2 = 3,6.10.19 e,
B) La esfera pierde protones. q3 = 9,6.10.19 e y q4 = 2,4.10.19 C.
C) La Tierra gana protones.
¿Cuáles de estas cargas están de acuerdo con la teoría
D) La esfera gana electrones. de la cuantización?
E) Todas son falsas.
B) ql Y q3
4.- En relación al siguiente experimento, el conduc-
tor M se encuentra inicialmente descargado: E) Todas

8.- Elige las palabras para completar correcta-mente


'. la oración: «Las líneas del campo eléctrico son ----
------------ a la superficie de los conductores y
A M B
,r- Conductor también a las superficies ----------------».

A) Paralelas, dieléctricas.
B) Perpendiculares, di eléctricas.
C) Perpendiculares, equipotenciales
D) Paralelas, equipotenciales.
E) Tangentes, ásperas.
) El sector A se carga positivamente.
9.- En el siguiente esquema, M y N son cuerpos
) El sector B se carga negati vamente. conductores. Inicialmente N tiene carga positiva y
M está descargado. Luego, es falso que:
) El campo en el interior de M es nulo.
384 Física - Primer Nivel Félix Aucallanchi V

12.- De las proposiciones:


B
1) Las superficies equipotenciales son perpendicu-
++
lares a las líneas de fuerza.

.0-
- N
+
+ 11) Al trasladar una carga entre dos puntos de una
misma superficie equipotencial, el trabajo eléc-
trico es nulo.
IlI) Al mover una carga en una trayectoria cerrada
dentro de un campo, el trabajo de la fuerza eléc-
trica es nula.
A) En A se induce carga positiva.
IV) El trabajo del agente externo y del campo son
B) En B se presenta una carga positiva igual al quc iguales si el movimiento de la carga se hace con
posee N. ';elocidad constante. ~
C) En el interior de M el campo es nulo. Indica cuantas afirmaciones son correctas:
D) El potencial en M es uniforme.
A) 1 B) 2 C) 3 D) 4 E) Ninguna
E) Si N toca a M, se descarga completamente.
13.- Para el esquema mostrado es cierto que:
10.- Dos esferas conductoras A y B se encuentran
inicialmente cargadas con -20 e y +8 e respectiva-
mente. Luego de ponerse en contacto procedemos a
A) Ep = O , Vp *O P
1,\
separarlos. Entonces: B) ER = O , VR = O i:
/a,~
/ : \
) Habrá un flujo de cargas hasta que los potencia- C) s; * O , Vp = O / : \
les se igualen. / : .\
D) ER =O VR = O / '
, \
) "A" perderá electrones. \
/ '
( ) "B" queda con carga negativa. E) e; = O V*O
p /
/ '
:
\
\

Indicar verdadero (V) o falso (F) +q~---~--~ce-q


R
A) VFF B) VFV C) VVV D) VVF E) FFF

11.- El esquema muestra las líneas de fuerza de Para reflexionar!


un campo eléctrico. Luego, es cierto que: Se tiene do esferas conductoras del mismo radio,
una con carga q y la otra descargada. A continuación
ve ponen en contacto y finalmente se separan. Se
estima que las esferas adquieren cargas iguales a q/2.
Ahora nos proponemos un valor de q. Si q = 13
A~ electrones, ¿Qué carga tiene cada e fcra",
--~--~--~--------~~B

A) EA> EB

B) VA < VB
C) q se mueve de B hacia A.

D) EA = EB

E) VA> VB
Electrostática 385

NIVEL 1 A) 20N

01.- Se tiene dos cargas puntuales de 16¡.tC y 40¡.tC; B)40N


separados por una distancia de 10 cm. ¿Con qué
fuerza (en N) se rechazan? C) 50N

A) 576 B) 144
D) 60 N
C)36 D) 1,4 E) 3,6
E) 80N
02.- Dos cuerpos cargados que están separados una
distanciade IOcm, se repelen con una fuerza de 5,4N.
Si uno de los cuerpos está cargado con 0,3 ¡.tC. De- 08.- Hallar la fuerza total (en N ) que soporta la
terminar el valor de la carga (en ¡.tC) del otro cuerpo. cargaq3;siq¡==+40¡.tC; q2=-40¡.tC y q3=+I¡.tC

A) 10 B)20 C) 30 D) 40 E) 50 q¡ q2 q3

03.- Dos cargas se repelen con una fuerza de 16 N


¿Con qué fuerza se repelerán cuando su distancia de
separación se cuadruplíque?
ct·~·~·;~·=St·~~·~:·~
A)9 B) 27 C) 36 D) 45 E) 50
A) 1 N B) 2 N C) 3N D) 4N E) 8 N
09.- Calcular la intensidad de campo eléctrico (en
04.- Dos cargas iguales que distan 30 mm, están si- kN/C) de una carga de 8 nC a una distancia de 6cm.
tuados en el aire y se rechazan con una fuerza de 360
N. Calcular el valor de las cargas (en ¡.tC) A) 10 B)20 C) 30 D) 40 E) 50
A)36 B)6 C)90 D) 60 E)3 10.- Determinar la intensidad de campo eléctrico (en
kN/C) resultante en el punto "A" si Q = +45 ¡.tC
05.- La fuerza de atracción entre dos cargas es 30 N.
¿Cuál será el nuevo valor de la fuerza (en N) si una se A) 5 Q ,
duplica, la otra se triplica y la distancia se reduce a la ,,, -,,
,,
mitad? B) 45./2 ,, ,
,, ,
, ,
,-,
A) 240 B) 560 C) 360 D) 720 E) 450 C) 4,5./2 3m ¡ ,,
,, ,,
,, ,,
06.- En la figura las esferitas A y B tienen cargas de ,,
,,
D)9./2 -, ,,
igual magnitud y de signos contrarios.Si "B" está en
equilibrio y su masa es 10 gramos, hállese la carga
(en ¡.tC) (g = 10 mls2)
E) 225 A D.................::'
3m
Q

A)4 11.- Localizar un punto (en cm) a partir de la carga


de la izquierda, en la recta que los une, donde la
B)3 intensidad de campo es nulo d = 50 cm.

C)2
4Q 9Q
D) I ......................
;¡ <)
E)6
A) 10 B)20 C)30 D)40 E) 50
07.- Las esferas iguales pesan 10"N Y tienen cargas
q == 60 ¡.tC, si d == 90 cm. Determinar la tensión 12.- Determinar la tensión en la cuerda, si se sabe
(en N) de la cuerda. que el sistema está en equilibrio. Las esferas tienen
386 Física-Primer nivel Félix Aucallanchi V.

masas iguales m = 3 kg, Y cargas de signos opues- 17.- Se tienen dos cargas eléctricas Q y -2Q separa-
tos q = 6 ¡.te; además el campo uniforme tiene inten- dos una distancia de 30clII. Hallar a qué distancia (en
sidad E = 5.106 N/e cm) de" Q" en la línea que los une el potencial es nulo.

:J:
A) 120 N A)5 B) 10 C)9 O) 12 E) 15

18.- ¿Qué valor debe tener Q3 para que el potencial


B) 100 N
,T

~tl
resultante en A sea cero? (Q, = 2 e y Q2 = -8 C)
e) 60N ' x
E A)4 e A
O) 90 N B) 5 e
E) 40N
C) -6 e
13.- Una esferita de masa m = 10 g está suspendida O) 7 e
de un hilo de seda, dentro de un campo eléctrico
E = 25 kN/C. Determinar el valor de la carga" q" E) -1 e
(en ¡..te). (g = 10 m/s2)

A)2 19.- Si en condiciones de equilibrio se traslada una


e
carga de q = +2 m desde el infinito hasta el punto
B)4 P determine el trabajo efectuado. Si : Q, = +6 ¡.te ,
Q2 = -15 ¡.te y Q) = +9 ¡.te
C)5
A) 270 J Q2 Q3
0)6 ... ..._~ ....
B) 18 J
E)8 ------------~C>
14.- Enlos vértices de un triángulo rectángulo se C) 27 J 30 cm
han colocado dos cargas eléctricas de magnitud
Q, = -125.10.8 e y Q1 = +27.10.8 C. Separados una O) 72 J
distancia de 0,4 m como muestra la figura. Determi-
nar la intensidad del campo eléctrico resultante E) 135 J
6L...... 40 cm
._---_.
,-----.-- ------_.-.- ..
p
(en kN le) en el vértice A. Q.

A) 270 Q. 20.- En cada vértice de una cuadrado de lado 2 m


se coloca una carga puntual de Q = +5 11 C. Hallar el
B)300 potencial (en V) en la intersección de las diagonales.
4m¡
C) 360
A) 80 2 B) 180 2 C) 80 O) 180 E) 90
O) 450
._?~o6 . 21.- Sabiendo que la di ferencia de potencial en-
Q2 A
E) 720 tre B y A es VA - V B = 600 V. Calcular la distancia
x (Q = 5 ¡.tc).
15.- Hallar el trabajo (en 1) requerido para acercar
entre si dos protones a una distancia de 2,56.10.30 m A) 20 m
desde una separación infinita.
B) 30 m
A)9 B) 27 C) 45 D)75 E) 90
Q
16.- Se tienen dos cargas puntuales Q, = 16 «e y
Q, = 9 11 e, separados por una distancia de 70 cm. O) 50 m
Cálcular el potencial eléctrico resultante (en V) en el 50 =>.
punto donde la intensidad de campo eléctrico es nulo. ····B
E) 75 m
A) 270· B) 360 C) 500 O) 630 E)80
Electrostática 3'b1

NIVEL 2 26.- La figura muestra dos esferas cargadas y ubica-


das en los vértices A y B de un triángulo ABC.
22.- Si cada carga pesa mg, hallar el valor de" q" para Calcular el valor de la carga B para que la intensidad
que el sistema esté en equilibrio (considerar superfi- de campo resultante en C sea horizontal, BC = 6 l1l
cies lisas). k = Constante de Coulomb. Y la carga en A es 125 ~c.

A) Jmglk A) -20 ~e
C
B) 220 ~e
B) J2mg 1k """,,"~ <, """'"
C) 216 ~e
C) Jmgl2k 1m 'i7~ J9.~'
O) 20 ~C
~ A B
0)2Jmglk ~
E) -400 ~e
E) (1/2) Jmg 1 k 27.- En la figura se muestra un ascensor que sube
con una aceleración constante a = 0,2 mls2. En el
techo se encuentra suspendida una esferita de masa
23.- Determinar la fuerza resultante sobre la carga
m = 60 g Y carga q = +200 ~e dentro de un campo
q3 = 5 ~c. Si ql = q2 = 80 ~e y d = 30 cm eléctrico homogéneo E = 3 kNIC. Determinar el án-
(2) gulo "0"
A) 24 N

B) 36 N
~C) A) 30°

B) 37°
C) 42 N

0)48 N
C)45° al
O) 53°
E) 54 N C>

24.- En la figura se muestran dos cargas eléctricas


q = 40 ~e y q; = 30 ~c. Hallar el valor de la carga Q 28.- El potencial que genera una carga a una distancia
p~ra mantener-en equilibrio al bloque de 7,5 kg. Des- determinada es 500 V ¿Cuál será el potencial que
preciar el peso de las cargas (g = 10 mls2) genera a una distancia diez veces mayor?

~e

º6
A)2 A) 70V B) 30V C) 50V O) 80V E) N.A.
,,/ ,
B) 3 ~e ,/" ',15 cm 29.- El trabajo realizado por una fuerza externa en
"m -, trasladar una carga q = 3 e de B hasta A es 90 J.
C)4~e Entonces, la diferencia de potencial VB -VA es .
O) 5 ~e O:~S;~o____ _~~_o¡~'O A) -36 V B) -30V C) -25V O) -40V E) -35 V
(1) (2)
E) 6~e 30.- El potencial eléctrico a una cierta distancia de
una carga puntual es de 600 V Y la intensidad de
25.- La figura muestra campo eléctrico es de 200 Nle ¿Qué valor tiene la
una esfera con carga carga (en ~e) ?
q = /0-4 C. Si la tensión
en el hilo de seda es A)2 B) 0,2 C) 0,3 D) 0,2 E) 0,3
20 N. Determinar la
intensidad de campo 31.- Se tiene 27 gotas de mercurio esféricas e idénti-
eléctrico homogéneo en cas cada una con un potencial de 5 V, al juntarlas
kNIC. para formar una gota mayor, también esférica ¿Cuál
será su potencial en voltios?
A)50 B) 100 C) 200 D) 400 E) 600
A) 50 B) 45 C) 40 O) 35 E) 30
388 Flstca-Primer nivel Félix Aucallanchi V.

NIVEL 3 36.- Calcular el trabajo realizado(en 1) para trans-


portar una carga de prueba q = 4. l 0-4 desde "A"
32.- Una partícula de masa m = 32 g Y carga q = +20 hasta "B", siendo Q, = +6¡.t ey Q2 = -4 Jl e
¡.tC, ingresa con una velocidad v = 2,5 mis en un
x
campo eléctrico uniforme E = 32 kN/C tal como se
muestra en la figura. Determinar el alcance horizontal
"x" si h = 6 km y g = 10 mls2
ct-.;.~ ;.~ _;~Q
Q¡ A B Q2

A) 25111
A) 900 B) 90 C) 9 O) -900 E)-9
B) 50 m

C) 80 m 37.- Hallar la diferencia de potencial entre las dos


, placas separadas de 5 cm, en donde está suspendida
O) 100 m \ una pequeña esfera de 3 Jl C de carga y 17,6 m N de
f· .·· x- · j
peso.
E) l 000111
A) 100 V
33.- Determinar la carga de la esferita B, si ambas
esferitas se encuentran en equilibrio suspendidos por
B) 110 V
hilos aislantes y sometidas a la acción del campo
eléctrico de intensidad E = 6.105 N/C
C) 200 V
A) 6¡.tC +
O) 220 V
B) 16¡.tC 1--------<[>

C) -8 ¡.tC E E) 250 V
1--------<[>

O) 18 ¡.tC
38.- Se tiene tres cargas eléctricas q = 311 e, coloca-
A 30cm B das en los vértices de un triángulo equilátero de 1111
E) 10 ¡.te -<ll---=-='------==----
de lado. Calcular el trabajo necesario para transpor-
34.- Sabiendo que el sistema se encuentra en equili- tar desde el infinito hasta el centro del triángulo a
brio, determinar la deformación en el resorte plástico velocidad constante una carga q = 3 C
(k = 15 Nrcm), sabiendo que 111 = 4 kg, q = +60 ¡.t C
y E= 5.105 toc A) 1241 B) 234 N C) 243 N

A)5C/Il 0)324 N E)214 N


B)4c/Il
39.- Una carga puntual es trasladada desde" A"
C) 3 CII! hasta "M" por un agente externo. Se sabe que
QB = -12 e , QD = +40 e y que el trabajo realiza-
O) 2 CII! do por dicho agente es 36 k1 ¿Cuál es el valor y
signo de la carga "q"?
E) l CIII

A) 4 Jle
35.- Calcular el trabajo que se debe realizar, sobre una B ······\······N C
carga de 20 ¡.tC para moverla desde "A" hasta "O" B) -3 JlC
siguiendo la trayectoria A BCO frente a una carga t
,r

BO/A
Q = 800 ¡.tC en reposo C) -2 f.le

A) 3 1 O) 241
6m
O) l ¡.te ,g;/
B) 6 1 E) 361
/3m E) 5 ¡.te
A _/....
" .
8m
.
·····OD
C , '
/0
,, ''
C) 121 '. /3m
Q 'O'
Capacidad eléctrica

OBJETIVOS
1.- Conocer la propiedad denominada ea-
pacitancia de los cond uctores, y que
consiste en almacenar cargas eléctricas.

2.- Estudiar las distintas formas de capa-


citores con o sin dieléctricos, así como
el modo de acoplarlas: En serie y en
paralelo.

uchos acontecimientos personales o públicos han quedado


grabados en fotografías, las que utilizaron un poderoso tlash
para mejorar la iluminación durante la breve exposición de la i-
mágen que se quiso grabar. Este resplandor que proviene de la cámara se
hace posible porque un diminuto dispositivo eléctrico pudo almacenar gran
cantidad de carga proveniente de una batería casera, y que se llama capaci-
tar, muy empleado también en los dispositivos de encedido de máquinas, BENJAMIN FRANKLlN
motores, beepers, celulares, computadoras, radio receptores, televisores, ...,etc.
(1706-1790)
CONCEPTO DE CAPACIDAD ELECTRICA
Según lo visto en el capítulo anterior, sabemos que cuando propor- Fué el primer científico
nacido en América (EEUU).
cionamos una determinada carga a un conductor, comprobamos que su Provino de una familia
potencial aumenta si dicha carga es positiva, y disminuye si esa carga es modesta; fué al colegio
r negativa; pero todo este proceso no es al azar, sino por el contrario, obedece solo hasta el primer grado
a una propiedad que manifiesta el mismo conductor. Ante ésto surge la idea de primaria. Se constituyó
en un consumado ptntot.
de introducir una nueva magnitud física que es propia para cada conductor, escntor. político, diplomá-
a la que llamaremos capacidad.o.capacitancia eléctrica, la cual nos ttco. inventor, filósofo y
señalará la proporción en que varía el potencial de dicho cuerpo cuando científico. Estudióla electrI-
gana o pierde carga eléctrica. cidad estática, dió la pri-
mera explicación científi-
CAPACIDAD ELECTRICA DE UN ca del funcionamiento de
la botella de Leyden,
CONDUCTOR AISLADO investigó la electricidad
atmosférica e inventó el
Cuando un conductor está libre de la intluencia eléctrica de otros pararrayos, dió el nombre
cuerpos, se dice que se encuentra aislado. Bajo estas condiciones se de- de carga positiva a la que
[itte COI/lO capacidad del conductor a aquella magnitud fisica escalar adquiere el vidrio al ser
frotado. Asimismo,hizo im-
que /lOS indica la cantidad de carga eléctrica que debe ganar o perder el portantes descubrimien-
conductor para elevar o disminuir su potencial el! una unidad. Por ello, tos en los campos de la
su valor se determina así: medicina, fertilizantes,
tooocrotk: y luminiscencia
. Carga oceanica.
CapacIdad = Potencial

/
390 Física - Primer Nivel Félíx Aucallanchí V.

IMPORTANTE

El potencial V de cual-
quier conductor se mide
=>
Ic=~1 (21.1)
(+)

siempre con relación a En el S.I. fa capacidad se mide en


Tierra,el que como se sabe faradio (F), de modo que: l Faradio
tiene potencial cero.
= 1 coulomb/voltio. Por ser elfara-
/'T é_--o
+ + + +
++
+
L-)V

Conductor (Ti )
dio una unidad muy grande, más se aislado erra--=-
usa el microfaradio: 1 flF = 10-6 F. Fig .21-1
riIID CAPACIDAD ELECTRICA DE UNA
r!I!B ESFERA CONDUCTORA
PARA RECORDAR Un minucioso estudio de la ca-
Dado que la carga Q y el pacidad que poseen los conductores
potencial V de un conduc- nos permitirá descubrir que ella de-
tor son entre sí directamen- pende fundamentalmente de la forma y
te proporcionales, al hacer tamaño de éstos, independientemente
una gráfica Carga -vs- Poten-
cial, se obtiene una recta de estar o no cargados. Una muestra
cuya pendiente es la propia de esta propiedad lo constituye la esfera
ca-pacidad C del conduc- conductora de la Fig. 21.2, en donde la
tor. capacidad eléctrica solo depende del
Q radio de aquella. Veamos:

Q Q ~"R Fig.21.2
Cesf = V = Q => CesF. k.. (21.2)
esf k - .e
Qo -----
: tg9=C
eR
I
I
I
11II CAPACITORES ELECTRICOS
I Llamaremos capacito res o condensadores eléctricos a aquel
par de conductores que manifiestan una propiedad capacitiva al encontrar-
se uno cerca del otro. Se caracterizan
PARA NO OLVIDAR porque al cargarse lo hacen de manera
... que cada uno presenta la misma carga + -", ..
Los capacltores se repre- +~+ .........
-l'

.--.:';..•-Q
sentan simbólicamente pero de signos diferentes, surgiendo
# + - .....
por dos barras paralelas. entre ellos un campo eléctrico que llena ..., ".::'.+ : ":;:.;"
el espacio que los separa, y debido al + -

-11- potencial propio de cada conductor


existe entre los dos una diferencia de
potencial, comprobándose que el mó-
A B

dulo de la carga (Q) de cualquiera de


ellos es directamente proporcional con
DEBES SABER QUE: la diferencia de potencial (V AB )
existente. Así pues: ,
Una batería es un acumu- ,
,
lador de energía química ,
que suministra entre sus ,
, ,
bor-nes una diferencia de +:--_.,;"
po-tenclal permanente. ,
l .....
,
- ....
¡

Se simboliza así: (21.3)


.. ~
-IIIII~ siendo C la capacidad del condensador (capacitor).
Fig.21.3
Capacidad Eléctrica 391

DA CAPACITOR DE PLACAS PLANAS


DENSIDAD SUPERFICIAL
DE CARGA
f!IIJ y PARALELAS
Se define como lo contt-
Así como un conductor aislado tiene una capacidad que depende dad de cargo que existe
básicamente de su forma y tamaño (ver item 21.3), los capacitores tie- por codo unidad de área
nen también una capacidad que depende de su forma y de sus dimensio- en uno superficie determi·
nodo. Enel S.I.,estamagni-
nes; ésto lo explicaremos en base al capacitar de la Fig. 21.4. tud se expreso en cou-
12) Si aumentamos el área común (A) tomottnetto cuadrado.
de las placas, podemos alma-cenar
más carga en el capacitor, y así
aumentamos su capacidad (Co)'
2º) Si disminuímos la distancia (d) en- (a) = C/m2
tre placas se incrementa la induc- +d
ción de cargas, y con ello aumenta -t- •
la capacidad del capacitar. Luego: "--------'-Q
A ~A] OJO!
Co a. d ~ ~ (21.4) Fig. 21.4
En el condensador de lo
donde Eo es una constante física llamada remlitividad eléctrica del Flg. 21.4 se ha generado
vacío. En el S.l. su valor es: Eo = 8,85.10-1 Flm. un campo eléctrico unifor-

m CAPACITORES CON DIELECTRICOS


Cuando introducimos un dieléctrico entre las placas de un capaci-
me por causo de uno distri-
bución igualmente unttor-
me de los cargos en los
placas. Suvolar en el vacío
viene dado así :
tor cargado, el die1éctrico se polariza (ver item 20.6), lo cual provoca una
re-ducción en la carga original de las placas. Este efecto se aprovecha en
agregar más carga al capacitar, lo que se interpreta como un aumento de su
capacidad. Así, si Co y Cd son las capacidades del capacitar en vacío y con
dieléctrico respectivamente, se verifica que: Cd> Co' lo cual permite definir ao: Densidad superficial de
un número adimensional llamado constante del dieléctrico (K ), tal que: cargo de uno de los
placas y sin signo.

(21.5) t le= ~ I donde: K ~ 1

(a) Capacitor conectado a la Bateria (b) Capacitor desconectado de la Baterla


y+Qo I ~+Qo CUIDADO!!
+++ ++++++++ 1+ + + + + + + + + + ++1 Cuando los capacitores
están ocupados totalmen-
te por un dleléctrlco de
constante K, el campo
Baterla eléctrico se ve sensible-
mente alterado:
..• ----------- 1------- - - -I Coso (A).- Capacitar co-
-!lo -Qo
nectado o lo boterla:
Capacidad CI = JeCo Capacidad C¡=JeCo
Ed=K Eo
Yoltaje _ Yd = Yo 'Yoltaje Yd ~ YolIc
Carga Neta Qd = JeOo Carga Neta Q,,=Qo Coso(B).-Capacitar desco-
1 Carga Inducida: Q, = (K 1)00 Carga Inducida: (1(-1)
Q¡=l-¡cQo
nectada de lo botería.
*) 00= Carga del Capacitor en Yacío
Fig21.5
r
392 Física-Primer Nivel Félix Aucallanchi V.

Constantes
Oieléctricas mENERGIA EN UN CAPACITOR CARGADO (U)
MorenO! ~ Como sabemos, el hecho de cargar un condensador implica reali-
Vacío 1,CXJOOzar un trabajo contra las fuerzas de re ulsión de las ro ias car as a
instaladas. Este trabajo se convierte
Aire seco 1,1XXJ6
en energía que el capacitor lo alma-
Aceite de silicio 2,5 cena entre sus placas bajo la forma de
Aceite de 4,0 campo eléctrico. Esto justifica que el
transformador foco de la Fig. 21.6, al cerrar y abrir el
Agua pura 80,0 interruptor S se encienda y se apague
Mica 7.0 intermitentemente. Esta energía viene
dadapo_r: ,
Papel paraflnado 2,3
Pol/etileno
Porcelana
2,3
6-10
I U = icvzl (21.6)
Fig2L6 •
Teflón
Cera
2,1
5,8 11I ASOCIACION DE CAPACITORES
Goma 2-3 a) En Serie.- Dos o más capacitores se encuentran en serie, si se acoplan
Hielo (a-18°C] 3,2 uno a continuación de otro formando una rama. En la Fig. 21.7, la car-
Vidrio 5-10 ga (q ) que sale de la batería se trans-
Kerosene 2,0 mite fiacia todas las placas y por igual
gracias a la inducción eléctrica. Asi-
mismo la tensión (Vt) de la batería se
reparte entre todos a la manera de una
IMPORTANTE cascada. Luego se verifica que:
La capacidad equivalen- 1º) qT=ql=q2=q3
te de un circuito es aquella
que es capaz de sustituira 2º) VT = VI + V2 + V3
un conjunto de capacftores
_1_ = _1 +'_1_+_1_
y almacenar la misma car-
ga con el mismo voltaje
que experimenta el
3º) Ceq < C2 C3
circuito. donde Ceq es la capacidad equivalente del sistema.

b) En paralela-Si conectamos las placas de un condensador a un mis-mo


borne (polo) de modo que se observe una derivación en el camino
de las cargas, se dirá que el sistema
está acoplado en paralelo, verifi-
cándose que la carga total (qT) se
reparte entre todos los capacitores.
Asimismo se observará que la tensión;'
que todos soportan es la misma.
Luego:
PARA NO OLVIDAR!
1º) qT = ql + q2 + q3
La capacidad equivalen-
te de dos capacitores en 2º) VT= VI = V2 = V3
serie se obtiene así:
3º) Ceq= CI +C2 + C3
Producto
eeq Suma Observación.- Siempre se cumplirá
que: (Ceq)serie < (Ceq)paralelo' Fig2L8

/
Capacidad Eléctrica 393

JlROBLEMAS RESUELTOS
Probo 1.- Se tiene dos placas paralelas de igual área A separadas una distancia d. ¿Qué sucede
con la capacitancia si el área aumenta en 150 % Yla distancia disminuye en 75 %?
A)No varía 8) Esdiez veces la inicial C) Esdos veces la inicial
D) Escuatro veces la inicial E)N.A.
Resolución.-
De acuerdo con los datos podemos reconocer que:
150
A. =0 Af =A+IOO A=2,5A
d;
Al inicio { =d Al final
1 df =d-
75
IOO
d=0,25d

EoAf co(2,5A) coA


........ (1) ef=-d-= 025d =10 7 ........ (2)
f '

De (1) en (2): Cr = 10 CI RPTA.B

Probo 2.- Se tiene un condensador plano con susplacas conectadas a una pila que produce una
tensión Vo' Si se duplica la distancia d entre las placas del condensador plano, la carga
acumuldda en susplacas:
A) Aumenta en un factor 2 D) No varía
8) Disminuye en un factor 1/4 E)Aumenta en un factor 4
C) Disminuye en un factor 1/2 UNI91
Resolución.-
Según los datos se puede establecer que la capacidad inicial del condensador es: e = f.oAld,de modo que al conectarse
a la pila la carga almacenada será: q. = ev (1). Ahora, al duplicarse la distancia entre placas, la nueva capacidad
= =
del condensador será er f.~ d 02, d~ modo que la carga que almacenará vendrá dada por:
I

qr=ervo=2Vo
e (2)

y de (1) en (2): qr = q¡l2 RPTA. e

Probo 3.- Encontrar la diferencia de potencial en


41JF 121JF
voltios en el capacitor de 12J.1F y la que
existe entre x e y (V xl -x ---'-: 11-:
--11--;-
A) 12:50 8)48:48 C)4:32
D) 16: 64 E) 36: 60
48 v
Resolución.-
De los datos reconocemos que los capacitores están conec- el e2
tados en serie y que por lo tanto debe cumplise que las
cargas q, y q2 de ambos deben ser iguales, tal como se
indicó en el item (21.8a).
MI-~
-:Ih
q2 = q, ~ e2 V2 = el VI
:c
~ (121110 V2 = (41110 (48 V) ~ V2 = 16 V
Y según lo indicado en el item 21.8b tendremos que el voltaje total entre x e y estará dado así:
394 Física - Primer Nivel Félix Aucallanchi V.

RPTA.D

Probo 4.- Trescondensadores, de 3 faradios, 8 faradios y 3 faradios están conectados en serie. Su


capacidad equivalente es: .
A) 0,83 a) 1,20 C]8,30 O) 12 E)6 UNMSM87

Resolución.-
Según los datos tenemos: el = 3 F , e2 = 6 F Y e3 = 3 F. Luego, por tratarse de una conexión en serie, utiliza-
remos lo establecido en el item 21.8a:

RPTA.B

Probo 5.- Determinar la carga en J.1Cque en con-


junto almacenan los dos capacitores de
lo derecha (q = 100 J.1C].
A) 24 0)36

a) 96 E) 50

C]60
Resolución.-
Reduciendo los dos capacitores de la derecha encontraremos que su equivalente viene dada así:
ex = e2 + e3 =3 + 5 => ex = 8 IlF
A continuación, reconocemos que los capacitores mostra-
dos experimentan el mismo voltaje por estar en paralelo,
luego, deben experimentar el mismo voltaje.

el} (2¡.tF)
=> ql = ( e2 ,,= 8¡.tF s,
1
=> ql=¡q"
Seguidamente calcularemos la carga qx' en base a lo establecido en el item 21.8b:

ql + qx = q => ¡qx + qx = 120 => s, = 96 Jle RPTA. B

Probo 6.- Se tiene dos condensadores C1 y C2 cargados a potenciales diferentes: V1 = 300 Vy


V = 100 V respectivamente. Luego se unen en paralelo, resultando que la diferencia
de potencial entre las placas de los condensadores es 250 V.Determinar la relación C/
C2 de las capacidades de los condensadores.
A) 2,6 a) 5,0 C] 4,2 O) 3,8 E)3,0 UNI 75
Resolución.-

Aprovechando la relación (21.1) establecemos que las cargas de los condensadores al inicio son:
ql¡ = el VI = 300 e, y q2¡ = e2 V2 = 100 e2
Capacidad Eléctrica 395

y después de cerrar los interruptores, los conden-


sadores quedan en paralelo, presentando ambos el
mismo voltaje V3, luego procediendo como en el paso
anterior, tendremos:

qlf= el V3 = 250 el ' y , q2f= e2v3 = 250 e2


Fmalmente utilizaremos el principio de conservación de la
carga para poder relacionar las cargas en ambos estados:

~q = ~q =::} qll + qli = qlf+ q2f


antes después
Jt V3 ~
=::}

=::}
300 el + 100 e2 = 250 el + 250 e2
50 el = 150 e2
l!lf y2f
Después

RPTA.E

Probo 7.- Un condensador de placas paralelas


de 1 I1F de capacidad es cargado
con 8.1[]<> coulombios. Este conden-
sador se conecta a un condensador
de 311Fdescargado, según la figura.
La carga en coulomblos que al final
adquiere el condensador de 311Fserá:
Al 8. 1{)'Ó C. O) 8/3. 1{)'Ó C.

B) 2. 1{)'Ó C. E) 4. 10-6 C.
C) 6. 1{)'Ó C. UNI89
ResoluciÓn.-

Utilizando el mismo procedimiento del problema anterior diremos que al cerrar los interruptores, los condensa-
dores quedan en paralelo, por lo que podemos afirmar que los voltajes finales son iguales. Luego:

=::} =

(1)

A continuación aplicaremos el principio de conserva-


ción de la carga:

~q = ~q =::} qli + q2i = qlf + q2f (2), (q2i = O)


antes después

-6
qZf =6.10 e
RPTA. E
Física - Primer Nivel Félix Aucallanchi V.

Probo 8.- Cuatro capacitores iguales, de capa-


citancia C, se conectan del modo
indicado; se p;':fe encontrar la
capacidad equive 'ente entre a y b. a

A)C/4 8)2C C)4C


O) 2 C/5 E)3 C
Resolución.- (l)a~-+@-I~ ••
Señalando todos los nudos por letras (a, o b) como en la
Fig. I observaremos que en el circuito, los capacitores
quedan entre dos nudos tal como se indica en la Fig. 2,
Luego, reconocemos que todos los capacitores han
quedado en paralelo, por lo que la capacidad equivalente
estará dada por lo establecido en el item 21 .Sb,
(2) A~
Cc = C + C + C + C
~~
RPTA.C e

Probo 9.- Determinar la energía en joules que


almacena el circuitode condenscxJores, 40 ¡¡F

A) 3. 1O-3J
50 ¡¡F
8)2.1O-3J ~
20 V
C)4.1O-3J

Resolución.-
O)5.1O-4J
E)ó.10-4J -G"'" 60 ¡¡F
45¡¡F

En primer lugar reduciremos los condensadores, utilizando la misma regla de puntos que empleamos en el
ejercicio anterior, con lo cual podemos establecer los siguientes circuitos:

40 40

-=- 45 -=- :25 50\

-r x y -r \-~f-T)
y 25 Y y~--
60 Y 60 Y

(1) (2) (3) (4)


En (1) se aplica la regla de los puntos y se descubre que los condensadores están en paralelo,

En (2): C = 25 + 45+ 50 :=) C = 120 IlF


11111 I 1
En (3): C= 40 +C+ 60 = 40 + 120 + 60 :=) Ce = 20JlF
e
Capacidad Eléctrica 3fJ7

y en (4), determinaremos la energía que almacena el capacitor equivalente que es el mismo que almacena todo
el circuito, para lo cual emplearemos la relación (21.6):

::::} u= 4.10-3J RPTA.C

Probo 10.- En un condensador de placas planas y paralelas, en el voco. se dispara un electrón de la


placa posiffva hacia la negatrva. Sila diferencia de potencial entre las placas es de 1ooV. y
la separación entre ellas es de 1cm, ¿Cuál debe ser su energía cinética inicial para que el
electrón apenas llegue a la placa negativa? Carga del electrón e = 1,6 . 10-19 C.
A)1,6.10-17j 8)3,2.10-17j C)0,B.10-17j 0)1,0.1O-17j E)1,.6.1O-19j
UNI88
Resolución.-
Debemos reconocer qu~ durante el movimiento del electrón
(q) el campo eléctrico (E) desarrolla un trabajo desde A hasta +
+
B (W; -> B) el cual vendrá dado por la relación (20.11). q~
Asimismo podemos notar que el campo trabajo antes menciona A ()--o--------------- B
produce un cambio en la energía cinética por lo que v¡
emplearemos el Teorema del trabajo y la energía cinética dada +
por la relación (11.5). Luego: +

Wneto --(WC A->B )

::::} - Eci = (- 1,6. 10-19 C) (100 V)

Eci = 1,6.10- 17 J RPTA.A

Probo 11.- La distancia entre las placas del conden-


sador plano mostrado es 6 cm. Si las cargas
q1 = -2 C y q2 = + 1 C tienen igual masa,
-r-.. ··+ + + +
la distancia que recorre cada una cuando d
se cruzan es respectivamente:
I ~q¡
A) 3; 3 cm 0)4,5; 1,5cm
.L ..=- ====
8)4; 2 cm E) 1,5; 4,5 cm
C)2;4cm
Resolución.-
Cuando las cargas se mueven por efecto del campo experimentan fuerzas netas que le producen aceleraciones,
las que guardan entre sí la siguiente relación:

!!tl-litl
liii - q2 (1)

y por Cinemática diremos que las distancias recorri-


das estarán relacionadas entre sí de modo que:

1 -

T:l 2
2
lt
:1 = ::::} :21 =1 :21 1 ..... (2)
2 i1a21t

/"
Física - Primer Nivel Félix Aucallanchi V.

Luego, de (1) en (2): ~=I~I=I~~~I~ d¡=2d2 •.•••• (*)

Y según el gráfico: =
d, + d2 6 cm (**)

Finalmente de (*) y (**): RPTA.B

Prob. 12.- Unconductor posee uno capacidad eléctrico de 12 J.1F se encuentro cargado con 36
J.1C tEn cuánto variará su potencial eléctrico absoluto si su gorga se Incremento hasta
60 J.1F?

A] 2 V B] 1 V C] 3 V O] 4 V E] N.A

Resolución>
Utilizando la fórmula (21. ) para la capacidad eléctrica, despejaremos el potencial (V) y tendremos para caso
10 siguiente:

-~-~
V¡-C-12J.1F ~ V¡=3V

_9z._~
V- C -12J.1F ~ V2=5V

Luego: V=V2-V¡=5V-3V

~ V=2V RPTA.A

Probo 13.- SIen lo naturaleza existiera uno esfera conductora del tamaño de lo tierra, ¿cuál sería
el tamaño de su copaciddd eléctrico en faradios?
(considere: radio terrestre = 6471 km).
A]l15J.1F B] 716J.1F C] 719J.1F O] 718J.1F E] 720J.1F

ResoluciÓn.-

Utilizando directamente la fórmula (21.2) para el calculo de la capacidad de una esfera coonductora, tendremos:

C = Ji. = 6471.103m = 7l9.1O·6[-C-]


esf k ~9 N m 2 (N.m)
e 91u--·-
2
-e
C
=
Cesf 719.10.6 [(f)] = 719.10.6 (f) == 719.10.6 F
Cesr = 719 J.1F RPTA.C

Nota: El resultado nos indica que el faradio es una unidad de capacidad grande.

/'
Capacidad Eléctrica 399

, 2~,AUTOEVALUACIÓN ",

1.- La capacidad eléctrica de un conductor es indepen- 6.- Dados los siguientes capacitores, señalar la
diente de: relación correcta de capacidades.
A) Su volumen D) Sus dmensiones
B) Su forma E) La carga que almacena
C) Su superficie
2.- Dado el gráfico carga-vs-potencial de un conduc-
tor, se afirma que:
( ) Su capacidad es de 3F (C)
q
( ) La energía almacenada p
hasta P es de 24 J 12
I

) El campo eléctrico I A) CI > C2>C3 D) C3 > C2 > CI


I
tiene una intensidad de I
B) CI = C2 = C3 E) C3 < C2 < CI
ION/C. : V(voltio)
4 C) C2<C3 <CI
Señalar verdadero (V) o falso (F)
7.- Manteniéndolo conectado a una batería, se aproxi-
A) VVV B) VVF C) FVV D) FFV E) FFF
man las placas de un capacitor plano. Luego:
3.- La capacidad de un condensador de caras planas 1) La carga disminuye.
y paralelas aumentará si: 11) El campo eléctrico aumenta.
A) Disminuímos el área de sus placas. I1I) La energía almacenada aumenta.
B) Aumentamos la distancia entre placas.
Señalar la(s) correcta(s):
C) Aumentamos el voltaje entre placas.
A) III B) 11 C) I D) 1 Y 11 E) 11Y III
D) Aumentamos la carga en cada placa.
E) Introducimos un di eléctrico. 8.- Se carga un capacitor plano uniendo sus armadu-
ras a los bomes de una batería. Suponiendo que se
4.- Elige las palabras que completan mejor la oración: desligara al capacitor de la batería para luego acercar
«El dieléctrico de un capacitor cargado genera un -------- sus armaduras, es cierto que:
eléctrico interior debido a las cargas ------------ en él ».
A) La capacidad disminuye.
A) Campo, móviles
B) El voltaje aumenta.
B) Exceso, polarizadas
C) Campo, inducidas C) La energía aumenta.
D) Voltaje, móviles D) La carga disminuye.
E) Desequilibrio, móviles. E) El campo permanece constante.
S.- Si C es la capacidad de un capacitor y Vla diferen- 9.- Luego de cargar un capacitor plano con aire entre
cia de potencial que experimenta, el gráfico correcto sus armaduras, se ha desconectado de la batería. A
de C-vs- V será: continuación se le introduce un dieléctrico, y se
afirma que:
) La capacidad aumenta.
) La carga neta disminuye.
) El voltaje aum -nta,

L- D)I ~ ) El campo neto disminuye.


C)

LC~ V v
E)N.A ( )

A) 5
La energía se mantiene igual.
¿Cuántas afirmaciones
B) 1
son incorrectas?
C) 2 D) 3 E) 4

/
400 Física-Primer nivel Félix Aucallanchí V.

rOTA: Por razones de simplificación operativa, para 7.- Se cargan tres capacitores de 1¡l F a tensiones de
el presente capítulo consideraremos que todas las 100; 200 Y 300 V. Se conectan luego en paralelo
capacidades se dan en microfaradios (¡l F). ¿Cuál es la tensión resultante en V?

A) 350 B) 200 C) 100 D) 400 El 250


IVELl
8.- Hallar la relación entre la capacidad equivalente
del sistema mostrado y la capacidad equivalente
1.- Un conductor presenta una carga q = 8.10-3 ey cuando M y N se unen mediante un conductor.
un potencial eléctrico V = 20 V. Se pide encontrar el
valor de la capacidad en microfaradios:
A) 1/2
A) 550 B)300 C) 100 D) 400 E) 200
B) 2/3

l~~
2.- Un conductor tiene una capacidad de 5¡lF y posee
C) 3/2
una carga de 60 ¡lC ¿En cuánto varía su potencial si
su carga se reduce a la tercera parte?
D) l
Al -8V B) 6V C) -60V D) 40V E)-5V
E) 1/4 C e
3.- Se tiene una esfera conductora cuyo radio es r y su 9.- Determinar la capacidad equivalente (en ¡l F) del
capacidad es igual a 30 ~tF.¿Qué capacidad (en ¡.tF)
sistema mostrado, entre los puntos A y B. Todos
tendrá otra esfera conductora cuyo radio sea 2r/3? los condensadores tienen igual capacidad e = 3 ¡l F
All5 B) 16 C) 17 D) 18 E) 20
A) 2
4.- ¿Cuál sería la capacidad de la Tierra si estuviera
B)3
hecha de un material buen conductor? Considerar:
Radio terrestre = 6 372 km. C)4
Al 154 B) 160 C) 708 D) 185 E) 200
D)5 B "-- ..L..---i

5.- Si el condensador mostrado tiene una capacidad E)6


de 600 ¡lF. ¿Cuál será su capacidad si ttiplicamos las
áreas y duplicamos la distancia entre placas. 10.- Hallar la capacidad equivalente entrex e y.Todas
las capacidades están expresadas en ¡l F
A) 200
A)2
B) 300 1"
d B)3
C)4OO ¡
C)4
D) 500
D)6 y <>-o4--..:!.lI--.:::.I
E) 900
E) 12
6.- Con 3 condensadores de 36 ¡l F cada uno, se U.- Se tienen tres condensadores de 5 ¡l F , 3 ¡l F Y
puede lograr capacidades de : Marcar 4 ¡l F, los dos primeros en paralelo y el conjunto en
la incorrecta. serie con el último. Si la carga en el de 3 ¡lF es de 300
¡lC Calcular la tensión (en V) en los extremos del
All2 B)24 C)60
conjunto.
DlI08 E)54 D) 500 E) 150
A) 100 B) 200 C) 300

/
I

. Capacidad Eléctrica 401

NIVEL 2 A)15
B)16
12.- Determinar la carga (en ¡.t C) que almacena el
circuito mostrado en la figura. C) 18
A) 10 D)20
B)20 E)21
C) 30
17.- En la figura mostrada, calcular la capacidad
D) 40 equivalente (en ¡.t F) entre x e y.
E) 50 A)2
13.- Determinar la capacidad equivalente(en ¡.t F) del B)3
sistema de condensadores mostrado respecto de los
terminales A y B.Todas las capacidades están enu F C)4
y
A)5 D) 5
B) 10
E)6
C) 15

D)20 18.- Para el circuito mostrado calcular la capacidad


(en ¡.t F) del condensador e para que la capacidad
E)25 equivalente vista desde a y b sea igual a la de dicho
condensador.
14.- Determinar la capacidad equivalente (en ¡.t F) del
sistema de condensadores mostrado, respecto de los A)5 D)2
terminales M y N.Todas las capacidades están en ¡.t F
B)4 E) 1
A)6
C)3
B)10 b 0-----'------'
C)12 19.- Hallar la capacidad" C' (en ¡.tF) de los capacitores
mostrados, si la diferencia de potencial, entre A y B
D) 15 es de 90 V Y entre A y D es de 225 V. -

E)18 A)5
B)4
15.- Para el acoplamiento de condensadores mostra-
do, determinar la carga(en ¡.t C) que almacena el siste- C)3
ma, si se sabe que éste tiene una energía almacenada
de 3 ~. Las capacidades están en ¡.tF. D)2

A)4 E) 1

B)6 20.- Las placas de un capacitor plano cargado son


puestas a Tierra por turno. Entonces se puede
C)8 afirmar que:
D)3 A) Solo la placa (+) se descarga.
E)5 B) Solo la placa (-) se descarga.
C) El capacitor se descargará.
16.- Calcular la capacidad equivalente (en ¡.tF) del
sistema de condesadores mostrado entre x e y, si D) El capacitor se recarga.
todos los condensadores son iguales a e = 12 ¡.tF. E) Ambas placas conservan su carga inicial.
402 Física-Primer nivel Félix AucaJ/anchi V.

NIVEL 3 e
25.- La energía almacenada (en ~ por 2 = 8 ¡.tF es
576 ¡.L/. Calcular la carga (en ¡.tC) en el condensador
21.- Hallar la capacidad equivalente (en ¡.t F) entre =
de capacidad C3 6 ¡.tF
los puntos a y b. Todos los conden-sadores son de
igual capacidad e= 5 ¡.tF. A) 12

A) 20 B)36

B) 15 C)72

C) 12 D)24

D) lO E) 48

E) 18 26.- Si se cortocircuita los puntos Q y N, calcular la


diferencia de potencial (en V) entre los puntos A y B.
22.- Determinar la capacidad equivalente (en ¡.tF) con los
dieléctricos dados. Si el condensador en el vacío tiene e Q
una capacidad C = 3 ¡.tF, además las constantes

r'~hN >B
dieléctricas son: KI = 3 Y K2 = 5
6~~4C

A) 11 A
1 T 1
r
60 V 6C 6C
B)12 '"
~~4C
4C
C) 13
CD 0 d
D) 14 J- A)5 B) 10 C) 15 D) 20 E) 25

27.- En el circuito mostrado, hallar la diferencia de


E) 15 potencial entre los puntos A y B del circuito. Todas
las capacidades son iguales.
23.- ¿Cuál es la capacidad (en ¡.t F) del condensador
mostrado si las constantes de los dieléctricos, son A) 10 V
K I = 3 YK} = 5, si la capacidad en el vacío es Co = 80 ¡.tF ¡--11
B)20 V
.±L60V ~ l
A) 100
A, I C)30 V f-1~
T
B)200

C) 300 CD D)40 V

F) 50 V
T
B
T
~ d
D)400
0
E) 500
I
1 28.- En el circuito mostrado se cierra SI y luego se
vuelve a abrir para cerrar S2' Hallar la carga en el capaci-
tor de 2 ¡.tF (en ¡.tC).

A)18
24.- Se tiene un capacitador plano de capacidad
eléctrica 22 ¡.tF cuando en sus armaduras existe aire. B)30
Calcular la capacidad del capacitor (en ¡.t F) cuando
en su interior se coloque una lámina de constante C) 12
dieléctrica K = 5 pegado a una de sus armaduras,
D) 15
siendo su espesor la tercera parte de la separación
entre las placas. E) 20
A)30 B) 25 C) 20 D) 15 E) 10
0)
L.:..J E(ectrotfinámica
ORJE1TVOS
1.- Conocer los conceptos básicos de
corriente eléctrica y resistencia
eléctrica, y su relación con la tensión.

2.- Estudiar los principios básicos de


transformación de energía eléctrica
en otras formas de energía.

3.- Analizar el principio de funciona-


miento de un circuito eléctrico.

1 inmenso avance de la tecnología mundial se fué labrando


a mediados del siglo pasado y hoy que disfrutamos de dicho
avance debemos reconocer que ello se debe y en gran medida,
al aprovechamiento de la electricidad. La gran mayoría de los aparatos
inventados por el hombre utilizan la electricidad, la misma que puede ser
empleada en pequeños o grandes circuitos. Así vemos que en los diminutos
espacios de los CHIPS o MICROCHIPS de las computadoras existe un GEORGE SIMON
movimiento de cargas eléctricas, que hace que éstos puedan funcionar de OHM
una forma por demás asombrosa. De igual modo es un bello espectáculo ver
cómo al atardecer nuestra ciudad empieza a iluminarse artificiaImente gracias (1787-1854)
a una gran corriente que circula por todos los cables de alumbrado público.
Nació en Bavlera-
11I CORRIENTE ELECTRICA
Alemania. Fué profesor
de Matemática en Colo-
nia. En 1827 llegó a publi-
Cuando logramos establecer un campo eléctrico en el interior de car mediante un folleto el
conductor, comprobaremos que los electrones libres iniciarán un mo- trabajo más Importante de
movimiento en sentido opuesto al su vida, cuyo título fué: ~EI
Sentido Real circuito galvánico exa-
campo. Llamaremos corriente eléc- minado matemáticamen-
trica en el conductor al flujo de elec- te», lo que más tarde se
trones que se produce debido a un denominó Ley de OHM,
base fundamental para el
campo eléctrico. Si el conductor es + desarrollo de la Electro-
un líquido o un gas, la corriente se tecnia. Aún cuando tales
debe principalmente al movimiento de estudios hayan sido una
iones positivos y/o iones negativos. colaboraclon Importante
Se comprueba que una carga negativa para la teoría de los
circuitos eléctricos y sus
que se mueve en cierto sentido aplicaciones, en su época
equivale a otra carga positiva de igual + este aporte fué recibido
valor que se mueve en sentido con frialdad por la comu-
contrario. Esto permite establecer el nidad científica, pero final-
Sentido Convencional mente fué reconocido, y
sentido convencional de la corriente por ello recibió una
que usaremos de aquí en adelante. medalla honorífica por la
Fig22.1 Real Sociedad de Londres.
Física - Primer Nivel Félix A(,Jcallanchi V.

DEBES SABER QUE:


• INTENSIDAD DE CORRIENfE ELECfRICA (i)
Cuando el sentido de la
corrIente es sIempre el Supongamos hipotéticamente
mIsmo, ésta se denomina la siguiente experiencia: Considere-
corriente contínua, y si su mos un observador que puede con-
valor permanece cons-
tante, su representación
tar las cargas que pasan a travéz de la
en el gráfico i-vs-t es así: sección recta (A) de un conductor que

J-q l
i+ lleva corriente. Sea q la carga total
que contó y t el tiempo que emplea-
ron en cruzar dicha sección; enton-
ces, se define la intensidad de corrien-
teicomo:
Area = Carga
i=~ (22.1) Fig 22.2
De este modo i nos dá la cantidad de carga que pasa a través de la
PARA NO OLVIDAR
sección recta del conductor en cada unidad de tiempo.
SI el campo eléctrico del Unidades: q = coulomb, t = segundos => i = ampere (A) => 1 A = 1Os
conductor cambia de sen-
tido, entonces también
cam-blará de sentido el GENERADORESELECTRICOS
fTlOVI-mlentode las corgos.
Así, llamaremos corriente
Denominamos así a todo sistema físico que puede transformar
alterna a aquella que cualquier tipo de energía: mecánica, térmica, magnética, luminosa,
cambia de sentido a trCNés nuclear, química, .... ,etc., en energía eléctrica. Todo generador debe
del tiempo. El tipo de realizar un trabajo para mantener en movimiento a las cargas eléctricas.
corriente alterno más cono-
cido es aquella que varía Así tenemos que una pila seca, una batería, son generadores eléctricos,
slnusoldalmente con una los cuales presentan siempre dos polos: uno positivo y el otro negativo.
frecuencia de 60 nz. como
la que llega a nuestros do- • FUERZA ELECTROMOTRIZ (E)
mielllos,y que se gráfIca así:
Cuando instalamos un alambre..
conductor entre los polos de un gene- a) Sistema },(ecállico
rador, se establece en su interior un
movimiento de cargas, los cuales
salen del polo positivo y retornan por
el polo negativo. Desde este último
lugar el generador realiza un trabajo
W para conducir una cantidad de
carga q hasta el polo positivo. de (-)
modo que éstas vuelvan a reiniciar
su recorrido. Luego, se define la
MPORTANTE b) Sistema Eléctrico
fuerza electromotríz (jem = & ) del ge- ......--c:>
Una pila seca presenta nerador como:
dos polos eléctricos,y se sim- 'LA
boliza así: W
&=- (22.2)
q
&:- -<J. Fuente de
Tensión •
Así, queda establecido que lafem nos
dá la cantidad de trabajo o energía
que proporciona un generador a cada
unidad de carga. Las unidades S.I.:
(&) = voltio = joulelcoulomb Fig22.3
Electrodinámica 405

MUY INTERESANTE
• RESISTENCIA ELECTRICA
A) Pilas en serie. - Se
Sobre un plano inclinado con r-----=-----------,
colocan una a continuación
tachuelas (Fig. 22.4.a) se han liberado ' a) de otra con sus polos
dos esferillas. Estas experimentan ¡ :1':.-, alternados, generándose un
cierta dificultad al descender, la cual ,¿/f-¡-=-., <,
voltaje mayor.
se verá incrementada si aumentamos ,"./
......--' '_',
el número de tachuelas. Esta es a i
aproximadamente la misma dificultad I c::c::a ••••••• - ••.
que experimentan los electrones al
viajar dentro de un conductor. Lla- ( ---A./VVV'-
maremos resistencia eléctrica de un Simbologia ~A~
conductor a aquella magnitud física - ..Jf-'(Y v~ I
de tipo escalar que nos informa del ~A B) Pilas en paralelo.- En
grado de dificultad que ofrece dicho I este caso lospolos del mismo
cuerpo al paso de las cargas eléctricas <- +======!::.L-===;=~¡W signo se conectan entre sí,
generándose dos únicos
por su interior. Fig 22.4
polos. Laspilas deben tener
11I LEY DE POULLIETT de preferencia el mismo
voltaje.
Esta ley experimental establece que: «La resistencia de un
conductor es directamente proporcional con su longitud e
inversamente proporcional con el área de su sección recta »,

R (J. ¡ => ~ R = P~ ] (22.3)


donde p es la constante de proporcionalidad conocida con el nombre
de resistividad eléctrica cuyo valor depende del tipo de material.
Unidades: (L) =
metro(m); =
(A) metro cuadradotmri,
(p ) = ohmio-metro (n-m), y (R) = ohmio (n).

_ VARIACION DE LA RESISTENCIA CON LA


_ TEMPERATURA
Al aumentar la temperatura de un conductor se incrementa su agitación
electrónica, aumentando por consiguiente la dificultad en el transporte de la
comente, y por lo tanto un aumento de su resistencia eléctrica. Así pues, con-
cluímos que la resistencia depende directamente de la temperatura de trabajo.
SORPRENDENTE!
IR2=Rl <Í+aT¡t\T) 1 (22.4) ~ La resistencia eléctrica de
un conductor metálico
donde R I Y R2 son las resistencias puede disminuirsemediante
a las temperaturas TI y T2 respecti- enfriamiento. Así,a tempe-
raturas cercanas al cero
vamente, y ex Ti es el coeficiente de absoluto (O K)la resistencia
temperatura de la resistencia medi- eléctrica desciende a
da a la temperatura TI' cero, denominándose a
dicho estado: Supercon-

!~=~I (22.5)

siendo T la temperatura a la cual la


~~----7U--~---~
-273°T
l Cero absoluto
T
I
T2 7t C)
0
ductividad. Una corriente
creada en un circuito
superconductor existirá de
manera indéfinida. El
inferido plomo y el mercurio pre-
resistencia del conductor es nula; sentan superconductivi-
ex se expresa en ac-I o K-l. Fig22.5 dada 4,2 K.
Física - Primer Nivel Félix Aucallanchi V.

RESISTlVIDADES
ELECTRICAS (p] IIILEYDEOHM
(A 20° C] Si entre los extremos de un con-
du. 'or se establece una diferencia de ~
Material . 10·a n- potencial, se generará un campo eléc- V. : -oC> I vb
I i I
trico que posibilitará la aparición de I I
Aluminio 2.8 una corriente eléctrica. George Simon I I
Carbono 3.6 V I I
Ohm descubrió que: «La intensidad v. I
Cobre 1.7
Cromo 2.7
de la corriente en un conductor es
directamente proporcional con la di-
I I
I
I
v=V.-Vb I
Cobalto 9.7 ferencia de potencial de sus extremos,
Constan tan 49 e inversamente proporcional con su Vb
Hierro 9 resistenc ia ». ---_!_------------_:
d
Latón 7 ~ (22.6)
Manganina
Mercurio
4
96
L1LJ Flg 22.6
Observación.- La corriente siempre fluirá del extremo de mayor potencial
Oro 2.4 hacia el extremo de menor potencial.
Plata
Plomo
Tungsteno
1.6
22
6
&11 TEOREMA DE LATRAYECTORIA
Cuando una corriente (i) recorre una rama de un circuito, se
dice que su potencial (V) experimenta un aumento al pasar de un polo
negativo (-) a un polo positivo (+),
y experimenta una disminución
DEBES SABER QUE: cuando pasa de un polo positivo a
otro negativo. Luego, en base a la
Al plantear las ecuaclones Fig. 22.7, recorriendo la rama de "a"
de circuitos es importante
haber señalado la polaridad
hacia "b", el teorema establece que:
de las resistencias. El signo
positivo se Indicará por el
!Va+l:&+l:iR=Vb j (22.7)
extremo por donde Ingresa Fig.22.7
b corriente. ~ Va + &} -&2 - ts, - iR2 = Vb

lID! EFECTO JOULE


De entre la innumerable cantidad de experimentos realizados por
Joule en su afán de encontrar el equivalente mecánico del calor, descubrió
que cada vez que circula una corriente por una resistencia, ésta convierte
ALGO MAS: la energía eléctrica en energía térmica. V
A) Energía Eléctrica.- La cantidad de - II +
Al aplicar el Teorema de energía (W) que se extrae de una
la Trayectoria se debe elegir resistencia (R) en donde circula una
el borne de partida. y a
continuación el camino a
corriente (i) durante un tiempo (t)
seguir. SI al avanzar en- determinado, viene dado or:
contramos un signo poro un
voltaje determinado (& o iR).
V2
W =Vit = ¡2Rt =-t (22.8)
en la ecuación deberá R
anotarse el signo contrario.
B) Potencia Eléctrica.- La rapidez con
Puedes ensayarlo con el
ejemplo de la Flg. 22.7. Y que se extrae energía de una fuente
,¡ ¡ \ "-R
deberás obtener la misma de em o de una resistencia viene dada así:
ecuación que se ha 2
propuesto debajo
relación [22.7).
de la PoI = Vi = ¡2R = -1f (22.9)
Fig27.~
Electrodinámica 4(J7

CIl!CUITUS r=Lr=CTl!ICUS ATENC/ON

La resistencia equivalen-
Cuando reunimos una fuente de te de un circuito es aque-
tensión con resistencias y conductores, " Conductores lla que es capaz de susti-
la corriente eléctrica circula siguiendo +:.k----»-_ ./ tuir a un conjunto de resis-
un camino definido por ellos. Llamaremos tencias y generar la misma
co- rriente experImentan-
circuito eléctrico al conjunto de dispo-
sitivos por los cuales circula una corrien-
v do el mismo voltaje de
todo el circuito.
te de manera permanente.
Se puede comprobar que la 4 Fuente de
corriente busca siempre el camino que ~Fuerza Interruptor
Electromotriz
le ofrezca menor resistencia.
Fig22.9

11II CLASES DE CIRCUITO~


A) Circuito Serie.- En este tipo de cir- i----:=======.1¡-i
cuitos las resistencias se acoplan una
a continuación de la otra, de manera OJO!
que forma un único camino para la co- Siun grupo de resistencias
rriente. Pueden verificarse las siguien- se acopla primero en se-
tes propiedades: rie y luego en paralelo,
se obtendrá una mayor
lº) La corriente es la misma en todas re- sistencia equivalente
+ en el primer caso. En la
las resistencias.
mayoría de las instalacio-
iT = i1 = i2 = i3 nes domésticas e indus-
triales, las conexiones se
2º) El voltaje de la fuente se hacen en paralelo. Luego,
distribuye en forma de cascada en cuando todas trabajan, la
resistencia del circuito dis-
todas las resistencias. minuye, lo cual produce a

vt (;,)
VT=V1+V2+V3 su vez un mayor consumo
Circuito Equivalente: de corriente y de energía,
3º) La resistencia equivalente del cir- dado que el voltaje es
siempre el mismo.
cuito viene dada por la suma de las re- ~R.
sistencias participantes.
Re =Rl +R2 +R3 Fig22.10
B) Circuito Paralelo.- En este circuito Ias
resistencias se acoplan de manera que ¡riT
sus bornes están unidos entre sí, de ill ¡i2

manera que todos quedan conectados + + +


+
VI V2 V3
directamente a la fuente. La corriente VT Rz -
RI
tiene en esta conexión varios caminos IMPORTANTE
para circular, verificándose por tanto
las siguientes propiedades: Cuando se tienen dos

vf (;,) ~~
Circuito Equivalente: resistencIas R1 Y R, en
lº) La corriente total está dada por la paralelo la resisteñcia
suma de las comentes en cada resistencia. equivalente se obtiene así:
iT = i1 + i2 + i3
2º) Todas las resistencias experi-men-
tan el mismo voltaje. Fig22.11

-,
403 Física-Primer Nivel Félix Aucallanchi V.

PARA RECORDAR: VT =V1 =V2= V3


Un galvanómetro es un 3º) La resistencia equivalente será:
instrumento que se utiliza 1 1 1 1
para detectar la presen-
cia de una corrIente o
Re = R;
+ R¡ + R3
para medlrla, par mllf pe-
queña que ésta sea. 11I INSTRUMENTOS DE MEDIDA
A) Voltímetro.- Este aparato permite medir la diferencia de potencial
entre dos puntos de un circuito. El modo de instalación es en paralelo
con la resistencia de la que se pretende +I _
medir la diferencia de potencial que í¡ I

experimenta. De este modo una


pequeñísima corriente iy se desvía hacia R
el voltímetro, con lo cual éste puede
funcionar y dar la medida correcta. Para
que la presencia del voltímetro no pro- v
duzca cambios apreciables en el circui- Vo = Voltaje Original
to, dicho instrumento posee una gran VAB = Voltaje Medido
resistencia. Así, un voltímetro ideal es V.
ATENC/ON aquel cuya resistencia interna es infi- VAB= l+#Rv

Determinadosamperímetros nitamente grande (Ry ~ 00). Fig 22.12


están calibrados en mili- B) Amperímetro.- Este instrumento permite determinar la intensidad de
amperios o en m/cro-
amperios, y por ello son corriente que circula por una resistencia. El modo de instalación es en
conocidos como miliam- serie con la resistencia cuya corriente que conduce intentamos medir.
perímetros y microompe- Debido a la resistencia interna que
rímetros respectivamente. presenta el amperímetro, la lectura
obtenida difiere un poco de la
corriente original, pues el instrumento
ha incrementado la resistencia VA +0: :V
equivalente del circuito. Por ello.durante
la fabricación de estos aparatos se bus- i. = Corriente Original
ca darle la menor resistencia posible. iA = Corriente Medida
Así, un amperímetro ideal es aquel que
posee una resistencia infinitamente pe- iA= llR/R
queña (RA ~O). Fig22.13
C) Ohmímetro.- Este aparato se utiliza para medir resistencias, y se
conecta en paralelo con la resistencia que se pretende medir. Viene con
ENTERATE! una pequeña batería y una resistencia variable. Generalmente, el
ohmímetro está junto a un voltímetro y
1)EI vatímetro es un instru- un amperímetro; instnunentos de este tipo
mento coyo lectura nos se denominan multímetros. Es posible
dá directamente la po- medir resistencias por medio de un R=?
tencia en vatios desa-
rrollada en una rama de voltímetro y un amperímetro; así, el pri-
un circuito. mero nos .dá el voltaje (V AB ) Y el se-
gundo la intensidad de la corriente (iA)'
2)EI contador eléctrico es
el aparato instalado en Luego la resistencia desconocida se
nuestros domIcilios y calcula utilizando la Ley de Ohm:
centros de trabajo, que
sirve para medir la R=V:"-B
energla consumIda en
kilowaft-hora (kWh). LA Fig22.14
Electrodinámica 400

11II PUENTE DE WHEATSTONE


MUY IMPORTANTE
Si en alguna conexIón se
Designamos con este nombre al circuito constituído por cinco verIfica la relacIón (22.10],
resistencias acopladas según como se muestra en la Fig. 22.15, en el automáticamente se puede
afirmar que la resistencia Rs
cual aparece una galvanómetro (G), que nos indicará si existe o no corriente está desenerglzada, dado
porla rama CD. SiR4 es una resistencia e . que por ella no cIrcula
variable, podemos graduarla hasta un ~~ corriente. Luego, su retIro
cierto valor en el cual el galvanómetro no alterará en lo absoluto
la resistencIa equivalente
indique cero, es decir: is = O, con lo cual del circuito.
se verificará que Ve = VD; luego, se
podrá establecer la Siguiente ecuación:

(22.10)
Fig22.15

&11 LEYES DE KIRCHHOFF INTERESANTE


Las pilas y las baterías
El físico alemán Gustav R. Kirchhoff en 1845 extendió la Ley presentan además de una
de Ohm a circuitos más complejos de dos y tres dimensiones (en placas fuerza electromotríz e: una
y en volúmenes), para 10 cual estableció las siguientes leyes: resIstencia Interna r. de
modo que entre suspolos a
1r.l!) Ley de las corrientes.- Se le lIama ybexlsteunad.d.p.: Vab < S.
también Ley de los nudos, y establece
que: «La corriente total que llega a
un nudo es igual a la corriente total
que sale de él ».

[ Dnegan =~SaleDJ (22.11)


Del ejemplo de la Fig. 22.16 tenemos:
i¡ + i3 + is = i2 + i4
Fig22.16

2da) Ley de los voltajes.- Se le conoce ,-------,------,


también con el nombre de Ley de las
, Rt
Mallas, y se basa en el Principio de
Conservación de la Energía. Establece a
que: «La suma de los voltajes a lo largo CUIDADO!
del circuito es igual a cero ».
Al apl/car la 2QQ Ley de
Kirchhoff debe tenerse en
f' 1:.V = O! (22.12) cuenta que los voltajes
I (& ° iR] deberán escribirse
con un slf¡lnoadelante. Este
Del ejemplo de la Fig. 22.17 recorre-
mos el circuito desde "a" y en sentido sIgno sera el primero que se
encuentra durante el re-
horario, siguiendo el camino cerrado corrido del circuIto, el cual
"abcda" : a su vez puede ser horario
° antihorario. Esteha sIdo el
crIterio para elaborar la
Fig22.17 ecuacIón para el ejemplo
de la Fig. 22.17.
410 Física - Primer Nivel Félíx Aucallanchi V.

nl08LEMASRESUELTOS

Probo 1.- El número de electrones que pasan por una sección transversal de un alambre por el cual
circula una corriente de 0,2 A durante 16 segundos es: (Carga del electrón = - 1,6.10-19 C)
A) 1.106 B) 2 .1019 C) 1,6.10-19 O) 32 .1010 E) 32 .1019 UNI 93 - 1
Resolución.-
Calcularemos primero la carga q que cruzó la sección indicada del conductor, para lo cual emplearemos la
relación (22.1):
q = i, t =(0,2 A) (16 s) ~ q = 3,2 coulombs
A continuación determinaremos el número (N) de electrones conducidos en la carga q obtenida, para lo cual
utilizaremos la siguiente regla de tres simple:

1e ~
N ~
1,6. 10-
3,2 C
19
c} N = (1 e) (3,2 C)
(),6. 10-19 C)
~ N = 2.1019 e
RPTA.B

Probo 2.- En una instalación eléctrica se reemplaza una resistencia de 12 ohm fabricada con un
alambre de nlquel de longitud "t' y sección "S"por otro alambre de constantan de igual
longitud y sección. La nueva resistencia será de:
(reslstivldad de Niquel = 0,12 crvrurvrrtm: reslstMdad del constantan = 0,50 ohm.mrr1/m)
A) 25 ohm B) 50 ohm C) 144 ohm O) 14,0 ohm E) 5 ohm UNI84 -1
Resolución.-
Utilizando la relación (22.3) para la resistencia de un alambre tendremos:

a) Niquel: RN¡=Jl..¡¡· t ~ t = (ft (1)

b) Constantan: Rconst. = Pconst.· S1 (2)

R 1 (0,50 n. mm2¡ m)(12 n)


Reemplazando (1) en (2): l\::onst. = PConst. ( P.4.¡ = (0,12 n . mm2 ¡m) -

RCOllSl = 50 n RPTA.B

Probo 3.- Un metro de alambre de cobre pesa 1 gramo y tiene una resistencia de O,15 n ¿Cuál será
la longitud de un alambre del mismo material que pesa una tonelada y cuya resistencia
sea 6 kn?
A) 21<r(1 B) 20 km C)40km 0)200 km E) 400 km UNI83-2
Resolución.-
Este ejercicio nos permitirá deducir una fórmula para la resistencia de un alambre principalmente en función
de su longitud y su masa, para ello supondremos que el conductor es recto, uniforme y homogéneo de longitud
L y sección transversalA. Ahora en base a la relación (15.1) para la densidad (D) encontramos que la masa (m)
del conductor está dada así: .
m
m = DV = DLA A=- (*)
DL
Electrodinámica 411

A continuación emplearemos la relación (22.3) para la resistencia de un alambre:


.É 2
R=P L ,yde(*)·. R-p
- (mI LD L) --" - V. h
......, R-pr.. rn: . (**)
A

y finalmente, para las condiciones del problema, se trata de comparar las resistencias de dos alambres del
mismo material, es decir p y D iguales; luego, de (**) tendremos:

&=L;/m2=(L2)2 ~ 2 R2
RI L2I / mI LI . m2
L2 = ~
-.-.LI
mI RI

103 kg. 6.103 Q


Si reemplazamos datos, obtenemos: Lz = J!:3 .lm RPTA.D
io kg. 0,15Q

Probo 4.- En la figura se muestran tres


generadores de corriente
contínua. ¿Cuál es el voltaje F + _._--- - + ~--+ - G
entre los puntos F y G?
A)2V D)5V

B)3V E) 1 V

C]4V UNI82-2
Resolución.-
Aprovechando las reglas o convenciones establecidas en el Teorema de la Trayectoria, iniciaremos nuestro
recorrido en F y terminaremos en G, de modo que se establece la siguiente relación para los voltajes:

VF = Potencial del borne F


V. +LV=V. {
F G VG = Potencial del borne G
En la sumatoria, V tendrá signo negativo si pasamos de un polo positi vo (+ ta un polo negati vo (-) y viceversa,
tendrá signo positivo si pasamos de un polo negativo (-) a un polo positivo(+); veamos:

VF + (- 1 V + 2 V - 3 V) = Vo RPTA.A

Probo 5.- Al conectar a un toma corriente de 220 vctts. una plancha se obtiene una corriente de
---- 8 amperios. Si la plancha fuera conectada 0110 votts. ¿qué corriente circularía por
ella? (en amperios).
A)4 B) 8 C] 16 D)6 E)3 UNI83 -2
Resolución.-
Dado que la plancha es una resistencia que no cambia con el voltaje, aplicaremos la Ley de Ohm para ambos
casos, pero en particular la relación que nos dá la resistencia:

VI V2 ~
R=-=- =} i2 = 4A RPTA. A
il i2

-,
412 Física - Primer Nivel Félíx Aucallanchi V.

Probo 6.- En el circuito, ¿CUól es el valor de R R

I]1730
(aproximadamente) de manera
que la resistencia de entrada
entre los terminales sea de 173 U?
A} 58 D} 173

B}87 E} 346

C} 100 UNI84 - 2
Resolución.-

A continuación presentamos las reducciones de las resistencias, según el tipo de conexión establecido, donde
además: r = 173 U. (Nótese que se reduce el circuito desde su parte más interna hacia afuera).

x u .... _--~SERIE
..__ .... _- •......
x ,R
;

u :
R '040 ••• __ .,

! R(R+r)
! 2R+r
_._-'
y y y y y
(1) (2) (3)

De (3) podemos deducir que la resistencia equivalente Re entre los borne s x e y del circuito, será:
R (R+r) .. , R(R+ r)
Re=R+ 2 R ' Y por condición del problema: r = R + --'-~
+r 2R+r
R(R+r)
:::) r-R= 2R+r :::) (r-R)(2R+r)=R(R+r) :::) 2rR+r2-2R2-Rr=R2+rR

r 173
R=-r:; = 1,73 R=l00n RPTA.C -

Probo 7.- Se conectan tres resistencia de


Igual longitud y sección (L= 1m
S= 1mm2J por medio de alam-
bres conductores a una batería
como se muestra en la figura. si
lass resistencia tIene una
resistividad r = t5.1(j8 U-m, la
resistencIa equivalente seró
iguala:

A) 0,01 U D}0,04 U
B} 0,02 U E} 0,05 U
C}O,03 U'
UNI92

,
Electrodinámica 413

Resolución.-
Calculemos primero el valor de la resistencia R de
cada alambre por medio de la relación (22.3);
recordando además que: 1 mm2 = 10-6 m2:
L -8 1m Y__~~-;JVVVV'J~~
R = P S = (1,5. 10 nm) 10-6 m2

~ R = 0,015 n
Ahora, de la instalación indicada se puede reconocer x ('
que al instalarse los puntos x - y - z, siendo x e y los
bomes de ingreso, la resistencia equi valente estará dada
así: R
R .2R 2
RXy=R//2R ~ Rxy = R + 2 R = '3 R

Rxy=o,Oln RPTA.A
y
-----c:. SERIE: 2R

Probo 8.- Se reo/iza e/ siguiente experimento:


aJ Se unen los terminales 1y 2 Yse mide la corriente que pasa por el amperímetro A: I = 3 n
bJ Se conecta una resistencia de 3 n entre 1 y 2 Y se vuelve a medir la corriente que
pasa por A: l' = 1 A.
R
Determine el valor de (1, y R
AJ & = 4,5 V R = 1,5 n
8J & = 3,0 V R = 1,0 n
C] & = 9,0 V R = 3,0 n
DJ & = 6,0 V R = 2,0 n
1_& _----:2
EJ Faltan datos UNI 91
Resolución.-
De acuerdo con la condición del problema, se tiene
dos casos:
(A)
Jll) CircuitoA.- Aquí la corrienteil = 3A, fluye debido
a la alimentación directa del voltaje & sobre la
resistenciaR; luego, por la ley de Ohm (\-iR), se tendrá:
2
& = i.R ~ & = 3R (1)
1
211) Circuito B.- En este circuito la resistencia
equivalente es (R+ 3 n) y la corriente que fluye es (B)
i2 = 1A. Luego, aplicando nuevamente ViR,tendre- H1
mos:

&=i2(R+3) ~ &=I(R+3) (2)


2
Resolviendo (1) y (2) obtenemos: =
(1, 4,5 V Y R = 1,5 n RPTA. A

Nota aclaratoria» En los circuitos A y B se ha obviado la presencia del amperímetro, porque ésta solo nos
indica la presencia y el valor de una corriente.

,
414 Física - Primer Nivel Félix Aucallanchí V.

Probo 9.- El amperímetro del circuito que se muestra


en la figura marca 0,55 A. SiR = 24000.
Y R es desconocido, hallar el valor de R
sabiendo que la diferencia de potencias
en los extremos de I{ 1 es de 120 volt.
A
A] 120 O O] 200 O
B] 260 O E] 240 O
C] 280 O UNMSM 94 R

Resolución.-

De acuerdo con los datos y el esquema adjunto, R¡


aplicaremos ViR en la resistencia R¡:

V=i¡.R¡ ==> 120V=i¡.(24000)

Ahora, del nudo P, podemos reconocer que la


==>i¡=0,05A

corriente i se divide en dos: i¡ e i2; luego, deberá


cumplirse que:
i
---<>
p -- - v Q

i¡ + i2 =i ==> 0,05 + i2 = 0,55 ==> i2 = 0,50 A R

Por último, volvemos aplicar ViR en la resistencia inferior R que buscamos, la que por estar en paralelo con R 1
experimenta el mismo voltaje V; luego:

V = ir R ==> 120 V = (0,50 A) R ==> R = 240 a RPTA. E

Probo 10.- En la figura adjunta el amperímetro 2ohmios


(A] indica una corriente de 3
amperios. La diferencia de poten-
cial entre los terminales de la
resistencia de 2 ohmios es, en voltios:

A] 9 O] 4

B] 6 E] 12
4ohmios
C] 18 UNMSM93
Resolución.-

Sea V la diferencia de potencial entre los terminales b y e de la resistencia de 2 O; luego, ésta es la misma que
existe en la resistencia de 4 O, dado que aquellas están conectadas en paralelo, Si reducimos estas resistencias
y colocamos su equivalente R entre los terminales b y e, se conservará la diferencia de potencial V y se
observará que la corriente en el amperímetro no ha cambiado su valor original: i = 3 A.
Luego, calculando el valor deR:

40.20 4
R= =-0
40+20 3 a
y ahora, aplicando ViR, tendremos:

V = iR = (3A{~ O) ==> V = 4 voltios RPTA.D

,
Electrodinámica 415

Probo 11.- En el circuito que se muestra en la figura determinar la corriente que circula por la
resistencia de 6 ohmios.
A) 5,8A
35V
B) 2,5A
C)3,4A 60
O) 4,7 A
e
E) N.A.

UNFV87
Resolución.-

11111111-------, 1111111--------, - ~1111r-+-----=Jl •


3SV 35V 35V

40

(2) (3)

90.180
En (1): R¡ = 90+180 ~ R¡ =60
En (2): x es la corriente que buscamos y que es igual para las dos resistencias por ser ellas de igual valor ..
. . 60.R¡
ASimismo: ~ = 60+ R¡ ~ R2 = 30

En (3), aplicando ViR tendremos: 35 V = i (40 + R2) ~ i = 5A


Finalmente reconocemos por el circuito (2) que: 2x = i ~ x = 2,5 A RPTA. B

Probo 12.- Cuando se abre el interruptor


S en el circuito mostrado, la
Intensidad de corriente es de s
3 A Y cuando se cierra, la
intensidad de corriente es
18 A. Los valores de la 100
diferencia de potencial entre
los terminales de la batería y
de la resistencia R serán:
R
A) 10 V; 5 O
v
B) 18 V; 9 O 1
<r-
C) 20 V; 2 O
0)36 V; 20
E) 40 V; 4 O UNI93-2
416 Física - Primer Nivel Fé/ix Aucallanchi V.

Resolución.-

1) Interruptor abierto.- Aplicamos ViR a todo el


circuito:
ti, = /1 (R + 10 O) ~ ti, =? (R + 10) .... (1)
2) Interruptor cerrado.- Vemos que la resistencia de
10 O queda cortocircuitada, y por tanto fuera de
servicio, ello quiere decir que el circuito queda con
una resistencia: R, si aplicamos ViR, tendremos
que:
ti, = /2R ~ ti, = 18 R..... (2)

Resolviendo (1) y (2) encontramos que: R

ti,=36V y R=20 RPTA.D

Probo 13.- En el circuito de la figura sucede que


si la llave S se abre y T se cierra, el
voltímetro marca 12 voltios; si S se
cierra y Tse abre el vottímetromarca
16 voltios. Si ambas llaves se cierran,
el voltímetro indicará en voltios:

A)O
8)9
C) 12

Resolución.-

T
'-------'
(1)

Circuito (1): Aquí el interruptor S está abierto y T está cerrado, luego: il = R:V = ill
12 V
= 4 A
Y aplicando ViR en todo el circuito: ti, = 4 (r + 3) (1)

V2 12 V 8
Circuito (2): Aquí el interruptor T está abierto y S está cerrado, entonces: i2 = R = 60 ="3 A
2

Y aplicando ViR a todo el circuito: ti, ="38 (r +6) (2)

Resolviendo (1) y (2): ti, = 24 V Y r =3O


Circuito (3): Aquí los dos interruptores están cerrados y el voltímetro indica el voltaje V para las dos
resistencias (RI y R2) que quedan en paralelo; luego:
Electrodinámica 417

R¡. R2 3Q. 6Q
R=---= => R=2Q
R¡ + R2 9Q
{1, 24
Circuito (4): Deducimos que: i = r+R = sn =
V
4,8 A

Y aplicando ViR en R: V = iR = (4,8 A) (2 Q) => V = 9,6 voltios RPTA.B

Probo 14.- Una cocinilla eléctrica posee la siguiente descripción garantizada por el fabricante:
220 voltios, 880 vatios. La energía consumida en kW . h durante 10 horas de
funcionamiento es:
A] 4,0 B] 0,25 C]4 D]8,8 E] N.A. UNFV87
ResoluCión.-
Con los datos del ejercicio calcularemos la energía eléctrica (W) que se consume aplicando direcramente la
relación (10.7) para lo cual solo convertiremos los vatios en kW: Pot = 880 W = 0,88 kW; luego:

W=Pot.t=(O,88kW)(lOh) => W=8,8kW.h RPTA.D

Probo 15.- ¿Qué corrIente circula por una /interna que usa pila de 3 Vy una bombilla de O,50 W?
A]O,214A B]O,166A C]0,119A D] 0,081 A E] 0,064 A
Resolución.-
Reconociendo los datos: V = 3 voltios, y Pot = 0,5 W, aplicaremos la relación (22.9) para la potencia eléctrica
de donde obtendremos la corriente (i) solicitada:

Pot = Vi => 0,5 W = (3 V) i => i '" 0,166 A RPTA.B

Probo 16.- En el diagrama la lámpara de


resistencia R 1 = 120 Q está
conectada a una resIstencia
variable R2' La potencia eléctrica
disipada txx la lámpara
cuando R2 = 120 Q está dada por:
"T
120V

A] 7200J
B] 30 W
D] 120 W
E] 480J "--- __ 1
C] 30 J UNMSM 86
Resolución.-
Dado que R¡ = R2 = 120 Q, deducimos que ambas
resistencias experimentan el mismo voltaje:
V¡ = V2 = 60 voltios, dado. que ellos están en serie. Así,
la potencia disipada por la lámpara será:
V? (60 V)2
Pot =- = => PoI = 30 W RPTA. B
R¡ 120 n

Probo 17.- Cuando dos resistenciasiguales son conectpdas en paralelo a una batería la potencia
total disipada por las resistenciases 40 W Siestasresistencias se conectan en serie con
la misma batería la potencia disipada será:
A] 160 W B] 80 W C] 20 W D] 10 W E] 5 W UN194-1
418 Física - Primer Nivel Félix AucalJanchí \/.

Resolución.-

De acuerdo con el circuito en paralelo indicado tenemos +


que la potencia que consume el sistema estará dado
Rt ,
por la relación (22.9), siendo la resistencia equivalente: \ ¡ i
'-' R :; y
R¡ =R.R/2R = R/2. Luego: :
L.._.._. F.'.F'. ._ ••__.J
V2 V2 V2
POli =- => 40 W =- => 20 W ..... (1)
R¡ R/2 R
Y según el circuito en serie tendremos que laresistencia ·······!
r .. ············R·· __
equivalente será: R2 = R + R = 2R. Luego, la potencia
+
que consume el sistema estará dada por:

POl2
2
= V~ = 2R
V 2
..... (2)
tJr y

Finalmente de (1) en (2): ·······..jf··········¡

Po12= 10 W RPTA.D

Probo 18.-Tres resistencias iguales se conectan en serie, y cuando se aplica una cierta diferencia
- de potencial a la combinación ésta consume una potencia total de 10 W. ¿Qué
potencia seconsumirá si las tres resistencias se conectan en paralelo a la misma
diferencia de potencial?

A)90W B)45W C)15W O) 135 W E) 5,3 W UNMSM91


Resolución.-

Procediendo de un modo similar al del problema anterior, tendremos:


a) Cuando están en serie.- La resistencia equivalente del sistema esR¡ = 3R, luego la potencia estará dada por:

v2 V2 v 2
POI, = -R¡ =>
.
10 W =-
3R
=> R =30 W (1)

b) Cuando están en paralelo.- La resistencia equivalente es: R2 = R/3, entonces la potencia será:

Pot
v2
=-=-
V2
=> Pot2 = 3(VR2) ....... (2)
2 R2 R/3

y reemplazando (1) en (2): POl2 =90 W RPTA.A

Probo 19.- Se desea calentar agua de 25°C a 75°C (m = 60 g) por medio de la resistencia R 1
en el circuito mostrado. ¿Cuánto tiempo debe circular corriente por dicha resistencia?
Si: Rl = 25 Q, R2 = 5 Q, Y V = 240 voltios.

A) 7,81 s
B) 3,00 s
C) 1,54 s
0)8,25 s
E) 0,25 s UNFV83
Electrodinámica 419

Resolucién,»
Calcularemos primero la cantidad de calor (Q) que se requiere para calentar el agua, recurriendo por ello a la
relación (17.4) del calor sensible:

Q = (mCe Li 1)H 0= cal • (75' C - 2.J-


60 g • I -----o--c '" C) ::::} Q= 3000cal
2 g.

Sea W la energía eléctrica que R2 convierte en calor (Q), de manera que al expresar ésta energía en joules,
mediante la siguiente regla de tres simple obtendremos:

I 1~ 0,24 cal } W = (1 1)(3 000 cal)


::::} W =125001
W ~ 3000 cal 0,24 cal

Ahora determinaremos la intensidad de corriente ique circula por R1, para lo cual utilizaremos ViR para todo
el circuito:
V = i (R 1 + R2) ::::} 240 V = i (25 Q + 5 Q) ::::} i = 8 A

Finalmente emplearemos la relación (22.8) para la energía eléctrica, de donde despejaremos el tiempo (r)
solicitado:
RPTA.A

Probo 20.- Enel siguiente circuito se in-


dica la potencia consu- ~
mida por cada elemento. I ¡-----,------,-------,
Luego, la corriente eléctri-
ca I extraída de la fuente
es aproximadamente:
220 V
A) 1,08A O)O,38A

B) 4,93A E) 0,21 A

C) 10,22 A UNI86
Resolución> 1
---<>
Dado que se conocen las potencias que consumen los + RESISTENCIA
elementos por separado, podemos decir que la potencia_ EQUlVALENTE
total que consume el sistema será:
Pot = 60 W + l 000 W + 25 W ::::} Pot = 1 085 W t
V
Pot= 1 085 W
/
Esta potencia es proporcionada al circuito por una
corriente total (J) que se nos solicita calcular, y a un

t
voltaje V = 220 voltios.

Luego, empleando la relación (22.9) tendremos:

Pat = V.I ::::} l 085 W = (220 V) l ::::} 1 •••4,93 A RPTA.B

Probo 21.- Un calentador eléctrico tiene una resistencia de 20 Q. ¿Cuántos de tales calentadores
pueden ser conectados (en paralelo) a un tclnacorriente casero de 220 voltios, sin
que el medidor pase de 25 amperios?
A) 1 B)2 C)3 0)4 E) 5 UNI83-2
420 Ftsica- Primer Nivel Félix Aucallanchi V.

Resolución.- i=2SA
--{:>
Con los datos: V = 220 voltios, e i 25 A, = +
encontraremos la resistencia equivalente (R ) de los
N calentadores instalados: e

V 220 V
Re=¡=TsA ::::}Re= 8,8 n 220 V

R R R R
y dado que los calentadores tienen la misma resistencia
R = 20 n, diremos que su resistencia equivalente estará
dada por:

Re= Z ::::} 8,8 n=2~ n ::::} N= 2,27 N resistencias


Así, el máximo número de calentadores eléctricos que se pueden instalar sin pasar la corriente dada será:

N=2 RPTA.B
Observacián.« Si el número de calentadores es mayor que dos (2), la resistencia equivalente (R~ se reducirá,
y la corriente 1 será superior a los 25 A.

Probo 22.- Un galvanómetro tiene uno resistencia interno de 1ny codo dMsión de su escalo represento
1 mA. Se quiere utilizar el mismo instrumento poro medir amperios en lo mismo escala.
AproJdmadamente, ¿Que resistencia hay que colocar en paralelo en el instrumento?
A) 0,001 n B)0,05n C)1,OOn D)5,On E)10n UNI83-1
Resolución.-

Según los datos y condiciones podemos decir que


cuando el galvanómetro marque por ejemplo I mA
en una escala, es que la corriente que mide el
_l_mA~~~ ~GQD~ _
instrumento es I A; lo que sucede es que el
resto: 1A • 1mA = I 000 mA • 1 mA = 999 mA, se
derivan hacia la resistencia de protección R2. Así,
por tratarse de una instalación en paralelo, y
usando ViR,tendremos:
ilR, = i2R2 ::::} 1 mA .1 n = (999 mA) R2 -c-
lA
Rz",O,OOl n RPTA.A

Probo 23.- En el circuito en serie. 2~In 40 6!i...!0 20 l~ In

Calcular Vea: a+~II-.-d----,NN'---e-+~II -.-f---'


A) . 9 voltios

B) ·51 voltios
C) 10 voltios
30
g
so
O) 51 voltios
E) N.A. UNFV89-2
Resolución>
Inspeccionando las baterías del circuito, podemos notar que éstas presentan resistencias internas conocidas;
luego, procediendo a separarlas, el circuito quedaría así:
Electrodinámica 421

an 50 j

Es notorio que todas las baterías generan corriente en el mismo sentido, por ello la fuerza electromotríz (&) del
sistema será: ti, = 24 V + 6 V + 12 V = 42 V. Luego, la corriente del circuito será:
ti, 42 V 7 &=42 V •
i=-=--=-A +
R2 18 n 3
A continuación calcularemos la d.d.p. entre a y e,
recorriendo la rama de "a" hacia "e" y aplicando el Teorema
de la Trayectoria tendremos:

Va - 24 V + iR 1 + iR2 = Ve => Va - Ve = 24 V - (R 1 + R2) i


7
=> Va - Ve = 24 V - (2 n + 4 n) 3" A => Va- Ve = 10 V RPTA. e R= 180

Probo 24.- Una resistencia de 5 ohm está &,r


conectada %S bornes de una
+ ~-~~----------------~
batería de r.e.m 12 voltios cuya a -<1--- b
resistencia interna es 1 ohm. Los
potencia/es en los puntos "a" y "b"
son respectivamente:
A)2V,10V O) 10V, 2V R
E) N.A. e
B) 10V, OV
d
C)OV,10V UNFV90
Resolución.-

Separando la resistencia interna de la batería de ésta, el


circuito presenta el aspecto que se indica. Luego, la
b
corriente del circuito será:
. ti, 12 V
a
ir
+ +n
I=R+r=Sn+ln => i=2A
A continuación determinaremos los potenciales de "a" y "d'
respecto de la Tierra, donde el potencial es cero, utilizando:
1) Rama abc: Va - iR = Ve +
iR
- e
ji
d -
=> Va - (2A) (S n) = O V => Va = 10 V VT=O
\
RPTA.B
422 Física - Primer Nivel Félix Aucallanchi V.

Probo 25.- Dado el siguiente circuito; determinar 54V 42V


la potencia que consume R = 4 Q.

Resolución>
A) 500 W
B) 120 W
C]300 W

Sea i2 la corriente que pasa por R = 4 Q. Luego:


O) 180 W

E) 324 W
E:I3
~~:
1) Nudo "x": i2 = i, + i3 (1)

2) Malla "T': 3 i, + 4 i2 = 54 (2) + +


3) Malla "JI": 4 i2 + 2 i3 = 42 (3) 3i1
Resolviendo (1), (2) Y (3) encontramos: i2 = 9 A,luego: +
Pot = ii R = (9 A)2 (4 Q) => Pot = 324 W RPTA. E

r4t' it ~

,.".'221M AUTOEVALUACION

1.- Indique verdadero (V) o falso (F)


( ) Para que circule corriente por un conductor macizo,
el campo eléctrico en su interior debe ser nulo.
( ) La corriente que circula en un conductor sólido
se debe básicamente al movimiento de sus
electrones libres.
( ) El sentido convencional de la corriente es siempre
del mismo sentido que el campo eléctrico.
A) FVV B) FFV C) FFF D) VFV E) VYF

2.- Señalar la proposición incorrecta:


1) La cantidad de carga que pasa por la sección trans-
versal de un conductor en cada unidad de tiempo se
ha convenido en llamar intensidad de corriente.
4.- ¿Cuál de las siguientes afirmaciones es cierta?
11) La cantidad de carga que pasa por la sección trans-
versal de un conductor en un segundo se ha 1) Una sustancia será mejor conductora de
convenido en llamar amperio. electricidad cuando menor sea el valor de su
I1I) Si la corriente en un circuito es de 5 amperes, resisti vidad eléctrica.
quiere decir que a través de la sección transversal 11) La resistividad es independiente de la masa del
del circuito pasan 5 coulombs en l segundo. conductor.
A) YYF B) YFV C) YFF D) FYY

3.- ¿Cuál de las siguientes gráficas representan mejor


E) FFV III) La resistividad
adimencionado.
B)IIyIlI
eléctrica

C)IyIl
,
es una constante

D) I E) II
la corriente real en un conductor metálico? A) I Y II
Electrodinámica 423

5.- Elige las palabras que completen mejor la siguien- A) YA = YB = Ye x----,-------r---,


te oración: «Si deseamos disminuir la resistencias de
un alambre debemos su sección recta B) YA> YB> Ye
y su longitud».
C) YA < YB < Ye
A) Reducir, reducir ~ D) Aumentar, reducir D) YB < YA < Ye
B) Reducir, aumentar E) Aumentar, aumentar ,r----~---L--~
E) YB> YA> Ye
C) Aumentar, conservar
10.- ¿Cómo deben permanecer los interruptores K y
6.- Señalar la relación correcta de los coeficientes de Q para obtener la máxima resistencia equivalente en
temperatura de la resistencia a O°C para los el circuito mostrado?
e
conductoores A, B Y cuyas resistencia varian con
la temperatura, tal como se indica en el siguiente A) K = cerrado
gráfico.
Q = abierto R
A) <XA = <XB = <Xe R e
B B) K = cerrado
B) <XA > <XB > <Xe = cerrado
A Q
C) <XA < <XB = <Xe R R ~
C) K = abierto
D) <XA = <XB> <Xe
Q = abierto R
E) <XA < <XB < <Xe o D)K = abierto
7.- ¿Cuál de los siguientes gráficos representa mejor Q = cerrado
la dependencia de la tensión (V) con la intensidad (i)
que experimenta un cuerpo conductor? E) N.A

11.- En el circuito mostrado, indique la lectura del


amperímetro ideal.

A) Cero
B)2V/R

C) V/2R

D)V/R

E) 3V/2R

12.- ¿En qué caso se disipa más potencia?


8.- Sabiendo queR¡ = 3 R2 encontrarla relación i¡li2
para el sistema indicado.
A)3
B)4/3

i ¡rr'---.J\NV\/V'----,
A)
i.-~t- R R R
B)
.ti R

C)2
D) 1/3
E) 2/3
9.- Señale la relación correcta de las iluminaciones
(Y) de los focos. Considere que los focos son iguales
C~:-d R

R
.f~"
A AU

R
(Vxy = 220V) E)
R

R ,
A) B)
424 Física-Primer nivel Félix Aucallanchi V.

PROBLEMASPRO'UESTOS
Nota: Todas las resistencias están en Ohmios (Q) A) 2 V B) 6 V C) 9 V D) 12 V E) 24 V

NIVEL 1 8.- Cierto alambre metálico de longitud L tiene una


resistencia eléctrica de 80 Q, si se formara un alambre
1.- En un calentador eléctrico ordinario, la tensión es más grueso del mismo material con la misma cantidad
220 Vy su resistencia 44 Q. Determinar la cantidad de metal de longitud U2 ¿Cuál será la resistencia
de carga (en C) que cruza dicho calentador en 2mín. eléctrica (en Q) de este nuevo alambre?
A)5 B) 200 C) 500 D) 600 E) 900 A) 10 B) 20 C) 40 D) 60 E) 160
2.- Determinar la caída de la tensión (en V) a lo largo 9.- Dos alambres de Nicrom de exactamente la misma
de un alambre de cobre de 314 m de longitud y 2 mm compasión tienen el mismo peso, pero uno de ellos
de diámetro, si por ella pasa una corriente de 5A
es 5 veces más largo que el otro. Si la resistencia
A)5 B)8 C) 8,5 D) 10 E) 16
eléctrica del más corto es R¡ = 5Q ¿Cuál es la
resistencia eléctrica (en Q) del otro? •
3.- Un alambre tiene una resistencia de 10 Q, si se
estira hasta triplicar su longitud permaneciendo A) 25 B) 50 C) 75 D) 100 E) 125
constante su volumen y su resistividad, determinar
la nueva resistencia (en Q). 10.- Se tiene dos baterías de iguales características
que al conectarse en serie tienen una duración de Idía.
A)9 B) 30 C) 90 D) 120 E) 1509 ¿Qué tiempo (en días) duran, si se conectan en
paralelo? Suponer en ambos casos que alimentan a
4.- El bobinado de un motor eléctrico es de alambre la misma resistencia.
de cobre si su resistencia antes de empezar a trabajar
es de 50 Q a 20°Cy después de trabajar varias horas, A) 1/2 B) I C)2 D)3 E)4
su resistencia se elevó hasta 70 Q. Determinar su
temperatura final en 0c. (aeu =4.10-3 °C-l a 20°C) 11.- Hallar la corriente (en A) que circula por un
calentador de 20Q para que en 10min caliente 432g
A) 20 B) 80 C) 100 D) 120 E) 60 de agua desde 20°C hasta 80 °C
5.- Calcular el costo (en S/.)de funcionamiento de A) 15 B) 12 C)9 D)5 E)3
una refrigeradora que durante 24 horas está conectado
a una línea de 220 Vy absorve una corriente de 2,5 A. 12.- Tres resistencias iguales conectadas en serie
El precio del kW-h es de SI. 5,00 consumen una potencia total de 10 W cuando se
conectan a una diferencia potencial ¿Qué potencia
A) 60 B) 66 C) 75 D) 80 E) 90
(en W) consumirán si se conectan en paralelo a la
6.- Si por "3R" atraviesa una corriente de 2A, misma diferencia de potencial?
determinar la potencia (en W) disipada por la
resistencia de 10 Q. A)30 B) 50 C)60 D)90 E) 120

A) 300 13.- Si la fuente tiene f.e.m. de S = ISO V Y la resisten-


cia de 6 Q disipa una potencia de ISO W, hallar la
B)320 potencia (en W) que disipa la resistencia "R"
2R
C)330
A) 360
D) 360
10 B)300
E) 400
C)260 60 R
7.- Una batería logra mantener en forma permanente
una corriente continua de intensidad 1 = 2,5 mA D) 220
durante 4 h, al cabo de los cuales la batería agota toda
su energía disponible que es de 432 J ¿Cuál era la E) 180 6
;uerza electromotriz de la batería?
Electrodinámica 425

NIVEL 2 A) 36 B) 72 C) 18 D)40 E) 12

14.- En el circuito mostrado, calcular la tensión (en V) 19.- Calcular la resistencia equivalente (en Q) entre
que entrega la fuente, si la corriente que circula por la bornesx e y.
resistencia de 5 Q, es de lOA. Las resistencias están en Q. 18
A)5
A) 60 2,5
B)90 B)8
y
C) 100 C)6
V 10 5 40
D) 125
D)9
E) 220
E) 10
15.- Hallar el valor deR (en Q) para que la resistencia 3
de 5Q disipe una potencia de 20 W
A) 10 20.- Calcular la resistencia equivalente (en Q) entre

50vrfR
R
los bornes a y b.
B) 12
24
C) 16 A)4 •
D) 20 B)5
E)25 a b
C)9
16.- Durante cuánto tiempo (en min) debe circular
corriente a través de un alambre de 5 Q de resistencia D) 4,5
que está incrustrado a un trozo de 48 g de hielo a
-60°C para que lo vaporice totalmente al aplicar 50 V E) 5,4 4
a los extremos del alambre.
21.- Calcular en qué relación se encuentran las
A)2 B)3 C)5 D)6 E)9
resistencias Ra y R c!' si éstas son resistencias
17.- Al cabo de qué tiempo (en horas) después de equivalentes de~ circ~lto mostrado desde ab y cd
cerrar el interruptor hervirá el agua, que inicialmente respectivamente.
estaba a 40°C, siendo su masa 576 g.
A) 1/2
2
A) 1 10V a --1~ e
B)2
B)2
rl
S 7 12 3
C)3
C)3 9/16
D)5 b d
D) 4 5
E)6
E)5
22.- Calcular la resistencia equivalente (en Q) entre
18.- Calcular la potencia (en W) que disipa la los bornesxey. I
resistencia "R", la resistencia equivalente entre los
puntos A y B es 34QJ II Y la diferencia de potencial
entre los mismos puntos es 12 V.
A~.--.---.r-----~---.--~
4
A) 1 B)2 C)3 D)4 E)5

23.- En el circuito mostrado, determine R (en Q)


entre los puntos "x" e "y" Todas las resistencia son
B 0-&-_--'--_-' iguales a R = 4 Q
426 Física-Primer nivel Félix Aucallanchi V.

A) I NIVEL 3
B)3 28.- Calcular la potencia que es capaz de disipar el
conjunto de resistencias mostrados, si todas las
C)5 resistencias son iguales a R = 2 Q
D)7 A) 50 W
E)9
B) 100 W

24.- Determine la resistencia equivalente (en Q) entre C) 150 W 20V


A y B si los valores de cada una de las resistencias se
dan en ohmios. D) 200 W
A)5 2
E) 250 W
B)4
29.- En el circuito eléctrico mostrado las resistencias
C)3 son iguales a "R". Si la llave "S" está abierta, la
resistencia equivalente entre los puntos A y B es
D)2 igual a 9Q. Determinar la resistencia equivalente
2 3 (en Q) cuando "S" está cerrado, entre A y B
E) I
A)3
25.- Un foco cuya resistencia es de R = 29 Q es
alimentada por una asociación de pilas en serie, de B)4
modo que la corriente a través del filamento del foco
es de O,5A ¿Cuántas pilas de f.e.m. = 1,5 V Y C)6
resistencia interna r = 0,1 Q deben usarse?
D)8
A)5 B) 10 C) 15 D) 20 E) 25
E) 10
26.- La caída de tensión en la resistencia R es 0,5
voltios. Determinar la caída de tensión en la resistencia 30.- Determinar la resistencia equivalente (en Q)
de "8 R" entre A y B. Todas las resistencias son iguales a
R= 17 Q.
A) 2 V R
A) 5 Q
B) 3 V 8R
B) 3 Q A
2R
C)4 V
C) 8 Q
D) 5 V
D) 10 Q
E) 6 V
E) 25 Q
27.- En el circuito mostrado en la figura, hallar la
corriente que pasa por la resistencia de R = 10 Q. 31.- Determinar la resistdncia equivalente (en Q)
Todas las resistencias se dan en ohmios. entre los puntos A y B. Todas están en Q.

A)IA A)2

B) 2A B)3

C) 3A 30V C)4

D)4A D)5

E) 5A E)6
Electrodinámica 427

32.- Si todas las resistencias son iguales a R = 26 n. 18 V R 8 V


Hallar la corriente que entrega la fuente. A o-IJ f-----J/IIImr--ll--<> B
A)IA
A) 10 B)50 C) 100 D) 150 E) 200
B) 2 A
C) 3 A

D)4 A
54Vtm 37.- Calcular la diferencia
puntos "a" y "b"
de potencial entre los

E) 5 A ~~\~ A) -10 V

B) 20 V
33.- Una terma eléctrica usa un sistema resistivo
como se muestra en la figura ¿Qué tiempo debe C) -15 V
circular la corriente para lograr hervir I080g de agua
que inicialmente a 20 ·C? D) 25 V
120V
A) 50 s E) 55 V

B) 60 s 38.- En el circuito mostrado, hallar la corriente "1"
C) 80 s A) 5 A 2 1
D) 100 s

I~I24V
B) 4A

'1k
E) 120 s C) 3 A 16 V

D) 2A
34.- En el circuito mostrado hallar la corriente" 1" ,2'AYlAJ
que circula por 2 n E) lA 1
A) 2A 60 V 39.- A partir del circuito eléctrico que se muestra,
determine el valor de la potencia eléctrica disipada
B) 5A

2E:·f "1
por la resistencia R = I n
C) 6A A) 114 W
D) 8 A
E) 10 A
20 V
B) 112

C) 110 W
W
rW~ In

25 V R
35.- Determinar la potencia eléctrica que disipa la D) 225 W
resistencia de 4 n
E) 196 W
A) I W 15 V 6 2
B)3 W rHYm~ 40.- Del circuito mostrado, determine el potencial
eléctrico en el puntb "A". Las resistencias están en
C) 6 W

D) 9 W
4LJ:jIOV n.
A) 1 V

10 V 3 B) 2 V
E) 12 W
C) 3 V
36.- Un circuito tiene la sección AB, mostrada en la
figura; sabiendo que la d.d.p. entre los puntos A y B D)4 V
es 20 V ( VA - V B = 20 V), determinar la potencia
(en W) disipada por la resistencia R = 6 n. E) 5 V
428 Física - Primer Nivel Félix Aucallanchi V.

Lectura N° 4

La resistencia eléctrica de un conductor metálico puede disminuirse mediante enfriamiento;


pero: ¿Hasta qué punto puede hacerse esta disminución? Aunque parezca extraño, puede irse
hasta los extremos que se desee con algunos materiales denominados superconductores.

Recordemos la relación (22.4): R


T2
= RTI (l + aT1 L'l.1)

Al sustituir L'l. T = __1_ obtendremos que R = O.


aTI T2

ASÍ, habríamos hecho lo posible por lograr un estado por ahora hipotético en el que los
cuerpos pierden toda su resistencia eléctrica. Esto significa que tal situación aparece si la
temperatura final es notablemente baja, pues como se sabe, aT1 es muy pequeña.

Es en 1911 que el físico holandés H. Kammerlingh Onnes observó que cuando se enfriaba
una muestra de mercurio con helio líquido, su resistencia desaparecía en forma brusca, y
aparentemente por completo a 4,2 K. Asimismo, estimó que la resistencia en el estado
superconductor era cuanto mucho 10-12 de la resistencia en el estado normal.

Hace unos 40 años, en el MIT (Instituto Tecnológico de Massachusetts) se encontró que


una corriente de varios cientos de amperios en un anillo de plomo superconductor no mostraba
cambio en la intensidad de la corriente durante un periodo de por 10 menos un año; ésto es una
gran evidencia de que la resistencia eléctrica en el estado superconductor es en efecto cero.
«n
Se ha encontrado que por 10 menos 22 elementos y
cientos de aleaciones y compuestos intermetálicos son
R
0,15
Q
.
superconductores con temperaturas de transición que varían
0,10-
..
desde esencialmente menos de 1K. La temperatura de transición,
o crítica, es aquella en la cual una sustancia se vuelve 0,075-
superconductora, y es característica del material particular que
se considera; depende además en cierto grado tanto de la pureza 0,05 -

química como de la perfección metalúrgica de la muestra que se


ensaya. La Fig. 1 muestra cómo la resistencia de un material
0,0251- 10"0
<. r(K)
superconductor cae bruscamente hasta cero cuando alcanza su 0,00 410 4,30 440
temperatura de transición, y la Tabla 1 contiene una lista de algunas Fig 1
sustancias superconductoras con sus correspondientes-
TABLA!
temperaturas de transición.
APLICACIONES DE LA SUPERCONDUCTIVIDAD ALGUNAS SUSTANCIAS
SUPERCONDUCTORAS
En la actualidad, la superconductividad se emplea en: Sustancia Temperatura de
transición (K)
* Los viajes espaciales, especialmente en los viajes hacia
planetas lejanos, dado que las corrientes generadas en los Aluminio 1,2
circuitos ocasionan un consumo de energía mínimo. Indio 3,4
* La Informática. Estaño 3,7
* Aceleradores de partículas. Plomo 7,2
* Generadores de fabulosos campos magnéticos, etc. Niobio 9,3
E Iectromaon etismo
OBJETIVOS
1.- Establecer las principales caracte-
rísticas y leyes que describen los
fenómenos magnéticos.
2.- Conocer los efectos magnéticos de
las cargas móviles.
3.- Comprender el fenómeno de la
inducción electromagnética y la
generación de corriente alterna.

n la actualidad se sabe que la propiedad lIamadamagnetisnw que


poseen determinados cuerpos y que consiste en atraer pedaci-
tos de hierro (limaduras) es simplemente una manifestación de
la electricidad en movimiento. Se cree que el magnetismo fué descubierto en
la región de Magnesia, antigua ciudad del reino de Lidia en Asia Menor. Las
atracciones magnéticas fueron estudiadas por Tales de Mileto (640-546 a
N.E.), Y las repulsiones por Roger Bacón (1214-1294). Los primeros tratados KARL F. GAÜSS
sobre el magnetismo fueron publicados en 126g (Epístola de Magnete) y en
1600 (De Magnete) por Pedro Peregrino y William Gilbert respectivamente. (1777-1855)
Todos estos estudios contribuyeron a desarrollar una rama de la Física
llamada Magnetostática, la que inicialmente se encontraba desconectada Este brillante científico
vino al mundo en 1777 en
de la electricidad, con la cual hoy forma la ciencia denominada la ciudad alemana de
Electromagnetismo. Brunswlck. Estudió en
Gottlnga, en cuya univer-
sidad se dedica a la
mIMAN Astronomía y enseña Ma-
Llamamos así a todos aquellos temáticas. A partir de
1833, y junto con Wilhelm
cuerpos que tienen la propiedad de
atraer limaduras de hierro. Entre los '¡Il:::::"'----- Polos..-----'"
~) ::::::I:~ Weber, desarrolla toda to
teoría matemática del
cuerpos que poseen magnetísmo en Magnetismo.
forma natural tenemos a la magnetita, EsInventor del mar:;;netls-
mineral de hierro cuya fórmula química mo bifilar y del telegrafo
eléctrico. Todavía en la
es Fe304 = Oxido Ferroso Férrico. Segunda Guerra Mundial
Los imanes tienen determinadas se usó con éxito su cintu-
regiones denominadas polos magnéticos, rón de desmagnetlzación,
donde parece concentrarse más su con el que muchos barcos
se libraron de las minas
magnetismo del modo como se muestra magnéticas.
en la Fig. 23.1a. Si suspendemos una barra Es muy conocido en
imán desde su centro de gravedad veremos Física el Teorema de
que al quedar en reposo se orienta en la Gaüss,que tiene una gran
direccióngeográficaNorte-Sur(verFig.23.lb). L-~_~~ -=:---=-="--, aplicación el campo de
Fig.23.1 la electricidad.
430 Física - Primer Nivel Félix Aucallanchi V.

MUY IMPORTANTE Llamaremos polo norte (O positi va)


al extremo que señala al norte geográfico,
Si partimos un imán y y polo sur (o negativo) al que señala al
continuamos con este pro-
ceso, comprobaremos s.ir geográfico. Los imanes pueden ser
que cada fragmento con- n••turales o artificiales preparados por el
tinúa manteniendo sus dos hombre, y al elaborarIos se les suele dar
polos, lo que probaría la
inseparabilidad de los po- la forma de barra, aguja o herradura.
los,Inclusoa nivelmolecular. Cuando el imán es recto, puede L
N s
verificarse de modo experimental que
9,>-----
I 9
---__1'9
= ª¡
I () cada polo está de su extremo más ~d~ ~d~
cercano a la distancia: d = L/12, '-- -=--=~
siendo L = longitud de la barra. Fig.23.2
§$ () §§
s:::::Js=J s=J s:::::J
()
11I CARGA MAGNETICA ( q*)
INTERESANTE Denominamos así a aquella magnitud física escalar que va asociada
a todo polo magnético, y nos indica de un modo directo el nivel de
La imantaclón de un magnetismo que posee. En una barra imán se verifica que los dos polos
cuerpo se sustenta en el
orden que manifiestan los tienen la misma carga magnética pero de signos diferentes. En el S.l. se
«Imanes mctecutares», el expresa en Ampere. metro = A.m.
mismo que se pierde si
elevamos su temperatura. • INTERACCIONES MAGNETOSTATICAS
Se denomina Temperatura
de Curie a aquella en la que A) Ley Cualitativa- En forma experimental podemos comprobar
un imán pierde su
imantación. En el caso del que: «Dos polos de la misma naturaleza (o nombre¡ se repelen,
fierro, esta temperatura es y de naturaleza diferente (o nombres diferentes) se atraen».
de 750"C.
B) Ley Cuantitativa.- Esta ley permite cuantificar una de las fuerzas
naturales más importantes de la naturaleza, y fué descubierta por
DEBES SABER QUE:
Charles A. Coulomb el mismo año que publicó su trabajo
En el sistema antiguo electrostático equivalente. De acuerdo con esta ley se establece
M.K.S. racionalizado, la que: «Dos cargas magnéticas se atraen o se repelen con fuerzas
masa magnética (m) se de igual intensidad pero de direcciones opuestas, y cuyos valores
expresaba en weber (IVbJ la son directamente proporcionales con el producto de las cargas
cual está relacionada con
su nueva denominación: magnéticas, pero inversamente proporcional con el cuadrado
carga magnética (q*) en el de la distancia que las separa».
S.I. por medio de la
siguiente expresión: Ql*·Q2 *

q*= k: ·=}I m =q*.km I => F = km' 2


d
(23.1)

Endonde al sustituirsolo las !lo


unidades correspondientes, donde: km = 41t = l O-7 N/A 2 r-- ~- .".....,-=---

tendremos:

I Wb<>(1 Al») (¿- J Observaciones.» I~


. F
la) La relación (23.1) solo se utiliza I q'"
r
[1 WJ<>IIt I
para el caso de cargas magnéticaspuntuales. 1
..
¿Puedes encontrar la
equivalencia entre 1 Wb y
2/l) La constante fJo se conoce como
permeabilidad magnética deLvacío,
y en eLS./. es igual a 4rc.JO·7 N/A2.
l Fig.23.3'
1 A. m?
Electromagnetismo 431

11I CAMPO MAGNETICO


NO OLVIDAR!

La misma regla para


Así como las cargas eléctricas, los polos magnéticos interactúan dibujar el V.!?ctor campo
entre sí a través de sus campos magnéticos. De este modo, llamaremos eléctrico ( E) se emplea
campo magnético a aquella región de espacio que rodea a todo polo para graftcar el vectot
magnético, y que posee propiedades magnéticas, las cuales se campo magnético (BL
manifiestan con fuerzas de atracción o repulsión contra todo polo Observa:
colocado en su interior.
A) Intensidad de Campo Magnético (ii).- Se le llama también
inducción magnética, viene a ser una magnitud vectorial que para +q*
un punto el campo se define como la fuerza que recibiría la unidad de
carga magnética colocada en dicho punto. Así pues, su valor se ®
determina así:
-q*

lB = :. I (23.2) o •
La unidad deB en el S.l.es el testa
(1), y se define así: l T = 1 N/A.m
Si la carga magnética Q* que CUIDADO!
genera el campo es puntual, entonces
Al reemplazar datos en las
se cumplirá que en un punto P, la ~ d
relacIones (23.1Jy (23.3Jno
intensidad del campo estará dado así: se deben IncluIr los.sIgnos
de las cargas magnéticas,
- Q* porque lo que se determIna
IBI=km-z (23.3) con dIchas relacIones es el
d Fig.23.4 módulo de la fuerza IFI y
del campo 181.
B) Líneas de Fuerza.- Históricamente se sabe que el concepto de
línea de fuerza fue inicialmente propuesta por Michael Faraday para
describir los campos eléctricos. Así pues, como en el caso de las
cargas eléctricas, los campos magnéticos se grafican por medio de
líneas que salen de los polos norte e ingresan a los polos sur, notándose
ahora que las líneas de fuerza se cierran sobre sí misma, lo que no ocurría
en los campos eléctricos. ATENCION

11II FLUJO MAGNETICO (el» Analizando la relación


(23.4) y la Flg. (23.5J
Una mayor o menor concentración de las líneas de fuerza nos concluiremos que:
permite tener una idea de lo intenso que es el campo magnético en 1¡¡) Existeflujo máximo sIlos
dichas regiones. Se define así el flujo magnético como aquella líneas cortan perpendi-
magnitud escalar que nos indica el número cularmente a la super-
de líneas de fuerza que atraviesa una ficIe A; ésto es:
superficie imaginaria. Su valor resulta ser
directamente proporcional con la 6= 0° => <l>máx = B.A
componente del campo (B ) normal a una
2QJ ExIste flujo mínImo sI
superficie y con el área del a misma. Así: ninguna línea logra

=
<l> Bl..A, ó I fI) =BAcosO I (23.4) atravesar la superficIe
A; ésto ocurre cuando
las líneas son parolelas
La unidad de <l>en el S.I. es el weber; a dicha superfIcIe, y:
tal que: l Wb= l Tm2. Asimismo:
l Wb < > 108 líneas de fuerza. Fig.23.2
432 Física - Primer Nivel Félix Aucallanchi V.

DEBES SABER QUE:

Existe la magnItud
11I PERMEABILIDAD MAGNETICA
denomInada: Campo Los materiales se diferencian
Magnetizante (il) que unos de otros por su mayor o menor a)
depende de la geometría cualidad de concentrar las líneas de
del sIstema pero no del fuerza de un campo magnético. Se
medio, y es en buena
cuenta el agente que define la permeabilidad magnética
promueve la magnetlzaclón relativa (!lr) de una sustancia como el
de los cuerpos. Su valor se cociente: b)
encuentra así:
Flujo con núcleo
~ Ilr = Flujo sin núcleo
~
La unidad de (H) es el
(Impere por metro = Alm. Enla (23.5)
Fig23.6
mayona de la bibliografía
publicada, incluso hasta la

década pasada, denomI- Asimismo, se define la permeabilidad
nan a H intensIdad de cam-
po magnético, nombre
magnética absoluta (!la) de una I Pa = Po' Pr 1 (23.6)
que en la actualidad se sustancia a la relación: ..
reserva para B.
• MAGNETISMO TERRESTRE
La orientación de los imanes en el aire se debe a que la Tierra se
comporta como un poderoso imán, cuyo norte magnético se ubica
cerca del sur geográfico, y
PERMEABILlDADES el sur magnético cerca del
RELATIVAS norte geográfico. Se llama
inclinación magnética al
ángulo que forma un imán
Material J.I,
con la horizontal en un punto
Fierro MagnétIco 200 de la Tierra, y declinación
Niquel 100 magnética al ángulo
Permalloy 8 oao formado por el imán con un
Numetal 20 000 meridiano geográfico,
ambos muy usados en
Ferrita 1 500
navegación.

PROBLEMAS RESUELTOS Fig23.7

Probo 1.- En la figura se muestra una barra imán


de 36 cm de longitud cuyos polos
magnéticos tienen una carga magnética
de ó.1[J3 A.m. Frente a él se encuentra un
polo sur cuya carga magnética es de
8.1[J3 A.m. ¿Qué fuerza neta en newtons
==~~~( s
}·-----o
-'-17cm-

experimenta la barra?
Al98 Bl139,2 C] 19,2 D) 100,8 E) 120
Electromagnetismo 433

Resolución.-

Aprovechando la propiedad de los polos en imánes


de barra en la Fig. 23.2 (d = U12), podemos elaborar
el D.CL de atracción y repulsión, donde las fuerzas
vienen dadas por la relación (23.1):
Fr.'-':._--'--__
30 cm
N'""-'~u-9
'~17 cm
-7 (6.103) (8.103) 3cmj; : \:3 cm
F¡ = 10 2 ~ F¡ = 120 N
(0,2) L~=O,2m-+
3 3
-7 (6.10 ) (8.10 )
F2 = 10 (0,5)2 ~ F2 = 19,2 N

Luego la fuerza resultante que experimerita la barra viene dada así:

R = F¡ - F2 = 120 - 19,2 ~ R = 100,8 N RPTA.D

Probo 2.- La barra Imán mostrada es uniforme •


y homogénea se encuentra en
equilibrio en la posición Indicada,
siendo su carga magnética de 300
Arn. y su masa Igual a 0,31 kg. SI el
campo magnético es uniforme y de
Intensidad B= 1(J2 T;¿Cuál es la tensión
que se presenta en el cable?

A) 3 N B) 4 N C) 5 N O) ó N E)l N

Resolución.-

Resolveremos el problemas en base al D.C.L. elaborado con los datos proporcionados, en donde la fuerza
magnética (F) que experimentan los polos del imán la encontramos en base a la relación (23.2):
F = q*B = (300 A. m) (l0·2n ~ F=3N
A continuación aplicamos la 2da Condición de F
Equilibrio tomando como centro de momentos
el extremo "O": k1x
0'-: --....,S,.----f--

antihcr.uio
(P=mg = 3 N) r
bocario
. ..
~ 3N(a)+(4r). 12a=P( a) + 3 N (11 a) ~ a~ Sa ~-Sa---é-al.

~ m= 1 ~ T=5m=5N RPTA.C

Probo 3.- Determinar la Inducción magnética


(B) (en teslas) en el punto F! debIdo a o~
los polos magnéticos mostradas, si:
q; = + 7 500Am yq; = - 4 500Am
\v>

\~\
A)O,B 0)0,5

8)0,7 E)OA G S_c."! 5.3_O /'-.p


C)O,ó (1)
434 Física - Primer Nivel Félix Auca/lanchi V

Resolución.-

En base a la relación (23.3) y al esquema indicado


encontraremos la inducción inagnética que genera cada
polo: BI y ~ ,ya continuación emplearemos la relación
(3.1) para calcular la inducción magnética total en P.
,,
Veamos:
\~=5.10-2m
,,
B =k iL=10-7 (7500) :} BI =0,3T
1m2 -2 2
dI (5.10)

Luego: BT = J B( + Bi + 2 BI B2 .cos 127 0

Probo 4.- Unabarra imán de carga magnética


q* = 3000 A.m se ha suspendido de
supolo norte del modo indicado en
la figura. Si el campo magnético
uniforme es de intensidad B = 0.01 T
Y la masa de la barra es de 8 kg.
¿Cuál es la medida del ángulo a. que
define la posición de equilibrio?

Resolución.-
,
En base al D.C.L. de la barra, reconocemos que:

F = q*.B = (3 OOOA.m) (0,01 1) = 30 N

Y P = mg = (8 kg) (10 m/s2) = 80 N


Luego, tomando como centro de momentos el polo norte
(N) de la barra, aplicaremos la 2da Condición de
Equilibrio:

¿IMI = ¿IMI
sentido sentido ~ P. (X sen 0.) = F (2xcoso.)
horario antihor a e¡o

2F 2 _ 30N 3
~ tg a. = -P = ~ a. = 37° RPTA. C
80N - 4
Electromagnetismo 435

Probo 5.- Dado el prisma de base rectangular z


cuyas dimensiones son las indicadas
está en una región que presenta un
campo magnético uniforme a = 0,5 T
¿Cuál es el flujo que sale de la
pirámide?

A) 1,2Wb D)0,8Wb
,, - --- y
a) 1 Wb E)OWb
~;(I
C)0,6Wb
Resolución.- z
R
De acuerdo con lo visto en el item 23.5,
sabemos que el flujo magnético (<1»solicitado
es el que sale por la cara PQR, y según el
esquema adjunto, reconocemos que el área
proyectada normalmente al campo es el del
triángulo ORS, por lo tanto empleando la
relación (23.4) tendremos que:
y
= BA.l
<1> = (0,5 n (12 m2)
<1>= 0,6 Wb RPTA.C

Probo 6.- En determinado lugar de la tierra la intensidad del campo magnético es 2.10-4 T. Sila
inclinación magnética es 6fr y la declinación magnética es 3JO, calcular el módulo de
la componente del campo magnético paralela al eje norte-sur geográfico.
A) 5.104 T a) 8.10-5 T C)4.1O-4 T D) 5.10-5 T E) 6.1(J5 T

Resolución.-
Según datos del problema y por lo visto en el item
23.7, reconocemos que: Li = 60°, y LO = 37°.

Luego, el campo solícitado Bj, viene dado así:


s N

BH = B cos i (Campo horizontal)

~ B,,= BH COS 0= B cos i. cos O


436 Física - Primer Nivel Félix Aucallanchi V.

MUY IMPORTANTE
Una de las reglas que
más difusIón tiene es la Podríamos decir con mucha justicia que el despegue tecnológico
regla de la mano de- de la humanidad se dió a partir del descubrimiento de que una carga en
recha, útil en todos los movimiento produce a la vez un campo eléctrico y un campo magnético;
puntos de e/ectromag- de este modo el fenómeno del magnetismo se constituyó en una
net/smo. Al ser utilIzado
para determinar el sentIdo propiedad del movimiento de aquellas.
de las líneas del campo se
procede
conductor
a coger el
de manera
11I EXPERIENCIA DE OESTERD
que el dedo pulgar señale U n año antes de la Independencia del Perú (1820), el físico danés
el sentIdo de la corriente, Hans Christian Oersted descubrió de manera casual que al hacer
entonces los dedos restan- circular una corriente lograba desviar la aguja imantada de una brújula,
tes cerrarán la mano en el lo que probaba que el movimiento de cargas eléctricas genera alrededor
mIsmo sentIdo de las líneas
de fuerza. de éstas un campo magnético. Pudo notar y comprobar que estas líneas
envuelven al alambre que conduce las cargas, y el sentido de éstas
viene dado or la re la de la mano derecha.

ATENC/ON!
Cuando las líneas del
campo cortan perpendi-
cularmente la hoja del di-
buJo, se señalan por puntos e) ('m.r"®x x x x x x x x x x x
sisalen, y por aspas sllngre- PJ X X X X X X X X X X X x
san a dicha hoja Entra
(Flg. 23.8c). -<>-r
••••••••••••
(iJ)Y····· ••••••

Fig23.8

PARA RECORDAR • LEY DE BIOT-SAVART


SIun conductor se grafica Pocas semanas después de conocerse el descubrimiento de
de punta, la corriente Oersted, los físicos Jean B.Biot y Félix Savart investigaron sobre la
puede salir (un punto) o intensidad de los campos creados por corrientes. A estos trabajos se
entrar (un aspa) en él. En sumaron los aportes de André M. Ampere y Pierre S. Laplace.
tales casos las líneas del
campo magnético se repre- A) Para un segmento de
sentan así: corriente.- Cuando un segmento iJ
íneaSdel conductor RS conduce una co-
rriente de intensidad i, genera un
~,.mp"
campo magnético tal queen un punto
P, el vector B será normal al plano
~ formado por el segmento y el punto,
y su módulo estará dado por:
LíneaSdel

~oamp. !B=lO-7f(cos a + cos P)] (23.7)

donde d es la distancia de P al seg-


~
mentoRS.
Electromagnetismo 437

B) Para una recta de corriente.- Se demuestra que toda corriente INTERESANTE


que transportaun conductor«infinitamente
SI en la relación (23.7)
largo» genera un campo magnético
hacemos <X = 90° Y ~ = 0°,
cuya intensidad es directamente estaremos frente al caso de
proporcional con la corriente (i), una semlrecta, en cuyo
pero inversamente proporcional con la caso el campo magnético
distancia (ti) al conductor. viene dado así:

B
Ilo
= -.- i [
~ :B " -7 i]
=,,2.1,0 -; (23.8)
B = 110 .i. = 10-7 .i.
P 41t d d
21t d.. . d '. Fig23.10
C) Para un Arco de Corriente.- Un conductor en forma de arco de radio
r, subtendido por un ángulo central
e, producirá un campo magnético a B
su alrededor de manera que en el
centro de curvatura la intensidad B de
dicho campo estará dado por:

t B;=* 10·7~1 (23.9)



donde e se expresa en radianes. Fig 23.11 ' OJO!
D) Para una Espira Circular de corriente.- Cuando un conductor
En el esquema anterior se
bajo la forma de un aro presenta una verifica que el campo
corriente, ésta genera un campo magnétIco en R es nulo.
magnético en todo el espacio que lo ¿Podríasjustiflcarlo?
rodea, el cual tendrá un valor máximo en
el centro de la espira, y viene dado por:

[;0 = 21&.10·7 ¿1 (23.10) DEBES SABER QUE:

Y en un punto P del eje: El físico francés André M.


Ampere propuso su Teoría
fVldect.krdela~
.,'" • 2
J.T en la que considera que a
Bp = 21t.10 2 2 3/2 (23.11) nivel molecular existen
, (x +r) . Fig23.l2 corrIentes elementales
producidaspor elmcNlmJento
E) Para un solenoide> Se Barna también bobina, y es un conjunto de de los electrones, las que
generrn romposrnc.Jgléttcos
arrollamientos por donde circula una al modo de las espiras
corriente, creando en su espacio cIrculares. En estado de
interior un campo magnético debido a desmagnetlzaclón estos
+---L---- •.. campos estón orientados al
la superposición de los campos
azar. pero sometidas a un
individuales que genera cada espira, de campo magnétIco exterIor
modo que éstos se refuerzan, dado que (Inductor); ellas tIenden a
en todas la corriente tiene el mismo orientarse paralelamente al
sentido. Puede comprobarse que: campo. Esto es lo que
ocurre con las sustancIas
=
B centro 2 B extremo ferromagnétlcas,
magnetIzan en el mIsmo
que se

sentIdo del campo exterIor.


lloiN , las dlamagnétlcas en
BceDtro= L -Ilotn (23.12) contra y lasparamagnéttcas
en forma transversal al
donde n = N/L es la densidad lineal de '-'-~~_~~~~ __ ---.J campo.
espiras. Fig 23.13
438 Física - Primer Nivel Félix Aucallanchi V.

INTERESANTE!
mFUERZAMAGNETICASOBREUNACARGAMÓVIL
SI una carga se mueve Debido a que una carga en movimiento genera su propio campo
dentro de un campo mixto
(magnético y eléctrico), magnético, al ingresar a otro campo magnético se produce una interacción
experimentará una fuerza entre ellos, lo cual origina fuerzas de naturaleza magnética, cuya dirección
por cada campo, de será normal al planoqueformanla velocidad (v) y él campo (8), y cuando
modo que a la resultante
de ellas se le denomina: la carga es positiva, su sentido viene dado por la regla de la mano derecha.
Fuerza de Lorentz.
I F:qvBsenO! (23.13)

donde:
- -
1Fml=qvB y IFel=qE

~V

l)Fmáx= qvB <=> 9=90" Regla de la


ATENC/ON! 2)Fmín= O ee- 9=0° MD1W Derecha
Las reglas. de la mano
derecha y de la mano Iz-
quierda dan directamente Fig23.14
el sentido de la fuerza,
siempre que la carga sea FUERZA MAGNETICA SOBRE UNA
positiva. Si ésta fuera CORRIENTE RECTILINEA
negativa, se procederá
del mismo modo, y solo se Cuando un conductor se encuen-
tomará el sentido opuesto tra dentro de un campo magnético, cada
al Indicado por el pulgar.
una de las cargas que él conduce
experimentan fuerzas cuya resultante
será normal al plano que formen el con-
MUY IMPORTANTE ductor y el campo magnético. Su sentido
viene dado por la regla de la mano
Cuando un cuadro con-
ductor que esrecorrido por derecha, y su módulo se determina así:
una corriente se orienta sene
perpendicularmente a las (23.14) Fig 23.15
líneas de fuerza, secumple
que dos de las fuerzas que
experimentan forman una
r:nn CORRIENTES
rtI!t
FUÉRZA MAGNETICA-ENTRE
RECTILINEAS
DOS' .
cuota. el cual le permite
girar respecto de un eje. Si dos alambres paralelos conducen
Estesimple hecho se utiliza corriente eléctrica, entonces los campos
para la construcción de los
motores eléctricos. magnéticos que ambos producen interac-
túan entre sí originando fuerzas de atrac-
ción si las corrientes tienen el mismo sen-
tido, y fuerzas de repulsión si aquellas tie-
nen sentidos opuestos. Estas fuerzas son
de igualmódulo pero de direcciones contra-
rias, pues constituyen una pareja de
acción y reacción. El valor de estas fuerzas
se determina así: L- ~=___=""~

[i:2:1~;'i11.Ll (23.15) Fig23.16


Electromagnetismo 439

El descubrimiento de que una corriente eléctrica genera un campo


magnético estimuló la reflexión ¿Puede un campo magnético producir
corriente? Muchos científicos trabajaron en tomo a este asunto, y
casi simultáneamente el año 1831 este problema fué resuelto
independientemente por Michael Faraday, el norteamericano Joseph
Henry, y el ruso Heinrich F.E. Lenz. De este modo se descubría el
fenómeno de la inducción electromagnética, el mismo que permitió
superar la energía eléctrica que proporcionan las pilas o baterías,
encontrándose un modo más adecuado de producir enormes cantidades
de energía eléctrica, así como su transmisión a grandes distancias.

11III FUERZA ELECTROMOTRIZ INDUCIDA MICHAEL FARADAY


A) Para una barra metálica móvil.- (1791-1867)
Cuando un conductor se mueve Este famoso físico Inglés
dentro de un campo magnético, sus se Inició como vendedor
electrones libres experimentan una de libros. Después de es-
fuerza de naturaleza magnética que cuchar a Humprey Davy
.x ; en la Real Academia de
las hará desplazarse hacia un extremo, Londres, comenzó a Inte-
y como es lógico el extremo opuesto resarse en las Investiga-
quedará cargado positivamente. Esto ciones científicas, y empe-
se interpreta como un diferencia de x zó a estudiar Físicay Quími-
x ca por su cuenta. En 1813
potencial 0¡, al que llamaremos fem Davy lo admitió como su
inducida, y cuyo valor viene dado así: &,=V.-Vb=~L ayudante, y así empezó su
Ibj= 'o/¡B.L=vBL brillante carrera, convir-
(23.16) tiéndose en uno de los
físicos experimentales más
brillantes de la historia. Son
Debe anotarse que la velocidad v, el campo B y muchas suscontribuciones
perpendiculares entre sí. a la Física en: Electricidad,
Mar;netísmo, Electromar;-
B) Para una espira conductora.- Si netisrno. Optlca y tamblen
desplazamos una barra conductora en Química con el estudio
de la Electrólisis.
sobre dos carriles metálicos.1
dentro de un campo magnético B MUY INTERESANTE!
(F i g. 23. 1 8) se observará que la
Faraday propuso una se-
0¡ inducida permite la aparición de rie de experimentos en
una corriente i (inducida). Además donde al hacer variar el
puede notarse que durante el flujo magnético que atra-
movimiento de la barra, el número viesa una espira conduc-
tora, producía una lem
de líneas encerradas en el circuito se Inducida, y por ende una
va reduciendo gradualmente, ésto es, corriente Inducida. Entre
el flujo magnético <l>que atraviesa estos experimentos pode-
el circuito está variando con el mos citar:
tiempo, y cuanto más rápido es esta 1) Moviendo un Imán fren-
variación es mayor la corriente. Todo &¡ = vBL = Af! = ~
te a una espira con-
ductora.
ésto nos conduce a establecer que: (A¡ - Ar)B ct>¡ -4>( ~ 2) Moviendo una espira
s,I /),J 61 =- 61 con relación a un Imán.
(23.17) 3) Variando la corriente en
Fig.23.18 una espira vecina a otra
desenergizada.
440 Física - Primer Nivel Félix Aucallanchí V.

ATENCION
1) En lo Flg. 23.190, 01
11II LEYDELENZ
acercar el polo norte Apoyándose en el principio de acción y reacción, el ruso H.F.E.
del Imán, el flujo externo Lenz comprobó que: «El sentido de la corriente producida por lafuerza
(<l» que atravIeso lo electromotri: inducida es tal que el campo que ella genera tiende a
espIro va en aumento;
luego, en ésto se compensar la variación del flujo magnético que atraviesa el circuito».
genera un flujo InducIdo Gracias a esta ley se explica el signo menos (-) de la Ley de Faraday
(<l>j que deberá sal/r de anterior.
lo espIro poro oponerse
01 aumento de aquél.
Esteflujo lo produce uno
corrIente InducIdo, que
visto desde P es de e)
sentIdo antihorarlo.
2 )AI cerrar el Interruptor S
(Flg. 23. 19b) se genero
casIInstantáneamente un
flujo en lo espIra "1", cu-
L-:3 (1) ~
;1 ~

;¡ : Corriente Inducida
yos líneas van de A hacIa cI>¡ : FlJ4jo Magnético Inducido
B y en aumento; luego,
en lo espIra "2" se gene- Fig23.19
rará un flujo Inducido que
deberá ser de B hacia A
oponiéndose 01aumento _ AUTOINDUCCION
de aquel. Estosupone lo
existenciade uno corrien- Es un hecho comprobado que cuando instalamos una bobina, en
te Inducldo(i) que encen- ella se genera una fuerza contraelectromotriz, que como se describe
derá temporalmente el en la Fig. 23.20, ella produce una disminución en el voltaje del foco,
foco. por lo cual éste reduce su
3)Cuando movemos el iluminación. Esto se interpreta
cursor e (Flg. 23. 19c) se
va reduciendo lo resisten- también como una caída de
cia del circuito, y por potencial &ai en la bobina, el cual solo
consiguiente lo corriente existirá si la corriente que recorre la ~-
i va en aumento. Esto o / I \
bobina es variable en el tiempo, es
su vez do lugar o un au-
mento del flujo que sale decir, no existe si la corriente es
del circuito (1) e Ingreso continua y constante. Se establece
01cIrcuito (2). en donde así que en una bobina se autoinduce
se genera un flujo Induci- unafem cuando lo recorre una corrien-
do que deberá salir de lo
hoja poro oponerse o te variable, y su valor viene dado por:
aquel. Este flujo solo se
explico por lo existencia Fig23.20
de uno corriente Inducido (23.18)
i2 de sentido antlhorario.
donde L es la inductancia de la bobina que se expresa en henry (H):
IMPORTANTE IR =
1 V/(I Als).

En uno bobIno de N espi- _ ENERGIA ALMACENADA EN UNA BOBINA


ras, en lo que codo uno
genera un flujo <l> debido o Cuando una bobina o solenoide es recorrida por una corriente
lo corriente i que los circu- eléctrica (i) se genera en él un campo magnético, el cual es una forma
lo, el coeficiente de
autolnducción L viene dado de energía que se almacena en aquella mientras exista una corriente
así: que la recorra. La energía (Um) almacenada vendrá dada por:

(23.19)
=> 1 henry = 1 weber/ampere
Electromagnetismo 441

DEBES SABER' QUE:


_GENERADO~SELECrROMAGNÉTICOS "
Como se sabe. la tensIón
El movimiento mecánico de un conductor dentro de un' campo y corrIente que llega a
magnético estacionario es el fundamento para convertir energía mecánica nuestros domIcilios es
en energía eléctrica, tal es el fin de un dinamo o central eléctrica (hidroeléctrica, alterna. es decir. cambia
contínuamente de sentido.
termoeléctrica, etc). ~ManiVe1a Cuando se habla del valor
A) f.e.m. alterna.- Cuando hacemos .J de un voltaje alterno. no
girar (mecánicamente) una espira :c1. nos referImos a su valor de
conductora dentro de un campo . pico. sIno a suvalor eficaz.
Así. la tensIón eficaz es
magnético, observaremos que el flujo aquel voltaje constante
magnético <1> que atraviesa la espira (continuo) que debe
de área A varía a medida que ésta va aplicarse a una resistencia
para desarrollar en él el
girando con velocidad angular co. mismo efecto térmico que
Luego,severificanlassiguientesrebiones: produce elvoltaje alterno.
<1> = BA cose , donde e = rot
&, __ Ll<l> _ - BA.Ll (cosrot) _ Vpico . E= Em
~.- M - M Veficaz - fi. . ó. fi.
~ &¡ = BAro sen (rot)
(23.20) . ¡pico.
Fig23.21 leficaz = fi. .0:
B) Corriente alterna> Si alimentamos una resistencia con una tensión
alterna obtendríamos una corriente que oscilará en fase con aquella,
tal como se indica en la Fig. 23.21b. Esto se debe a que la resistencia
es solo un elemento de consumo y no de variación de la tensión. Su
valor viene dado por:
(23.21) i.sé f.t donde (23.22)

_ TRANSFORMApO:R.ES
~sc:n(Q)t)
Denominamos así a aquellos dispositivos compuestos por un
núcleo de fierro, el cual es alimentado por una corriente variable (i ¡) que
genera en el interior un flujo
magnético también variable, el mismo
que produce en el arrollamiento "2" una
fuerza electromotríz inducida (&2)'
verificándose una proporción directa
entre las tensiones (&¡ y &2) con .el
número de espiras (N¡ y N2) de los
arrollamientos, tal que:

I:~:~]
.OJO!

En matemática se
(23.23) demostrará que:

y si no existen pérdidas en el (Piimario),.


transformador.Por¡ =Pot2 ~ &¡.i¡ =&2.i2 lírn!L'1COS(oot)}
(Sec¡uudario) M --oo.senwt
M-') ,

(23.24)
Fig23.22
442 Física - Primer Nivel Félix Aucallanchi V.

PROBLEMAS RESUELTOS
Probo 1.- Dos alambres A V 1:\ rectos muy largos y paralelos A B
llevan la misma cou.ente en el mismo sentido. ¿En
qué re<;Jión
sehallan lospuntos en loscuáles el campo
magnetico es cero?
A) Sobre el alambre B.
B)A la izquierda del alambre A.
1~ ~1
C) Sobre el alambre A.
D)A la derecha del alambre B.
E)Entre los alambres A y B.
Resolución.-

Recordando que el campo (B) creado por una


corriente rectilínea es inversa mente proporcional con
la distancia, concluímos que los puntos en donde el
t~A t~
campo total es nulo, son aquellos que se encuentran xBA x
equidistantes de los conductores dado que las x x
corrientes tienen igual intensidad. Asimismo debe
verificarse que los campos creados por cada
x xBB
x x
corriente (8A Y SB) deben tener direcciones
x x
opuestas y estas características solo se presentan
únicamente en la región comprendida entre los dos A B
alambres.

RPTA.E

Probo 2.- La figura muestra 2 conductores (1 y 2) rectilíneos e infinitamente largos con corrientes
I y 21.La distancia entre ellos es ó cm. Encontrar la distancia a partir del conductor (1)
donde el campo magnético es nulo.
A) Enel punto medio entre los alambres.
B)2 cm a la derecha del conductor 1.
C) 4 cm a la derecha del conductor 1. A~ . ~¡ll
D) ó cm a la izquierda del conductor l.
E)12 cm a la derecha del conductor 1. (1) (2) UNI84 -1
Resolución.-
Dado que> i, < iz' conc1uímos que el punto donde
el campo es nulo debe estar más próximo al
conductor (1). Según esta observación Y.de
acuerdo con el esquema adjunto, estableceremos
la siguiente igualdad:
1 (21)
110 ~ = 110 (x + 6)

=> x + 6= 2x => x = 6 cm RPTA. D


Observacién.: Los sentidos de las líneas del
campo se han determinado, usando la regla
de la mano derecha.
Electromagnetismo 443

Probo 3.- Dado dos alambres rectos muy largos perpendiculares al papel, y separados una
distancia a, transportan corrientes de igual intensidad I y de igual sentido. Entrando al
papel (ver figura]. Entonces si se evalúa el campo magnético resultante en el punto P
de la figura con d < 0/2, el resultado cualitativamente es:
A] a, con 8> O
8] 8] con 8> O

C]BI con 8<0


D] 8) con 8 < O a ----e-

E] a{í+7) con 8 > O UNI86

Resolución.-
De acuerdo con la regla de la mano derecha (colocamos
el pulgar hacia dentro de la hoja) ~escu~imos que en
el punto P indicado, los campos BI y B2 tienen sen-
tidos opuestos. Utilizando ahora la condición del pro-
blema: d < aI2, concluímos que P está más cerca al
conductor (1). Luego: (~

IBII >IB21 '"'''' (il = i2 = l)

De este modo al calcular el campo total en P tendre-


mos que:

~ BI + B2 = ( - BI + B2 ) j => BI + 82 = Bj ,y de (*) : B<O RPTA.D


'-----.,---"
B

Probo 4.- Una carga puntual de magnitud q que se mueve paralelamente y entre dos cables
muy largos que transportan corrientes /1 e 12' según se muestra en la figura. Siel peso de
la carga q es despreciable, ¿Qué relación guardan entre sílas corrientes para que la
partícula continúe moviéndose como al inicio?
A] 11 =12
11
8]11>12 - .r··-~==C>-===
d
-~ -- ()--<>v
C]11<12
i.d
D] Depende del signo de q.
~===
E]No es posible que continúe
moviéndose en tal forma.
UNI89
Resolución.-
Cuando la carga + q se desplaza de manera paralela a los conductores, resulta que su velocidad ¡; es perpendicular
a los dos campos magnéticos BI y B2' los cuales generaran en la carga fuerzas magnéticas J3¡ y F2.
444 Física - Primer Nivel Félix Aucallanchi V.

las mismas que de acuerdo con la regla de los tres


dedos de la mano izquierda, serán colineales y de
direcciones opuestas. Ahora, para que la carga continúe
moviéndose en línea recta paralela a los conductores,
deberá cumplirse que las fuerzas se equilibren entre
sí. De este modo se establecerá que:
If) 1= IF21 (*)

y empleando la relación (23.13) tendremos en (*):


qvñ, = qvB2 => BI = B2
/1 /2
=> Ilo
d = Ilo
d => I1 =1 1 RPTA. A

Probo 5.- La figura muestra un conductor rectilíneo, delgado y muy largo colocado sobre el eje
Z por el que cIrcula una corrIente en el sentido de + Z. Síen un cierto Instante un protón
se encuentra en el punto P (3; 4; O) con una velocidad v paralela al eje + Y.¿Qué
dirección tiene la fuerza magnética que actúa sobre la carga?
A) La dirección es paralela al vector a z
en ese punto.
a) la dirección es perpendicular al vector
a en ese punto. y
C) La dIrección es paralela gl eje Z.
D) La dirección es paralela al eje X. +e -
E) La dirección es OP UNFV 92 x ~
P(3;4;O)
Resolución.-
Z,
Dado que el campo magnético (B) se
encuentra que pasa por P, y está en el plano 1
x - y, diremos que la fuerza magnética (F)
que actúa sobre la carga + q es perpendicular
alplano formado por la velocidad (v) y al campo
(B), los cuales están en el planox - y, por ello
la fuerza será paralela al eje + Z, puesto que
según la regla de los tres dedos de la mano
izquierda, el pulgar apunta hacia arriba.

RPTA.C

Probo 6.- En la figura se muestra un alambre


largo por el cual circula una IZ
I
corriente l. Enel punto P se lanza una
partícula cargada positivamente con +q~
una velocidad v 5e9.únla dirección del P1
I
eje y positivo. ¿Cual es la dirección de I
la fuerza magnética en P? I
I
A) + Y D) + Z
a) + x E)-Z
y
C) -x UNI92
Electromagnetismo 445

Resolución.-
Reconociendo que en P el campo magnético (B ) está
en la dirección +x, comprobamos que: v
1. jj. Luego,
por tratarse de una carga positiva (+ q), la regla de la
mano izquierda o de la mano derecha nos permite des-
cubrir que la fuerza magnética tiene dirección - Z.

RPTA.E

Prob. 7.- Sobre la carga puntual + q que se mueve con velocidad constante y actúan loscampos
magnéticos By 2 B , los tres vectores v, B y 2 B estón en un mismo plano. La magnitud
y el sentido de la fuerza total que actúa sobre la ccrqo + q son respectivamente:
A) (1/2) qvB; - Z
+Z 1r---------7
B)2 qvB; +Z
C) (1/2) qvB;
O)( J3 /2) qvB;
E)( J3 /2) qvB ; - Z
+Z
+Z
UNI94-2
+q~:
e-300

Resolución.-
De acuerdo con la regla de los tres dedos de ~ many
izquierda, es fácil reconocer que las fuerzas F¡ y Fi
originadas por los campos; 2B y B respectivamente,
+Zlr-----~----------~
tienen direcciones paralelas al eje Z, pero sus sentidos
opuestos. Luego, utilizando la relación (23.13) para la
fuerza magnética, tendremos:

F¡ = q v (2 B) sen 300 ~ F¡ = + ( qvB) k

F2 = qvBsen300 ~ ~ = {~qVB)k

- - - -
Por lo tanto: R = F¡ + F2 = (qvB) k -"2l (qvB) k
-

~ R={~qvB)k RPTA. e

y
Probo 8.- Un cubo de arista se encuentra
situado en un lugar donde existe un
campo magnético B uniforme de 0,5
testa. según el dibujo. Sobre el cubo
se coloca un alambre abcd a través
del cual circula una corriente eléctrica
de 2 A de intensidad. La fuerza que I~ B
I~
actúa sobre el tramo bc es:
a ----- d X
A)O,5Ní B)-1,ONj j C)2,ONk
z
O) J2 N (/ - k) E) 2 J2 N T .UNI 93 - 2
r
446 Fisica-Primer Nivel Fé/ix Aucallanchi V.

Resolución.-
De acuerdo con el esquema adjunto, podemos
notar ,.9.ue la longitud del conductor es:
e B
1 = .J2 m!-y es afectado por un campo
magnético B que con la corriente (y también
con el conductor) forma un ángulo de 45°.
Luego, utilizando la regla de la mano izquierda
descubrimos que la fuerza magnética (F) que b c.¿:-----II--
experimenta el conductor es paralela al eje -Y.
Ahora, utilizando la relación (23.14) tendremos: . f.Jít1l~

F = B.i.L sen 45° = (0,5 T) (2 A) (.fi.m). ~ ~

F=IN =::} F=-lN j RPTA.B

Probo 9.- En la figura, siI es un imán y E una espira y el imán se está alejando de la espira a una
velocidad v, según la Ley de inducción de Faraday, podemos afirmar que:

Al No existe corriente, puesto que no


hay fuente.

Bl El séntído de la corriente es según


la flecha 1.

C] Se induce una fuerza de repul-


sión sobre el imán.

D} Se induce una corriente de senti-


do contrario en el imán.
El El sentido de la corriente es la flecha 2. UNMSM89
Resolución.-
De acuerdo con el esquema adjunto podemos
reconocer que el tlujo externo (<1> ) que cruza el
interior de la espira va reduciéndose a medida que
el imán se aleja. Luego, según la Ley de Faraday en
aquella deberá existir un tlujo inducido producido
por una corriente "i" también inducida, cuyosentido
deberá generar un flujo inducido (<1>,). Según la Ley
de Lenz este flujo deberá oponerse a la reducción
de <1>0' y ello solo se logrará si el inducido va en el
mismo sentido que aquel. Ahora por la regla de la
mano derecha la corriente deberá tener sentido
OBSERVADOR
antihorario para el observador indicado.

Sentido 1 RPTA.B
Electromagnetismo 447

1.- Señala lo correcto: 6.- Si los puntos (.) y las aspas (x) representan líneas
(1) Los imanes solo tienen dos polos. del campo saliendo y entrando a la hoja
respectivamente, se pide indicar el esquema correcto
(Il) La imantación de los cuerpos es independiente para el campo magnético generado por i.
de la temperatura.
(III) Si martillamos un imán, éste reduce su imantación.
A) B)

::~f
IV) Es imposible aislar un monopolo magnético.

A) III Y IV B) I Y Il C) 11Y III D) I Y IV E) N.A.


2.- En relación al magnetismo terrestre se afirma que:
x x ~ x x E) N.A
) La inclinación y declinación magnética tienen un
valor definido para cada punto de la superficie C) D)

:~I: : ::~t
terrestre.
) El polo norte magnético coincide exactamente con
el polo sur geográfico. . 'x x i
x x ~ .
) La inclinación magnética del polo norte magnético . x x
es ~Oo.
Indicar verdadero (V) o falso (F): 7.- Sabiendo que las espiras mostradas se encuentran
en planos paralelos en los que circulan corrientes
A) VVV B) VFV C) VFF D) FVV E) FVF iguales en los sentidos indicados, será cierto que en
3.- Dadas las siguientes listas, señalar la relación (1) y (11) existe respectivamente:
incorrecta:
1) Inducción magnética a) wb
Il) Flujo magnético b) N/A2
I1I) Permeabilidad absoluta c) Alm
IV) Campo magnetizante d) testa A) Atracción-Atracción B) Atracción-Repulsión
A) I-d B) II-a C) III-c D) III-b E) IV-c C) Repulsión-Repulsión D) Repulsión-Atracción
E) N.A.
4.- Elige las palabras que completen mejor la siguiente
oración: «Las de fuerza del campo 8.- ¿Cuál de los esquemas siguientes representa
magnético son ». correctamente a la fuerza magnética (F) , el campo
A) Tensiones; nulas D) Líneas; cerradas magnético (B) y la velocidad (v)?
B) Líneas; abiertas
C) Curvas; nulas
E) Curvas; absolutas

5.- Sabiendo que los esquemas muestran las líneas


del campo magnético de la corriente i, indicar
A)

+q
k- 9
F
_
V
B) Fk-
_q 9
C) vt
~
]J

verdadero (V) o falso (F):


B

A) FFV
a)~b)~

B) FVF C) VFF
')~

D) VVF E) FVV
D)T >¡:
B

F
+q
E)
448 Física - Primer Nivel Félix Auca/lanchl \1.

9.- Dado el siguiente esquema en donde las barras 13.- En la figura se muestra dos bobinas A y B sobre
de igual longitud conducen la misma corriente, se un núcleo de hierro. Al disminuir la resistenciaR 1 del
afinnaque: circuito se tendrá que:

A) Va - Vb> O

B) Va - Vb = O

C) Va - Vb < O
(I) (II) (III) D) El signo de
(V - V) de-
( ) La fuerza en I es hacia arriba. pe~de d'el va-
( ) La fuerza en 11entra a la hoja. lor deR2• v
( ) La fuerza en 1II es nula. E) N.A.

Señalar verdadero (V) o falso (F) 14.- Se tienen dos bobinas 1 y 2 inmóviles alejadas
una de otra y un imán con sus polos indicados,
A) VVF B) FVV C) FFV D) VFV E) VFF moviéndose hacia la derecha con velocidad
constante v. Acerca del sentido de las corrientes
10.- Si ubicamos un observador en e, en la espira inducidas i1 e i2 indicadas en la figura, se puede
conductora se inducirá una corriente: afirmar que:
1) Nula si ninguno
se mueve.

11) Horario, si se
mueve hacia el
imán. A
m N S

I1I) Antihorario si el
imán se aleja.

Indica lo correcto:

A) I B) 11 C) III D) Ninguna E) Todas


A) Sólo i1 está correcto.
11.- ¿Cómo deberá moverse el cuadro conductor
B) Sólo i2 está correcto.
mostrado para que se genere una corriente inducida
en ella? C) Ambos están incorrectos.
A) Hacia la derecha. D) Ambos están correctos.
B) Hacia dentro de la hoja. I E) No existen tales corrientes.
I
C) Hacia arriba. '0 li
I
D) Girando respecto I
deO.
iQ
E) Rotando respecto
del ejePQ.

12.- En relación a las unidades físicas, es falso que:

A) T= Wb/m2 B) H = V.s/A C) J = H.A2


D) V= Wb/s E) A.m = NIT
Electromagnetismo 449

PROBLEMAS PROPUESTOS
NIVEL 1

1.- Dos polos norte 600 Am y 800 Am se colocan a


2 cm de distancia ¿Cuál es la fuerza (en N) de repul-
sión que existe entre ellos? O)~

A) 20 B) 60 C)90 O) 120 E) 150


E) N.A.
2.- ¿A qué distancia (en cm) habrá que colocar dos
cargas magnéticas de 2.103 Am y 3.103 Am, para
que la fuerza de repulsión entre ellas valga 15 N? 6.- Una brújula es colocada en un campo magnético
uniforme. La posición que toma es :
A) 10 B) 15 C)20 0)25 E)30

3.- Indicar el enunciado verdadero:

A) En el magnetismo terrestre, los polos nortes geo-


gráficos y magnéticos coinciden.

B) El magnetismo de una sustancia resulta básica-


mente del movimiento de las cargas eléctricas conte-
nidas en ellas.
••
C) Puede encontrarse un imán de un solo polo mag-
nético. C)-<J>-- O)-<J>--
O) Solo hay sustancias pararnagnéticas.
••
E) El magnetismo de una sustancia resulta la atrac-
ción graviiacional. E) N.A.

4.- Si u n imán se aproxima a las cercanías de otro en 7.- En la figura se muestra una barra de imán de 36cIII
forma de herradura y se deja libre en la posición de longitud cuyos polos magnéticos tienen una car-
mostrada, entonces despreciando el peso de dicho ga magnética de ± 25.103/\/11. Frente a él se encuen-
imán, podemos afirmar espccto al imán recto que: tra un polo sur cuya carga magnética -4103 AIII ¿Qué
fuerza neta (en N) experimenta la barra?
A) Es atraído al polo Sur
Lot- _~,--_~~m----mO
B) Es atraído al polo Norte 1-17cm-f

C) No se mueve A) 40 B) 210 C) 250 O) 290 E) 300

O) Gira en sentido horario 8.- Determinar la intensidad del campo cantidad re-
sultante en el punto P, si se sabe que sus cargas son
E) Gira en sentido antihorario de: ±50.106 A1/1
A) 12 T
s.- ¿Indicar cuál de los espectros magnéticos es el P:
correcto? B) 20 T

Al --00- -E:} Bl
C) 24 T

D) 32

E) 42 T
T 1-
-r------
j30cm

E=_::J
96cm ----1-
-l50 Fisica-Primer nivel Fé/ix Auca/lanchi V.

9.- Dos polos n0I1e A y 8 de cargas magnéticas iguales IVEI,2


a q* y 3q* respectivamente, se encuentran distanciados
'20C/I/ ¿Cuál es la carga magnética (enAIII) del mayor, si 1.+.-La barra imán de 12 C/I/ de lonuitud, se cncuen-
la fuerza con se rechazan es de '+,8 N) tra en equilibrio en la posición indicada siendo su
carga magnética de ±300;\/I/ y su peso de 3 N. Si el
A) 800 8) I 600 C) 2 .+00 campo magnético es uniforme y de intensidad B = 10.1 T
D) 3 200 E) 4 000 ¿Cuál es la tensión que se presenta en el cable?
A) 3 N
10.-Tres polos magnéticos están ubicados en una misma
recta siendo sus cargas magnéticas : q~* = + 1 600;\111 ; 8)4N
qB• = -1 200;\/1/ Y qc* = -900 1\/1/ t. uál es la fuerza
magnética resultante (en N) sobre el polo "8"'1 C) 5 N
A) 19,2 D) 6N

B) 12.7 A B C E)7N
C) 16,5 --------------0 15.- Determinar el flujo magnético que pasa a través
D) 16,7
~- 10 cm -1-- 20 cm -l- de la cara PQRS, si se sabe que el campo magnético
uniforme es de 400T. Además PS = 60clI/; PQ = 50c/I/;
E) 21,9 N PT = 30 C/I/. z
11.- Determinar la intensidad de campo maanético A) 18 Wh
resultante e~ el punto "P". Si:
q; = 90.10) A/I/
qt
= -160.10'3/\11/ Y
8) 27 Wh

A) 10 T C) 54 Wb

13) 100T D) 72 Wb

0200 T E) 90 Wb x s
D) .fi T
16.- Calcular a qué distancia (en cm) de un conductor
E)IO.fiT infinitamente largo; por el cual pasa por una corriente
de 50;\, la intensidad de campo magnético es 2.1 O·~T
12.- Un imán recto de carga q* = ±5 ;\/11 y peso 50 A)3 8)4 C)5 D)6 E)9
Newtows, es ubicado dentro de un campo uni forme
8 = 5T. Determinar el valor del ángulo "a" para que 17.- Hallar el campo magnético (en ¡l T) en un punto
el imán quede en equilibrio en la posición mostrada. situado a una distancia de 2 CIII de un conductor
infinitamente largo por el cual fluye una corriente de
A) 30° 6 amperios.
8) 37°

C)45°

D) 53°

E) 60°
=~ A) 40 8)50 C) 60 D)70

1R.- La fisura muestra dos conductores infinitamente


largos y ~que transportan corrientes I1 = lOA /\
1, = 40 A, la distancia entre ellos es de 12 ClI/.
Encontrar la distancia a partir del conductor izquierdo
donde el campo magnético es nulo.
E) 10

A) 5 D)6c/II E) 2 CIII

--
13.- Del sistema en equilibrio, CII/ 8)4c/II C)3clII
determinar el peso (en N) del
imán recto. Si la carga de cada 19.- La figura muestra dos con-
polo es de ±3 AIII y el campo
uniforme es de 41', además
<l> = 37°.
~B
••
$:---
,---
,
ductorcs infinitamente
transportan corrientes
largos que
1 y 31.
Encontrar la distancia a partir del
conductor Izquierdo donde el Ilr
I j r,
campo magnético resultante es
Al72 8)36 C) 32 D) 12 E) 40 nulo.
Electromagnetismo 451

A)2cm A) 10-7 T

B)3cm 1
8-------
------
----
------
-----@
TI B) 2.10-7 T

C) 3_10-7 T
I?'
2m 'iI~ M
C)4cm
1 10cm ir-
~

D)5cm

E) 6cm

20.- Determinar el campo magnético en el grupo punto


D) 4.10-7 T

E) 5.10-7 T 1~
24.- En la figura se muestra las secciones de tres
"P", generada por el conductor si es recorrido por conductores infinitamente largos recorridos por
una corriente de 60 A corrientes. Las distancias AB = BC = 6 cm, además
P
'2
'1 = e 13= 2'1_ Determinar el punto sobre la recta
AC donde el campo es cero.
A) 20 ~T r

,/: ,
B) 70 ~T A) Icm
15cm/ i 12
" : cm B) 2 cm
C) 120 ~T
;\ b 1 C)3cm
D) 140 ~T

E)21O~T
tL==============~=1
+--25cm --+ D) 4 cm

E) 5 cm
21.- Calcular la intensidad de corriente, para que la
intensidad de campo magnético en el punto "P" sea 25.- Hallar el campo magnético en el centro de un
de 12.10-4 T Y entrando a la hoja. espira circular de un conductor de radio igual a n:cm
y por el cual fluye una corriente y 1 amperio.
A) 200 A

B) 300 A
PC?pB A) 10-5 T B) 2.10-5 T C) 3.'10-5 T

, D) 4.10-5 T E) 5.10-5 T
C) 400 A ¡ Scm
,, 26.- Dos espiras cada uno de radio 2n: cm se
D) 500 A disponen en un ángulo recto una con respecto a la
00 •• ·1•....
__ h
-'-.1..-_--,1 ... Ñ otra con un céntro común. Si las corrientes que
E) 600 A circulan por las espiras son respectivamente iguales
a 3 y 4 amperios. ¿Cuál es la magnitud del campo
22.- La figura muestra dos conductores rectilíneos magnético (en ~1)en el centro común?
infinitamente largos, donde: '1 = 30 A Y r! = 50 A.
Determinar el campo magnético resultante en el punto A) 20 B)30 C) 40 D) 50 E) 60
Jlp"
27.- Un solenoide de 40 cm de longitud posee en
A) 4.10-4 T total 200 espiras y son circuladas por una corriente
I1 12 cuya intensidad es de 4A; se desea saber qué flujo
B) 8_10-4 T (en ~ Wb) magnético sale de su extremo norte, si el
!?_!!l!!l_~ P área de su sección recta (vacía) es de 50 tt: cm".
C) 16.10-4 T
A) 10 B)20 C)30 D) 40 E)50
D) 32.10-4 T
- ---- 20 mm ---- -
28.- Calcular el flujo magnético (en Wb) de un toroide
E) 64.10-4 T de radio interior y exterior iguales a 16 cm y 20 cm
respectivamente. Por sus 360 espiras circulan 25 A Y
23.- En la figura mostrada los alambres paralelos en el núcleo existe una sustancia ferromágnetica de
infinitamente largos conducen I= 5A. Hallar el campo permeabilidad u, = 2 000.
magnético resultante en el punto "M"
A) 0,1 B) 0,2 C)0,3 D) 0,4 E) 0,5
452 Fisica-Pritner nivel FéJixAucallanchi V.

NIVEL 3 33.- Un avión de propulsión a chorro vuela horizon-


talmente a razón de 960 km/li y a una altura tal que
29.- La fuerza de un campo magnético de intensidad la inducción magnética de la tierra es vertical y su
B = 2 teslas ejerce sobre una carga de I¡,tC que entra magnitud es de 6. J 0-5 T, si la di ferencia de potencial
perpendicular a dicho campo es de I N. Calcular la entre los extremos de sus alas es 0,32 V, calcular la
'rapidez (en mIs) de ingreso de la carga al campo. longitud (en m) de envergadura de sus alas.

A) 5.105 B) 6.105 C) 8.105 A)S B) 10 C) 15 D) 20 E) 30

D) 9.105 E) 15.105 34.- Una barra de 2 m de longitud se deja caer de una


altura de 20m en forma horizontal, y lo hace cortan-
30.- UnaPoartícula con cargaq = 2 /lC y masa do las líneas de fuerza del campo magnético terres-
m = 4.10--9 kg Ingresa perpendicularmente a una tre en la dirección Este a Oeste, siendo el campo de
región donde existe un campo magnético uni forme una intensidad BH = 0,5 T ¡.Cuál es la f.e.m.indueida
B = 0,2 T con una velocidad de 104 m/s. Calcular la (en V). que se presenta entre los extremos de la barra
intensidad y dirección del campo eléctrico (en kNIC) alllcgar a la tierra? (g = 10 mls2)
necesario para que la partícula atraviese la región del
campo magnético sin desviarse. A) 1 B)2 C)3 D)4 E)5
A) I (J-) o 35.- Encuéntrese la potencia disipada por la resisten- ••••
B cia de 2Q cuando la barra conductora de 0,5 m se
B) 2 (1) desliza pegada a los rieles con una velocidad 10 m/s
en el interior de un campo magnético uniforme de 0,4

.. -
e) 3 (J-) T. Despreciar las resistencias de la barra y los rieles.
q
D) 4 (i) A) I W o

EJE
E) 5 (i) B)2 W B
31.- Una partícula cuya carga es q = 5 e es impul- 2 ---=-v
sada desde P con una velocidad v = I 600 m/s en
forma radial, alejándose de un conductor infinito por
C) 3 W

D) 5 W
. . . .
el cual circula una corriente 1 = 200 A ¿Qué fuerza
magnética experimenta la partícula cn dicha posi- E) 6 W
ción? (d = 4 cm)
36.- Un transformador que cuenta con 2 000 espiras
A) 9 N en el primario y 20 en el secundario tiene una fuerza
electromotriz de 220 Ven el primario y una corrien-
B) 8N te de 5 A. Hallar la potencia (en kW) de salida en el
secundario.
e) 6N
A) 1 B)0,5 C) 1,1 D) 50 E) 100
D)S N
37.- Calcular la intensidad de corriente que circula
E)4N por la resistencia indicada (R = I1 Q) si la tensión
en el primario del transformador es de 220 V, NI =
32.- En la figura se muestra 100 espiras y N 2 = 150 espiras.
xA x 0
un alambre AeD doblado en
C, por la cual circula unaco-
rriente 1 = lOA, si 8 = 60° y
x tI: x x
B A) 10 A
~
11
el campo es B = 10 T ¿Cuál xC x x x B) 20A

El
es la fuerza que actúa sobre x :8 x x
dicho alambre, si A = 5 C/1/ ~ e) 30,1
y CD = 3 cm? x x xD x N2~ R
,,
D) 40A ,,
,
A)3N B) 5 N C) 7 N D) 9 N E) 6N E) 50A
Ondas E{ectromagnéticas
!1 Luz
ORlE1TVOS
1.- Conocer el origen de las ondas electro-
magnéticas, su espectro de variabilidad,
y la doble naturaleza de la luz.
2.- Comprender los fenómenos de ilumina-
ción, reflexión y refracción descritos por
la Optica Geométrica.
3.- Entender los principales fenómenos on-
dulatorios de la luz descritos por la Op-
tica Física.
•••

odos quienes hemos venido al mundo en "buenas condicio


nes" de salud somos seres privilegiados porque nues-
tros ojos nos permiten apreciar todo aquello que nos rodea,
y ésto en gran medida se debe a la existencia de la luz. Es gracias a este
hecho que podemos estudiar nuestra naturaleza de un modo directo,
mediante su observación. Se dice con gran acierto que la luz es para el JAMES C. MAXWELL
ojo lo que el sonido es para el oído. A semejanza del sonido, la luz es
una perturbación (onda), pero ésta se distingue claramente de las otras, (1831-1879)
porque para propagarse no necesita de ningún medio mecánico, pues
Físico Inglés. Nació en
se ha descubierto que la luz es una onda electromagnética que se puede Edimburgo, en 1831. Estudió
propagar a través del espacio vacío, y como tal la estudiaremos primero. en su ciudad natal, y más
tarde en Cambridge. En
CAMPO ELECTRICO INDUCIDO Londres explica la cátedra
de Física y Astronomía del
Según como se explica en el item 23.12, es un hecho conocido que King's College. A los 40 años
la variación de un campo magnético induce una corriente eléctrica en una pasa a desempeñar la cá-
espira conductora. Pues bien, para que 1----,:::;==:::;:;==::;-1 tedra de Física experimen-
esta corriente se haya producido, dentro tal de la Universidad de
Cambridge, luer;¡o de no-
del conductor debió aparecer un campo ber sido citsciputo de
eléctrico en el mismo sentido de la Michael Faraday. En 1865
corriente inducida. En la Fig. 24.1 se propone su «Teoría Electro-
muestra una espira conductora y un magnética» que para el
estudio del electromag-
campo magnético B que está netismo es solo compara-
aumentando hacia dentro de la hoja, de ble con las leyes de la Me-
modo que se induce una corriente i cánica de Newtor; abrien-
do así de par en par las
de sentido antihorario, y ésto nos
permite trazar algunas líneas del
x x x
x x x x x ----~ puertas a la técnica de las
telecomunicaciones de
campo eléctrico inducido E. En nuestros días. Su propuesta
conclusión: «Toda variación de un
campo magnético en una región de L__
espacio, genera en dicho lugar un
-======~,---::-:~
Fig, 24.1
de la existencia de las on-
das electromagnéticas
confirmada
mente por
experimental·
el alemán
fué

campo eléctrico inducido». Heinrich Hertz.


454 Física - Primer Nivel Félix Aucallanchi V.

POSTULADO DE MAXWELL • CAMPO MAGNETICO INDUCIDO


!'...SI
un campo magnético
( B ) variable c.r:...ea
un cam- Como ya sabemos, una carga en movimiento produce a su alrede-
po eléctrico ( E ), entonc~s dor un campo magnético, y puede
un campo eléctrico (E) notarse que ello está asociado al hecho
variable creará tamQlén un de que en cada punto del espacio
campo magnético B ». el campo eléctrico que genera la
carga está variando en el tiempo.
Esto puede verse también en el es-
quema de la Fig. 24.2., en donde el
aumento de las cargas en las pla-
cas produce una variación en el
campo eléctrico, lo que a su vez
permite la aparición de un campo
magnético inducido. Fig.24.2
DEBES SABER QUE:
Estas observaciones permite establecer la siguiente conclusión:
Cuando una corga eléc- «Si un campo eléctrico existente en cierta región del espacio sufre
trica (q) positiva o negativa
experimenta un movimien- una variación en el tiempo. ésto hará aparecer en dicha región un
to acelerado u oscuotono. campo magnético inducido». .
genera ondas electromag-
néticas, pues en coda pun- • CIRCUITO OSCILANTE
to del espacio que lo rodea
habrá un campo eléctrico
variable, y por lo tanto exis- Llamamos así a aquel circuito
tirá un campo magnético constituído por un capacitor inicial- J~ -<>
i
también variable. mente cargado conectado a una bo-
bina desenergizada. Al cerrar el inte-
t g~
rruptor S, la energía eléctrica se con- :L = i='"
vierte en energía magnética y vice- ~
versa. Si logramos que la frecuencia 1/ l\
de estas oscilaciones sea muy gran-
de, la energía del circuito se irá per- S
diendo debido a la radiación de on-
das electromagnéticas que salen de él. Fig.24.3
VELOCIDAD DE LAS ONDAS ONDAELECTROMAGNETICA
ELECTROMAGNETICAS
Si alimentamos dos bobinas NI y N2 con una fuente d~ corriente
Después de un minucioso alterna, se generará en ellos un campo magnético variable (B), el cual
análisismatemático, MOi'M'eII
encontró que la velocidad producirá en los alrededores del circuito un campo eléctrico inducido
con que se propagan las (E) también variable. De acuerdo con r--------------,
ondas electromagnéticas el Principio de Maxwell, este campo
viene dada por la relación:
eléctrico, por ser variable, generará
I 8 otro campo magnético inducido va-
c= ~ 3.10 mis riable, y éste a su vez otro campo
v f.o ·1.10 E
eléctrico, y así sucesivamente. Sor-
que como sabemos es la
prendente verdad!!.
x
velocidad de la luz en el Pues bien, estamos ante un con-
vacío. Nueve años después junto de perturbaciones que se van auto-
de la muerte de Maxvvell,el z
físicoalemán Heinrich Hertz generando en el espacio y de un modo
pudo confirmar experimen- variable, recibiendo el nombre de on- '-- ---'
talmente estos resultados. das electromagnéticas.En la Fig. 24.4 se Fig.24.4
Ondas Electromagnéticas y Luz 455

ha representado una onda electromagnética que se propaga en la dirección OJO!!


+x, observándose que los campos E y B son perpendiculares entre sí.
La longitud de onda (A) y
la frecuencia (¡] de las on-
• ESPECTRO ELECTROMAGNETICO das electromagnéticas se
Desde la época de Maxwell hasta nuestros días se han descubierto
varios tipos de onda electromagnética, los cl!fllesLa pesar de ser todos de
relacionan entre
promedio de la relación

{14.1) vistaen el capítulo de
,
la misma naturaleza (constituídos por campos E y B que oscilan en el tiem- Oscilaciones, tal que:
po y se propagan en el espacio), presentan en ciertas circunstancias ca- A..f= c
racterísticas diferentes. Veamos las características de algunos de ellos:
A) Rayos Gamma (y).- Según el cuadro del lado, éstas son las ondas
que corresponden a las más altas frecuencias. Este tipo de radia- ESPECTRO
ción lo emiten los núcleos atómicos al desintegrarse, como por' ELECTROMAGNETlCO
ejemplo en la explosión de una bomba atómica. Estos rayos produ-
cen daños irreparables a las células animales.
B) Rayos X.- Fueron descubiertos por el alemán Wilhelm Roentgen,
f(ilz) A(III)
ganando por ello el Nobel de Física en 1901. Estos rayos tienen la
propiedad de atravesar con facilidad las sustancias de baja densi- lO" Rayos 11)"
101\ Cosmiros 1O";
dad (músculos del hombre), y son absorbidos por cuerpos de alta
10" 11)"
densidad (huesos). Esta propiedad es empleada en las radiografías. Rayos 11)'\
lO"
C) Radiación Ultravioleta.- Estas radiaciones tienen una mayor fre- 10'0
Y 10'"
cuencia que la radiación violeta: Hasta 1018 Hz, Estos rayos son 10" 10'"

:~::
Rayos _1O.'O=IÁ
emitidos por átomos excitados, como los que produce una lámpa- X .,
ra de vapor de mercurio. No son visibles, pero pueden imprimir .,
-10 =1//111
ciertos tipos de placas fotográficas. Pueden dañar el ojo humano.
10"
10')
Uliraviotetu
-10
-10
.,
LUZ
D) Ondas Luminosas> Este grupo de ondas electromagnéticas tiene 10" 1C)"=I¡ltn
11!1"lI1"(~i(}
frecuencias del orden de 4,6.1014 Hz y 6,7.1014 Hz; y son capaces de 10-5
10:: (Calor) ~
estimular el ojo humano. En orden decreciente de frecuencias se com- 10" lO,
10 Microondus -10
ponen de las siguientes radiaciones: Violeta, azul, verde, amarillo, 10
10-'
10.,
anaranjado, rojo y añil; a todo el conjunto se le llama luz. 10 10-'
Radar 11
E) Radiación Infrarroja.- Estas se encuentran a continuación de la radia- 10' TV 10,=111I
ción roja, y la emiten en gran cantidad los cuerpos calientes. Esta lO' 10,
R"d;oFM -lO'
lMllz=IO'
es la radiación que recibimos de una fogata, y que nuestro cuerpo 105 \

tiene la propiedad de absorberlos. Es este tipo de transmisión de


RlIditllli{lOiáll ,
-1O:1klll
10: -10
Ondas
calor que hemos denominado radiación en el Capítulo 17. lkHz=IO 10,5
Eléctricas
10' (Lineus de 10,
F) Microondas.- Llamamos así a aquella radiación de frecuencias un
110' Trammilití,,) -10
tanto menores: De 108 Hz a 1012 Hz. Se utilizan mucho en las tele-
comunicaciones; transmisiones telefónicas, celulares, TV (vía sa-
télite), ... , etc. En la actualidad también se utilizan en la cocina, con
el nombre de horno de microondas.
G) Ondas de Radio> Son las que poseen las más bajas frecuencias, como
máximo 108 Hz. Reciben este nombre porque se emplean en las
estaciones de radiofusión. En las antenas de estas estaciones se
logran acelerar los electrones de manera oscilatoria y continua, lo
cual permite la generación de ondas electromagnéticas, las mis-
mas que logran transportar los mensajes o programas de la estación a
los radioreceptores, en donde las ondas aceleran los electrones de
la antena receptora, y estos estímulos eléctricos son finalmente
convertidos en imágenes y/o sonido.
456 Física - Primer Nivel Félix Aucallanchi V.

EL ERROR DE NEWTON! UVTICÁ f7I:UMI:Tl?ICÁ . f

A pesar que con la teoría De aquí en adelante estudiaremos todos aquellos fenómenos lu-
corpuscular se podían expli- minosos en donde el tamaño de los cuerpos iluminados es notoria-
car los fenómenos de re-
flexión, refracción y disper- mente mayor que la longitud de onda de la radiación luminosa. Para tales
sIón de la luz, esta incurría casos solo nos dedicaremos a describir el comportamiento de los rayos
en el error de asegurar que luminosos.
la luz se movía más a prisa
en el agua que en el aire.
EaIINATURALEZADELALUZ
A) Teoría Corpuscular.- Tratando de descubrir qué es la luz, Newton
consideró que ella era una emisión de pequeñísimos corpúsculos
que salían de los cuerpos luminosos, incidían. sobre los demás cuer-
EL ERROR DE HÜYGENS! pos, y luego de rebotar en ellos llegaban a nuestros ojos, estimulán-
dolos con choques, los cuales producían el fenómeno de la visión.
Siendo aún asombroso el
parecido que hay entre los B) Teoría Ondulatoria.- Esta teoría fué sustentada por Christian
fenómenos ondulatorlos de Hüygens, quien sostuvo que la luz era una emisión de ondas simi-
la luz y el scotoo. fué un lares a las del sonido. Esta teoría tuvo finalmente mayor acogida
e.rot de la Teoría
Ondulatorio el asegurar que dado que permitía explicar los fenómenos propios de las ondas como
las ondas luminosas eran la interferencia, difracción y la polarización. Maxwell reforzó aún más
longitudinales, pues según esta teoría al afirmar que la luz era una onda electromagnética.
como lo prueba la polari-
zación de la luz, ella está C) Teoría Actual.- A principios de este siglo se descubrió que la luz
compuesta por ondas trans- estaba constituida por un flujo de pequeños paquetes de energía
versales. llamados latones, y que permiten explicar el fenómeno Fotoeléctri-
co. Esto nos obliga a aceptar una doble naturaleza para la luz: «Es
onda y es partícula a la vez». Como onda se propaga y como par-
tícula interacciona con los cuerpos que ilumina. Esta teoría fue
sustentada por Max Planck y Albert Einstein .

• CLASIFICACION OPTICA DE LOS CUERPOS


PARA NO OLVIDAR!!
a) Cuerpos Luminosos.- Son aquellos que producen luz propia. Por
1] Llamamos rayo de luz a ejemplo: el Sol, las estrellas, el foco de una lámpara, etc.
la línea que le sirve de
dirección de propagación b) Cuerpos I1uminados.- Llamamos así a aquellos cuerpos en donde
a uno radiación luminoso. incide o llega la luz de otro cuerpo. Un cuerpo iluminado puede ser visible.
2] Denominamos haz lumi- e) Cuerpos Transparentes.- Estos se caracterizan porque dejan pa-
noso al conjunto de rayos sar la luz por el interior de su masa, de modo que podemos ver lo
lumInosos emitidos por que hay detrás de ellos.
una fuente. Pueden ser:
d) Cuerpos Opacos.- Todos estos cuerpos impiden el paso de la luz
a) Convergentes. a través de su masa, y debido a ello producen sombra (oscuridad)
b) Divergentes.
detrás de ellos.
e) Paralelos.
e) Cuerpos Translúcidos.- En estos cuerpos la luz puede atravesarlos
parcialmente, de modo que es posible ver objetos detrás de ellos.

INTENSIDAD LUMINOSA (1).


Denominados así a aquella magnitud física escalar considerada
como fundamental en el S.l. y que se define por medios subjetivos, dado
(e) que sl1 recurre a la apreciación de un observador medio. La intensidad
luminosa compara esa parte del flujo radiante que logramos ver (luz)
H
o\'
con la que emite el platino a su temperatura de fusión (2042 K) por cada
centímetro cuadrado. Por definición, esta emisión es igual a 60 candelas,
Ondas Electromagnéticas y Luz 457

siendo la candela (al) la unidad bási- S O "{B RA


ca de la Intensidad luminosa en el
S.l. Si colocamos un cuerpo
opaco (O) entre uno panta-
lla y uno fuente (F)puntual.
se formaró en aquello uno
>1.. Energía qmiITadia 1 cm sombra (S)cuyo silueta seró
60 de Platino a 2 042 K Igualo lo del objeto.
Platino
Fundente
Fig.24.5
.'ANGULO SOLIDO(O)
Cuando trabajamos con figu-
Casquete Esférico '
ras espaciales es común recurrir a
un tipo especial de ángulo espacial
llamado ángulo sólido, cuya carac-
terística es la de limitar una cierta
región de espacio por medio de una
superficie cónica o piramidal. En
la Fig. 24.6 se ha trazado una su- PENUMBRA
perficie esférica cuyo centro de
curvatura se encuentra en 0, desde Si el tamaño de lo fuente
la cual el ángulo sólido (n) (F) de luz es comparable
con lo del cuerpo opaco
intersecta a la esfera de radio r, li- (O), entonces lo sombra se
mitando una superficie de ésta y de compone de dos portes:
*) A_ =41tl Uno totalmente oscuro y lo
área A. La medida del ángulo sóli-
do viene dada por la relación: => Q••r.ra =41t sr otro porclalmentelluminodo
o penumbra. Solo desde lo
zona de penumbra puede
(24.1) Fig.24.6 verse parcIalmente o lo
fuente lumInoso, tal como
ocurre en los eclipses:
En el S.1. el ángulo sólido se mide en estereoradián (sr)

_FLuJO~~
Cuando estudiamos la forma
cómo el Sol irradia su energía al
espacio circundante, encontramos
que el 40% de ella se irradia en
el espectro visible (energía lumi-
nosa). El 55% en la región
infrarroja (calor) y el 5% restan- PARA RECORDAR
te en la zona ultravioleta. Llama- 1) El estereorrodlón (sr) es el
remos flujo radiante a la cantidad á-guIos51ldoquesubttende
de energía que emite o recibe por uno superficie esférico de
unidad de tiempo una superficie creo Igud 01coxrox: del
rodio de /oesfera.
por medio de ondas electromag-
néticas. En el S.l. se medirá en: Fig.24.7 2) Lo romo de /o Físicoque
joule/segundo = watt. estudotrxbs Iosferómeros'
luminosos o partir de lo
Observación.- En el ejemplo de la fig 24.7, el flujo radiante del foco incan- cuontlficoción de su ener-
descente coincide en valor con la potencia eléctrica que éste consume du- o go sederaniXJFo1orretrb.
rante su funcionamiento.
458 Física - Primer Nivel .Félíx Aucallanchi V.

SENSACION LUMINOSA
Uomomos asío lo respues-
lID FLUJO LUMINOSO (~J
to del ojo humano cuando Esta es una magnitud escalar, y es la componente del flujo radiante
se estImulo de algún modo que es capaz de producir sensación luminosa. Evidentemente también se
o lo retIna del mismo, yo seo
por medios mecánicos (gol- puede medir en vatios. La unidad del flujo luminoso es en realidad el
pe) o por uno radiación lumen (1m) en vez del vatio; si bien ambas son unidades de potencia
electromagnético (luz). (energía con respecto al tiempo), el lumen no se relaciona de manera
tan sencilla con el vatio; a determinada longitud de onda, por ejemplo
a 5 540 A, que corresponde al color verde-amarillo, se registra la máxima
LUMEN sensibilidad del ojo huma-
no, y en tales condiciones:
Eslo unidad de flujo lumi- Un vatio de flujo radiante
noso, y se define como lo equivale a 685 lumens de
cantidad de luz que Irradia
uno fuente de uno candela flujo luminoso. Para otras
o través de un ángulo sóli- longitudes de onda se esta-
do Igualo un esfereorradlán. blecen otras equivalencias. Foco
Si deseamos asociar
el flujo luminoso (<1>L) con 1= Icandela
TABLADE RENDIMIENTOS la intensidad luminosa (J)
del foco, encontraremos que
ambas son directamente
Lámparo 1/
L
(lmlW) proporcionales:
Más pequeño (1.5 W7 2 <1>Lex/ EneIS.J
Wa/from/olVOCÍO (24 W7 10 Il/umen= 1 candela. 1 estereorradiánl
Asimismo, el flujo luminoso
Wolframio/gas (60 W7 14
dependerá de la región de Fig.24.8
Wo!frcrn/olgas (100 W7 16
espacio en que se irradia, es
Fluorescente (30 W7 50 decir, depende del ángulo sólido (Q): <1>L ex Q
Fluorescente (100 W7 44
Wolframio
Lucolox
(500 W7 20
105
Luego, podemos establecer que: ¡tt L = 1.0 1 (24.2)

EJID RENDIMIENI'ODE UN FOCO WMINOSO (7lJ


CUIDADO!! Se define el rendimiento luminoso como la relación existente entre
el flujo luminoso (<1>J Yel flujo radiante total (Pot) emitido por un foco de luz.
Engeneral, cuanto mayor
es el rendimiento, menor es
el tiempo de vida útil que
tiene lo lámpara. Por ejem-
plo, el foco de uno como-
I T/L =~t I (24.3)

ra fotográfico puede durar De este modo, el rendimiento de un foco en el S.I. se expresará en lumen
de 5 o 10 horas, los domés-
tIcos duran unos 1 000 ho- por watt (lm/W). Para la luz monocromática de longitud de onda igual
ras, los lámparas f1uores- a 5 540 A el rendimiento teórico es de 685 W.
centes alcanzan los 10 000
horas, y los de vapor de
mercurio 16000 horas. Es-
tos duraciones disminuyen
EIIII ILUMmACIO~ (1)
Esta magnitud fotométrica es de naturaleza escalar, y expresa la
drástlcamente silos lámpa-
ras experimentan variacio- cantidad de flujo luminoso que recibe la unidad de superficie. Así, su
nes frecuentes en su tensión valor medio se encuentra por medio de la siguiente relación:
eléctrico.
I y=~ ¡ (24.4)
Ondas Electromagnéticas y Luz 459

La unidad de iluminación puede ser el watt/m2; sin embargo, en la prácti- LUX


ca, se emplea el lux (Ix), el cual se define en función de las magnitu-
. Es la unidad de ilumina-
des fotométricas; de este modo: 1 lux = 1 lumen/m-, ción, y se define como el
flujo luminoso de un lumen
que Incide sobre una super-
ficie de un metro cuadra-
do.

INTERESANTE
En el ejemplo de la Flg.
24.100, el flujo luminoso
Irradiado txx la vela {fuen-
te de luz] en una dirección
determinada y a 1m de dis-
tancia puede iluminar una
loseta. A 2 m de distancia
Fig.24.9 el flujo luminoso debe divi-
dirse entre 4 losetas Iguales,
por lo que cado uno presen-
_ILUMINACION PUNTpAL ta 1/4de la iluminación que
experimenta la primera lo-
seta. A 3 m de distancia, el
Cuando el flujo luminoso o radiación de luz proviene de un pun- flujo se divide entre 9 lose-
to como en el ejemplo de la Fig. 24.lOa, notamos que para un mismo tas: por ello cada loceta
ángulo sólido (n) dicho flujo puede iluminar con menos intensidad a presenta 1/9 de la ilumina-
las superficies más alejadas, y según como se ve en la figura, la ilumi- ción que experimenta la
primera. De ésto se dedu-
nación se va reduciendo con el cuadrado de la distancia (d). Asimismo, ceque:
se puede asegurar que la iluminación aumentará si aumentamos la in-
tensidad luminosa (l) del foco, y aún más si los rayos luminosos inci-
den normalmente a la superficie; es decir, la iluminación sólo depen-
de de los rayos normales a dicha superficie, lo que nos conduce al uso
del coseno del ángulo (9) de incidencia; por tanto, la iluminación (Y)
en un punto de la superficie estará dada por la siguiente relación:

I Y=f,:~1 (24.5)
ILUMINACIONES
RECOMENDADAS

s:
a) b)

Salas Y{Ix]

J~~l6
\
• Normal Espectáculos 100
Rayo\!i ;-'
Lectura {bibliotecas] 325
Aulas 325
Dibujo 540
Laboratorios 3 200
Mafricería {tal/eres] 5 400
Operaciones 5400 a
{clínicas] 10300

Fig.24.10
Física - Primer Nivel Félix Aucallanchi V.

JlROBLEMAS RESUELTOS (FA PARTE)


Probo 1.- ¿En qué longitud de .svio podemos sintonizar una estación de radio que emite señales
a una frecuencia de 12 MHz?
Al 100 m B} 75 m C} 12 m D} 300 m' El 25 m UNMSM 89
ResoluciÓn.-
De acuerdo con los datos se tiene.j' = 12 MHz = 12.106 Hz; y por teoría se sabe que la velocidad de las ondas
electromagnéticas es: e = 3.108 mis. Luego, la longitud de onda buscada la podemos encontrar utilizando la
siguiente relación:
8
Lf =e ~ A = ~ = 3.10 m1s 11.= 25 m RPTA.E
f 12.106Hz

Probo 2.- Un haz de luz roja (iongitud de onda 7 000 A) se propaga en el vacío e incide en una
región R. En su camino se cruza con un haz de luz violeta (longitud de onda 4 000 Al de
la misma intensidad. La frecuencia de la luz que llega a R es (aproximadamente):
A) 6.1014 Hz B} 7,5.1014 Hz C} 7.1010 Hz D} 1.1.1010 Hz El 4,3.1014 Hz UN! 91
ResoluciÓn.-
Recordando que los rayos luminosos son de naturaleza
ondulatoria, diremos que ellos deberán satisfacer el Prin-
cipio de Superposición de Ondas visto en el item 14.4.
Por esta razón, afirmaremos que el cruce de los rayos
violeta no alteran el paso de los rayos rojo; por lo tanto,
la radiación luminosa que llega a R es de color rojo, cuya
frecuencia la encontraremos en base a la relación utili-
zada en el ejercicio anterior.
8
e 3.10 mis
y como 1 ti = 10-10 m
f = "f. = 7 000 A
3. 108 mis
4,3 . 1014 Hz RPTA. E
! ".

Probo 3.- La longitud de onda y frecuencia de dos ondas electromagnéticas que se propagan en el
mismo medio son Al vt, para una onda y~yf2para la otra. ¿Cuál de /os siguientesenunckxios
es correcto?

A} Si fA..jAl} =2 entonces (~~ )= 3 D} Para (f/f2) = 1/6, se tendrá que fA..j/"l} =6

B} Si fA..j/"l} = 1, será: U~)= 3 E} (f¡lf21 =4 implica que (~~ )= 3


e} Se tiene que fA.¡I/..21= %' cuando (f¡lf21 ., 2/3.
Resolución.-
Todas las radiaciones electromagnéticas que se propagan en un mismo medio tendrán la misma velocidad.
Luego, dos de ellas deberán relacionarse entre sí del siguiente modo:
/..1!1=/..2h
!:L 12
/"2-!1 ;ó ;
u:».
/..1-12
Ondas Electromagnéticas y Luz 461

Así pues, analizando las alternativas A, B, C y E, vemos que ninguna de ellas verifica las igualdades obtenidas.
Sin embargo, notamos que en "C":
2:L_1 fz_1
1.2-2 ~
fl
-
2
;ó; RPTA.C

Probo 4.- En el punto A se coloca una fuen-


te luminosa de 62,5 candelas, yen
el punto B otra de 22,5 candelas. .r sm K 5m~ "
Viendo el fotómetro, ¿Cuál será su ~ - - -i - - -' rmn1
lectura? •.--- LLJ
B . .
A] 1OJ51ux D]3,75Iux

B]8,5Iux E] 1,5251ux

C] 85 lux UNMSM 85

Resolución.-
De acuerdo con los datos y empleando la relación (24.5) para la iluminación (Y), tendremos:

Para "A": Y =~=62,5cd ~ YA=0,625lux


A d!
(lOm/
"" v __lB _ 22,5 cd __ v O9 l
P ara B : -s - - -<' lB = , ux
d~ (sm/
Finalmente, la iluminación total que mide el fotómetro estará dado por la siguiente suma escalar:

YT = YA + YB = 0,625 + 0,9 ~ YT = 1,525 lx RPTA.E

Probo 5.- Una lámpara A de 16 candelas y otra B de 9 candelas distan entre sí140 cm. ¿A qué
distancia de la lámpara A hay que poner una pantalla para que esté Igualmente ilumi-
nada por ambos focos?,
Al 90 cm Bl 70 cm C] 80 cm D] 6n cm El N.A. UNFV 87
Resolución.-
De acuerdo con el esquema adjunto y los datos del
problema, se reconocen los siguientes datos:

11 = 16cd, 12 = 9 cd, d = 140cm

Luego, de acuerdo con la condición del problema y


utilizando la relación (24.5) para la iluminación (Y),
tendremos:
+----<
I

140 - x)--+---
16cd 9cd
,
(140-x)2 - x2 •..•
----d= 140cm--- .•

4 3
----- ~ x=60cm RPTA.D
140- x x
462 Física - Primer Nivel Félix Aucollonchi V.

I
Probo 6.- En el gráfico se muestran dos lámpa-
ras de Igual intensidad luminosa. Si la
Iluminación tot~1 en el punto A es de
3,04 tx: ¿cuál es la intensidad I de las
lámparas?

A}50cd C} 80 cd E} 90cd

B}60cd D} 10cd

Resolución.-

De acuerdo con los datos del ejercicio y en base al


siguiente adjunto reconocemos que:

/I =x /2 =x
Foco (1):
{
di = 10 m Foco (2): d: = 6 m
9 = 53° 19 = 0°
Luego, la iluminación total en A se encuentra su-
perponiendo las iluminaciones que cada foco pro-
duce de manera independiente: <, t,
----------
------t:~
(2):
¡dz=6 m
,,

::) x = 90 cd RPTA.E

Probo 7.- Una lámpara Incandescente tiene una intensidad luminosa de 35 candelas. Calcular el
flujo luminoso que Irradia {1t '" 22/1}.

A} 110 1m B} 220 1m C}330lm D} 440 1m E} 550 1m

Resolución>
Recordando que el ángulo sólido que subtiende una esfera (todo el espacio) es igual a 41t estereorradianes,
calcularemos el flujo luminoso de la lámpara a partir de la relación (24.2).

<l>L = m. = J.41t = J . 4(2217)


Reemplazando datos: <l>L = 35 . 4 (22/7) ::) <l>L = 440 lm RPTA.D
Ondas Electromagnéticas y Luz 463

DENSIDAD OPTlCA
Con este nombre se le
Entre los fenómenos ópticos con los que estamos más familiariza- conoce también al índice
dos tenemos la reflexión en los espejos y la refracción en los lentes. Todos de refracción de un medio.
de alguna forma hemos tenido la curiosidad de saber por qué detrás de un Un mayor valor de esta den-
espejo plano aparece siempre una imágen «similar» a nosotros. Asimismo, sidad sugiere una disminu-
ción en la rapidez del mo-
nos ha causado sorpresa el poder aumentar el tamaño de los objetos por vimiento de la luz. El valor
medio de una lupa; pues bien, estos fenómenos están muy relacionados de la densidad óptica de un
entre sí, y es lo que explicaremos a continuación y con mucho detalle. medIo transparente se cal-
cula para una longitud de
onda determInada. Se ha
BlINDICEDEREFRACCIONDEUNMEDIO(n) descubIerto que:

Cuando la luz viaja en el vacío


manifiesta una velocidad e, y cuando
lo hace dentro de un cuerpo transpa-
rente, su velocidad es v < c; ésto per-
[na ~ I
mite definir el índice de refracción
de dicha sustancia del siguiente INDICES DE REFRACCION
modo: MedIdas con luz amarll/9
de IiqXYde!'lXilo{A = 5893A)
(24.5)
SustancIa n
Observaciones.« Cada sustancia trans- Aceite 1,51
parente tiene su propio índice de refrac- Fig.24.11 Agua 1,33
ción. Se observa que n ;:::1, Y que v= c/n.
Aire 1.0003

11I REFRACCIONDELALUZ Azúcar


Cuarzo
1.56
1.54
Cuando la luz pasa de un medio transparente a otro de diferente
índice de refracción, la dirección de su movimiento experimenta una DIamante 2.42
sensible desviación así, como también su velocidad de propagación; GlicerIna 1.47
cuando ésto ocurre se dice que la luz se ha refractado. Un sencillo HIelo 1.31
análisis nos permitirá demostrar que a causa de este fenómeno la lon-
Sal de cocIna 1.54
gitud de onda de la luz cambia, pero no así su frecuencia.
Vidrio Crown 1.50
A) Leyes de la Refracción de la Luz
VIdrIo Fllnt 1.7
In! Ley.- «El rayo incidente,
el rayo reflejado y la nor-
mal trazada en el punto de
incidencia a la interfase
están contenidas en un mis-
mo plano». MUY INTERESANTE!
2;!Ii! Ley.- Fué descubierta por
Willebrord Snell, y establece DebIdo al fenómeno de re-
fracción de la luz. una cucha-
que: ra dentro de un lIOSOCO'l agtKJ
nos parece quebrada: el fon-
In1sen91 = nz seD9l1 (24.6) do de una piscina nos pare-
ce poco profundo: en un día
B) Angulo Límite (L).- Cuando caluroso el aire parece ele-
los rayos inciden con ángulos varse, yen las ccneteios nos
crecientes, según la Fig. 24.15, parece ver chacras de
agua. Asimismo, alatardecer
los rayos refractados se alejan el Sol nos parece más grande
de la normal y se acercan a la Fig.24.12 v osxxx:
464 Física - Primer Nivel Félíx Aucalianchí V

MUY IMPORTANTE interfase. Existe un ángulo de "


incidencia "L" con el cual el rayo
Dé acuerdo con lo Flg. se pega a la interfase. Así, todo
24.13 podemos establecer
los siguientes relaciones: rayo que incide con un ángulo e > L
experimentará una reflexión total.
1) Cuando un rayo de luz
poso de un medio o otro
tres denso, éste se (24.8)
retracto acercándose o lo
normal. y si poso de un Fig.24.13
medio o otro menos den-
a)
so, el rayo refractado se
alejo de lo normal.

2) En los análisis de refrac-


ción es Indiferente reco-
nocer y diferenciar o los
rayos Incidente y
refractado.

OJO!
5) 6)

SI los rayos luminoSOSpo-


san de un medio o otro
menos denso, como en el
coso de lo Flg.. 24.15, se
noto que o medido que el
ángulo de Incidencia se
hace más grande, el hoz
refractado se alejo más de Fig.24.14 Fig.24.15
lo normal. y además se va
debilitando, mientras que el .,REFLEXIONDEIALUZ
hoz reflejado en el medio
más denso (vidrio) va en Si las ondas, luego de incidir sobre una superficie determinada
aumento hasta alcanzar el retornan al medio original de propagación cambiando la dirección de
ángulo límite, que para el
aire y el vidrio es 42°. su movimiento, pero manteniendo su misma rapidez, se dice que ex-
perimentan el fenómeno de reflexión.
A) Leyes de la Reflexión
TIPOS DE REFLEXION 1l:ll Ley) Fué descubierta por
Euclides, y establece que:
«El rayo incidente, el rayo
reflejado y la normal tra-
zada a la superficie de re-
~ flexión en el punto de in-
Reflexión regular
cidencia se encuentran en
un mismo plano».
2illl Ley) Fué descubierta por el
árabe Al Hazen, y establece
~ que los ángulos de incidencia
Reflexión difusa (i) y de reflexión (r) son
iguales entre sí, L-~-"-,-=",-,-~~~ __ ~~=--J
....••.
Fig.24.16
Ondas Electromagnéticas y Luz 465

B) Espejos IMAGEN "


En general Ilamamos espejo a toda superficie pulimentada so- Uamamos así a aquel pun.-
bre la que se produce únicamente reflexión regular. Cualquiera que sea la to o figura que se forma en
virtud a la intersección de
forma de las superficies, ésta divi- r-r- ~~~- __ ~
los rayos reflejados (o
de al espacio en una zona real (+) Zona refractados) o de sus pro-
ubicada delante del espejo, y la Virtua/(-) longaciones. Para los ojos
zona virtual (-) considerada detrás de un observador. la Ima-
gen es aparentemente
del espejo. La concurrencia de las __ el lugar de donde provienen
prolongaciones de los rayos refle-
jados forman figuras llamadas
imagenes. Una imagen será real o ~
virtual si aparece respectivamente en
la zona real o virtual. En los espe-
o
b

t
o
""""

",
<,

"'c.:__
{

-..!
:.~"
i; los rayos luminosos.

*) La imagen es real si se
forma por la Intersección
de los mismos rayos re-
flejados, apareciendo
jos planos la imagen es derecha, vir- f.<--- d ~"'¡k;-.--d--t frente al espejo (zona
tual, del mismo tamaño del objeto, L- .l:.- ~-=-' real).
y ambos equidistantes del espejo. Fig. 24.17
*) La Imagen es virtual si se
Espejos Angulares.- Son aquellos que están constituídos por dos espejos forma por la Intersección
planos que forman entre sí ángulo diedro. En ellos el número N de imáge- de las prolongaciones de
los rayos reflejados, apa-
nes completas depende del ángulo (9) que formen los espejos (Fig. 24.18). reciendo detrós del espe-
jo (zona virtual).
360°
N=ar-1 (24.9)

Espejos Paralelos.- En estos espejos la imagen de un objeto se consti- MUY INTERESANTE


tuye en objeto para el espejo que está frente a él, y la imagen proyec-
tada será el nuevo objeto para el espejo que está en frente, y así sucesiva- Para un par de espejos
mente, generándose infinitas imágenes (Fig. 24.19). angulares planos se verifica
el siguiente cuadro de da-
tos:

Angula N2 de lmagenes

_--- -__ rObjeto / , 18CJ' 1


1 ,'-'-
•...~~~
2
-"',·0 -: -: "
" 120' 2
/ ~--..--::-- <»;
.. ~t5~'_fti p-~-- /'2 90' 3

\,,~:.:::-:-- --
t
:
b
2'"'lntdgen
a
-¡;
;2-lmág ••
7~
&J'
4
5
13 """ ..
: mE, d .
:""EZ
51.~ 6
4SO 7
360°
~e---- dr"rn ---4-
8=90° =:;,N= 900 -1 40' 8
36° 9
:. N= 3 Imágenes
32.7" 10
O' 00

Fig.24.18 Fig.24.19
11I ESPEJOS ESFERICOS Nota. - Este cuadro ha sido
extraído del libro: FISICA de
los autcres: Irwin Genzer & Philip
Yougner. de Publicaciones
Son aquellas superficies reflectoras formadas sobre un casque-
Cultural SA edición 1975(M).
te esférico, y se llamará cóncavo si el espejo se forma en su interior,
y convexo st está en la parte externa del mismo.
466 Física - Primer Nivel Félix Aucallanchi V.

DEBES SABER QUE A) Elementos de un Espejo Esférico r=~-=--------'-'--"""'-~"""""'""""",""--------'

1) Centro de Curvatura (C).- Es el a)


SIhacemos un corte a una
esfera hueca obtendremos centro de la esfera que origina al
dos casquetes, y utilizando espejo.
a uno de ellos puliremos sus 2) Eje principal (EP) recta que pasa
dos superficies para así con-
seguir dos espejos: El Inte- por C, y divide la sección princi-
rior es el espejo cóncavo, y pal del espejo (AVB) en dos partes
el exterior es el convexo. iguales.
3) Vértice o Polo ill.- Intersección
del espejo con EP.
4) Radio de Curvatura (R).- Es el b)
radio de la esfera que dió origen
~ R
,, al espejo.
p
r
5) Foco Principal (F).- Punto de
EP, en donde convergen todos los
Espejo rayos que inciden en el espejo
cóncavo paralelamente al eje principal.
6) Distancia Focal (j).- Es la distan-
FOCO PRINCIPAL cia entre el foco principal y el es-
pejo. Fig.24.20
Todos los espejos esféricos
tienen un foco principal [F).
el cual se ubica sobre el (24.10)
eje principal, y equidistan-
te del centro de curvatura
(C) y el vértice (V) del espe- B) Rayos Principales
Jo.
La construcción de imágenes
a)
requiere del uso de por lo menos
Espejo cóncavo un par de rayos luminosos de tra-
yectoria definida. Así tenemos:
1) Rayo Paralelo (RP).- Llega al
espejo de forma paralela al eje
principal y se refleja pasando
por el foco principal.
Espejo convexo
2) Rayo focal (RF).- Este rayo b)
pasa por el foco antes de llegar
al espejo y se refleja de forma
paralela al eje principal.
3) Rayo central (RC).- Antes de lle-
CUIDADO! gar al espejo, pasa por el centro
de curvatura, y se refleja confundién-
Es frecuente olvidar que el
aumento (A) tiene valor y
dose con el rayo incidente.
signo, /os cuales nos Informan Observacién» Para el caso de espejos convexos (Fig 24.21b), las prolongaciones
de las características de la de los rayos RF y Re pasan por el foco y centro de curvatura respectivamente.
imagen. Veamos:
C) Ecuación de Descartes
L> 1: Imagen mayor
lA I~ = 1: Imagen Igual Cuando colocamos un objeto (O) frente a un espejo esférico, éste
< 1: Imagen menor producirá una imagen que podóa ubicarse delante o detrás de aquel, o
simplemente no producir ninguna imagen. El reconocer la ubicación y las
A«+): Imagen virtual
caracteósticas de la imagen (l) fué un problema resuelto por el sabio
(-) : Imagen real francés René Descartes, y establece que: «La inversa de la distancia
Ondas Electromagnéticas y Luz 467

[ocal (f) es igual a la suma de las inversas de la distancia del obje- MUY IMPORTANTE

l;=t+! I
to al espejo (o) y de la imagen al espejo (i]»,
(24.11) r----=-r....:..------- --------,
o
Las Imagenes que txxie-
mas ver directamente son
aquellas que aparecen tie-
trás de los espejos; ésto es:
. Las Imagenes virtuales se
De donde despejamos "i": ven directamente. Sin etn-
. f .o
1=--
bargo, las Imagenes que se
forman delante de los es-
o-f pejos sólo son visibles si co-
locamos una pantalla [su-
Observaciones.- Todas las perficie plana) en el lugar
distancias tienen signo, ob- donde se coloca la imagen
servándose que "o" es siem- real.
pre positiva.
*) En el caso [d) de la Fig.
24.3, el objeto colocado
Además: en el foco principal [F)no
Fig.24.22 proyecta Imagen. dado
::::}Espejo cóncavo

::::}Espejo convexo I
<+): Imagen real e inversa

): Imagen virtual y derecha.

D) Aumento (A).- Se define como la razón que existe entre la altura


de la imagen (h¡) con la altura del objeto (ho)'
que los rayos reflejados
son paralelos, y nunca se
Intersectan.

PERISCOPIOS
(24.13) ¡!-AI: ~ 1 ' también: r=~"9 (24.14) Son dispositivos
constituídospor dos prismas
en donde los rayos de luz
E) Construcción de Imágenes.- Los casos de (a) hasta (e) corres- experimentan reflexión
ponden a espejos cóncavos, y sólo el caso (f) muestra la construc- total, lo cual permite trans-
ción de una imá en de un es eio convexo. portar una Imagen de un
lugar a otro. Este es el prin-
cipio físico que explica la
posibilidad de enviar
o Imagenes por fibras ópti-
cas, muy utilizadas en Me-
dlcina y en Telecomunica-
ciones.
a) b)

d)

e) f)

INTERESANTE!
Los espejos convexos
[Flg. 24.23f] proyectan úni-
camente imagenes vlrtua-
les y más pequeñas que el
Fig.24.23 objeto.
468 Física - Primer Nivel Félix Aucallanchi V.

PROBLEMAS RESUELTOS (2M NR7f)

Probo 8.- Si un espejo plano gira un ángulo a, entonces el rayo reflejado se desvía respecto al
original en un ángulo de:
A) a B) 90 + 2a C) 2a O) 90 - a E) 3a UNMSM 81
Resolución.-
De acuerdo con el esquema adjunto podemos reconocer que la normal (N) gira exactamente el mismo ángulo
a que gira el espejo. Pueden reconocerse asimismo los ángulos de incidencia e} y etqUe forma el rayo
incidente (RI) con cada normal N} Y N2, respectivamente. Luego, aprovechando la 2l1a ey de la Reflexión
tendremos:

1) Angulo de reflexión inicial formado por N} YRR}:


x+ a=e} ..... (1)

2) Angulo de reflexión final formado por N2 y RR2:

x + ~= e2 ..•.•. (2)

3) Angulo formado por las normales N} YN2:

a = e2 - e} (3)

Reemplazando (1) y (2) en (3):

a = (x + ~) - (x + a) => p = 2a RPTA. e

Probo9.- ¿Bajo qué incidencia debe un rayo luminoso encontrar un espejo plano para que dicho
rayo esté igualmente Inclinado sobre el espejo y sobre el rayo reflejado?

A)60° B) 300 UNFV 82


Resolución.-
R.1.
Sea a el ángulo que forma el rayo incidente (RI) con el R.R.
espejo y con el rayo reflejado (RR). Luego, de acuerdo
con el esquema, el rayo RR debe formar el mismo ángulo
(a) con el espejo. Por esta razón se debe cumplir que:

3a = 180 0
=> RPTA.A

Prob.10.- Una persona tiene una estatura de 1,7 m, Y sus ojos están a 10 cm debajo de su cenit.
¿De qué altura mínima debe ser el espejo plano colocado en una pared vertical para
que pueda ver su imagen completa, ya qué altura del piso debe encontrarse su bor-
de inferior?

A) 85 cm; 70 cm. B) 80 cm; 75 cm C) 85 cm; 80 cm

D) 90 cm; 60 cm E) 82 cm; 80 cm
Ondas Electromagnéticas y Luz 469


Resolución.-

Utilizando la 2~ Ley de la Reflexión, y aprove-


chando las características de las imágenes en un
espejo plano, hemos elaborado el esquema adjun-
to, en donde podemos reconocer:

1) Ó OES ...., Ó OP'M'

x d h
h - 2d
~ x=- ~ x=85cm
2

2) ó P'EH ...., ó p'OP

_y_ =.!L ~ _h- a


h - a 2d Y- 2 ~ Y = 80 cm RPTA.C

Observación.- De acuerdo con estos resultados podemos establecer que:

Longitud mínima del espejo (x) + Altura del borde inferior (y) ~ :;0 h

Probo 11.- Un hombre de altura h esto


frente o un espejo de 1m de
ancho y de altura 3/4 ti. como
se muestro en /0 figuro. Poro
que e/ hombre puedo verse
de cuerpo entero tendrá que:
A) A/eJore/ espejo 1
3/4 h
B) Subir e/ espejo (1/4) h desde
et sueto.
C) Dejar el espeJe;>
como está.
~
1
D) Acercar el espejo

E) Subir e/ espejo (3/4) h desde


el suelo.

UNI 94-1
Resolución.-

Por lo visto en el problema anterior, vemos que la longitud del espejo dado: 3 h/4 = 75/1
garantizar que el hombre vea su imagen completa, pues como se sabe:
00 h, es insuficiente para

Altura del borde inferior + Longitud mínima del espejo ~ ::'71100 h (*)
Por ello, lo mas conveniente es elevar el espejo 1/4 h desde el piso, con lo cual se satisface plenamente la
condición (*).

RPTA. B
470 Física - Primer Nivel Félix Auca/lanchi V.

Probo 12.- (A qué distancia se forma la imagen de un objeto colocado a 180 cm del espejo,
cuyo radio mide 120 cm?
A) 60 cm B)90cm C) 120 cm O) 150 cm E) 180cm
Resolución.-
De acuerdo con los datos, se tiene que R =120 cm, y según la relación (24.9) la distancia focal será:
f = + R/2 = + 60cm. A continuación, utilizaremos la relación (24.11) para determinar la distancia de la imagen
al espejo.

. f.o (+60cm) (+180cm) •.•


tc:r---- o = 180 cm --
1=--=
0- f 180cm-60cm

i = + 90 cm RPTA. B oj
Observaci6n.- De acuerdo con el signo de
"i" deducimos que la imagen es real, inver- e
tida y mas pequeña.

IAI= 1 i. 1 = 1- 90cm 1= 1
o 180cm 2

Probo 13.- Un espejo cóncavo tiene una distancia focal de 20 cm. (A qué distancia del espejo
debe colocarse un objeto para que la imagen sea real y de doble tamaño que el
objeto?
A) 10 cm B) 30 cm C)40cm O) 50 cm E) 60 cm UNI88
Resolución.-
De acuerdo con la condición del problema y por lo
visto en el item 24.18E,la imagen será real y ma-
yor que el objeto si éste se encuentra entre el foco
y el centro de curvatura. Así, el aumento (A) ten-
drá signo negativo, y de acuerdo con los datos será: o
A = - 2.A continuación utilizaremos las relaciones e
(24.12) y (24.11) para encontrar la distancia (o)
solicitada:
11
I
A = .L ~ - 2 = .L ~ i = 20 '+-f=20-j
o o
.•. o '//'
y reconociendo que f = + 20 cm (Espejo cónca- ----- 'l"
vo), tendremos que:

1 1 1
+ + o = 18m RPTA. B
f o 3m 0/5 o

Probo 14.- Para un espejo esférico cóncavo de 6 m de radio, (A qué distancia del espejo hay que
colocar un objeto perpendicularmente el eje principal para obtener una imagen in-
vertida, cinco veces menor que el objeto?
A) - 10 m B) -12 m C) + 12 m O) + 18 m E) N.A.
UNFV 87
Ondas Electromagnéticas y Luz 471

Resolución.-

De acuerdo con lo expuesto en el item 24.18e, la imagen será real y de menor tamaño que el objeto si éste se
ubica más allá del centro de curvatura. De este modo podemos asegurar que el aumento (A) será negativo, y de
acuerdo con los datos su valor será: A = - 1/5. Ahora, por la relación (24.12) tendremos:

i 1 i . o
'" '-"r---
A=--=>--=-- => 1=- /,
o 5 o 5
Reconociendo que la distancia focal del espejo está
o
dada por la relación (24.9), tendremos que:
e 1
f=+R/2=+3m
y ahora, por la relación (24.10) encontraremos la
distancia (o) del objeto al espejo:

1. 1+1 _1_
o=18m RPTA.D
f 3m
°
Probo 15.- ¿Cuántos milímetros se aleja la imagen real de un objeto formado por un espejo
cóncavo esférico de radio 135 cm, siel objeto seacerca de 45 m a 30 m del espejo?
A)2,5 8)5,2 C]0,5 0)3,5 E) 4,2 UNI92
Resolución.-

A partir del esquema adjunto y empleando la rela-


ción (24.1) para la ubicación de la imágen, tendre- ~Ol= 4500 cm ~
mos que el desplazamiento de ésta estará dado así: -, l. +-02= 3000 cm---+
<,
x = i2 - i, (*)

135cm ( )
-2- 3000 cm e F
f·02
donde: i =--=7---<----~=69,05cm
2 02 - f

l35cm (000 )
. f.o, -2- 4 cm
Y• · '1
L -
-
-
01- f--7---"-----""""' = 68,53cm

Luego, reemplazando en (*), tendremos: x = 0,52 Cm = 5,2 mm RPTA. B


412 Física - Primer Nivel Félix Aucallanchi V.

LENTES CONVERGENTES
Los rayos que Inciden so-
11II PRISMAOPTICO
bre una de suscaras, y de Llamamos así a aquellos sólidos transparentes limitados por dos
forma paralela a su eje caras planas, en donde la luz que intente atravesarlo deberá experimentar
principaL se refractan con- dos refracciones, una por cada cara. Estos cuerpos producen imagenes
centrándose en un punto
del eje llamado foco prin- por refracción gracias a la concurrencia de las prolongaciones de los rayos
cipal. refractados. Los rayos incidentes (RI) al salir del prisma cambian su direc-
ción, y se llaman rayos emergentes (RE), los cuales forman entre sí un
ángulo 8 denominado ángulo
de desviación. Si hacemos
que el prisma gire haciendo
centro en O (ver Fig. 24.24),
se observará que el ángulo 8 000

va tomando distintos valores, y


de todos el mínimo (8.mín) se
presenta cuando el rayo
refractado interior (OE) es R.I.
perpendicular a la bisectríz del
LENTES DIVERGENTES ángulo de refringencia (A) del
En estos lentes, los rayos
prisma, de modo que se puede 3&min<=>i=~
que Inciden paralelos al eje establecer que: *) Cuando la desviación S es
principal se refractan de mínima se verifica que: t e
A

manera que susprolonga-


ciones se intersectan en un sen[ A+~mín] Fíg.24.24
plano del eje principal lla- (24.15)
mado foco principal de la 2 =n1 sen{AI2}
lente.

_LENTES
Si construímos un sóli-
do con varios prismas conse-
guiremos que la luz se
refracte dos veces, y gracias
a ello dispondríamos de un
cuerpo capaz de producir
imágenes por refracción, tal
PARA NO OLVIDAR
como apreciamos en la Fig.
En base a la relación 24.25. Llamamos lente a
(24.16) podemos decir que aquel cuerpo transparente li-
una lente es convergente o mitado por dos superficies,
divergente según el medio
donde se encuentre sumer- una de las cuales es siempre
gida. Esto se explica por el esférica. Por sus propiedades
hecho de que la distancia refractantes los lentes son uti-
focal (f] depende de los ín- lizados principalmente para
dices de refracción de la
lente {nJ y del medio (nj
corregir defectos visuales
que lo tixieo. Así: como el astigmatismo, mio- F.. 2425
pta, e h'tpermetropta., 19. .
~>no ~ f(+) : Lente con-
vergente. 1) Tipos de Lentes.- Las lentes pueden ser convergentes o positivas
(Fig. 24.26), Y divergentes o negativas (Fig. 24.27). Los primeros
'i. < no ~ 11-): LErl1e cJ.-€ygsn te se caracterizan por tener sus bordes delgados, y los segundos por
tenerlos gruesos.
Ondas Electromagnéticas y Luz 473

OJO
Lentes Convergentes Lentes Divergentes
En Id relación (24.17), la
C) C) regla de los signos para i, o
A) B) A) B) y f son las mIsmas que se
establecieron para el caso
de espejos. Del mismo
modo. el aumento (A) visto
para espejos tiene identica
aplicación para lentes.

A) Plano B) Biconvexo C) Menisco A) Plano B) Bicóncavo C) Menisco


Convexo Convergente cóncavo Divergente
Fig.24.26 Fig.24.27
2) Foco de una lente.- Toda lente presenta la propiedad de hacer concurrir o
dispersar, según sea el caso, a los rayos luminosos que inciden sobre cual- ATENC/ON!
quiera de sus caras. Lo mismo que vimos en espejos esféricos, estos
rayos refractados se reunen en puntos definidos ubicados en el eje La disposición de un obje-
de la lente, comprobándose to. de la lente y del obser-
vador. es sIempre así:
que existen dos de estos pun-
tos, uno a cada lado de la len- Objeto - Lente - ccsovoxx
te; por 10 tanto, diremos que
toda lente presenta dos focos (*] El cosetvcxx» a través de
E'e la lente nunca ve al ob-
principales. La distancia
jeto tal como es; sólo
focal (j) viene dada por la logra ver su Imagen.
siguiente expresión, l1ama- Para que esta imagen
da Ecuación del Fabricante: sea vista directamente
,/ (+) ~ Cara Convexa por el observador. ella
deberá formarse detrás
R" (_) ~ Cara Cóncava de la lente. es decir. en
el mismo sector en don-
(24.16) Fig. 24.28 de se ubica el objeto.

Observación.- no y nL son respectivamente los índices de refracción del


medio y de la lente.
3) Ecuación de los Focos Conjugados.- Para determinar la posición de
la imagen con relación a una lente basta con determinar la distancia
imagen (i), y ésta se relaciona con la distancia objeto (o) y la distan-
cia focal (j) del mismo modo que vimos en espejos esféricos, hecho
que fué descubierto por el científico alemán Karl Friederich Gaüss.

[[8]
r:
f =
11
(24.17)
-:
CUIDADO!!
De acuerdo con la Fig.
24.30, las lentes divergentes
Observación.- En las lentes el ob- O
siempre producen imáge-
jeto define la zona virtual de , -2P- - P;, -- nes vlrtuales. derechas y
modo que la zona real se ubica ' más pequeñas que elobje-
siempre al otro lado. Asimismo, para Z. V.(-) : ~ f I
to. EstaImagen siempre se
la construcción de imágenes se re-
curre a dos rayos principales, '--
~
~
o __ _$-
'i>_
i --l
__:=-__:_:_::_:'
ubica entre el foco princi-
pal (F) yel centro óptico (O).
los mismos que empleamos en espejos. Fig.24.29
474 Física - Primer Nivel Félix Aucallanchi V.

SIMBOLOGIA 4) Construcción de imagenes.- Los casos de (a) hasta (e) corres-


ponden a lentes convergentes, y el caso (f) es para una lente diver-
Se estila en casi todas las gente. En todos los casos se emplean símbolos para las lentes.
bibliografías utilizar símbo-
los para representar a las
lentes delgados, las que tie- a)
nen en consideración la for-
ma de los bordes, y así te-
nemos:
o

LENTE CONVERGENTE

e) d)

e) f)

[ --
.::::--
.••..
LENTE DIVERGENTE
1 ~ _~
o
F

o Fig.24.30
5) Potencia de una lente (C).- Se
le llama también poder conver-
gente, y viene a ser una carac-
terística de la lente. Se mide por
la inversa de la distancia focal, e,
y se expresa en dioptrías cuan-
OJO do la distancia f viene dada en
metros.
El punto O señalado en las
lentes es conocido con el
nombre de Centro óptico. (24.18) Fig.24.31

Observaci6n.- Cuando se tiene un conjunto de lentes delgados y en con-


tacto, el conjunto actúa como una lente cuya potencia viene dada por la
suma de las potencias de -las lentes que componen el sistema. Así, para el
ejemplo de la Fig. 24 3/ se tendrá que:
Ondas Electromagnéticas y Luz 475

nlO'LEMASIESUELTOS (JRA MlTE)

Probo 16.- Unrayo de luz monocromática incide del aire al agua, de modo que el rayo refleja-
do es perpendicular al rayo retractado. ¿Cuál tué el ángulo de incidencia? Indice de
refrooción del agua = 4/3.
A) 30" B) 37" E) 60"
Resolución.-
Rayo
Reflejado
De acuerdo con la condición del problema se ha
elaborado el esquema adjunto, en donde los ángu-
los de incidencia (a) y de refracción (~) resultan
ser complementarios: a + ~ = 90°. Y ahora, por la
Ley de Snell dada en la relación (24.6) tendremos
que:

4
naire' sena = nagua .sen~ ~ l . sena = "3 .cosa
4
~ tga= 3" ~ RPTA.D

Prob.17.- Determinar el ángulo de refrlngenc/a de un prisma transparente, si se sabe que la


desviación mínima que produce es de 30". Indlce de refracción del prisma = J2.
A) 10" O) 37" E) 60"

Resolución.-
Empleando la relación (24.15) que relaciona el ángulo de refringencia (A) del prisma con la desviación mínima:
0mín= 30°, Y el índice de refracción del prisma n2 = J2 , asumiendo asimismo que el índice de refracción del
medio que rodea al prisma es ni = 1; luego tendremos:

r A + 0mín lJ lr A + 30 10

sen l sen --- J () ( )


n2 = ni 2 ~ J2 = 1 2 ~ J2 .sen A = sen A + 15° (*)
sen(A/2) sen(A/2) 2 2

La expresión obtenida nos conduce a una ecuación trigonométrica cuyo desarrollo podría podría ser muy
extenso. Para evitar ésto, debo decirte que determinados problemas de exámenes tipo IBM pueden desarro-
llarse por tanteo empleando los valores dados en las alternativas, y para nuestro caso, de dichos valores, el que
satisface la relación (*) es:
A=60° RPTA.E

Probo 18.- Frente a una lente convergente delgada se coloca un objeto a una distancia de 50
cm. La imagen de este objeto aparece del otro lado a 60 cm de la lente. La distancia
tocal de la lente es aproximadamente:
A) 21,3 cm B) 30.0 cm C)25Acm 0)33 cm E) 24,1 cm
UNI85
476 Física - Primer Nivel Félix Aucallanchi v..

Resolución.-
Según los datos tenemos que la lente es convergente; luego, la distancia focalj'será de signo positivo. Asimis-
mo reconocemos la distancia del objeto a la lente: o = 50 cm, y la distancia de la imagen a la lente:
i = + 60 cm (Imágen real e invertida). Luego, empleando la relación (24.17). tendremos:
1 1 1 1 1 1
-=-+- => -=-+-
/ i o / 60 50
=> / •• 27;3cm RPTA. A
Observacián-De acuerdo con los datos y el
resultado obtenido, se puede asegurar que el Z.Y.(-)
objeto está ubicado entre F y 2F Y la imágen o
se forma en la zona real, ubicada más allá de
2F, y su tamaño es mayor que la del objeto: 2F
IL
L F
0= 50
IAI= -- i I = I---50
60 cm I = 1,2 ~2f=54,6
I o cm

Probo 19.- En la figura adjunta: SIlo distancia


del objeto O y la distancia de la Ima-
gen " de la lente delgada coñver-
gente es60 cm y 30'cm respectiva-
mente, ¿Cuál será el valor de la dis-
tancia focal de la lente?
A) 20 cm 0)45 cm
B)30cm E) 60 cm
C)90cm UNMSM 92
Resolución.-

De acuerdo con los datos tenemos: o = 60 cm, i = + 30 cm. Luego, utilizando la relación (24.10) tendremos:
l 1 l 1 1 1
-=-+- => -=-+- => / = 20 cm RPTA.A
/ i o / 30 60

Probo 20.- La lente mostrada en la figura es delgada, y tiene una distancia focal de f cm. Un
objeto "A"de 1 cm de altura tendrá una imágen cuya altura es en cm:
A) 1/3 B) 1/2 C) 1/6 O) 1 E) 2 UNI86
Resolución.-

De los datos y esquema original podemos reconocer que la distancia focal: o = 3f12.siendo/la distancia focal
de la lente. Luego, empleando la relación (24.11)encontraremos la distancia (1) de la imagen a la lente:

. /.0
1=--=
(n (3/12) => i;3/
0- / 3//2-/

A continuación emplearemos las relaciones (24. ) Y (24. ) para el aumento, en donde encontraremos la altura 1
de la imagen, siendo la altura del objeto 0= 1 cm:

IAI=I- ~I => h¡ I3/


ho = - 3/12
I => _1_=2
lcm
=> 1 =2 cm RPTA.E
Ondas Electromagnéticas y Luz 477

Probo 21.- Unobjeto luminoso está a cierta distancia L de una pantalla. Cuando un lente se coloca
a U2 de la pantalla (entre el objeto y la pantallaJ se produce la imagen real del objeto
luminoso sobre la pantalla. Diga el tipo de lente y su distancia focal.
A) Convergente, U4 B)Convergente, L/5 C) Convergente, L/3
O) Divergente, L/4 E) Divergente, L/3 UNI 94-2
Resolución.-
De acuerdo con el esquema adjunto podemos notar que la imagen se coloca al otro lado de la lente, el cual nos
indica que ésta es real, y por lo tanto la lente es convergente, pues solo ellas pueden producir imágenes de ese
tipo. Asimismo, reconocemos que:
0=1/2.; i=+1/2.

Luego, mediante la relación (24.10) encontraremos


la distancia focal:
Pared
1 1 1 1 1 L
7=0+i = L/2 + L/2 ~ 1=4 RPTA. A

Observación.- Por la equidistancia existente


entre el objeto y la imagen con relación a la
lente, podemos asegurar que éstos tienen el mis-
mo tamaño. pues ambos se ubican a la distan- L/2 U2
cia 2/ de la lente. (0)--- (i) -~I

Probo 22.- Un proyector de diapositivas se usa para obtener imagenes de 30 x 30 cm. Las
dlapositivas tienen 3 x 3 cm, y se encuentran a 20 cm de la lente convexa. iA qué
distancia se colocarán las imágenes? "
A) 0,1 m B)2m C)3m 0)1 m E)6m UNMSM 82
Resolución.-
Teniendo en cuenta que el aumento (A) se define en función de las dimensiones lineales del objeto y de la
h¡ 30cm
imagen, según los datos tendremos que: lA 1= - = -- = 10
ho 3cm
Ahora, empleando la relación (24.14), y reconociendo que la distancia (o) de los objetos a la lente se cono-
ce: o = 20 cm, estableceremos que: -' • .

1 . I= li I
= IO li 1 = 200 cm =2m RPTA. B
I
- -0 IAl =$ --
20 cm
~

Probo 23.- La distancia entre un foco (lámpara eléctrica) y una pantalla (plana) es d = 1 m.
¿ParaquéposicIones de una lente convergente, Interrr.~dio entre el foco y la pantalla,
con distancia tccct f = 21 cm, la imagen del filamento ncondesceme de la lámpara
se verá nítida en la pantalla?
A) 15 cm y 85 cm de la lámpara O) 70 cm y 30 cm de la lámpara
B)60 cm y 40 cm de la lámpara E)42 cm y 58 cm de la lámpara
C) 56 cm y 44 cm de la lámpara UNI 89
478 Física - Primer Nivel Félíx Aucallanchi V.

Resolución.-
De acuerdo con el esquema elaborado según condiciones del problema, podemos reconocer que la imagen
es real e invertida; por consiguiente las dis-
tancias (1) y (o) serán en centímetros:
= =
i 100 - x, o x, siendo x la distancia que
nos interesa calcular, la misma que deberá ser Pared
de signo positivo. Empleando la relación (24.17)
tendremos: Imagen

1 1 1 1 1 1
-=-+- ~ -=---+-
/ o i 21 l00-x x

=> (100 - x) x = 2100

~ x2 - l00x + 2100 = O

y descomponiendo en dos factores el primer


miembro establecemos que:

(x • 70) (x - 30) =O
=> Xl = 70 cm y Xl = 30 cm RPTA.D

Probo 24.- Unobjeto de 12 cm de altura seencuentra a una distancia 3f de una lente divergente
cuya distancIa focal esf. ¿Quéaltura tendrá la Imagen correspondiente?
A}3cm B}4cm C}6cm D}9cm E} 12 cm

Resolución.-
De acuerdo con los datos reconocemos que: o = 3/ Luego, en base a la relación (24.17) podemos encontrar la
distancia (i) de la imagen a la lente:

. (-f)o (-f)(3f) 3
I=O-(_f)= 3/+/ i=- ¡/ (Imagen virtual y derecha)

A continuación calcularemos la altura, empleando para ello las relaciones (24.13) y (24.14):

h¡ 1 -3/ /41 12cm


h.1 =--4 "=3cm
12cm = -31 ~ "., RPTA.A
Ondas Electromagnéticas y Luz 479

: 2.T" AUTOEVALUACIÓN

1.- Señala lo incorrecto. mano, son posibles gracias esencialmente al fenóme-


no de la _.. _ de los rayos X por los hue-
1) Todo campo magnético va asociado a un campo sos».
eléctrico.
A) Difracción D) Absorción
II) El campo magnético es siempre perpendicular al
B) Refracción E) Superposición
campo eléctrico.
C) Interferencia
1II) Toda variación de campo magnético produce un
campo eléctrico. 7.- Indicar verdadero (V) o falso (F):
IV) Toda variación de campo eléctrico produce un
( ) Las ondas electromagnéticas visibles están como
campo magnético.
prendidas entre 400 nm y 700 nm. -
A) I B) 11 C) I Y 11 D) III E) IV
- ( ) La luz se propaga rectilíneamente.
2.- Una onda electromagnética es: ( ) La luz está constituída por ondas longitudinales.

( ) Una perturbación de un campo magnético. A) VVV B) FFF C) VFF D) FVV E) VVF


( ) Una perturbación de un campo eléctrico. 8.- La relación correcta de las iluminaciones sobre A
( ) Originada por cargas eléctricas aceleradas. y B (YAIYB ) es como:
Indicar verdadero (V) o falso (F) A) '213
A)FFF B) VVV C) VVF D) VFF E) FFV B) 114
3.- No es una característica de una onda electro- C)419
magnética:
D) 3/2 ,,
A) Los campos E y ¡¡ son simultáneamente variables. ,, ,
E y ¡¡son perpendiculares E) 9/4
B) Los campos entre sí.
C) En el vacío se propaga con una velocidad de 3.1 OS
-+--2 m~ 1 m-4
mis.
D) Solo se propagan en el vacío. 9.- Señalar la relación correcta de las iluminaciones
(Y).
E) La longitud de onda es inversamente proporcio-
nal con la frecuencia. A) Y2< Y¡ = Y3
4.- Señala las palabras que completen mejor la si- B) Y2> Y¡ > Y3
guiente oración: «La luz es la parte del ,
_ electromagnético. C) Y2> Y3> Y¡

A) Oscura ; espacio D) Visible ; medio D) Y3= Y2= Y¡


B) Visible ; espectro E) N.A. E) Y¡ > Y2> Y3
C) Oculta ; medio

5.- Los rayos infrarrojos tienen longitudes de onda


comprendidas entre: 10.- El ángulo que forma el lápiz con su imágen cs.
A) 10.2 m y 10-3 m D) 10-8 m y 10-10m A) 37°
B) 10-5 m y 10-7 m E) 10-4m y 10-6 m B) 53°
C) IOmylO2m C) 60°
6.- Diga cuál es la palabra que completa mejor la D) 74°
siguiente frase: «Las radiografías que permiten a un Espejo
médico tener i rná gcnes de los huesos del cuerpo hu- E) 123°
480 Física - Primer Nivel Félix Auca/lanchi V.

11.- Si el ángulo formado por el poste P con su 1) Está entre 1 y 2


imagen es de 1I0°, ¿Cuál es la medida de a?
II) Es cóncavo
A) 45° P, o
III) Está entre 2 y 3
B) 15°

C) 25°
,,
D) 300 : Espejo
A) Solo 1 B) SoloIl C)SololII
E) 35° :.....rL Cl_
D) 1 Y II E) Ir Y III
12.- Dos espejos angulares proyectan cinco imá-
genes completas de la vela. Luego: 16.- Un rayo de luz cruza una placa de vidrio.
¿Cuál de las trayectorias indicadas tendrá el rayo
1) Necesariamen- emergente de la placa?
te 9 = 60°.
I1) 51 ° < 9 s 60°
IIl) Es posible que
9 = 55°

Señalar lo incorrecto:
A) B) C) D) E)

A) 1 B) II C) III D) 1 Y II E) II Y III 17.- En relación al siguiente esquema, en donde


las líneas indican la trayectoria de una luz
13.- Un espejo se aleja 10 cm de un objeto deteni-
monocromática, se propone:
do. ¿Qué distancia se habrá desplazado su imágen

r.
desde su posición original? (en cm). ( ) nI < n2
Espejo
11
( ) Al <A2
A) 25 r
B) 15 11
11 ( ) VI > v2
'.1/ 1
C) 5 'I!J?í: "--t> 11
/K 11 ( )fl <12
D) 10 :: Indicar verdadero
IJ
(V) o falso (F):
E) 20 • 10 cm •
14.- Si el espejo mostrado gira un ángulo de 10°,
¿Qué ángulo "9" se desviará el rayo reflejado res- A) VVFF B) FFVV C) VFVF D) FVFV. E) FFFV
pecto de su ubicación original?
18.- Si O es el objeto, y el esquema representa a
A) 20° una lente convergente, ¿Cuál es la imágen corres-
pondiente? F = Foco principal.
B) 10°

C) 15°
D) 30° o
A) B) C)
E) 5°
~-A __ ~ __ ~ __ ~ __ ~ __ -r~R
15.- Sabiendo que O es el objeto, e 1es su corres- E)
pondiente imágen (O > 1), entonces, en relación al
espejo se puede afirmar que:
Ondas Electromagnéticas y Luz 481

JlROBLEMAS JlROl'UEnOS

NIVEL 1 07.- ¿En qué relación se encuentran las intensidades


luminosas de los focos 1 Y 2, si la mancha de aceite
01.- Determinar la iluminación en el punto A, si la del fotómetro mostrado presenta la misma ilumina-
intensidad del foco es 1 = 8 cd. ción por ambas caras en la posición mostrada?

A) 160 Ix A) 1/2
2
B) 13 Ix B) 2/8 ,,1,;

C) 140 Ix
//,'
~/
,,~ ,
:
¡ 16 cm C)4/9 ;:O~
,
D) 6/9
D) 151x

E) 161x
/\53
0
b E) 7/3
+-20 cm+-30 cm-+
A
0.8.- Si en el fotómetro de Bünsen del problema
02.- Determina el flujo luminoso (<l>L) que incide
anterior'/2 = 90 cd. ¿Cuál es la intensidad (en cd) del
sobre las superficie indicada, cuya área es A = 5m2, foco" 1"?
si se sabe que la iluminación sobre ella es de 30 Ix.
A) 20 B)40 C) 60 D) 80 E) 100
A) 130 1m

B) 140/111
?'(:r-
;;'/,\," 0.9.- ¿Cuál es la iluminación (en kilolux) que presen-
ta la pantalla del fotómetro usado en el problema 08,

=,
, si la malla es de nylon?
C) 150 lm ,,,
, A) 1 B)6 C)9 D)2 E)4
D) 160/111
10.- En el espejo no podemos decir que se cumple:
E) 170lm 1. El aumento es -1
II. La imagen es virtual
03.- Si un foco irradia energía a razón de 200 W, I1I.La imagen es derecha.
siendo su rendimiento 2,5 Im/W, ¿Cuál es el flujo
luminoso (en 1m) que emite dicho foco? A)VVV B)VVF C)VFV
A) 400 B) 500 C) 600 D) 700 E) 800 D)FVV E)FVF
04.- ¿Cuál es el rendimiento (en Im/W) de una lámpa- 11.- Se muestra dos espejos planos que forman 110·
ra que irradia energía a razón de 100 W y produce un y un rayo que incide en el espejo reflejando también
flujo luminoso de 1,25. 104 1m? en el otro. Determine "9"
A) 121 B)122 C)123 D) 124 E) 125 A) io-
05.- ¿Cuál es el ángulo sólido (en sr) que subtiende la B)20·
superficie de una esfera?
C)30·
A) 4n B) 0,5 n C) 2n D) 5 n E) In
D)40·
06.- Un foco luminoso tiene una intensidad luminosa
1 = 70cd. ¿Cuál será el flujo luminoso (en 1m) que logra E) 50·
irradiar hacia todo el espacio que lo rodea? (n == 22/7).

A)8oo B)850 C)750 D)880 E) 890 12.- Dos espejos planos forman un ángulo de 600,
encuentre la medida de "9"
482 Física-Primer nivel Félix Aucal/anchi V.

A)90° NIVEL 2

B) 80° 17.- Empleando un espejo esférico cóncavo se


obtiene una imagen virtual, luego el tamaño de esta
C)6Qo imagen es:

D)40° A) Mayor que el tamaño del objeto

E) 20" B) De igual tamaño que el objeto

C) Menor que el tamaño que el objeto


13.- Se muestra un caja cúbica interiormente reflectora
y un rayo incidente que luego de 3 reflexiones energe D) De mayor o menor tamaño que el objeto
de la caja. Hállese "O"
E) Impredecible
A) 20°
18.- El tamaño de una imagen virtual es la mitad del
B) 30° tamaño del objeto ¿A qué distancia (en CII1) del vértice
del espejo convexo de 60 CII1 de radio se halla el
C)40° objeto?

D) 50° A) 15 B)20 C)30 D)45 E) 60

E) 60" 19.- El radio de curvatura de un espejo esférico


cóncavo es de 40cIII I.A qué distancia del espejo (en
CIII) debe colocarse el objeto para obtener una imagen
real cuya altura sea la mitad del objeto?
1-1.- Cuando un objeto se coloca a 60 CIII de un es-
pejo esférico se obtiene una Imagen derecha a 20cII1 A) 15 B)30 C) 60 D)80 E)90
del espejo, luego. son ciertas:
20.- Señalar como verdadera (V) o falsa (F) con
l La imagen es real respecto a los espejos esféricos.
11 El espejo es convexo 1). Las imágenes virtualcs siempre se forman detrás
del espejo.
111El aumento en dicha posición es 3
11). Los espejos convexos siempre dan imágenes más
A)VVF B) VFF C)FVV pequeñas.

D)FVF E)FFV III). Los espejos cóncavos siempre dan imágenes


reales.
15.- Un objeto es colocado a 6cIII de un espejo esfé-
rico obieniéndose una imagen invertida con un A)VFV B)VVV C)FVV
aumento de -5, luego son ciertas:
D)FFV E)VVF
1) La imagen del objeto es virtual
21.- Un espejo cóncavo de radio" R" puede empicar-
11) La imagen está a 30 CII! del espejo se como cocina solar colocando la parrilla en el eje
111) El espejo es cóncavo principal del espejo a una distancia "x" del vértice,
luego se cumplirá que:
A)VFV B) VVF C)FVV
A)x= R B)x =R/2 C)x>RI2
D)VFF E)FVF
D)x < R/2 E)x=O
16.- ¿Cuál es el radio de curvatura (en CIII) de un
22.- Considere que desde la superficie terrestre al centro
espejo de afeitar que da un aumento triple de un
del sol hay una distancia "S" y que el diámetro de éste
rostro a 30 CIII del vértice del espejo?
es "D" ¿Cuál será el diámetro de la imagen del sol
A) 60 B)70 C) 80 D) 90 E)95 cuando se emplea un espejo cóncavo de radio "R"?
Ondas Electromagnétiscas y Luz 483

A) OR/(2S - R) O) Cero 29.- Un haz de luz pasa por un medio donde /11 = 4
a otro cuyo índice es /12 = 1,4. Calcular la medida de]
B) SR/(O - R) E) OR/(2S + R) ángulo a indicado. .

C) OR/(S - R) A) 16°

23.- A través del espejo retrovisor un automovilista B) 30°


observa un camión con 1/20 de su tamaño real. Si se
sabe que la distancia focal del espejo es 40 cm, halle C)37°
la distancia (en m) entre el camión y el espejo en
dicho instante. O) 600

A) 5,6 B) 7,6 C) 9,6 O) 11,6 E) 13,6 E) 74°

24.- Un objeto se encuentra ubicado a una 30.- Delante de un lente convergente de 10 CIl1 de
distancia de 30 cm con respecto a un espejo distancia focal se coloca a 4 cm del mismo, un objeto
esférico cóncavo de 30 cm de radio de curvatura, luminoso se 12cm de tamaño de la imagen. Calcular
¿Cuál sería el radio de curvatura (en CIII) si se el tamaño (en cm) de la imagen.
desea tener una imagen a la quinta parte de la
imagen en el caso inicial? A) 20 B) lO C)30 O) 12 E) 15

A)5 B) 10 C) 15 O) 20 E)25 31.- ¿A qué distancia (en cm) de una lente biconvexa
se debe colocar un objeto para obtener aumento igual
25.- Frente a un espejo cóncavo de 60cII1 de radio de a 4; se sabe que el índice de refracción de la lente es
curvatura se coloca una vela de 20 cm de altura. Si 1,5 y que los radios de curvatura son 30 cm y 60
esta vela se ubica a 40 cm del espejo, calcular el cm?
tamaño (en cm) de la imagen.
A)5 B) 15 C)20 0)25' E)30
A) 20 B)30 C)40 0)50 E) 60
32.- Se tiene un lente biconvexa de radio dc curvatura
26.- La imagen real de un objeto producido por un 30 y 60 CII1, con índice de refracción 1,5. Hallar la
espejo cóncavo de 20cm de distancia fecal es cuatro potencia de la lente.
veces el tamaño del objeto ¿A qué distancia (en CII1)
se encuentra el objeto del espejo? A) 2 dioptrías B) 3 dioptrías C) 4 dioptrías

A) 20 B) 25 C) 30 0)35 E) 40 D) 4,5 dioptrías E) 2,5 dioptrías

27.- Calcular el ángulo de refracción de un rayo 33.- Un objeto es colocado a 60 cm de una lente
luminoso que está en un ambiente de índice 4 y pasa plano convexa, cuya cara esférica tiene un radio de
a un ambiente de índice 3 indiciendo con un ángulo 20 CIl1 e índice de refracción 1,5 ¿A qué distancia
de 37°. (en CIl1) dc la lente se forma la imagen?

A) 37° B)45° C) 53° O) 60° E) 300 A) 60 B)9 C) 120 D) 100 E) 150

28.- Encontrar un ángulo con la cual se refracta el 34.- Un cuerpo se encuentra a 60 cm de una lente
rayo de luz al pasar del vidrio al agua. bicóncava de 30 cm de distancia focal. ¿A qué dis-
tancia de la lente se forma la imágen y cuáles son sus
A) 30° características? •

B) 37° A) i = -5 cm; Virtual,derecha,de menor tamaño que el objeto,


B)i=-lOclll; Virtua dcrecha,de menortamaño qucelobjeto.
C)45°
C)i=-20clIl; Virtual,derecha,de rrerortamaño que elobjeto.
O) 53° O)i= -25clIl;Virtual.derecha,de mcnortamañoque elobjeto.
. agua
E) 600 E) i= -30cm; Virtual,derecha,de menortamañoquc elobjeto.
484 Física-Primer nivel Félix Auca/lanchi V.

NIVEL 3 A) 20 B)25 C)30 0)35 E) 40

35.- Una lente de 30 cm de distancia focal dá una 42.- Un espejo esférico cóncavo da una imagen real
imágen virtual 1,5 veces más pequeña que el objeto. cuyo tamaño es tres veces mayor que el objeto. De-
Determinar dónde está colocado el objeto. terminar la distancia focal del espejo, si la distancia
entre el objeto y su imagen es 20 cm.
A)o=llcm B)o=12cm C)o=14cm
A) 7,5 cm B) 8,0 cm C) 8,5 cm
O) 0= 15 cm E) 0= 17 cm
O) 9,Ocm E) Faltan datos
36.- Se tiene una lente biconvexa de radio de curvatu-
ra 30 y 60 cm con índice de refracción 1,5. Hallar la 43.- Un objeto luminoso se encuentra a 60 cm de un
potencia de la lente. espejo cóncavo. Si el objeto se acerca 10 cm al espe-
jo, la distancia entre éste y la imagen se hace 5/3
A) 1,4 dioptrias B) 2,5 dioptrias C) 3,7 dioptrias veces mayor. Calcular la distancia focal del espejo.
O) 4,2 dioptrias E) 5,9 dioptrlas
A) Es un espejo plano B) 9 km C) 1 mm
37.- Determinar los radios de curvatura de una lente O) 40 cm E) 60 cm
bicóncava cuyo índice de refracción es 1,5, Y su po-
tencia 5 dioptrias, sabiendo que uno de los radios es 44.- En un espejo esférico convexo se obtiene la ima-
el doble del otro. gen de un objeto reducida 10 veces, que dista 1,8 m
A)R¡=15cm;R2=30cm del espejo. Calcular el radio de curvatura del espejo.
B) R¡ = 20 cm; R2 = 40 cm A) Infinito B) \O cm C) Faltan datos
C) R ¡ = 40 cm ; R2 = 80 cm O) 15 cm E) 40 cm
D)R¡=IOcm;R2=20cm
45.- Un buzo estableció debajo del agua que la direc-
E) R ¡ = 30 cm ; R2 = 60 cm ción haciael Sol forma un ángulo de 37° con la verti-
cal. Al salir del agua notó que el Sol se encontraba
38.- Una lente bicóncava tiene 20 y 30 cm de radios más abajo respecto al horizonte. Definir en qué án-
de curvatura. Se coloca un objeto a 40 cm y se forma gulo cambió la dirección hacia el Sol para el buzo.
una imágen real a 50cm de la lente. Calcular su índice
de refracción. A) Es imposible definirlo O) 16°
B) Faltan datos E) 37°
A) 8,57 B) 0,23 C) 5,19 O) 1,54 E) 6,23
C) No cambió; es el mismo
39.- Alojo de un hombre penetra una radiación elec-
46.- En el fondo de un riachuelo yace una pequeña
tromagnética con una longitud de onda de 316 nm.
piedra. Un niño desea darle un golpe con un palo.
¿Percibirá el hombre esta radiación como luz?
Apuntando el niño, mantiene el palo en el aire bajo
A) La percibe como ondas ultravioletas un áegulo de 53° con la vertical. ¿A qué distancia de
B) La percibe corno rayos X la piedra se clavará el palo ea el fondo del riachuelo,
si su profundidad es de 36. cm?
C) La percibe como luz azul
O) La percibe corno luz roja A) Le dájusto a la piedra O) 18 cm
E) No la percibe B)2lcm E) Faltan datos
C) 15 cm
40.- Determinar entre qué valores deberá estar com-
prendido el ángulo diedro que forman dos espejos 47.- En el fondo de un. recipiente lleno de agua hay
plano de modo que el número de imágenes comple- un.espejo. Un individuo se inclina sobre el recipien-
tas visibles en ellos sea cuatro. te y ve la imagen de su ojo en el espejo a la distancia
A) 60° <El '.5,72° B) 55° < e '.5,60° C) e = 72° de visión óptimad = 25 cm, siendo la distancia des-
de el ojo hasta la superficie del agu,ah = 5 cm. Deter-
O) 72° < 8 '.5,80° E) N:.A. minar la profundidad del recipiente.
41.- La imagen real de un objeto producida por un A) Muy difícil D)9cm
espejo cóncavo de 20 cm de distancia focal es cuatro
B) Faltan datos E) \O cm
veces el tamaño del objeto. ¿A qué distancia (en cm)
se encuentra el tamaño del objeto? C) Los datos no son lógicos
Ondas Electromagnéticas y Luz 485

Lectura N° S

Existen dos aplicaciones tecnológicas de las lentes que son de gran importancia: El microsco-
pio, que nos permite visual izar los objetos pequeños, y el telescopio, que nos permite visualizar como
cercanos a los objetos muy distantes. En la presente lectura hablaremos de este último.
El telescopio, llamado también anteojo terrestre, holandés o de Galileo, es un anteojo
astronómico de modo que permita ver derechos los objetos observados; es el anteojo más antiguo
de cuantos se conocen. Se ignora quien lo inventó. Su teoría ha atribuído Roger Bacon (siglo
XVIII). Existe un documento del año 1 648 que menciona el empleo de "vidres de largua vista" por
los vigías de Mallorca. También se ha creído que un arquitecto de Gerona, hermano de Rogete de
Borgoña indujo los telescopios en España en el siglo XVI. Un fraile hijo de Rogete, los construía
hacia 1 500 de muy buena calidad. Otros afirman que se deben al alemán Santiago Merzu, pero más
se sabe que por casualidad fué inventado por un niño (hijo del óptico holandés Lippershey) que
jugaba con lentes en el taller de su padre (1 608). La noticia del descubrimiento llegó a Italia, y
Galileo, después de varios ensayos, logró encontrar la misma combinación. De aquí las denomina-
ciones de holandés y de Galileo que recibe este anteojo.
En esencia, los telescopios recogen y concentran energía luminosa para formar imágenes.
Esto se advierte sobre todo en los telescopios astronómicos que se usan para contemplar estrellas
y galaxias situadas a grandes distancias. Hay dos tipos generales de telescopio: Los de refracción
o dióptricos, que se basan en la refracción de la lente, y los de reflexión, que se basan en la reflexión
del espejo.
TELESCOPIO DEREFRACCION.-
Este aparato se asemeja a un micros-
copio, en cuanto está dotado de
objetivo y ocular; sin embargo, tie-
nen un objeto grande con una gran
longitud foca!. Los rayos paralelos
provenientes de un objeto lejano imagen
forman una imagen invertida en el intermedia
plano focal del objetivo; éste se en-
cuentra dentro del punto focal del
ocular que actúa como amplificador;
de ahí que el observador reciba al
final una imagen virtual amplifica-
da. En la Fig, 1 se muestra el esque-
ma de principio en que se basa el
telescopio de Galileo, que tiene
como ocular una lente divergente. Y> objetivo

Para formar imágenes de estrellas distantes es preciso que el telescopio reciba o recoja
suficiente energía luminosa. La cantidad de luz que entra en un telescopio de refracción puede
aumentar si se aumenta el tamaño del objetivo. Sin embargo, este método presenta limitaciones
físicas en el pulido de la lente, además que hay defectos materiales.
TELESCOPIO DE REFLEXION.- En este aparato el objetivo es un espejo cóncavo; recoge y enfoca
la luz. En los telescopio modernos, el espejo reflector es parabólico y con frecuencia
486 Física - Primer Nivel Fé/íx Aucallanchí V.

plateado. Existen dos tipos de telescopio según su visión sea lateral o directa. En los de visión
lateral, si el ocular estuviese frente al foco del espejo, en el eje óptico de este, la cabeza del
observador ocasionaría una enorme obstrucción a los rayos de luz incidentes. Para obviar tal
inconveniente se coloca en el cono de rayos reflejados por el espejo y antes de llegar al foco un
prisma de reflexión total o un espejo de perfil elíptico, llamado secundario; éste traslada la
imagen focal al exterior del tubo del telescopio, en donde se aplica el ocular. Este tipo de tele-
scopio se denomina newtoniano, por haberlo inventado Newton en I 688. Entre los telescopios
de visión directa cabe mencionar el gregoriano inventado por Gregory en 1 663, y el
cassegrainiano, que lo fué por Cassegrain en 1672. Ambos tienen el espejo principal agujerea-
dos en el centro, que está situado en el eje óptico frente a aquel, refleja el cono de rayos proce-
dentes del mismo a dicho agujero, concentrándolos en él y obligándoles a atravesarlo, con lo
que se forma el foco al otro lado, en donde está el ocular. En el modelo de Gregory el espejo
secundario es cóncavo, está colocado más allá del foco primero del espejo y proporciona imá-
genes derechas; el de Cassegrain es convexo, está colocado entre el espejo y su foco primero y
proporciona imágenes invertidas. Los dos tienen el espejo de sección circular.
Para hacer la imagen fácilmente accesible alojo humano o a una cámara, los rayos pueden
ser desviados mediante un espejo plano hacia el lado del tubo y observarse con la imagen ampli-
ficada por un ocular; pero si el telescopio es lo bastante grande, la observación puede realizarse
dentro del tubo del mismo. Esto se hace con el telescopio de los Hale Observatories, con un
diámetro de espejo de 5,1 m, situado en el Monte Palomar (California), tal como se muestra en
la Fig. 2. En ella la flecha muestra la ubicación del ocular.

Con telescopios de dimensiones gigantescas, y con radiotelescopios como el de Arecibo


(Puerto Rico), mostrado en la Fig, 3, se consiguen alcances del orden de 1025 m, los cuales han
sido capaces de ubicar al agujero negro más cercano a la Tierra, que se encuentra en la Constela-
ción de Sagitario.

Fig.3
Fig.2
Clave de respuestas 487

Creo oportuno señalar aquí algunas recomendaciones para quienes desean revisar el resultado
obtenido en la resolución de los ejercicios planteados en este texto, para lo cual dividiré las recomendaciones
según el caso:

1) Preguntas de autoevaluación.- Todas estas preguntas pueden absolverse en base a la teoría vista en el
capítulo correspondiente: Conceptos, definiciones, cuadros de valores, figuras y las recomendaciones
que aparecen en las columnas. Las primeras de cada serie son relativamente sencillas, pues solamente
apuntan a reforzar los conceptos y/o las definiciones. Las siguientes deberán analizarse de modo que toda
afirmación o negación planteada deberá responderse teniendo en cuenta si su grado de verdad cubre
todos los casos físicos posibles; de este modo, si encuentran por lo menos un caso donde tal proposición
deja de tener validez, entonces proceder a marcar lo que se solicita: Hacer ésto con mucho cuidado!.

2) Preguntas de Nivel l.- Estos ejercicios se han planteado con la finalidad de afirmar un concepto, principio
o una fórmula física. En todos los C~lSOS.el valor de la aceleración de la gravedad deberá considerarse
como 10 mls', Los valores allí planteados se han elegido de modo que las operaciones a realizar sean lo
suficientemente sencillas: Sumas. restas, multiplicaciones o divisiones. En lo posible, recomendamos
utilizar aquí los triángulos rectángulos piragóricos en lugar de las razones trigonométricas: Seno, coseno
y tangente, aunque ésto lo dejamos a criterio del profesor.

3) Preguntas de Nh'eI2.- Los problemas se plantean de modo que se da pordescontadoqueetesitidi~nté ya


superó con éxito las preguntas de Nivel l. Estos ejercicios apuntan principalmente a resolver casos físicos
puntuales, para lo cual deberán manejarse como mínimo fórmulas y ecuaciones con relativa facilidad. A
nuestro juicio, un estudiante que supera las preguntas de este nivel en una escala de evaluación de la A
hasta la E, tendrá calificación B.

4) Preguntas de Nivel 3.- Estas preguntas están dirigidas especialmente a aquel grupo de estudiantes que
tienen vocación por aquellas especialidades vinculadas en gran medida a las Ciencias e Ingeniería. El
nivel de dificultad es relativamente alto, por lo que deberán utilizarse todos los conocimientos adquiridos
en los capítulos anteriores inclusive, pero además demandan una buena dosis de imaginación. Recomiendo
tanto a los estudiantes como a los profesores el texto: "Problemas de Fiuc« y eó•• o Resolverlos", de la
misma colección, donde encontrarán ejercicios similares, que están completamente resueltos por uno, por
dos y hasta por tres métodos distintos.

BUENA SUERTE !!!

CAPITULO 1: INTRODUCCION NIVEL 1:1.1. Fl'cos9/E 1.2. E/" 1.3,pV/IITl.4. 111


A UTO 1;;VA LUACJON: 1.5.m/V 1.6. F(P/mM 1.7. Q/me 1.8. 1/lJlle! 1.9. R(b/a)
1.10. JR;~ 1.11. j2h/ g 1.12. L3 1.13. LT'¡ 1.14. LT'!
1.15. L1Mi1 1.16. L1Mi3 1.17. LM131·1 1.18. MI2J.I

1.19.JL-21.20. [b] = L7M-IT-2 1.21.[c1 = L-'MT2 1.22. rB] = L-IMil1.23. rh] = L1Mil 1.24.[GI =
L3M-1T-2 1.25. [R1 = L2~1T-19-IN-I 1.26. U] = T-1 1.27. [E] = L-.lM-IT~ll 1.28. [B] = L41.29. [k] = L2T
488

1.30. 14 1.31. 361.32. el / ~v¡ + v~ 1.33.3 1.34.71.35.5 1.36. x = 32; Y = 4 1.37. x = 240; y = 5 1.38. .r =
5;y=31.39.x= l;y=-11.40.t= 1 y t=5 1.41.t=6yt=-141.42.t=6yt=41.43.t=8yt=-21.44.
t=3yt= 11.45.J=401.46.t=2+J21.47.a=6;b= 101.48.a=20;b= 161.49.a= 12;b=91.50.
a = 20; b = 121.51. a = 150; b = 1201.52. a = 16; b = 121.53. a = 20..J2;b = 20 1.54. a = b = 101.55. a =
8; b = 4 1.56: a = 16; b = 81.57. a = 10; b = 5 1.58. a = 20fi; b = 20 1.59. a = 37°; ~ = 53° 1.60. a =
~ = 45° 1.61. a = 30°; ~ = 60° 1.62. a = 53°; ~ = 3?D1.63. a = 37°; ~ = 53° 1.64. a = 53°; ~ = 37° 1.65.
~ = 30°; a = 60° 1.66. a = 60°; ~ = 30° 1.67. a = 60°; ~ = 30° 1.68. a = ~ = 45° 1.69. a = ~ = 45° 1.70. a
= ~ = 45° 1.71. D 1.72. e 1.73. B 1.74. D 1.75. A 1.76. E 1.77. B 1.78. E 1.79. A 1.80. D
CAPITULO 2: MEDICIONES - ANALISIS DIMENSIONAL
AUTOEVALUACION:

NIVEL 1,2 Y3:

CAPITULO 3: ANALISIS VECTORIAL


AUTOEVALUACION:

NIVEL 12 Y3:

CAPITULO 4: CINEMATICA
AUTOEVALUACION:
Clave de respuestas 489

NIVEL 1, 2 Y 3:

CAPITULO 5: CAlDA LIBRE VERTICAL· MOVIMIENTO PARABOLICO

=1
A UTOEVALUACION:

=
NIVEL 1,2 Y 3:

CAPITULO 6: CINEMATICA CIRCULAR


AUTOEVALUAClON:

NIVEL 1 ,2 Y 3:

CAPITULO 7: ESTATICA 1
A UTOEVALUAClON:
~.

4sx:J

NIVEL 1,2 Y 3:

CAPITUL08: ESTATICAII
AUTOEVALUACION:

NIVEL 1, 2 Y 3:

CAPITUL09: DINAMICA LINEAL Y CIRCULAR


AUTOEVALUACION:

NIVEL 1, 2 Y 3:

CAPITULO 10: TRABAJO Y POTENCIA


A UTOEVALUACION:
Clave de respuestas 491

NIVEL 1, 2 Y 3:

I v'l V~I Vj I V~ I Vj ~O~l~l~tE{r~


I'~ I'j I'~ I 'j I'v~

I ;j I ~~ I ~; I ~; I ~~ ~fJ!K~PB
CAPITULO 11: ENERGIA
A UTOEyALUACION:

~
~
NIVEL 1,2 Y 3:

CAPITULO 12: CANTIDAD DE MOVIMIENTO


AUTOEVALUACION:

NIVEL 1,2 Y 3:

CAPITULO 13: GRAVITACIONUNIVERSAL


A UTOEVALUACION :

NIVEL 1
492

CAPITULO 14: OSCILACIONFS

A UTOEVALUACION:

NIVEL 1, 2 Y 3 :

CAPITULO 15: FLUIDOS

AUTOEVALUACION:

=
~
NIVEL 1 , 2 Y 3 :

CAPITULO 16: TERMOMETRIA - DILATACION


AUTOEVALUACION:

NIVEL 1,2 Y 3:
Clave de respuestas 493

CAPITIJLO 17: CALORIMETRIA


AUTOEVALUACION:

NIVEL 1 , 2 Y 3 :

CAPITIJLO 18: TEORIA CINETICA DE LOS GASES


AUTOEVALUAClON:

NIVEL 1 ,2 Y 3 :

CAPITUW 19: TERMODINAMICA


AUTOEVALUACION:

NIVEL 1 ,2 Y 3 :
494

CAPITUL020: ELECTROSTÁTICA

AUTOEVALUACION:

NIVEL 1 , 2 Y 3 :

CAPITULO 21: CAPACIDAD ELECTRICA


AUTOEVALUACION:

NIVEL 1 , 2 Y 3 :

CAPITULO 22: ELECTRODINAMICA


AUTOEVALUACION:

NIVEL 1 , 2 Y 3 :
Clave de respuestas 495

CAPITULO 23: ELECTROMAGNETISMO


AUTOEVALUAC/ON:

NIVELl,2Y3:

CAPITULO 24: ONDAS ELECTROMAGNETICAS y LUZ


AUTOEVALUAC/ON:

NIVELl,2Y3:

IMPORTANTE:
Las personas interesadas en adquirir todas nuestras publicaciones pueden acercarse
a nuestra oficina de ventas:

Jr. Ollero s 4976 Urb. Parque Naranjal - Los e livos

Telfs. 522-1634
865-7636
496

1.- Unidades de las Magnitudes Físicas y sus 14.- Física Fundamental - Tomos 1 y 2
Dimensiones Michel Valero
L. A. Sena Editorial orma - 1983
Editorial MIR-Moscú - 1977
15.- Física - Tomos 1, 2, 3 Y4
2.- Elementos de Física y Química A. V . Piórishkin, N.A. Ródina, G. Ya Miákishev,
Alberto Máistegui y Jorge A. Sabato B.B Bújovtsev. _
Editorial Kapelusz - 1979 Editorial MJR-Moscú - 1982
3.- Curso de Física: Orientación Universitaria 16.- Física, Tomos 1 y 2
Fernando Marín Alonso Physical Science Study Committee (PSSC)
Proyecto MT62 - Alhambra - 1978 Editorial Reverté S.A - 1973
4.- Manual Programado de Física Fundamental 17.- Física con Aplicaciones
Jay Orear Jerry D. Wilson
Editorial Lirnusa - Wiley S.A. - 1971 Editorial Mc. Graw - Hill - 1991
5.- Introducción a la Física - Tomos I y 11 18.- Física, Tomos 1 y 11
Marcelo Alonso y Virgilio Acosta Paul A. Tipler
Ediciones Cuitural - 1975 Editorial Reverté S.A - 1974
6.- Física Curso Elemental - Tomos 1, 11, III Y IV 19.- Física Clásica y Moderna
Marcelo Alonso. W. Edward Gettys, Frederick J. Keller, Malcolm
Cultural Centroamericana, S.A - 1968 1. Skove.
7.- Física I y 11: Introducción a la Física Editorial Mc. Graw - Hill, 1991
Alberto P. Maistegui, Jorge A. Sabato 20.- Física
Editorial Kapelusz - 1972 Donald E. Tilley, Walter Thumm
8.- Física General con Experimentos Sencillos Fondo Educativo Interamericano S.A., 1979
Beatriz Alvarenaa Alvares, Antonio Maximo 21.- Física - Fundamentos y Fronteras
Ribeiro da Luz. ~
Robert Stollberz, Faith Hill
Editorial HARLA - 1983 Publicaciones Cultural S.A. 1971
9.- Tratado de Física 22.-Física en Perspectiva
Kleiber Karsten, Alt. Eugene Hecht
Editorial Gustavo Gili, S.A.- 1962 Addison - Wesley Iberoamericana, 1987
10.- ¿La Física? - Pero si es muy Fácil 23.- Física
Fred Klinger Jrwing Genzer y Philip Yougner
Marcombo Boixaren Editores - 1970 Publicaciones Cultural S.A. 1975
11.- Física: Tomos 1 y 2 24.- Física Fundamental
Robert L. Weber - Kenneth V. Manning, Marsh W. Jay Orear
White. Editorial Limusa - Wiley, S.A., México, 1970
Editorial REVERTE - 1970
25.- Física Avanzada Nuffield - Guía del profesor
12.- Física
Copyright © by the Nuffield Foundation.
Wayne E. Hazen - Robert W. Pidd Editorial Reverté S.A., 1984
Editorial Norma, Colombia - 1969
26.- Física: Fundamentos y Aplicaciones
13.- Elementos de Física Clásica - Libros 1 y 2
Robert M. Eisberg: Lawrence S. Lerner
R. T. Weidner, R. L. Sells Editorial Me Graw - Hilllnc. U.S.A .. , 1981
Publicaciones Cultural S.A. - 1981
4CJ7

27.- Physics for You, OLevel Edition 41.- Algebra Vectorial en Ejemplos y Problemas
Keith lohnson G. Gosiántnikov, S. Reznichenko
Hutchinson Group, 1980 Editorial MIR-Moscú 1985
28.- Física Conceptual 42.- Mecánica Newtoniana
Paul G. Hewitt A.P. French. MIT - Physics Course
2da Edición 1995 Adisson - ,Wesley (Massachusetts Institute of Technology)
Editorial Reverté S.A. 1974
29.- Conceptos de Física
Paul G. Hewitt 43.- El Mundo de la Luz
Editorial Limusa - 1996 Alexander Efron
Editorial Bell S.A .. , Buenos Aires, 1971
30.- Física 1 y II
Serway - 3ra Edición - 1993 44.- ¿Qué es la Teoría de la Relatividad?
Mc Graw Hill L. Landau. Y. Rummer
Editorial Ricardo Aguilera, 1974
31.- Física 1 y 2: Principios y Problemas
Paul W. Zitzewitz, Robert F. Neff 45.- La Relatividad y los Quanta
Mc Graw Hill, 1995 Oscar Miró Quesada (RACSO)
CONCYTEC, 1983
32.- Enseñanza de la Física
. Enrique Loedel - Editorial Kapelusz 46.- La Explosión de la Relatividad
Bibliotecas de Ciencias de la Educación Martin Gardner
Biblioteca Científica SALVAT, 1987
33.- Física para estudiantes de ciencias e
Ingeniería Volúmenes I y II 47.- Algo Ameno e Interesante sobre Cosmogonía
Fishbane, Gasio Rowicz, Thornton A.N. Tomilin
1994 Prentice - Hall Hispanoamericana. S.A. Editorial MIR-Moscú 1979
34.- Física Creativa y Recreativa - Tomos 1, 2 Y 3 48.- Super Fuerza
Eliezer Braun Paul Davies
Editorial Trillas - 1983 Biblioteca Científica SALVAT, 1986
35.- Física Recreativa: La Feria Ambulante de la 49.- El Universo Desbocado
Física Paul Davies
Jearl D.; Walker Biblioteca Científica SALVAT, 1985
Noriega Editores - L1MUSA, 1990
50.- ¿Qué es la Mecánica Cuántica?
36.- Física Recreativa -Tomos 1 y 2 V.J.Ridnik
Yakov Perelman Editorial. MIR-Moscú 1977
Editorial MIR - Moscú, 1982
51.- El Universo de Stephen IJawking
37.- Paradojas y Sofismas Físicos John Boslough
V. Langue Biblioteca Científica SALVAT, J 986
Editorial MlR - Moscú, 1978
52.- Historia del Tiempo: Del Big Bang a los
38.- La Física en Preguntas Agujeros Negros
lean Marc Levy - Leblond y André Butoli Stephen W. Hawking
Alianza Editorial, J 988 Alianza Editorial, 1988
39.- Atlas de Física 53.- Biografía de la Física
J. Fernandez Ferrer George Gamow
Ediciones Jover, S.A. - J 966 Alianza Editorial, 1980
40.- Atlas de Prácticas de Física y Química 54.- Dios y la Nueva Física
E. Seba, y A. Roca Paul Davies - Biblioteca Científica SALVAT
Ediciones Jover, S.A. - J 988 Barcelona, 1986
498

A NUESTROS CI.IENTES
RACSO EDITORES pone en su conocimiento que ya están a la venta la siguiente
relación de obras :
1) FISICA - Primer Nivel
de Félix Aucallanchi V. (500 Pág.)
2) GEOMETRIA - Primer Nivel
de Ernesto Quispe R. (470 Pág.)
3) TRIGONOMETRIA - Primer Nivel
de Juan Carlos Sandoval P. '(500 Pág.)
4) MATEMATICA 1 - para secundaria
de J.C. Sandoval y A.Tori L. y Yuri Calle R. (286 Pág.)
5) MATEMATICA 2 - para secundaria
6) MATEMATICA 3 - para secundaría
7) MATEMATICA 4 - para secundaria
8) MATEMATICA 5 - para secundaria
9) PROBLE.MASDEALGEBRA .... ycómor
de A.Tori L. y 1. C. Ramos L. (650 Pág.)
lO)PROBLEMASDEARITMETICA •... ycómo
de Hernán Flores Velasco (720 Pág.)
11)PROBLEMAS DEGEOMETRIA •.w y cómo resolverlos
de Ernesto Quispe R. (820 Pág.)
I2)PROBLEMASDEFISICA ••.• ycómoresolverlos
de Félix Aucallanchí V. (866 Pág.)
13) PROBLEMAS DE RAZONAMIE TO MATEMATICO ... y cómo resolverlos
de Armando Tori Loza. .

'P'ROXIMAS PlIBI.ICACIONES
14) PROBLEMAS DE QUlM1CA .... ycómo resolverlos
15) PROBLEMAS DE TRIGONOMETRIA .... y cómo resolverlos
16)ALGEBRA ler. NIVEL
17) QUIMICA ler. NIVEL

Вам также может понравиться